Skip to content

Instantly share code, notes, and snippets.

@alexalemi
Created April 22, 2015 02:49
Show Gist options
  • Star 0 You must be signed in to star a gist
  • Fork 0 You must be signed in to fork a gist
  • Save alexalemi/358860973176564b5e75 to your computer and use it in GitHub Desktop.
Save alexalemi/358860973176564b5e75 to your computer and use it in GitHub Desktop.
Stackexchange answer for optimal path for fuel efficiency
Display the source blob
Display the rendered blob
Raw
{
"cells": [
{
"cell_type": "markdown",
"metadata": {},
"source": [
"# Most Fuel Efficient Road\n",
"\n",
"This notebook contains code and some discussion created to address the [stackexchange question](http://physics.stackexchange.com/questions/6220/can-a-car-get-better-mileage-driving-over-hills)\n",
"\n",
">##Can a car get better mileage driving over hills?\n",
">Two towns are at the same elevation and are connected by two roads of the same length. One road is flat, the other road goes up and down some hills. Will an automobile always get the best mileage driving between the two towns on the flat road versus the hilly one, if operated by a perfect driver with knowledge of the route? Is there any hilly road on which a better mileage can be achieved?"
]
},
{
"cell_type": "code",
"execution_count": 1,
"metadata": {
"collapsed": true
},
"outputs": [],
"source": [
"import numpy as np\n",
"import scipy as sp\n",
"import matplotlib.pyplot as plt\n",
"%matplotlib inline"
]
},
{
"cell_type": "markdown",
"metadata": {},
"source": [
"# Car Model"
]
},
{
"cell_type": "markdown",
"metadata": {},
"source": [
"We begin by defining some parameters for our car model, from an [EPA report](http://www.epa.gov/oms/models/ngm/420p05001.pdf) for a 2004 Honda Civic."
]
},
{
"cell_type": "code",
"execution_count": 2,
"metadata": {
"collapsed": true
},
"outputs": [],
"source": [
"A = 105.47 #N\n",
"B = 5.4276 #N/mps\n",
"C = 0.267 #N/mps^2\n",
"m = 1239 #kg\n",
"g = 9.8 #m/s^2\n",
"a = 3.19 #0.15 * g\n",
"b = 7.193 #0.6 * g\n",
"P0 = 6000 # Watts\n",
"\n",
"mphfact = 2.237\n",
"gallonperj = 9.478e-4 / 114000.0 / 0.34"
]
},
{
"cell_type": "markdown",
"metadata": {},
"source": [
"This hypothetical vehicle feels a friction force, a rotating friction, and air drag\n",
"\n",
"$$ f = A (v > 0) + B v + C v^2 $$\n",
"at constant velocity, the power consumed by the vehicle will be\n",
"$$ P = P_0 + Fv = P_0 + A v + B v^2 + C v^3 $$\n",
"if we are concerned with the energy to go some constant distance, we have that\n",
"$$ U = \\int dt\\, P = \\int dx\\, \\frac{dt}{dx} P = \\Delta x \\frac{P}{v} $$\n",
"so that our fuel efficiency is given by\n",
"$$ \\frac{\\Delta x}{U} = \\frac{v}{ P } $$\n",
"\n",
"using the numbers from the EPA report, we can see the fuel efficiency of this vehicle"
]
},
{
"cell_type": "code",
"execution_count": 90,
"metadata": {
"collapsed": false
},
"outputs": [
{
"data": {
"image/png": [
"iVBORw0KGgoAAAANSUhEUgAAAYwAAAEbCAYAAADJWrOxAAAABHNCSVQICAgIfAhkiAAAAAlwSFlz\n",
"AAALEgAACxIB0t1+/AAAIABJREFUeJzt3XmYHFW9//H3CUsIm2wSmDPFclkFZHFALwhYEVxQAwim\n",
"ZJFdURTUK9f1CkZQURGE6wYIwgVxKdaIEIEJKbYfCkEQwiYgwZojkTUk7ISc3x/nNKnpVE+6Oz19\n",
"ama+r+fpp7urTld/0jPp71TVqXOUtRYhhBBiacaFDiCEEGJkkIIhhBCiKVIwhBBCNEUKhhBCiKZI\n",
"wRBCCNEUKRhCCCGasnzoAEqpzYD7gT2stTf5ZXOBBwrNbrHWnhAinxBCCCd4wQBOAl6tWzbdWntE\n",
"iDBCCCHKBT0kpZTaCVgAPBUyhxBCiKULfQ5jKvDtkuVbKaWuUUrdrJQ6Wym1dpdzCSGEqBOsYCil\n",
"9gJmW2tNyerZQGKt3Q14Hpje1XBCCCGWoEKMJaWUGgfMAD5qrZ2nlHoMOKx20ruu7crAfGBna+0d\n",
"XY4qhBDCC7WHcRBwrbV2XmGZKmtorX0JeBbYsBvBhBBClAvVS2pXYBul1Af88/WAM5RSjwNnAvOt\n",
"tXcCKKVWBNYE/lW/kRkzZshQu0II0YY99tij9I/0IVlrg9+Ax4Dd/ePDgHML647HXZOxXP3r+vv7\n",
"bejs9bfNN9/8gtAZRkouySSZxkKuKmZq97sz6HUYSqntgBOAdYCTlFJXAJcDuyulbsYdppoPTLbW\n",
"vhEuafPWX3/9jUJnKFPFXJKpOZKpeVXMVcVM7QpaMKy1fwM+VrLqqG5nEUIIMbTQ12GMOn19fXNC\n",
"ZyhTxVySqTmSqXlVzFXFTO2SgtFhkydPnhM6Q5kq5pJMzZFMzatiripmapcUDCGEEE2RgtF5WegA\n",
"DWShA5TIQgcokYUOUCILHaBEFjpAA1noACWy0AE6JciV3p0yY8YM21ZfYiGEGMPa/e6UPQwhhBBN\n",
"kYIhhBCiKVIwhBBCNEUKhhBCiKZIweiwLMvi0BnKVDGXZGqOZGpeFXNVMVO7pGB0Xhw6QANx6AAl\n",
"4tABSsShA5SIQwcoEYcO0EAcOkCJOHSATpGCIYQQoilSMIQQQjRFCoYQQoimBB3eHEAptRlwP7CH\n",
"9XN6K6W2B34BvIGbnvVwa+2z4VKKMWPRIoyO1gY0sD6wNvAW3KyPawCrAOP9bUV/e8PfFvn7V4AX\n",
"gRf8bQHwtL89VXusTf5q1/5dQnRA8IIBnAS8+R/HT8k6DTjUWnujUmoqcBaQhInXsix0gAay0AFK\n",
"ZKHe2OhIAROBzYHN/P3G/7HKKm8DPtKlDE8D/wRy4PHC/ePa5MUJw7Ju5GlRFjpAA1noACWy0AE6\n",
"JehYUkqpnYBPAe8DDrPW3qSU2gc43Vq7iW/TC8wBJlprnym+XsaSEs0yOloO2ALYwd+2x+0xVNGr\n",
"wCPA3wu3h7TJXwmaSowa7X53ht7DmAocjSsYNTvh5vAGwFo7oJR6CXgHcH1X04kRzR9aejewG/Au\n",
"YOUmX/oCMAD8C3cI6XngOWAe7vDSq8Br/v513LnAcbj/T8sBK+EOXa3q79fATUP8Vtwhrtp9o/9/\n",
"44Gt/a1modHRQ8A9tZs2+b+b/PcI0RHBCoZSai9gtrXWKDWo0E3EzeNdNA9Yt1vZxMhldLQ+8EFc\n",
"3/eth27NAtxf7w/7+0eBXJu8/vev4/wez3rAhkAEbOBvm1D+u748i4vIgX4bc4HbgTuA27XJnyl5\n",
"nRAdE6RgKKXGAV8BPlqy2vrbEi8b1lBixDI6Wg3YA9gL6Bui6VzgrsJtjjZ5kGOy/hyF8bdBjI7W\n",
"xJ1X2QJ3buVtwEYlm1kP2NvfMDp6FF88gFna5C8NR3YxdoXawzgIuNZaO6+wrFYQnsQdYy5awy9f\n",
"QhzHWe1xX1/fHD8dYlZcXuMv0Y9LNiPtR2B7o6MtcZ0hPgCMf2PChNUWTZiw2putlFr0+rrrzl00\n",
"YcK1q/71r+cA/ywWiCzL4ofLh20I+++9+KLi4jeAn2528CF3AdsA2/rbNm9MmLDuoH8v9AC7jXv5\n",
"5QXLvfzys0ZHs4CbgZu1yZ8I/fOS9uHaT5o0ieK6/v7+kmZLF+Skt1LqLNwv/+t+0X8CD+JObl8A\n",
"nGat3dS3jYDHGCEnvbMsi8t+mKFVMVc7mYyOlgfeC3wc2K6kySLgL8B04JZWDy+NlM/JH9LaGnfO\n",
"7524z2KoPwAfwRWPm4D7tMkXdTpTFVQxVxUzjaiT3tbazxSfK6UeA77ge0mNB85USu3ur8s4Eris\n",
"vlhUWEw1u9HFVC9XTJOZjI5WxHV3PRz3l3S9h4Crgeu0yZ/uRqYuiqnL5A9p1U6An2d0NAHX82sn\n",
"YGfcIa2iTf3tCODfRkczgBnAvW0WjyUyVURM9XLFVC9TW4L2klJKbQecgOtBcpJS6gpr7ZlKqX2B\n",
"nyul3rxwL2BMEZDR0XhgH+AwXIeIooVAP/B7YHao8xFVoE3+MnCbv/2v0dF6uN5hu+KKyIqF5hNx\n",
"h4UPojPFQ4wRQQuGtfZvwMdKlt8N7NL9RKIqjI7GAR8GjmHJXkPzcEXicukZVE6bfC5wCXCJ0dHK\n",
"uMNWuwHvYfD1J/XFox+4Wpv8712OLEaA0NdhCLEEo6OdgC/iegkVPQNcBFzm/6IWTfC9pTIgMzo6\n",
"BdgR2BOYhBv2pGYicDBwsNHRI8A1wHRt8qe6m1hUlRQMURlGR73Al4Dd61Y9A5wPXCHjLy0bbfKF\n",
"wJ+BPxsdfZ/GxWNT4PPAsUZHd+DOD82UQj22ScHovCx0gAay0AFKZPBmz6dP4IaJGV9Y/wrwa+DC\n",
"Ll5TkHXpfVqRDcdGS4rHO4EP4U7SruSbjcNdJf8u4GWjo+uBK7nwgmHJ1AFZ6AAlstABOiXoWFLL\n",
"qordakVrjI7eDvwP7i/aoquBn8vwF91ndLQKruvyh3B7IGX/xx7FDRJ6tTb5812MJzpgRHWrFcL3\n",
"fjoWOIDBX0gPAd/TJr8vSDCBNvmLwFXAVUZHE3FX0H8Y2LjQbBPc4cPjjI5uAK4E7pReVqObFAzR\n",
"dUZHWwAnA/9RWPwKcDbwm7qhvUVAfg/vAqOj/8NdbLsv8H5ggm+yAu5K+w8AA0ZH04A/SO+10UkK\n",
"huga31X2E8BnGfy7dxvwfW3yJcZVEtXgr3G5F7jX6Oh0XNHYl8EDPPYCnwM+7bvnXoIbVXfkHvcW\n",
"g0jBEF1hdLQO8B3cMfGaV4DTcb2f5EtlhPCHrK4ArjA62hx3YeWHgNq4VsvjRgz+IPCQ0dGlwJ+k\n",
"h9XIJ3N6d1hWPphdcCFzGR1tj+vtVCwW98/97DE/0Sa/vErFooo/vypn0ib/uzb5qbjicCJuqJKi\n",
"LXCdGqYbHX3J6GjDbuSqkipmapcUjM6LQwdoIO72GxodKaOjg3BT7K7jFy8CzgOOXPDuXd7W7UxN\n",
"iEMHKBGHDlAiLj7RJn9Vm/wabfIjcRf/TcPtQdasirua/DKjo58ZHb3HD6A4rLkqIg4doFOkYIhh\n",
"4Yej+C6uJ03t0Oc84Fht8l/4awDEKKRN/pA2+cm4w1Q/xs1VXvQu4DRc8TjQd+MVI4AUDNFxvivm\n",
"ubgTozWzgYO1yW8Pk0p0mzb5fG3yi4H9geNwQ6sXDz/2AscD1/jDVTpATNECOektOspPavRj3LzV\n",
"NZcBp2mTvxYmlQjJX5txG3Cb0VEProB8FFjdN1kFd7jqAKOjG4HfAndV6dyWcKRgiI4xOtoddxiq\n",
"1kd/Ia677JXhUokq0Sb/F/ATo6NzcYesDmTx9LPjcGNaTQIeNDr6DXC9NvnrZdsS3Rf0kJRS6jil\n",
"VL9SaoZS6h6l1LGFdXOVUjMLt5NDZm1BFjpAA9lwbtzo6OPAj1hcLF4APr+UYjGsmdqUhQ5QIgsd\n",
"oES2LC/WJn9Zm/wy3BS7n8eNaVW0JXAS7mrzT/p5zoc91zDJQgfolKBjSSml7gb2tNY+rZTaFHgA\n",
"2NlaO0spdb619oihXi9jSYVndKSAzwBHFRcDX9QmfyxMKjESGR1tgpt698MMHoQS4FXgj7iRAB7v\n",
"drbRpt3vztAnvT9hrX0awFr7CPAci3dPRcX5K7f/m8HF4l7gCCkWolXa5I9qk38PVzB+BhTn4RiP\n",
"O/dxqdHRj4yOyuZzF8Ms9Ix7s2uPlVL7AwuA6/yirZRS1+CuHr0f+MYImtd71PNDkp+IOw5dcyvw\n",
"VW3yV8pfJcTSaZPPA843Ovo1sAfuuo7aNTsKd11DbHR0D25CrZtk/LHuCL2HgVJqG6XUfbjj3wda\n",
"a+f7VbOBxFq7G/A8MD1URjGYH2n2BwwuFtcB/y3FQnSKNvnr2uR/Ag4FPg3cUtdkW+BU3DS0+/vf\n",
"SzGMghcMa+1sa+3WuG5205RSO/nlR1lrX/DNpgLvqK0T4RgdrQj8EDc3dM3lwAnSm0UMB21yq01+\n",
"pzb5F3EnyacBxd+1DYCvA1cbHR3dwgly0aJKTaCklDoXGG+tPaRk3ZPAZ621l9aWzZgxw5588sk3\n",
"1p739fXNmTx58hwgi+M4q9+GH9MlLnnrTrY/HJgzjNtvt/1GhVxtbd/oaAVcsdgN4I0JE1Z7Yef/\n",
"fPTJIw6/nXHjlmjfxPZrmarw+RQzXVChPADz4jg+o0J5MoD613Qzzwpz5668xjXTt1559n1bLTd/\n",
"/mvLvfzyAnC/k8u9/PLTwFVPfO6z/3hhl5236kaeodpnWRbHcZyF/HlNmjSJ4rr+/v5vtdVhyFob\n",
"5AasBexXt+x/cX+tTgL6CstXxP1FsUuxfX9/vw2Vv9Ft5syZU0NnGI5cAz29Kwz09J420NM7q3A7\n",
"ZqCnV42mz0oyjaxMAz29qwz09B440NN71UBP76zHttn2X4XfzzsGenp/ONDTu618VoNv7X53hjwk\n",
"tTpwglJqAoBS6q24YZL7cbuYxxTaHgc8Avyl2yHFmye4v8vgw1DnA2fJ1bgiJG3yF7XJf4ubm+N/\n",
"Fq615tOF1Qo31eyvjI7O9QMeBj8MP5KF7CX1BG4ayBlKqdeANXCjmP4CN8bM7kqpm3E/9PnAZGut\n",
"9IToMj+i6Ldx//Fq/g8337YUC1EJvpfUtdkNN+y82SGHXY2bqOvdhSbb+9sc3/tqujb5qwGijmjB\n",
"Coa19lVct8wTS1bnDO7bLwLwF+Udj5t+s+Zi4KdSLEQljRuHNvkdwB3+QsBP4OYkr33XbQR8E/iM\n",
"0dHvgcu0yeeXbkssQXbPxFCOwvVKqUmBM6RYiJHAXwj4bWBv3F7xC4XV6+Cmk/2j0dF/GR2tFyLj\n",
"SCMFo/Oy0AEayFppbHS0P27Ij5prgR91uFi0lKlLstABSmShA5TIQgdoIKtfoE3+pDb5T4CPAGcy\n",
"+ArylXEXBl5pdHSS0dFm3cg0UlWqW22rZCyp4WF0tAfwfdz5I3ADw/2XXGchRgPfPfwDwCHAJiVN\n",
"bgMuBGaN1r3pkTqWlKgYo6MdgO+wuFjcB3xFioUYLfwV5H/EDa3+ReDOuiY74zrfXGh09P5hmkp2\n",
"RJL5MMSbjI4i3FALK/hFc3Cjzr4ULJQQw8RP7HQLcIvR0da4IUgmsfgP6bcB3wOM0dHFwFXa5C8H\n",
"CVsRsochADA6Wh03U94aftGzuPksnguXSoju0Ca/T5v8q/gRcXHDqb+5GvgKbm6OT4/loUekYIja\n",
"hXk/YPHQ8q8Bx/vZ0YQYM7TJc23y7wOTgV/iBj6tWQP4FK5n1deMjnpDZAxJCkaH+fFcKqdRLn+t\n",
"xdeA4sCOU7XJ7w2VKSTJ1JwqZoLO5dImf1ab/Gxcz6ofAsU/nsYDHwMuNzr6gT+cNeyZqkAKRufF\n",
"oQM0EDdYfiBuWIWaX2iTX9egbafFXXqfVsShA5SIQwcoEYcO0EDcyY35qWRT3Gja3wAeLKweh5uv\n",
"4/+Mjs42Onq3/wNsWDOFJCe9xzCjo52ALxQWXQ38KlAcISrLDz1yndHR9cCOuBPkOxea9PnbI37o\n",
"kWtHY89C2cMYo/yVracAtS6D9wLfHa39zoXoBD83xx3a5McBBwHXAAsLTTbFzd8zzejoEKOjVQLE\n",
"HDZSMMYgPzPZqSzuEfU07lqL18KlEmJk0Sb/uzb5ibhDuhcDxe7n6+L23q9e97xfvdPo6K0hMnaa\n",
"FIwxpnCSuzZH8kLga9rkTzV+lRCiEW3yudrkP8adIP8Z8Exh9aor3/237XFdck8wOto4SMgOkYLR\n",
"eVnoAA1k/v5juC6DNadrk9/d/ThANT+rLHSAElnoACWy0AEayEK9sTb5fG3y83GDHX4HeBxgnJsN\n",
"cHncfD+XGB2dbnS0Q4MT5JUWdCwppdRxuA9RAW8FzrHW/tSv2x53ef4buIvIDrfWPlt8vYwl1Rrf\n",
"/e88Fnd2+CPwbTlvIUTn+cmadsOdIN+upMm9wEXAjf6kete0+90ZupfUUcCe1tqnlVKbAg8opf4M\n",
"3IOb6P1Qa+2NSqmpwFkMHmpbtMDoaDXcMAe1n/mDwClSLIQYHn7okRuBG42OtsMVjuKslW/HXePx\n",
"T9+z6uqqT+oU+pDUJ6y1TwNYax8BnsNdbbwXsNBae6Nvdy6wn1Jq7SApRzi/6/tN3BAH4OYF+GrV\n",
"fzmFGC20yf+mTX487pDwlUCxy+0GuGs8rjI6OtLo6C0hMjYjaMGw1s6uPVZK7Q8sAK7DXXX8QKHd\n",
"AK4Hwju6nXGUmIK7wKjmZG1yEyqMEGOVNvkcbfLv4M4jno/7zqtZC/gsbuiR442O1g+RcSih9zBQ\n",
"Sm2jlLoP+BFwgLV2PjARN4930TxcVzXRAqOjLYH/Kiy6RJt8Rqg8QgjQJn9am/xnuJ5VpwNzC6sn\n",
"4EZguNLo6DtGR1uEyFgmeMGw1s621m6Nu/T+D0qpnQDrb/Uqf4K7SuPG+IuGTgFWeGPChNWAh4Az\n",
"wqZarEqfVY1kak4VM0E1cw2VSZv8RW3y3+Cu5TgReLiwejngg8DFRkc/Mzp6Z+ieVaFPer/JWnu3\n",
"Uupq4PO47mg71DVZA3iy/nVxHGe1x319fXMmT548B8iKy2v8Dy4ueftOtp+aZVn9uiB5Xtl4458u\n",
"9/zz7wJ4Y7XVVjZHHfHkq5ts8vWHsyzk51NsH/vPqip5apmmVigPuPN6VcqT+eWDXhMyT6H9m7kq\n",
"kufNTEO2N3kGXGN0NB34T+DQNyZMeO8i94ceuD+oP/rGGms880BywG9Xv/XW0+rPQQ61/UmTJlFc\n",
"19/fX9KsCdbaIDfc8br96pb9L3A5rqvtI4XlEe4Cs7WL7fv7+22o/I1uM2fOnBo6g7WWgZ7ePQZ6\n",
"emfVbnd99WsXhc5U1c9KMo2OTFXN1W6mgZ7etw309H5voKf39uL/ZX+7bqCn9+iBnt6129l2u9+d\n",
"IQ9JrQ6coJSaAKCUeiuuUPQD04HllVK7+7ZHApdZa58p3ZIYxOhoXVyvi5pr5n3wA4+GyiOEaJ02\n",
"+QPa5N/A7V38HnilsHot4GjcCfITjY4270amkIekngCuAmYopV7DHXI6z1r7cwCl1L7Az5VSb164\n",
"FyroSOIvFvoWUOuaNxfX1/u/g4USQrTN92g81ejobNy5jo/jOgaBm055b2Bvo6NZwG+AW/w1IB0X\n",
"rGBYa1/FneQ5scH6u4FduhpqdPg48C7/2AInapO/8PASp1WEECOJNvl84EKjo98A78X1pHp7ocmO\n",
"/pYbHf0W+KM2+UtLbql9wXtJjUJZqDc2OtoEOK6w6EJt8r/6x1n3Ey1VFjpAiSx0gBJZ6AAlstAB\n",
"GshCByiRdXJj2uQLtcmv0yY/AjgCd+1acWiRCDcH+TVGR1/o5PUcQceSWlYyltRifl7uC4At/aKH\n",
"gMNH4yQuQojBjI4m4oZO2g9YrW71ImAm7nDVPdrktt3vTtnDGD0OY3GxeA04QYqFEGODNvm/tcl/\n",
"AnwY+AF+pFyvNpXsecAFRkcfavd9pGCMAkZHmwKfKiz6hTb5P0LlEUKEoU3+kjb5JbjhgL4I/KWu\n",
"ydbASe1uvzIX7on2+ENRU1n8s7wXt+sphBijfC+pW4Bb/B+UB+IGdV1xWbYrexgj36EMPhT17W6P\n",
"rS+EqC5t8ke0yU/GHa76OdD27JpSMDqsm2PZ+F5RRxcWnaVNPqes7UgbYycUydScKmaCauaqSiZt\n",
"8ue0yX/F4Bk3WyIFo/PibryJ0dFyuAv0aoeiZuMmom8kHu5MbYhDBygRhw5QIg4doEQcOkADcegA\n",
"JeLQAYq0yRe2+1opGCPXAcBW/rEcihJCDDspGCOQvxDnmMKic7XJHwuVRwgxNkjBGGH8ePhfBVby\n",
"ix4BLgyXSAgxVkjBGHn2BHb1jy3wvWU5JimEEM2SgtF52XBt2OhodQaPOnupNvk9Tb4863yiZZaF\n",
"DlAiCx2gRBY6QIksdIAGstABSmShA3RKsLGklFLLA8fi5sBQuAtKvmmtvcGvnws8UHjJLdbaE4rb\n",
"GGtjSRkdfQM3Vgy4vtRTtMlfCBhJCDECtfvdGfJK717cdKzbWWsXKKX2BKYppTa31j4BTLfWHhEw\n",
"X6UYHW3H4mIB8EMpFkKIbmr6kFSSJLd3+L3nAydYaxcAWGv7cTNKyRwYdfw1F18rLLpRm3xmqDxC\n",
"iLGplT2MKEmS0smOPAu8ANyZpulNS9uYtfZZCheaKaVqh6Vql61vpZS6BjdU7/3AN8bwFK0fAzbz\n",
"j1/BzaAnhBBd1UrBmIgb5G6pkiS5EfhImqYvtrD99wBzrLW1YjMb+IK19gWl1A9x83y/s4XtjQpG\n",
"R2sz+JqL87TJ/x0qjxBi7Gqll9TuwA3AJGCtNE3HAWvjpgqcBmwPvBV4HzAB+E6zG1ZKrQR8l8K8\n",
"3dbao6y1tWP0U4F3KKV2aiFvEMMwbsyxwKr+8T8ZeviPhqoynk2RZGqOZGpeFXNVMVO7WikYJwEf\n",
"S9P0xjRN5wGkafpcmqYZbprAH6Rp+kyapjNwg1vt3cxG/aGoc4DTrbV3lbWx1r4EPAts2ELeUOJO\n",
"bcjoaFsGDxR2qjb5a21uLl72RB0Xhw5QIg4doEQcOkCJOHSABuLQAUrEoQN0SiuHpNatFYp6aZrO\n",
"S5Kkt/D8qSRJXm5yu6cBf7bWXqaUWhF36GtTYL619k4Av3xN4F/1L47jOKs97uvrmzN58uQ5QFZc\n",
"XuMrfVySoaPtsyybuqzb3+zgQ27GzctbM/Phiy8a//CS2252+8VcQT+fwqJapqrkqWWKK5QHYKOS\n",
"ZSHzLNE2dB5pP3T7SZMmUVzX399f0qwJ1tqmblOmTHlyypQpWzVYt9WUKVOeKjxfYcqUKQ8ubZu4\n",
"IS7OBFbBHXbZCjcC62HAuYV2x+OuyViu+Pr+/n7bbP5u3WbOnDm1E9sZ6OmdMtDTO8vfbh3o6e2p\n",
"Qq4qflaSSTJVOVcVM7X73dnKHsYVwA1JkvwM+DPuENHawM7AZ4FLAZIk2QB3zuGJoTamlNocOMU/\n",
"Pa6wairuXMnuSqmbcRf1zQcmW2vHxGisRkdr4j7TmvO1yZfYuxJCiG5qpWB8Gde189sl6/r9+lq7\n",
"jYFfDbUxa+3fGfocylEtZBttPofrTgwwAFwUMIsQQgAtFIw0TecnSbIH8AFcT6m1gWeA/jRNry+0\n",
"O67BJsaKbFlebHS0BW64lJofaZO/ukyJnKwD2+i0LHSAElnoACWy0AFKZKEDNJCFDlAiCx2gU4KN\n",
"JdUJo20sKT90+S+AHf2iW7XJvxAwkhBiFOrKWFJJkozDjWc0CVgHeBqYAVyRpunIrTzVsTuLi8Ub\n",
"wBkBswghxCCtjCX1FuAmIMVdeTzF318K3JQkyerDknCMMDpaASjuTVwqs+gJIaqklQv3TsP1B/8S\n",
"8G5gS3//ZeA//HrRvinABv7xC8AvA2YRQogltHJIah9glzRNH65bfluSJH8AbgM+1bFkY4jR0VuA\n",
"TxYW/VKbvPQiSSGECKWVPYy5JcUCAL98yOsuxoo2x435FFA7pDcAXNKxQF4Vx7ORTM2RTM2rYq4q\n",
"ZmpXKwXjqSRJ1ihbkSTJmoCpWzZWr6OIW2lsdLQhbvjymjOXYbyoocTDsM1lFYcOUCIOHaBEHDpA\n",
"iTh0gAbi0AFKxKEDdEorh6ROANIkSU4G7k7TdIE/0b097mTtl+vaHwuc15mYo9oXWPxzuJNR1Gdb\n",
"CDG6tFIwbvb3ewIkSVK//qN1y6Sb7VIYHe2I60oL7vP6sTa5fG5CiEpqpWD8GzgLN7ZTMz7depyx\n",
"w1+kV7wq/mpt8gdD5RFCiKVppWAMpGlaNo4U4C7qS9N0UeH5R5Yp2ej3XmBr//g13BXeQghRWU2f\n",
"9E7TdGmz3c2qa79jo4ajXLa0BkZHyzF4NNrfd2Ha1WyYt9+OLHSAElnoACWy0AFKZKEDNJCFDlAi\n",
"Cx2gU1oaSypJkl1wgw+uz+C9EwXsnabp2p2NN7SROpaU0dF+wDf80xeAfbTJnw8YSQgxhgz7WFJJ\n",
"knwROB14CTdK7aLCaoWbBKlpSqnlcT2p9vGvXxH4prX2Br9+e9xhmjdwc28cbq19tpX3qCKjo5WA\n",
"owuLLpBiIYQYCVo5h3EcbviKy8sGGkySpHQ+7iH0Ap8HtrPWLlBK7QlM8xMrPQNMAw611t6olJqK\n",
"O+G+RNesEehA3MCNAE8BvwuYRQghmtZKwXghTdPLhljfaq+o+cAJ1toFANbafqXUK7jxqV4HFlpr\n",
"b/RtzwXmKKXWttY+0+L7VIYfAuSwwqJfapO/EiqPEEK0opUrvf/uR6xtpKWT3NbaZ621F9eeK6Vq\n",
"h6WeBHYCHiy0HcAdCntHK+9RQYfj5i4HeBy3FyWEECNCKwXjS8BPkyTZP0mSrZIk2aBw2xA3reiy\n",
"eA8wB3eB4ESg/rj+PGDdZXyPYddo3Bijo4nAxwuLfq5N3rU5yqs4no1kao5kal4Vc1UxU7taKRhr\n",
"AbviBsabjftyr90eww133hal1ErAd3Enti3uquey7lsjoUdU3GD50bg9KID7gBu6kmaxuMvv14w4\n",
"dIAScegAJeLQAUrEoQM0EIcOUCIOHaBTWjmH8RPc4aKf4U5K13+hN7yobyj+UNQ5wOnW2tqJ8yeB\n",
"HeqaruGXjzhGRxsAxQsZfyJDgAghRppWCsYmwCZpmpaepE2S5D1tZjgN+LO19jKl1Iq4w1F3AAfU\n",
"GiilImDhShrpAAAX4klEQVRl3OB8g8RxnNUe9/X1zZk8efIcICsur/G7hnFJho62z7JsanF5tNGG\n",
"G6805/Hl/NO/aJPP6mYe/7SYK+jnU1hUy1SVPLVMcYXygJu4bMmNhMuzRNvQeaT90O0nTZpEcV1/\n",
"f39JsyZYa5u6TZky5aalrB/X7LZqN+CrwJm4azhWBbYCvoU7dDMH2N23+xbw+/rX9/f321bfc7hv\n",
"M2fOnFp8PtDTu/FAT+8dAz29s/xt2yrkqsJNMkmmsZCripna/e5s5RzGxUmSHDrE+llDrFuCv97i\n",
"FNz1HQtw3WxnuxpmXwP2Bb6vlLoZ1zvqM61sv0KOZvG5l1u1ye8JGUYIIdrVyiGp/wTelyTJl4EH\n",
"cENa1Chgw1be2Fr7d4Y46W6tvRvYpZVtVkRWe2B0tBnwvsK6s7ueZrEs4Hs3koUOUCILHaBEFjpA\n",
"iSx0gAay0AFKZKEDdErTY0klSfIq8C8W/7VcfKEC1kvTdKXOxhta1ceSMjo6FZjkn96oTX58yDxC\n",
"CAFdGEsKuD9N0/qeS29qY2iQUc3oaEsWFwsIu3chhBDLrJVzGN9YynqZMGmw4jmXGdrkfw+WRAgh\n",
"OqCV+TCmL2X97cseZ3QwOno77iJHcIfuzgkYRwghOqKVPQzRvOLe1rXa5I8GSyKEEB0iBaPD/vZf\n",
"XzoK16MM3JwhvwwY501VHM9GMjVHMjWvirmqmKldUjA6bOX77y/uXUzXJn88WJjB4tABSsShA5SI\n",
"QwcoEYcOUCIOHaCBOHSAEnHoAJ0iBaODjI62XuFfT/T6p4uA80LmEUKITpKC0VmfLDy+Tpv8n8GS\n",
"CCFEh0nB6BCjoy2A3fxTC/wqYBwhhOg4KRidc1Th8Qxt8n8ESyKEEMNACkYHGB1tArwXYNzLLy+g\n",
"mucustABSmShA5TIQgcokYUOUCILHaCBLHSAElnoAJ3S9FhSVVSVsaSMjr4HvN8/lTGjhBCV1u53\n",
"p+xhLCOjo40YPCLtuYGiCCHEsApaMJRSfUqpR5RSh9Utn6uUmlm4nRwqYxOOYPB8Fw+EDCOEEMOl\n",
"ldFqO0optTdwEPA8S84PPt1ae0T3U7XG6KgX+GBhURXPXQghREeE3MO4y1p7AG6mvZHqcKA2V/ft\n",
"MpueEGI0C1YwrLX5EKu3Ukpdo5S6WSl1tlJq7a4Fa5LR0brARwqLzoPqjhtTxVySqTmSqXlVzFXF\n",
"TO2q6knv2UBird0Nd8hqyKHVAzmIxYf0/gb81T+Og6RZujh0gBJx6AAl4tABSsShA5SIQwdoIA4d\n",
"oEQcOkCnVLJgWGuPstbW5gyfCrxDKbVTwEiDGB29Bdi/sOgCbfKR2z9ZCCGaEOykd7OstS8ppZ4F\n",
"NgTuqF8fx3FWe9zX1zdn8uTJc4CsuLzG7xrGJW/TUvuebd++yir33DvBP30EuLXYPsuyqcuy/WFq\n",
"X8xVhTzFTFXJU8sUVygPwEYly0LmWaJt6DzSfuj2kyZNoriuv7+/pFkTrLVBb8BM4NDC80lAX+H5\n",
"isDrwC71r+3v77fdzjvQ0zthoKe3f6Cnd5a/fai4fubMmVNDf6ZltyrmkkySaSzkqmKmdr87q3BI\n",
"SrH4OgaADYBjCs+Pw/0V/5duhhrC3sAa/vETwHUBswghRNcEKxhKqW2UUpcAWwGfU0qd7Vfd4Far\n",
"m5VStwB7AJOttW+EylpjdLQ8cEhh0UXa5AvrmmXdS9SSLHSAElnoACWy0AFKZKEDlMhCB2ggCx2g\n",
"RBY6QKfIWFItMDr6EHCSf/ocMFmb/JVuvb8QQnSCjCU1zIyOxgHFIUx+K8VCCDGWSMFo3q7AJv7x\n",
"S8ClAbMIIUTXScFogtGRwg0yWHOZNvlIHtJECCFaJgWjOdsDb/ePXwd+EzCLEEIEIQWjOZ8oPL5G\n",
"m/ypRg2rOm5MFXNJpuZIpuZVMVcVM7VLCsZSGB1tCOxeWPTrpbwkHr40yyQOHaBEHDpAiTh0gBJx\n",
"6AAl4tABGohDBygRhw7QKVIwlu4gFl9YeIs2+WMhwwghRChSMIZgdLQmg4cwX9rehRBCjFpSMIb2\n",
"MWC8f/wgcGfALEIIEZQUjAaMjsYDSWHRr2UIcyHEWCYFo7EPAWv6x/8Gmh0POBuWNMsuCx2gRBY6\n",
"QIksdIASWegAJbLQARrIQgcokYUO0CkyllQJPwxIyuJ5CM7QJpfzF0KIUUHGkuqsXVhcLF4ErgwX\n",
"RQghqkEKRrniEOZXaJO/0LClEEKMEUELhlKqTyn1iFLqsLrl2yulblNK3aKU+oNSaq1uZTI62hLo\n",
"80/fAH7XrfcWQogqCzmB0t7Al4HnAVtYviIwDfiatXZX4K/AWV2MVhwG5Hpt8rldfG8hhKiskHsY\n",
"d1lrDwDqR33dC1horb3RPz8X2E8ptfZwBzI6Wg94X2FRyye6qzpuTBVzSabmSKbmVTFXFTO1K1jB\n",
"sNbmDVbtBDxQaDeAm3/iHV2IdQCwnH98pzb5g21sI+5cnI6KQwcoEYcOUCIOHaBEHDpAiTh0gAbi\n",
"0AFKxKEDdEoVT3pPZMm9jnnAusP5pkZHqwD7FhZJN1ohhCioYsGwFM5pFAz33N0fBlb1jx8Hbh3m\n",
"9xNCiBFl+dABSjwJ7FC3bA2/fAlxHGe1x319fXMmT548B8iKy2v8scS4frl67fUbN3WHo2p+r02+\n",
"qFH7pW0/y7KprbRvdfttti/mqkKeYqaq5KlliiuUBxZfE1SVPEu0DZ1H2g/dftKkSRTX9fc3O3BF\n",
"HWtt0BswEzi08Hwf4JHC8whYCKxd/9r+/n7biQwDPb27DPT0zvK3bKCnd+V2tzVz5sypoT/TkZJL\n",
"MkmmsZCripna/e6swiEpxeDDTdOB5ZVStUmLjgQus9Y+M4wZDiw8nqZN/tIybCtbxizDJQsdoEQW\n",
"OkCJLHSAElnoACWy0AEayEIHKJGFDtApwcaSUkptA3wLeA8wB7jbWnu0X7c98HPchXPPAodba5+r\n",
"30YnxpIyOtoIuNQ/tcC+2uRmWbYphBBV1u53Z7BzGNba2cCUBuvuxo3n1A3Fcxc3SbEQQohyVTgk\n",
"FYzR0eoMnlHvt6GyCCFE1Y3pgoE7wb6Sf/wwMqOeEEI0NGYLhtHRcgyeUe93MqOeEEI0NmYLBu5k\n",
"+/r+8TzgT53YaFXHjaliLsnUHMnUvCrmqmKmdo3lglE82X25NvmrHdpu3KHtdFocOkCJOHSAEnHo\n",
"ACXi0AFKxKEDNBCHDlAiDh2gU8ZkwTA62oLFgxkuZHG3WiGEEA2MyYLB4L2LGdrkpcOOCCGEWGzM\n",
"FQyjo7WADxYWyYx6QgjRhDFXMHBDmK/gH8/WJr83ZBghhBgpxlTB8F1p9y8sSofhbbJh2GYnZKED\n",
"lMhCByiRhQ5QIgsdoEQWOkADWegAJbLQATol2FhSndDqeChGR5OAU/3T54APa5O/NizhhBCiotod\n",
"S2pM7WEweOyqK6VYCCFE88ZMwfCj0r7TP10EXBYujRBCjDxjpmAweO/iZm3yucGSCCHECFTFKVoB\n",
"UEpdAGxYt/gj1toXW92W0dHKuDm7a4bjZLcQQoxqVd7DsNbaSXW3louFtxewqn/8OHBHZyIuqarj\n",
"xlQxl2RqjmRqXhVzVTFTu6pcMDrC6Egx+HDUpdrki4bxLeNh3PayiEMHKBGHDlAiDh2gRBw6QIk4\n",
"dIAG4tABSsShA3RKpQuGUuoXSqmblFLXKKXiNjezA7Cpf/wK8MeOhBNCiDGmsucwgAeAGdbaO5VS\n",
"OwIzlVK7Wmv/1uJ2insX12iTL+hcRCGEGDsqu4dhrf2htfZO/3gWcDXw6Va2YXS0DjCpsOiSziUU\n",
"Qoixpcp7GPVyYKv6hXEcZ7XHfX19cyZPnjwHyPzy/Vj8b7z74Ysv0g9n2cEl286K23lzoTtZFbfa\n",
"PsuyqcO5/TbbF3NVIU8xU1Xy1DLFFcoDsFHJspB5lmgbOo+0H7r9pEmTKK7r7+8vadYEa20lb8BX\n",
"6p5fCJxTXNbf328bvX6gp3f5gZ7ePw309M7yt/d3I/fMmTPj0J/dSMklmSTTWMhVxUxDfXcOdavs\n",
"WFJKqSeAba21TymlNgbuAiZba2+utRlqPBSjoz2B7/unzwAf0SZ/fbhzCyFE1bU7llSVD0n9CLhC\n",
"KbUQWAX4XLFYNKF4svsKKRZCCLFsKlswrLWnAae181qjo02APv/0DeDyTuUSQoixqrIFYxk9Dnwd\n",
"t5fxnEzBKoQQy25UFgxt8oXA9cD1Rkcrhs4jhBCjQWWvw+iUbs95UdVxY6qYSzI1RzI1r4q5qpip\n",
"XaO+YAQQhw7QQBw6QIk4dIAScegAJeLQAUrEoQM0EIcOUCIOHaBTpGAIIYRoihQMIYQQTZGCIYQQ\n",
"oilSMIQQQjRFCkbnZaEDNJCFDlAiCx2gRBY6QIksdIASWegADWShA5TIQgfolMqOJdWMdsdDEUKI\n",
"sazd707ZwxBCCNEUKRhCCCGaIgVDCCFEUypbMJRSKymlLlBK3aaUukMp9b7QmYQQYiyrbMEApgLW\n",
"WrszcBDwO6XUumEjLV1Vx42pYi7J1BzJ1Lwq5qpipnZVsmAopcYBRwHnAVhrH8bNuPeJkLmaFIcO\n",
"0EAcOkCJOHSAEnHoACXi0AFKxKEDNBCHDlAiDh2gUypZMID/ANYGHiwsux/YMUyc5l111VUbhc5Q\n",
"poq5JFNzJFPzqpiripnaVdWCMdHfP19Y9jxQ+UNSd95550ahM5SpYi7J1BzJ1Lwq5qpipnZVtWDU\n",
"1F9VKBfpCSFEIFUtGLUpVdcoLFsD+HeALEIIIajo0CD+pPeTwL7W2lv8shnAH621P661mzFjRvXC\n",
"CyHECNDO0CCVLBgASqlTgInW2iOVUpsBtwFbWWufXMpLhRBCDIMqF4zxwFnAlsDywNettf1hUwkh\n",
"xNhV2YIhhBCiWqp60ntIVRk2RCnVp5R6RCl1WN3y7X22W5RSf1BKrdWlPMsrpb6olJqplMqUUv9P\n",
"KfXeCuQ6TinVr5SaoZS6Ryl1bOhMhfffTCn1ulJq99CZ/O/0zLrbKqFz+ff+pP99ukkpdXft8wqR\n",
"SSk1t+4zmq2UmlZYH+rn9zaf52al1F1KqW9WINNEpdSl/ud2j1Lq8GXKZK0dcTfg+8D5/vFmwDPA\n",
"ul3OsDfwO+BO4NDC8hWBx4H3+OdTgbRLmTYC/gGs5p/vCSwA1g+c625gHf94U+B13EWYwTIVsv0W\n",
"eAHYvQI/v/OHWBcy1xTgKmA5//xQvyxIJuCndc/PAA6pwOd0O3Cyf7yW/176cOBMNwAn+McR7nq2\n",
"d7WbadgDD8MHMA54Cti1sKwf+FKXc0T+fmZdwdgHeLTwvBdYCKzdhUxrAQfXLXsK+FjgXNvUPX8y\n",
"dCb/fjsB5wCPFQpGyM/p/CHWhcx1L7BzlTIV3nM54FFgldCZcH94fKjw/M/AV0Jl8u+zCNCFZZf6\n",
"3/m2Mo3EQ1KVGDbEWps3WLUT8ECh3QDwEvCOLmR61lp7ce25Ukrh/pJ40ud6sNC2m7lmFzLtj9vr\n",
"uY6An5U3Ffh23bKgmZRSv/CHD65RSsWhc/kBP7cGNveHFG9SSn0mZKY6ewK3WWtfrECmq3FHHlBK\n",
"bYI7+tEfMJP2908Xlj0JbI/7vmz5+2D5DgfshkbDhmwVIEuZicD8umXzCDOsyXuAOcDNwCEM/syg\n",
"i7mUUtsAvwdWBhJr7XylVLDPSim1FzDbWmtcXX1TyJ/fA8AMa+2dSqkdgZlKqV2ttX8LmGsjf/9x\n",
"4APAmsDtSqnn/HuH/l0/GLio8Dzkz++TwDSl1CPAW4BjrbV/VUodEyjTgL/vxe2FAayH+xlOpI3v\n",
"g5G4h1FT1WFDLEtmgy7nU0qtBHwXONy6fc6guay1s621WwMfBf6glNopVCZ/YehXgFNKVgf7nKy1\n",
"P7TW3ukfz8L9xfrpumzdzjXe359trV1orX0K+DVwZMBM7k2UWhnYGbe3WhPy9/xy4P9ZazcFdgC+\n",
"7zudBMlkrTW4PZwvgjspD7wXeKXWpNVMI7FgVH3YkCcZnA3/vGsXHPpDUecAp1tr76pKLgBr7d24\n",
"L8LPB8x0EHCttXZeYVntP0olPicvBzb0j0Ples7fF/9/PYH7qzX0Z7UPcJX/g6gmSCb/ZbwH7gR8\n",
"7RDPNOCYUJm8A4CVlVK3AicC5+JOdreVaSQWjEeBZ3EX9NVsBdwRJs4S7gC2qD1RSkW4wzB3djHD\n",
"acCfrbWXKaVW9BmC5FJKraWU2q9u8UvAKqEyAbsCH6l1y8Ttpp+hlLoiYCaUUl+pWzQRMP7x7YFy\n",
"PYz7eRUPVazjc4X+XT+IwYejCJhpRX//emHZQtweWrDPyZ/XPMpa+25r7YG4w1Ez2s403D0Hhuns\n",
"/ynAr/zjzXAndbrarbaQJQMOKzxfEXfeoNbr5lvA77uY56vAmbgv5FVxxfRboXLhjoHfBUzwz9+K\n",
"+wvns6E/q0LGYi+p8aEy4f5yf6t/vDHumPJuFcj1c+Ay/3gCrtfUwSF/friidXfJ8lC/5+Nxe4TH\n",
"+Oer4YrtcYE/pwvxPZ/8d8GDPltbv0/DHngYfzjn48aXugPYM0CGbYBLcLtwtwPnFNZtD/w/3Mnm\n",
"acCaXcq0Oa4bXf3txFC5/M/qJP++Ge6ajBNDf1b+vbfDdTNc4LN9IfDP73jgFp/lDpbsIh0q18r+\n",
"i+cu//7/XYFMxwBfa7AuVKY+4EbgJuCvuD9sxwXO9D1gtn/f6cCWy/I5ydAgQgghmjISz2EIIYQI\n",
"QAqGEEKIpkjBEEII0RQpGEIIIZoiBUMIIURTpGAIIYRoihQMIYQQTRmJo9UK0VVJkmwC7J+m6Q8D\n",
"vPc03AVWEbBRmqb/HKLtprjxlX6cpumiLkUUY4jsYQgxhCRJdsFdvXt7iPdP03Qf3KBxzfgH8Hbg\n",
"miRJVlxaYyFaJQVDiAaSJFkHN2TCd9M0zQJGaWoYbL9XcQTQA/x4WBOJMUkKhhCNfQ03+ugvQwdp\n",
"VpqmFvgO8JkkSTYPnUeMLnIOQ4xKSZKshJtTeVvcbGf/AHZL0/TFJElOBRLcqJ0Xpmn6xZLXK9ws\n",
"hf1pmi4sLN8LN6DbVsBv/HscDGyCG3r/ONzw+6fjBoNcFfhJmqZnFLZxB24+iYnAbsDJuFF818Pt\n",
"0Ryfpmn9bGgAUZIkP/LbfQvwO+B/Ss5X/MnfHwb8z1I/LCGaJHsYYlRK0/SVNE23x40c+gKwY5qm\n",
"L/p1XwauBb5eViy8LXBf4g/VbXd6mqY7AP8C3ge8nKbp7riCMR64EldQPpmm6Xa4mf1O9+dCatvY\n",
"CTjLP/0WsF+apm8HdgE+CFydJEnZ/82vA5/x/64pwJdxxar+3z4fN19F3PgTEqJ1UjDEaHc27pj+\n",
"3rUFSZKsBkwGLh7idZv6+0YzkCnguTRNLwRXoHDD3W8IXFnYQ7gcNwHRB0peD3Bymqbz/DYeBn6E\n",
"Kxz1k04B/F+aps/6trNwxex9DfI9Wfg3CNERUjDEaHcZ8AxwdGHZwcAf0jR9YYjXvcXfvzJEmwfq\n",
"nj9Tv9yfU3gGWL/BNu6ue/4Xfz+ppO19Je+3XoPtvszif4MQHSEFQ4xqaZq+BlwAvD9Jkg384qNx\n",
"c54PpXZeoFEPJYs71FW/jJLli2hwvjBN0wV1i2pFZ52S5k1vF/d/W67FEB0lBUOMBefgftePTpLk\n",
"nYBN03Rp8yk/6+9XHs5gSZKsXrdobX//9DJuehUWFx8hOkIKhhj1/LmBmcCRuLnEz27iZbXDSo0O\n",
"JXXK9nXPayfHZyzjdtdnyUNmQiwTKRhirDgbd7z/owx9shsAPwTHo7i528soGh+uql8+VNsvJEmy\n",
"JkCSJJsBXwJuBa5oYruly5IkmYjr4XV9g/cUoi0yp7cYE5IkWQHIgWlpmn66ydd8GXcdwwa+qyr+\n",
"kNbZwNuAF4HHgR2B84D34q6veBD4GTCrpG1fmqY2SZKpuCE/tgNOBTRur+AK3HUYtfc7v267ZwJX\n",
"AdNxXXkBHgY+nKbpE/41x+B6W22UpulTLX5UQjQkBUOIBpIkGQ/8FddNtqMXwNUKRpqmHd3LT5Jk\n",
"VdyhqDPSND2tk9sWQg5JCdFAmqavAnsB+yZJ8qnQeZbGF4vrgN9KsRDDQQqGEEPw5zJ2AkbCoZ3V\n",
"gW+kafqV0EHE6CSHpITosiRJZuHOWawL3AP8NE3T88KmEmLppGAIIYRoihySEkII0RQpGEIIIZoi\n",
"BUMIIURTpGAIIYRoihQMIYQQTZGCIYQQoin/H4vZwXlM57JQAAAAAElFTkSuQmCC\n"
],
"text/plain": [
"<matplotlib.figure.Figure at 0x7f0ba69b16d0>"
]
},
"metadata": {},
"output_type": "display_data"
}
],
"source": [
"vs = np.linspace(1, 90/mphfact, 300)\n",
"mpg = vs/( A*vs + B*vs**2 + C*vs**3 + P0 ) / 1609 / gallonperj\n",
"plt.plot( vs * mphfact, mpg, lw=3, alpha=0.8 );\n",
"plt.xlabel(\"v (mph)\");\n",
"plt.ylabel(\"mpg\");"
]
},
{
"cell_type": "markdown",
"metadata": {},
"source": [
"I've actually cheated a bit, and picked $P_0$ to give a maximum fuel efficiency near 50 mph. In either case, we now have a decent model of a car's fuel consumption.\n",
"\n",
"Or in the more useful gallons/100 miles"
]
},
{
"cell_type": "code",
"execution_count": 91,
"metadata": {
"collapsed": false
},
"outputs": [
{
"data": {
"image/png": [
"iVBORw0KGgoAAAANSUhEUgAAAYQAAAEbCAYAAADajfNFAAAABHNCSVQICAgIfAhkiAAAAAlwSFlz\n",
"AAALEgAACxIB0t1+/AAAIABJREFUeJzt3XmcHEXdx/FPhSOEEBAIR+iMRpH7NtwidLgFgo/olgqC\n",
"PHiBingAHoAEAUEFlQcvUB4EHwRK7nCIbEhzHyFcIocIRIYGDPeZyFXPH1VDJpOe2enZnqne3d/7\n",
"9ZrXznT3dH8zu+nfdHd1lbLWIoQQQowKHUAIIUQ5SEEQQggBSEEQQgjhSUEQQggBSEEQQgjhSUEQ\n",
"QggBwOKhNqyU+gPwvobJe1hrXwsQRwghRrxgBQGw1topAbcvhBCijpwyEkIIAQQuCEqp3yilrldK\n",
"XamUikNmEUKIkS7kKaMHgBnW2tlKqU2BmUqpbay19wTMJIQQI5YqS19GSqnzgOettV8JnUUIIUai\n",
"Ml1DqLJoqyMhhBA9ErLZ6eHW2p/UTVoFSBuXmzFjRjkOYYQQYojZYYcdVK43WGuDPICngJX88/cD\n",
"LwIfaVyuv7/fhsrY7LHmmmv+oXHaE6tNHPvEahNvf2K1iXc8sdrEWU+sNnGZ0JlCP8qYqay5JJNk\n",
"KvrRyb4z5Cmjk4CLlVIJYICvWmtvCJinbRMmTJjUOC1Kq68B//AvFbBh6EyhlTETlDOXZGqPZOqu\n",
"YKeMrLUnAyeH2n6X3A2s7Z9vBNwcMIsQQuRSpovKQ8bkyZPnNJl1d93zTXoQ5V0tMgVTxkxQzlyS\n",
"qT2SqbukIHRg6tSpc5rMuqvu+QZpVFmqB3GAlpmCKWMmKGcuydQeydRdUhAKFKXV54BH/csl6PF1\n",
"BCGEGAwpCJ1JWsybVfd8sy7nqJf0cFvtSkIHaCIJHSBDEjpAhiR0gAxJ6AAZktABilKaO5WbmTFj\n",
"hs3dljagNKpMAX7qX94XpdX9w6URQoxUnew75QiheHcCtSq7bhpVlgkZRggh2iUFoWBRWn0JeMi/\n",
"HEWPWxsJIUSnpCB0R/11hE2DpRBCiBykIHRHfUHYPFgKIYTIQQpCB5IkiQdY5G7gLf98jTSqjO9u\n",
"orYy9VwZM0E5c0mm9kim7pKC0Jm41cworb7Ownctb93VNE7cg23kFYcO0EQcOkCGOHSADHHoABni\n",
"0AEyxKEDFEUKQvfU92PUi4IghBCDIgWhe+oLwhZpVFksWBIhhGiDFITueQSY65+PAzYImEUIIQYk\n",
"BaFLorRqWfgoYatQWYQQoh1SEDqTtLlcfUH4cBdy1Eu6vP5OJKEDNJGEDpAhCR0gQxI6QIYkdIAM\n",
"SegARZG+jLoojSpjgRksGIho1yitPhswkhBihJC+jErGD6tZ3/x021BZhBBiIFIQui+pe75dqBBC\n",
"CDEQKQjdd13d8839aSQhhCgdKQhdFqXVp1jQ++kSyE1qQoiSkoLQgQ76Lqk/Ssj73raUsT+VMmaC\n",
"cuaSTO2RTN0lBaEzcc7lk7rnH06jyhLFRXlX3IV1DlYcOkATcegAGeLQATLEoQNkiEMHyBCHDlAU\n",
"KQi98TDwpH++DL0da1kIIdoiBaEH/F3LM+sm7RgqixBCNLP4wIuA1noxYHtcs8k1gOWAd4DngAeA\n",
"a4wxs5qvQQB/Bfbxz7dPo8qJUVp9I2QgIYSoN2BB0Fr3AT8F3ttiseO01ncCXzPG3FpUuGHmfiAF\n",
"Itxpoy2B64MmEkKIOi1PGWmtpwFnA1cCH8MdHSyLaz45GlgBWA/4DK53z0Rr/cku5i2LJO8b/Gmj\n",
"a+om7VxYGicpeH1FSEIHaCIJHSBDEjpAhiR0gAxJ6AAZktABitK0LyOt9R7AD4E9jTFPtLMyrfWH\n",
"gAuAnYwxjxQRcCj3ZdQojSprAef4l68DO0Vp9T8BIwkhhqmi+zIaBezcbjEAMMbcCewBrJQnxAjy\n",
"D2COf740sE24KEIIsbCm1xCMMZd1skJjzP2dxxneorRq06hyDfBFP2k3XG+oQggRXFutjJrRWsfA\n",
"RsBMY8y9hSQa/q5kQUH4cBpVVojS6vMhAwkhBOS4D0Fr/W2t9Tta6+P8688A1wI/B27XWk/pUsZh\n",
"JUqrVRZ0ib04sGvAOEII8a48N6btCWhjzJH+9RG4Tts2BY70jxGhgL5LLq97vscg1wWUsz+VMmaC\n",
"cuaSTO2RTN2VpyAsY4y5AEBrvTawLnCMMeZOY8xJwCqdBFBKraGUelMpNZQGj4kH+f5+oNa6aE3f\n",
"+miw4gLWUbQ4dIAm4tABMsShA2SIQwfIEIcOkCEOHaAoeQpC/bJ7Aa8Bl9ZN67T55A8H8d4hKUqr\n",
"r7Jw2+VCjhKEEGIw8hSEN7TWsdY6Ag4ELjHGzAPQWi8PLJl340qpzYBXgGfyvncYmF73fPc0qowO\n",
"lkQIIchXEH6KayJZBVYFfgagtZ4KXAXc1sH2pwHHdPC+4eB2XFcW4O7+lg7vhBBBtV0Q/PWDjwCH\n",
"A1saY+7ys8bhCsIpeTaslPoocJ+1Nh1w4WEoSqvvABfVTfpEqCxCCAE570MwxtwM3Nww7U95N6qU\n",
"GoUrLB/P+96SSApaz3TgINzvYcM0qqwRpdWHA2cqUhI6QBNJ6AAZktABMiShA2RIQgfIkIQOUJSm\n",
"fRk141sY7QiMM8acoLXeGLjXGPNO2xtV6rPARGvtif71Y8D+1trrGpedMWOGPfbYY9+dPnny5DlT\n",
"p06dAyRxHCeNy/smYHHGZku5fBpVfsSCju4uePicP94aMo8sL8vL8kNz+SlTplA/r7+//+i8fRm1\n",
"XRC01qOA04ED/KSnjTGraa3/D1gH2NEY80JbG1Xqt8D6wJt+0pbAg8Aca+1CRw3DqXO7LGlUmQyc\n",
"5l/OA3aL0uorASMJIYaBoju3a/RdXPPIabhmp7Wd/364dvVtXxy21h5ord3GWjvFWjsFeBo4pLEY\n",
"jBB3Av/0z8fguhkXQoiey1MQ9gWmGGN+aIy5BHgDwJ8qOhJ3wTkXpdRGSqkLgPHAD5VSh+Rdx1Dn\n",
"x0k4t27Sp9OoslioPEKIkSvXmMrGmAeaTH+TDjrKs9beY639pLV2nLU2ttbmaqk0jPwFeNE/XxWQ\n",
"fqGEED2XpyAspbVeMWuG1no8bljIEaHovkv8IDkX1E3aO+86ytifShkzQTlzSab2SKbuylMQrgau\n",
"0VrvqrUeC6C1VlrrTXDt6a/oRsCSiruwzj+z4CL7hmlU2STn++Ni4xQiDh2giTh0gAxx6AAZ4tAB\n",
"MsShA2SIQwcoSp6CcCTuoueVwEvABsB8YDZubOWjCk83gkRp9TncZ1vz36GyCCFGpjx3Kj8LbA4c\n",
"jet24TFcMTgKd+dyW01ORUtnAbX7ObZOo8raIcMIIUaWvHcqvwIc6x+iYFFafTyNKv0suFHtv4Hv\n",
"BIwkhBhBcrUyakVrfWpR6xrh/lD3fPs0qqweKogQYmRpeoSgtd4OaLdfC8XIGgoy6daKo7T6jzSq\n",
"3Ahsg/tcDwQOC5lpEJLQAZpIQgfIkIQOkCEJHSBDEjpAhiR0gKI07bpCa91230SeNcYUfkPVcO+6\n",
"IosfQe2cukmfi9Lq30PlEUIMPZ3sO1tdQ3gE+ALuW2o7fpdnw6K5KK0+lEaVa4Cd/KSvAF8NGEkI\n",
"MQK0Kgj9xphFeh9tRmvdX0AescBvge2BxYAt0qiyWZRWZwXOJIQYxppeVDbGHJRzXRcPMouoE6XV\n",
"fwGX1036ahpVRtSpMyFEbxXWygg4scB1Ced3LLh7eX2G0R2RQojyaVkQtNY/1lp/yz9/TGv9qP9Z\n",
"/3hUa/0YsG5PEpdAr/ouidLq07guLWoOSaPK6JCZ8ihjJihnLsnUHsnUnk7PJgx0hKCBqf75csB1\n",
"GY/r/c/XOgkwRMU93NYZwMv++URgnybLxT1Jk08cOkATcegAGeLQATLEoQNkiEMHyBCHDlCTRpVR\n",
"aVSJWfh+prYNdKfyesBb/vm/jDFN+9fRWt/dSQDRWpRWX0qjym9YcMfyAWlUuSJKq/8OmUsIUR5+\n",
"DJWdgf2Bjm9mbVkQjDGv1738xADr2qvTEGJAFwEfB9YElgIOAb4fNJEQIrg0qiyJG8lyP9wZhEHJ\n",
"07ndowMscsQgs4gmorT6NnBS3aSd/VjMQogRKI0qY9KosjdwKe7LYX0xeB04u5P15urcTmu9DLAZ\n",
"MKHhvQrYsZMAoj1RWr0zjSpXA7v4Sd9Lo8o+fnAdIcQIkEaVcbhru58B3tMw+2XgPOD8KK2+9OCM\n",
"GblHoGy7IGitt8e1eFm+ySLt9ns0HCSBtvs/uLGrlwYmAZ8Hfu3nJWEitZSEDtBEEjpAhiR0gAxJ\n",
"6AAZktABMiTd3kAaVVbAjaTYB4xtmP0srqubi6K0OqjGPU37Mmqktb4H16Lojz5AY19HVxhj1htM\n",
"mCwjsS+jVtKo8kngu/7l27h+jh4MGEkI0SVpVIlwLQv3xF0/rPcUbgyV6VlnCoruy6jRUsaYg5vN\n",
"1Fr/Ks+GRccuwvVxNBnXrcXRaVTZL0qrb7Z+mxBiqEijynrAvrjuaxqv9c4BzgSujtLqWxQoT0H4\n",
"1wDz7xhMENGeKK2+k0aV44Bzcd8Y1gA+B/w+aDAhxKCkUWUUsDWuEGQ1GnkQVwgS39CkcHm6rjhe\n",
"a/0/WuvxTeb/tohAYmBRWq2y4NoBwBf8NwohxBCTRpUl06iyJ+6C8C9YtBjchuvteN8orc7oVjGA\n",
"fEcI9+LGU56rtX4WeBXXusj6nxOKjydaOB/XsmtD3O/xeN/qaCTdMS7EkOVbDO2FazHU+EX7beCv\n",
"wP9FafWhXmXKc4TwK1yT06v8o9ZlxYjruqIMfZf4bwlH4Qozb48Zsw6uKWppLsCX4XPKUsZckqk9\n",
"wyFTGlUmplHl27jejA9m4WLwOq7F0MeitHpUL4sB5DtC2B5YzxjzeNbMEdZ1RUwJmr9FaTVNo8qP\n",
"gB+9M2bMuMXmzdsVd3g5PXC0mpgSfE4ZYsqXK0YytSNmCGbyX9Qm444GtmXRgceexV0XvChKq68U\n",
"H7E9eQrC482KgfehwYYR+UVp9a9pVNkCqI1f8Z00qtwfpdVHQuYSQoDvnXgXXCFYI2ORx3BN+f8S\n",
"pdU3epktS55TRjdqrTdoMV+G0AznpLeXHfeif74UcHIaVZYLGUiIkSyNKuPTqHIg7rTQD1i0GNyM\n",
"O130qSitXlaGYgD5jhAeBM7VWs8EHsCfu/ak64qAorQ677bf/KZ/4nE/2gQYg+vX5Pg0qhzSzRYJ\n",
"QoiFpVFlXeDTuJ5HG/ev83EF4rworc7pcbS25CkItWalzQbCGUldV5TOvHXWeR7XCuwnftKWwNeA\n",
"3P2ZCCHap+bNWyyNKrvhupXIOovyNGCAS6K0+nLG/NLIUxAeAHZj0YshNVcMPs6QkYQOkCGJ0mqS\n",
"RpUzcH0cAeybRpVHo7Qa6iJzEmi7A0lCB8iQhA6QIQkdIEMSOkCN71Zir0njxu0GfDRjkbtx9xbM\n",
"HCpH6nn6MjrKGHNsi/lfMcb8utn8TklfRvn4ux1PwrVkANee+ZtRWr05XCohhgc/EM2WuB5Ht2bR\n",
"L8hvAv2400J/73G8hXSy72y7IIQiBSG/NKqMxXVlUbuQNQ84MPQfqBBDVRpV3oPrYO4TQJSxyFPA\n",
"hcBlUVp9vpfZmul253aFU0odDHwMV2VXAk631v4yZKbhIEqrr6VR5eu4fk9WxV1o/kUaVT4fpdVW\n",
"TYeFEJ6/d2Aj3GiFOwFLZix2M3ABcNNQOS3UStCCgDvXvaO19lml1AeBB5RSt1lrZwXONeRFafWZ\n",
"NKocjDtSWA43jsWv06jypSitPhk2nRDl5cce2B33ZXVSxiIvAZcBF0Zp9YkeRuu60AXhs9baZwGs\n",
"tf9USr0AvA+QglCAKK0+lkaVb+I6wlsKd7Rwmi8KT4VNJ0R5+GsDmwP/BWxH9r7xPtzRwDXDdaTC\n",
"PDemFc5ae1/tuVLqE8AruA6dSm0o9acSpdV7gUOB2o0vE3BFYdVQmUIrYy7J1J6iM6VRZZU0qnwJ\n",
"NzbxqcAOLFwMXsNdG9gvSqv7R2n18sZiUMbPqVNBCwKAUmp9pdTfcS1jPmOtLXU7XS8OHSBD3GxG\n",
"lFZvxRWF2iA6qwGnp1HlvaEyBRaHDpAhDh0gQxw6QIZ4sCtIo8oSaVTZPo0qp+BuFPsS7ui53j3A\n",
"D4Fdo7R6QpRW7+9mprLIfcpIa70kgDGmkFut/VHCekqpjYGrlFJ7yjWE4kVp9eY0qhyKK7xL4IrC\n",
"79Oo8nUZglMMd/4C8bq4awO74K6rNXoRdz/VpVFafbSH8UpjwIKgtV4Md/F3H2BjYBk//VXgLlxX\n",
"rWcYYxrHWM7FWnu3UuoK4Ou4EYNEwaK0epPvdvcnuGsKK+BOHx0apVUpwmLYSaPKKrgbancn+wIx\n",
"uB6CLwGuK0ufQqG0LAha62WBq4EtcOfS5gAv47qpWA7YFHcD1AFa612MMW2f7lFKrQDE1tqL6ia/\n",
"jmsNs5A4jpPa88mTJ8+ZOnXqHCCpn17jz+fFGZssdPkkSaZ1c/0dLN+YKXP5h8/545Jj75j9t/F/\n",
"OndX9eabowEYNeqi+z+zzy/XPfecHxecvz5T6M+nXpwkSVyiPNBkZxUwzyLLhs7T7vKjXnllifF/\n",
"Ou/l5a6//n24MVzebYv/9pgx494ZM2bcO0sv/eq8ddb+x8vxdg/9Z9KkV4A34zhepBgMhX9vbdaU\n",
"KVOon9ff35+xWGstb0zTWv8St9M/DLjZGPN2w/zFcHfr/RSYZYw5uO0NKzUJuBjY2lo7Tym1Em5c\n",
"5h9ba9+947mMN6YlSTItjuNpoXPUy5spjSqrA7/E3f9Rcxbw66LaU5fxc4Jy5pJM7WmWKY0qi+Na\n",
"Ce2CG7tlTMbbXwdm4K4b3BWl1UGd1RgoU2jduDFtJ2CzZt/8fYG4QWu9M25nnsdTuIFcZiil3gDe\n",
"A5wB/CbnekJIQgfIkORZOEqrj6RR5fO4MVw/4Cd/DvhAGlWOLGgozlyZeigJHSBDEjpAhiR0gAxJ\n",
"7YnvpmUjXBHYEbcPaWSB23HXBmZGaXVeNzMNdQMdIdxrjNmwnRXlWTaPMh4hDCe+m4vjgI/UTa4C\n",
"3+318H1CDMRfHF4bVwR2AlZpsuhjuCOBv0Rp9d89ilcqnew7B2p2+ob/9t+SX+bNgZYT5eOPBA7F\n",
"nS6qqQBnplHlk2Uao1mMXGlUeX8aVb6Muyfgj8BnWbQYzMU1ctkP0FFaPWukFoNODXTK6CfAFVrr\n",
"i3A3jD2Cu6gMsCywOq5KfwLXCkkMQf6awalpVPkHcASwNK7flu8Cm6VR5cQorb4QMqMYWfwXkTVx\n",
"1wN2oHkLoRdx1wWuBu4u6rrASNWyIBhjjNZ6LPAz3OAPWV4EvmSMMUWHE70VpdWr06jyIHAiC3pK\n",
"3QHYJI0qx0Vp9fpw6cRw568JrA9MwRWCrF5FwV0cnokrArdHafWt3iQc/trq/toXhV2BTVjQKuVZ\n",
"YDbwF2PM690KKNcQes8PDP4t3JFfvSuAX8jRgiiK70NoY1wBmAKs3GTR+cBNuDMVNw7XvoSK1LXu\n",
"r40xtf48Luwk2HDTrA17SEVm8v/ZTkijyvXAUcB4P2t3YJs0qpyK6/e95eF5GT8nKGeukZQpjSrL\n",
"4JqrbwN8mOy7hsGN234DcC1wS5RW5/tMpSoGZfzddSp41xVDVEz5mprFFJzJ39n8KeBwXKsOcP95\n",
"jwSmplHlZwMMulN4poLElC9XzDDOlEaVCq4l20dwZxqa7XteBK7DXRe4I+PO4cIyFSimfJk6Upqu\n",
"K0Q5RWn1JeCINKpchSsMq/lZGwFnpVHlr8CvorSahsooysffKLYhC4rApBaLP4PboV4L3DkcBpoZ\n",
"qoJ1XSGGliit3phGlTuAA3DN+mp/OzsD26dRxQBnyvWFkcsPOr8VbszhTfFfHpu4H7gRuB54KEqr\n",
"5R7Ld4QY6AjhR7h+QLZj4K4rjgfa7rpCDD1RWp2PG3XtcuCruBZI4P6O9gb2SqPKRcA50v57+PM3\n",
"NW6KKwBbARNbLD4f14ncjbiLws90P6HIK2TXFWKI8uMyfyeNKhsCh+BOH4HrQXVvQKdRZfroY49Z\n",
"ljgOlFIUzbc+2wBXBDbFNRFttQ95Gtcy6AZglrQMKr+BCsJ/2jkNZIx5WWs9v6BMQ0ESOkCGpNcb\n",
"jNLqvWlU+QLuCPKLwFp+1uLAx6MTTlwmPeroxYA/41qJlOXccBI6QIYkdIBGi73wwk1pVJmM2/lP\n",
"xhWDJVq8ZT7ui+GtwC3A4104FZQUvL4iJKEDFGWgvozuAL5vjGk5rKU/QjjBGDO54HxyH8IQ4e8s\n",
"3Rp3jWGjrEVwzZavjNLqs73MJtqTRpVxuJ3+hriWQBvg7lhv5UHcqaBbgHtH+ngCZdKN+xCk6wrR\n",
"Fv9N8KY0qtwMfAh34XlrFvRFH+EGP/paGlVuB64Ekiitdu2mRtGcvyv4vbidf+3xgZZvcubgbki9\n",
"A9ci6LluZRS9J11XiEL5wjAbmJ1GlYm4Lwt7suDmo1G4i5BbAvPTqHIT7pD7piitSiu1LvBHb+OB\n",
"dXA9ha6HO//f7IawenPwv09gthSA4U26rhBd5y9G7gzsgTsXneUt4E5cK5TbgEelKWJ+fue/Mm7n\n",
"XysAawMrtvH2t4GHgL/hBpm/U07vDV2d7DvbKgghSUEYXtKosiruy8VutD5F8Swwyz/uAarSk+XC\n",
"0qiyLO4zXN3//KB/njVQTJYXgXv942/A/V0aQEYE0LW+jMTCyth3yVDJFKXVp4E/pFHlLNxObDv/\n",
"WK/h7eOBj/oHwMtpVLkPqD0ejNLq80XlCq1ZJt/52wTc+f6J/uf7cTv+8Y3LtzAP9+3/AdyF4PsY\n",
"oBXQUPqcQipjpk5JQehMTPmamsUMoUx+R/SIf/xvGlVWxnVxsDluYPRlG96yLO4i9da1CWlUebFu\n",
"HY/gznc/ATw7QBPXprl6ze/wx4/95jc+le6z7xK4QV8msGDnP4H8/09fxe38H2RBAah20Ow3piSf\n",
"U50YydQ1hRUErfXlxpg9ilqfGFmitDoX36Ou30muhSsOG+MugGadBnkP7ppE43WJt9Ko8jSuqeuT\n",
"uNNPzwPPAS+MPubo5dJ99l0OeK3ovvR99rG4QYbGAcv7nMvXPd6D2/GvjPuWP2qls85eDVcI83gD\n",
"N1TkI8Cj/vEI8JScXhOdKPIIofGQX4iO+G+y9/tH7UJphGsXvz7ub211YEyTVSyO+4ad2ZXChFNO\n",
"XQ0/hnQaVf6D66frdf/zTdwF7rf9o/a8tt7F/M/6x9J1j6U6+ke39gzwOO7o53H/eBR4okQ3+4lh\n",
"oGlB0FqfievErh21Zm1CFM6fXnrCP66Cd9vRT2DBRdXVcWNBR7hv4e0a7R8rFBg5r+feWmGFJZZ4\n",
"/vkE+Ld/PAFUcTt9udAreqLVEcJeuEPedpW7uZIYVvwpkdQ/bqifl0aVMbhuulfDFY0VcTv8FYAV\n",
"3llm7DI8//yruG/0owqOZllwtPEa8IJ/vFj3/AXcaax/A89EafWNJEmmxXE8reAsQuTSqiAciuuG\n",
"4NMsuNu0lRsGXmTYSEIHyJCEDpAhCbFR/426dqF5EbVWIf5U1FK4wrAM7hRU7TTQYix8eggWnD56\n",
"q+7xNq4FT+200/wOz98nHbyn25LQATIkoQNkSEIHKMpAfRnNAP7XGHPOQCvSWj9mjHl/keFA7kMQ\n",
"QohOdLLvHOhw+YvAFK11O4fVX8+zYSGEEOUyUF9GjwJfaGdFxpjphSQSQggRRNEX1IQQQgxRUhCE\n",
"EEIALQqC1vpkrfVheVamtVZa6z9qrfcefLTySpIkDp2hkWRqXxlzSab2SKbuanWEcDSwv9b6fK31\n",
"+gOtSGsdA9cDY40xfyooX1nFoQNkiEMHyBCHDtBEHDpAhjh0gAxx6AAZ4tABMsShAxSl6UVlY8yr\n",
"WuspwHnAvVrrp3AdZs3FjZ2qcO23JwDr4vpnORM4qNuhhRBCFG+gVkZztdY7ALsDn8P1/xLXLWJx\n",
"PUxeCJxujJnVnZhCCCG6bcDO7YwxFrjcP9BaL4Mbeu8d4AVjzPyuJhRCCNETLQuC1non4FpjzLs9\n",
"KhpjXsX1ty6EEGIYGegI4SrgJa315cBFwNVFHREopRYHvgZ8DHc9YkngSGvttUWsv8uS0AEyJKED\n",
"ZEhCB2giCR0gQxI6QIYkdIAMSegAGZLQAYoyUF9G43E77L2A7XEdef0FVxwuN8a83PGGlZoEXAts\n",
"ZK19RSm1I3AxsJa19snactKXkRBC5Ff4mMrGmGeBM4AztNbjcBeX9wJ+C4zWWl+LKw6XGGOeyZn3\n",
"ZeAoa+0rANbafqXUfGAr3EVqIYQQPdTyCKEZrfVoYGdccZiKa3J6M644XGSMeTx3EKUUrs/4qdba\n",
"62vT5QhBCCHyK/wIoRljzH+A6cB0rfXiwLa44nAocDKuH/m8tgPm1BcDIYQQvTPoMZWNMW/hrgVc\n",
"q7U+GDcwei5KqaWA44H9B5tHCCFEZ9ru3E5rPV5rva3WenX/enGt9TSt9cVa6wPB3bNgjLktTwB/\n",
"quh04GfW2rvyvDeUMvZdIpnaV8Zckqk9kqm78hwhfBs3NsJhuKEJjwB+ALwETNVaW2PMaR1kOBm4\n",
"1Vp7oVJqSWAVa221foE4jpPa88mTJ8+ZOnXqHCCpn17jfzlxxnaKXH5akiSN80LmiYG4IVPoPDRk\n",
"KkOemtiPYVyWPACTyGi+GDBP4qcv9J6QefzyC2UqQR7qM4XMM2XKFOrn9ff3Zyw2AGttW4++vr5Z\n",
"fX19a/nni/X19T3d19d3ZV9f36i+vr4t+/r67mp3XbUH8B3gFGAsbkzbdYGj65fp7++3edfb7cfM\n",
"mTOnhc4gmYZXLskkmYp+dLLvzDMewihjzEP++ZbAysBPjDHvGGNuxd1c1jal1JrACcDBwCu4Zqj3\n",
"4fpHEkII0WN5ThnVL6uBp4Dr6qa9TQ7W2n8gA/QIIURp5CkIT2qtjwb+CXwe+I3v+A4/XoLcKyCE\n",
"EENYnoLwfeAaYAXgCeAkAK31EbgLzp1cUB6qktABMiShA2RIQgdoIgkdIEMSOkCGJHSADEnoABmS\n",
"0AGKkutOZd/19drA/caY1/20DYAVgb8ZY54rOqDcqSyEEPl1/U5l3/X1HQ3T/gagtf4QUHhBEEII\n",
"0RtFXtRxGINlAAASXElEQVQ9o8B1CSGE6LG2jxC01mNwfRXthBtHufZei7ugPKHwdEIIIXomzymj\n",
"U4BPAzcCt+CG0Kw3tahQQgghei/PKaNdgY2NMbsZY/Yzxuxf/wCqA7x/2Chj3yWSqX1lzCWZ2iOZ\n",
"uitPQXjGGPNos5nGmI0LyDNUxKEDZIhDB8gQhw7QRBw6QIY4dIAMcegAGeLQATLEoQMUJU9BuEtr\n",
"PanZTK31tEGnEUIIEUyeawjTgXO11hcBfwderZungH2AacVFE0II0Ut5CsLF/ucWTeZLp3RCCDGE\n",
"5SkIj+DGQ2h259vvBh9HCCFEKHkKQr8x5rpmM7XW5xeQZ6hIQgfIkIQOkCEJHaCJJHSADEnoABmS\n",
"0AEyJKEDZEhCByhKrr6MQpC+jIQQIr+u92WktV4S+CLunoSVgbnAlcDvjTFv5lmXEEKIcmm72anW\n",
"eixwA3AqsDuwmf/5K+AGrfXSXUkohBCiJ/LchzANGAf0AROBpfzPPj/9mKLDCSGE6J08p4w+Dmxn\n",
"jEnrpj0JXKi1vgV39HBYkeGEEEL0Tp4jhDcbisG7jDFPAm8UE6n8yth3iWRqXxlzSab2SKbuylMQ\n",
"RmutV8ya4acvVUykISEOHSBDHDpAhjh0gCbi0AEyxKEDZIhDB8gQhw6QIQ4doCh5ThldAfxFa304\n",
"cLMx5j9a69HA1sCPcV1bCCGEGKLyHCH8ABgDzABe11q/CrzuX4/x84UQQgxRbRcEY8wLwJa4Hf9t\n",
"wFP+51HAVsaYF7uSUAghRE/kujHNGPMqcJx/CCGEGEbynDJqSWt9alHrGgKS0AEyJKEDZEhCB2gi\n",
"CR0gQxI6QIYkdIAMSegAGZLQAYrStC8jrfV2tN+ltcJ1X7FGUcFqpC8jIYTIr+i+jGbm3H65e8kT\n",
"QgjRUquCMND4B41kPAQhhBjCWhWEluMfNNJa9xeQRwghRCBNLyobYw7Kua5fDTKLEEKIgAprZQSc\n",
"XeC6Sq2MfZdIpvaVMZdkao9k6q6mp4y01jPJ18qo8BZGJRZTvqZmMZKpXTHlyxUjmdoRI5m6ptUR\n",
"wqa4Hf2oNh7SLFQIIYa4VheV/2mMmdLuirTWd3USQCk1GTgfONZae1Yn6xBCCDF4rQrC/jnXlXd5\n",
"lFJ7AnsDLyH3MQghRFCtWhndk3NdX+1g+3dZaz8NvNzBe4UQQhQoV+d2WmsFbIK7gDy6bpYCdsm7\n",
"cWttNe97SiIJHSBDEjpAhiR0gCaS0AEyJKEDZEhCB8iQhA6QIQkdoChN+zJqpLVeCbgU1wV2JmNM\n",
"R81YlVIzgTOttYs0XZW+jIQQIr9O9p15duDHA/cA6wIPAO8HPgBsA/wV+GaeDQshhCiXPKeMNgc2\n",
"N8a8obV+wxjzLz99jtZ6L9zRwymFJwTiOE5qzydPnjxn6tSpc4CkfnqNv0kkzliNLC/Ly/Ky/LBd\n",
"fsqUKdTP6+/voDcha21bj76+vtl1z+9rNT/vA9ez6n5Z8/r7+22n65WHPOQhj5H66GTfmeeU0eJa\n",
"66X98+e01jvXZmittwBWzV+O3qWQm9uEECKoPAXhTmC6v7h8AXCZ1voyrfUluKvs1+TduFJqfaXU\n",
"n3HXJb6qlDot7zpCKGPfJZKpfWXMJZnaI5m6K09B+ClwMe4GstOAPwEfBaYC1wGH5t24tfY+a22f\n",
"tXZla+3m1tov511HIHHoABni0AEyxKEDNBGHDpAhDh0gQxw6QIY4dIAMcegARWn7orIx5n7g/rpJ\n",
"B2itvwIsbox5tfBkQggheqrtIwSt9QGN04wx86UYCCHE8JDnlNHBXUshhBAiuDz3IWyktX7bP69v\n",
"EWTrpr0CzAaON8bMKCCfEEKIHslzhPBjYB5wA/A/wA+BU/3rucBxuFHTlgCu1lrvWGzUUklCB8iQ\n",
"hA6QIQkdoIkkdIAMSegAGZLQATIkoQNkSEIHKEqevoxOAmYZY87PmPdpYC1jzDH+9X7A540x2w02\n",
"oPRlJIQQ+XW7L6Nts4oBgDHmPFwT1Nrrs4H35QkihBAirDwF4b3NZvhusSc1TH6pk0BCCCHCyFMQ\n",
"ntJan1rXfQUAWuuxuGsKT9ZN24SFx0sQQghRcnlaGR0BXAbsp7W+D3geWBHYAFgK2ANAa30Mrivs\n",
"84qNKoQQopvaPkIwxlyJGxXtAWArYHdgC+BeYEdjzNV+0UuAPYGji41aHmXsu0Qyta+MuSRTeyRT\n",
"d+UaQtPfWzDDnyZaHnjBGPNawzJ3FZivrGLK19QsRjK1K6Z8uWIkUztiJFPX5CoINb4IvDbggkII\n",
"IYaMjsZAFkIIMfxIQRBCCAFIQRBCCOFJQehMEjpAhiR0gAxJ6ABNJKEDZEhCB8iQhA6QIQkdIEMS\n",
"OkBR2u7LKBTpy0gIIfLrdl9GQgghhjEpCEIIIQApCEIIITwpCEIIIQApCB0pY98lkql9Zcwlmdoj\n",
"mbpLCkJn4tABMsShA2SIQwdoIg4dIEMcOkCGOHSADHHoABni0AGKIgVBCCEEIAVBCCGEJwVBCCEE\n",
"IAVBCCGEJwWhM0noABmS0AEyJKEDNJGEDpAhCR0gQxI6QIYkdIAMSegARZG+jIQQYhiSvoyEEEJ0\n",
"TAqCEEIIQAqCEEIIL2hBUEotpZT6g1LqFqXULKXUTiHzCCHESBb6CGEaYK21WwF7A+cppVYOG2lg\n",
"Zey7RDK1r4y5JFN7JFN3BSsISqlRwOeBMwCstQ8DdwGfDZUphzh0gAxx6AAZ4tABmohDB8gQhw6Q\n",
"IQ4dIEMcOkCGOHSAooQ8QvgAsCLwYN20+4FNw8Rp3/Tp0yeFztBIMrWvjLkkU3skU3eFLAir+J8v\n",
"1U17CSj9KaPZs2dPCp2hkWRqXxlzSab2SKbuCn0NAaDxzji5CU0IIQIIWRDm+p/vqZv2HuDfAbII\n",
"IcSIF6zrCn9ReS7wX9baG/20GcDl1tqf15abMWNGufvWEEKIksrbdUXQvoyUUicAq1hrD1BKrQHc\n",
"AqxrrZ07wFuFEEIULHRBGA38FlgbWBz4nrW2P1ggIYQYwUrf26kQQojeKEMro0xl6dZCKTVZKfVP\n",
"pdTnGqZv7LPdqJS6TCm1Qg+yLK6U+oZSaqZSKlFK3ayU2j5kprptH6yU6ldKzVBK3auU+loZcvnt\n",
"r6GUelMptW3ITP7veWbDY2zITHXb/oL/e7peKXV37bMKlUkp9XTD53SfUurSuvmhcq3j89yglLpL\n",
"KXVkCTKtopS6wP/u7lVK7d9xJmttKR/AicCZ/vkawHPAyj3OsCdwHjAb2K9u+pLAv4Dt/OtpgOlB\n",
"nknAo8A4/3pH4BVgQqhMddnuBsb75x8E3sTdZBg0l9/mucCrwLaBf39ntpgX7HMC+oDpwGL+9X5+\n",
"WshMv2x4/Qtg3xJ8VrcDx/rnK/j90u6BM10LHOWfV3D3c23RSaauh+3wHzgKeAbYpm5aP/CtHueo\n",
"+J8zGwrCx4BH6l5PBN4CVuxynhWAfRqmPQN8MlSmuu2t3/B6bklybQacDjxWVxBC/f7ObDEv2OcE\n",
"/A3YqkyZGnIsBjwCjA2dC/fFYre617cChwf8m5oIvANEddMu8H/zuTOV9ZRRKbq1sNZWm8zaDHig\n",
"brkngNeBD3U5z/PW2nNqr5VSCvctYK7P9GDdsj3JVLe9++pyfQJ35PJXAn1WdaYBxzRMC5ZJKfUb\n",
"f2h/pVIqDp3Jdya5HrCmP913vVLqwJCZMuwI3GKtfa0Eua7AnTlAKbU67uxFf8BMkf/5bN20ucDG\n",
"uP1lrn3C4l0IWIRm3VqsGyBLllWAlxumvUjvu93YDpgD3ADsy8KfV88zKaXWB84Hlga0tfZlpVSw\n",
"z0op9VHgPmtt6mrnu0JlegCYYa2drZTaFJiplNrGWntPwEyT/M9PAbsAywO3K6Ve8Nsuw9/5PsAf\n",
"616H/P/3BeBSpdQ/geWAr1lr71RKHRQo0xP+50TcURTAqrjf4yrk3CeU9QihpqzdWlgWzQY9zKeU\n",
"Wgo4HtjfuuPB4JmstfdZa9cDPg5cppTaLFQuf+Pj4cAJGbODZLLW/sRaO9s/vwP3bfPLDbl6mgkY\n",
"7X+eZq19y1r7DPB/wAEBMy3YkFJLA1vhjjZrQv6tXwTcbK39ILAJcKJv2BHqbyrFHaF8A9xFb2B7\n",
"YH5tkTyZyloQyt6txVwWzoZ/3ZMb6vypotOBn1lr7ypDpnrW2rtxO7uvB8y1N3C1tfbFumm1/whl\n",
"+ayqwPv881CZXvA/6/9vPYX7xlmGz+ljwHT/pacmSC6/s90Bd4G7dgrmUuCgUJm8TwNLK6VuAn4A\n",
"/B53MTl3prIWhEeA53E3rNWsC8wKE2cRs4C1ai+UUhXcaZLZPdr+ycCt1toLlVJL+u0Hy6SUWkEp\n",
"tVfD5NeBsQFzbQPsUWu2iDuM/oVS6uJQmZRShzdMWgVI/fPbQ2QCHsb9rupPI4z3uUL/nYMr7H9s\n",
"mBYq15L+55t1097CHWUF+6z8tcXPW2s/bK39DO500YyOMnX7qvwgrp6fAPyvf74G7qJJT5ud1mVJ\n",
"gM/VvV4Sd+6+1mrlaOD8HmX5DnAKbme7DK5QHh040yTc4EZj/OuVcN9QvhIyV0PG+lZGo0Nkwn3z\n",
"Xsk/fz/ufO5HQmby2/o1cKF/PgbX6mif0L87XGG6O2N6kFz+d1QFDvKvx+EK6sGB//+djW855PcH\n",
"D/psuf+mevKLHcSHfyauf6NZwI4BMqwP/Bl3iHU7cHrdvI2Bm3EXdC8Flu9BnjVxTcwaHz8Ilanu\n",
"d/VDv+0Ed0/CD0J+VnXb3gjXDO8Vn+2QgL+/bwM3+hyzWLQJcajf39J+p3KX3/6hoTP5bR8EfLfJ\n",
"vFCf1WTgOuB64E7cF9dRgTP9CLjPb/cqYO1OPyfpukIIIQRQ3msIQgghekwKghBCCEAKghBCCE8K\n",
"ghBCCEAKghBCCE8KghBCCEAKghBCCK+svZ0K0TNa69WBTxhjfhJg25fibh6qAJOMMY+3WPaDuL59\n",
"fm6MeadHEcUIIkcIYkTTWm+Nu/P09hDbN8Z8DNchWTseBTYArtRaLznQwkLkJQVBjFha6/G42/mP\n",
"N8YkAaO01UWyPyr4b2A14OddTSRGJCkIYiT7Lq7nyt+FDtIuY4wFjgMO1FqvGTqPGF7kGoIYkrTW\n",
"S+HGs90QN1LVo8BHjDGvaa1/Cmhcj49nG2O+kfF+hRtlrt8Y81bd9I/iOgtbF/iT38Y+wOq4btkP\n",
"xnXN/jNcZ4PLAKcaY35Rt45ZuPEEVgE+AhyL6wF2VdwRybeNMY0jWQFUtNYn+fUuB5wHHJFxveAv\n",
"/ufngCMG/LCEaJMcIYghyRgz3xizMa7XyVeBTY0xr/l5hwFXA9/LKgbeWrid9EMN673KGLMJ8CSw\n",
"EzDPGLMtriCMBi7BFYwvGGM2wo3K9jN/LaK2js2A3/qXRwN7GWM2ALYGdgWu0Fpn/d/7HnCg/3f1\n",
"AYfhilHjv/1l3HgFcfNPSIj8pCCIoe403Dn1PWsTtNbjgKnAOS3e90H/s9noUQp4wRhzNrgChOsK\n",
"/X3AJXXf8C/CDTCzS8b7AY41xrzo1/EwcBKuMDQOKARwljHmeb/sHbhitVOTfHPr/g1CFEIKghjq\n",
"LgSeA75UN20f4DJjzKst3rec/zm/xTIPNLx+rnG6P6f/HDChyTrubnh9m/85JWPZv2dsb9Um653H\n",
"gn+DEIWQgiCGNGPMG8AfgJ211u/1k7+EG3O6ldp5+WYtfCzuVFTjNDKmv0OT63HGmFcaJtWKyviM\n",
"xdteL+7/rtyLIAolBUEMB6fj/pa/pLXeHLDGmIHGsn3e/1y6m8G01ss2TFrR/3x2kKsey4LiIkQh\n",
"pCCIIc+fm58JHIAbx/m0Nt5WO+3T7FRPUTZueF27+DxjkOudwKKntIQYFCkIYrg4DXe+/eO0vpgM\n",
"gO8i4hHcuNlZFM1PJzVOb7XsIVrr5QG01msA3wJuAi5uY72Z07TWq+BaSF3TZJtCdETGVBbDgtZ6\n",
"CaAKXGqM+XKb7zkM147/vb4pJ/6U02nAOsBrwL+ATYEzgO1x9xc8CPwKuCNj2cnGGKu1nobrkmIj\n",
"4KdAhPtWfzHuPoTa9s5sWO8pwHTcYOmr+6gPA7sbY57y7zkI11ppkjHmmZwflRBNSUEQI5bWejRw\n",
"J64ZaaE3eNUKgjGm0KNwrfUyuFNFvzDGnFzkuoWQU0ZixDLG/Af4KPBfWusvhs4zEF8M/gqcK8VA\n",
"dIMUBDGi+WsJmwFD4dTLssD3jTGHhw4ihic5ZSREwbTWd+CuGawM3Av80hhzRthUQgxMCoIQQghA\n",
"ThkJIYTwpCAIIYQApCAIIYTwpCAIIYQApCAIIYTwpCAIIYQA4P8BjkknnSiMywgAAAAASUVORK5C\n",
"YII=\n"
],
"text/plain": [
"<matplotlib.figure.Figure at 0x7f0ba68ac590>"
]
},
"metadata": {},
"output_type": "display_data"
}
],
"source": [
"plt.plot( vs * mphfact, 100./mpg, lw=3, alpha=0.8 );\n",
"plt.xlabel(\"v (mph)\");\n",
"plt.ylabel(\"gallons/(100 miles)\");\n",
"plt.ylim((0,5));"
]
},
{
"cell_type": "markdown",
"metadata": {},
"source": [
"The peak efficiency of our model car is:"
]
},
{
"cell_type": "code",
"execution_count": 97,
"metadata": {
"collapsed": false
},
"outputs": [
{
"data": {
"text/plain": [
"41.421056713604862"
]
},
"execution_count": 97,
"metadata": {},
"output_type": "execute_result"
}
],
"source": [
"from scipy.optimize import minimize\n",
"minimize( lambda v: -v/( A*v + B*v**2 + C*v**3 + P0 ), 25 ).x[0] * mphfact"
]
},
{
"cell_type": "markdown",
"metadata": {},
"source": [
"# Dynamic Programming for Flat Terrain"
]
},
{
"cell_type": "markdown",
"metadata": {},
"source": [
"We need to define the acceleration and fuel consumption of our car"
]
},
{
"cell_type": "code",
"execution_count": 229,
"metadata": {
"collapsed": true
},
"outputs": [],
"source": [
"# the acceleration of our car\n",
"def f(v,u):\n",
" return u - A/m*(v>0) - B/m*v - C/m*v**2 \n",
"\n",
"#the power consumption of the car\n",
"def r(v,u):\n",
" return m*v*u*(u>0) + P0"
]
},
{
"cell_type": "markdown",
"metadata": {},
"source": [
"Now we use dynamic programming to find an optimal control trajectory for our vehicle, on a grid of positions and possible velocities."
]
},
{
"cell_type": "code",
"execution_count": 230,
"metadata": {
"collapsed": false
},
"outputs": [
{
"name": "stderr",
"output_type": "stream",
"text": [
"/home/alemi/anaconda/lib/python2.7/site-packages/IPython/kernel/__main__.py:26: RuntimeWarning: divide by zero encountered in divide\n"
]
}
],
"source": [
"# define our grid\n",
"XNUM = 161\n",
"VNUM = 100\n",
"VMAX = 50/mphfact\n",
"dx = 10\n",
"\n",
"# create our dynamic programming variables\n",
"costs = np.zeros((XNUM, VNUM))\n",
"uvals = np.zeros((XNUM, VNUM))\n",
"args = np.zeros((XNUM, VNUM))\n",
"vs = np.linspace(0, VMAX, VNUM)\n",
"\n",
"# set the end conditions\n",
"costs[-1,:] = np.inf\n",
"costs[-1,0] = 0\n",
"\n",
"# fill in all of the other values\n",
"for i in xrange(costs.shape[0]-2,-1,-1):\n",
" # for each position going backwards\n",
" for j,vim1 in enumerate(vs):\n",
" # for each of our discritized velocities, compute the optimal cost\n",
" v = 0.5*(vs + vim1) #candidate average velocities\n",
" dv = vs - vim1 # candidate differences in velocities\n",
" u = ( v * dv + 0.5 * (dv)**2 )/dx + A/m * (v>0) + B/m * v + C/m * v**2 # candidate values of u\n",
" u[np.isnan(u)] = 0 # fix broken values at 0\n",
" dcost = r(v, u)/v # compute all of the candidate costs\n",
" # apply our physical constraints on u\n",
" constraint = np.piecewise( u, [ u < -b, (u <= a)*(u >= -b), u > a ], [ np.inf, 0, np.inf ])\n",
" # all possible new costs\n",
" newcost = costs[i+1,:] + dcost*dx + constraint\n",
" # take the best one\n",
" costs[i,j] = newcost.min()\n",
" uvals[i,j] = u[newcost.argmin()]\n",
" args[i,j] = newcost.argmin()"
]
},
{
"cell_type": "markdown",
"metadata": {},
"source": [
"Having computed all of the possible costs for all possible $x$s and $v$s,"
]
},
{
"cell_type": "code",
"execution_count": 231,
"metadata": {
"collapsed": false
},
"outputs": [
{
"data": {
"image/png": [
"iVBORw0KGgoAAAANSUhEUgAAAaEAAAEbCAYAAABz+TvRAAAABHNCSVQICAgIfAhkiAAAAAlwSFlz\n",
"AAALEgAACxIB0t1+/AAAIABJREFUeJzsvXn8NUdV5/8+9xvDjtkwyggim5CECUtQE5VFQIVRRMU7\n",
"ERADUQZBZBnAGRQIKsM4BoRBkwgBw8wvQa+/iCCBMRBCIJpoAkENBAlRMMoSQhbCsCTPvWf+qKWr\n",
"Tlf17ftdnu/y1Od53df36dNV1VV9u2/VqfM554iq0tDQ0NDQsB2YbHcHGhoaGhoOXLRJqKGhoaFh\n",
"29AmoYaGhoaGbUObhBoaGhoatg1tEmpoaGho2Da0SaihoaGhYdvQJqEVICLHisgVInKLiFy43f3Z\n",
"TIjI40TkMhH5mIh8WkT+dLv7tNfg7+/nRWSx3X2xEJHTReRqEVmIyCM3ob37iMhLN6NvlfaL7+Jm\n",
"j2OjEJH7ish/FpH2W1vBrrkxInK4f+g+7x+wq/2x/XxTRO65FX1Q1b9T1YcAlwM71sFKRE5Z5QUU\n",
"kW8H3gG8VVUfDJwA/MBW9W+rISInicgvFOR3FJF/FpHf2Y5+qerDgTPYxmdHRB7sn49vTeWq+svA\n",
"L4bDDV7jBOAi4G830s4Qau/iZo5jk/BPwIOA94jIwdvdmZ2IXTMJqeqX/UN3hhedrKoPsR/gc/uh\n",
"O7IfrrERvAJYZRX4fcAdgQsAVPU6YEsm8v2Ek/zHYh/wGeCL+7EvFsL2Pj8Pxj0f31o4t+F+icgR\n",
"wDuBV6vqBzfa3phLjpRtC1R1ATwDuDvwe9vcnR2Jg7a7A+vAsgfsGcB1+6MjOxyrvIiH+r/fDAJV\n",
"3be53dl+qOqtwKO3ux87BFv1Q/1fgNuAN29R+7sOqqoi8tvA20XkDar6qe3u007CrtGElsFvMbxS\n",
"VT+oqt8Qkd8u7Q17m8eXReSfC238exF5l4h8RkQ+JSJ/KyI/vcF+HSoivy8i/yQify8i/yAifyIi\n",
"jzflThCR80Xks94m8zci8mRTZiIiv+Hb+ajffnyHiDzRn3+CiFzhiz872aJ82kD/3gm8yh++x5f/\n",
"b2P7JSJ383W+7Le6fsCX/7i/969ccn+OFpE/8+3/o4j8nYg8e6D9J4rIxSJyld+aPTVsc4SywMOA\n",
"45Lxv15E7u2/+xvS715EXp709Vki8jr/vX9BRM7x398j/ZhC/37YjOGuIvK7/ju53F/zPBE5dmjs\n",
"S+5LtB/5e/oB/+x8SUTOtNtpIvICf1/C9S+xz66InEH/u75CRO5qLn+ov8blIvIvIvImEbnDiD4L\n",
"8PPAhaVFTHg+/b39W//dvcaP8QoReaYvd5SI/JG/1x/x39ubRORuK9zCUv8OFZE3insXr/LP8++K\n",
"yJ2SMqlN6Sf9ffhI7T6IyENE5L1JXy/zYzrCXP7/+L+9beIDHqq6qz7AKcACeGRB/koje6Qv+wgj\n",
"/yPgn4zswcBXgbOAiZf9DDAHnmbKfhD4wIi+3hm4Evgb4DAvuytwPnBFUu5HgFvT/gNPxm0fvSCR\n",
"/VfgC8C3+eMJ8HrcS59edwG8YoV7+gu+zj2NfFS/knt6M3Am8C1eduZQP4B/D9ziy4V7/kNe9vpC\n",
"+18H/hS4nZd9n7/mH5uyF9a+n8p3/11+/FcCD/Kye/p+fAD4n0nZNwJfBu6cyI4BPg/cK5H9PHAj\n",
"8B2l53fk9/JK36/zgUO87H7AvwIXh3vm5dcBT0iOj/X9fPyY79qfe5Q/d0kYC3Avf49fPqK/D6g9\n",
"e8Bj/HPzxkT2H3Hv3NyU/RXc1vAd/fHBwOnAXwOy7F1MxvGIRBbexcuAw73s3wGfwL2fBydlw+9G\n",
"eh/uDXwlvQ++zevJ34/vAb6E+c3x5/4F+Kux7+WB8tn2Dqzc4W4Suhq4Ivl8nv4k1HsYvfws4J+N\n",
"7ALg/4aXPZF/wL/06Qvfe/Arff0Nf/1HGfnDgYuT46ttf7z8L4CvJS/Nu4F/TF9E4DuA55h6q05C\n",
"J1GehJb16whzTxckP7o4u8PhA9d9v3+x72Lkb/RtHVNo/x6m7Ou9/Lgx30/lu7+Xb+ONRn6el6fj\n",
"/F4ve3Qiux3w3YVrfR74z6Xnd8Vn/YeM/AVe/uREdlSh/h8DfzHmuzbvy0sK9+FDI/r7477+swvn\n",
"LsVNZnc08g/Sn4SOAO5mZGGCe2ih/phJKLyLjzZlf8bLnzviPrwnvQ/Acb7cUwv3+D6Fe3A58MUx\n",
"3/2B9NnN23EZMYENsI5E5I641c+VqnqTOf33OKPiUeto+gm+T5ekQlW9TFV/0F/7/sB9cC+pxSXA\n",
"7XEvBcBf4lbCfyMivygi366qn1fV09bRt0Gs2K+AL6vq58OBqt6sql+utH8HX//jqnpLoX2AHzPy\n",
"G1X1WiP7G//3h9k4rjLH1wPXq+r1RgZu8gdAVb8JHO234P4hbHMBhwH33YR+fcwchzGn9q01Efnf\n",
"fkvw7/z1H7fO63/cHH+ZZLwDCFuE30iFfrvre3Hv19dMnY/Rt099BThRRC4SkSv9WM715+43oh8l\n",
"hHfxr4289qxB/z5cT34fPoFboP6hiLzBb5uuqepZqnpNob2vUyaEHNDYzZNQBlV9lar+5jqrH4q7\n",
"F98jhvIN/AfcFtjh62j3bsDX/Y/UUBlwL7pF+MH7NgBVfSNu5XYzbnvi30Tk/SLy0HX0bRnG9Mvu\n",
"0dvJZAiH4e75Ku1/pVA21F/P92PxVXOsBVnw8YmkHhF5OvAu3A/asZozNW+30U4VJukw5iP89Y/y\n",
"174zTus+1l//XbjFwqoojXkMiSncGzupBOLLDYU6dtEHbkH5uziG3TF+LE/w59Z7P2vvYvaOGQze\n",
"Bz+hPhz3Lj4Z+DDunXyliJTu14TuHjV47JlJqIK5/2tfiruQa003+LIf0T7t+36qendVvWgd1/8S\n",
"cAcZ9g/4kv9b+hENxs3I9lPVd6jq43D72S/BGeEvsIbqTcBK/fJYhXF1A+6FXKX9QwplQ/3rC+c2\n",
"A2PG9CzgFlX9bXWU3FXqLu9Anzhgx/wUHMX+t1Q1naj3N1U5TDJ3NPIbce9b6bs+ND0Qkdvj7Gnn\n",
"q+r56akN9i28i3YSqz1ro6CqX1TVlwDfidPGP4qz5f1GofidKC+6Dmjs9UkobA3ZFfXR6YGqfh23\n",
"t3yMnTDEeX7P/MuxKs7DvTzHmzaP9xqMqKNrfhr4fs8uSnE8ToW/0Nd7jYjcy/f5OlV9HfDbOBX/\n",
"Xkm9ffjvVkSOEJHHrNrxVfqVVluh/VD/GBG5S6F9gPca+bdK3xH5BP/3gkR2G7AWDkTkSebHZ5Vt\n",
"2zFlb4dZ4YrIGuXV9XrwYHNsxxzGZvt694LsNv83PB8PE5ExW1xj7kPYzsy27lT1/+K2dR+UMtE8\n",
"jjVtH0RZY7j7iOsPIbyLJxh5eNbeM7Kd2FcReZCI/Do4GrY6v6gfx+1UlJiR30F/y/eAx26ehMas\n",
"jP4JuAa3vxxeumfhViS2/ouAOwCn+h8QROQw4A+Bz6vqN0z5Mdd/A27f+Hd8W6HN38UZOMMD/Ss4\n",
"zeblsXFHr3088OuqGlaY3w+8JKj6/of1B4Br/XXScYcf65/FserGwI5pbL9q9ZfhRbiX+nXJPf9B\n",
"HIPrDar6CVP+q8B/CxOKiHy/L/snqvrRpNw1vt9hq+oc3MS8rJ+rOD6m8nNxE+TzE9krcFthm6GN\n",
"PF9EDgXwE8aLgL/CRbnA/10AvyYdXf1HcCtze/1/8n/v6RcXf4Cz16RYlwOoqv4L7t4fUzj9Mtz9\n",
"+O9hUSMiJ+LYZGkbXwXeB/yoiHyfL3dH3P1ctW+p/A04G8/viMjhvt27A7+Ji+xw5sh2U9nhwH8W\n",
"kXS834vbafnLrJLIkbjF8PsqfT1wsV2MCBxL6ULzuVNy/sG4fe6LcXvb96Fjwc3p2HEPXXKdh+GM\n",
"kZ/B7dn+Eo6m+01f/0eSskcBf4b7Uf8ojs3youT8sb7OLf5zBQmDq3L9Q4HfB/4ZR3K4AkNv9uWO\n",
"xz24n8W9yJeSsJ98mZ8A/hz4B9/OJ4CzgXubck8APoUz+v7t0D3Cebd/1t/Tj/t2v3Nsv3Aax8dw\n",
"2wzf8PVfu8JzcBTuR/yzvs9/T5lddZa/h4/Eaa1X+Wfhd/GU8KTsffx3fhWOlvvzOIrtFaafPwI8\n",
"x4977vvwFtziLB3TR3E/Lr2y/noT3ET9T34MHwBe6p+jLwMf9eUup3t+rwBOWnJvTsFNLg/C+Zn8\n",
"A24L7s3AXU3Zn/F9/jd//dNwLMZv+v4/LCn7B3TP4znAtwCvxr1T4d06xZe13+3DlvT5JTg7z10L\n",
"537U9+UL/jl6JW4SWJhyR/jv+9+Aj+B+uJ/v78VngTfRfxc/ipv8TjfjSCnhhwD/04/9Ktzz/Lsk\n",
"jD3czsKY+/BQ3CT0Gn/tj/gyVwD/qTD2X8axb+82dP8OxI/4G7TfISJ/pKrPqJw7GPcAPF1VLxKR\n",
"U3AU1On+7GPDzoGInIXzDfvu7e7L/oJ/7l+hqrtmx8JrqR8F/lxVf31E+Tfi3vM9yxoTkTvjJr3X\n",
"q+prt7s/Ow079eF+PLBPOzLAmcBPBzW64YDF9qyYGkZDHfvs8cCTROSXgtxHFnhVocqDcVrEnoSf\n",
"gM4H3t4moDK2dRLyITI+JCLvEZFHJaceTmLAU9V/xTlHbgUVuWH3YMcEpmyoQ51t6OF0DEtw5JkX\n",
"ishDgsBT24/HbQXuVdwVeJmqbllai92O7QxgehVwgap+RESOAy4UkR9U1b8DjqTvE3ITm8c2atgl\n",
"EBcv7Hwc0eJO3nfrZapqmXN7CiJyOY5coX7Mv6+qb9nmbo2GOh+aP09E/4iL4PB2EfkmblL6LPBj\n",
"qnpBoYk9AVX9HPsnsv+uxbbZhCxE5I+BG1T1OSLyZpyx8KnJ+X/B/fj8f9vWyYaGhoaGTcVOSuVw\n",
"LV1onOvo+0YcgnEou+CCC3bGDNrQ0NBQwWMe85gNbyOv+lu3GdfcX9i2SUhEXqqq/yMRHYmjZIKj\n",
"Ff/HpOw9cF7YPQPmXW5+ELJQZO6CI8jCf1cL5+s2ifJFJpf5PmThzwUn9/k+X9YHWtBQ9tbuWOdG\n",
"5o7V/2VRkHtZiG6vPpCDBrl3YQltLPRWFqGMrzPXW7O64fy+RB5kt3FrVqaT3+aP3bjmzKNs7mXh\n",
"3K2+T3NZZOdvZR7/P/c8gVt9++E4yqUr15X1ZfzX1MnDcff3Vs1lwXvxVhV/nySev83Iwt+afN9i\n",
"gtp2FpPsWI1cF9LVmU+yMrrwf+d5XRYSZYQy4edkn5fHskFOR8HwdWSfF0R5KOsEsuga7sqGdyGU\n",
"CTd0EeViyoRn3h5LemzL+r9akbNYoFaWBZagd161UMeUtX8zGf1ztToLf1NrZRf027d1F7aul5//\n",
"vvPZLDz6h8eFR7zwAx/YtGvuD2wnMeGFfr8fEflu4InA//bn/g9wkIg8wh8/EzhXK8EwAbSlcD+g\n",
"sba8SEPDrsZi5L/dhu3cjjsVeIeI7MNFMHiuqn4YHM1TRJ4EnCYic1xMqpNKjehE4ipNZQKTsHLz\n",
"50PEmcnEyxfxOCwqJawM/UQWvDKifOJ/4hZ0Go5z8O/oWkETEvNzqPMoC2WDxiOhbFzkdvJJ1AKc\n",
"RjLxZcNiM+gHudzLzE9y0ITWjHzOvCcLD/FaWJ+EfkiQC2HtMo/t5pp/0G5SeU8muUZkJ5F5QRZe\n",
"ry4eT2hDOrmElamTTbovyMi1e129THzdsJyZGzmTTtOR8AWF54wgD53VTh7XR/FmxvbSsvEJKQSt\n",
"ic+k/Y3xA1QUCQq8l8Wy9jrdTYlKRuz3nK2FXSzqiB/NUKdSVsJ7Wzg/dG63YafY7zcb2zYJec58\n",
"lTevqh+jH+epoaGIiWicYBoa9iIWe9RNbicRE9YNnUi35g4rOa/5dNrOPJcvQMK+sV8tdatBoxmF\n",
"Nui0mm65nGs18TFJtJ2ezK/l429mQTPqtKKDoiwdYEnbsbKAmrykus8r6vxatnObakXuCkV5ssDv\n",
"yuar/q5n+Qu21hXpFIhwMmgmwT7jz01EO1ksE+p0GpCFGlnMR2A0o0Uiiwgail+ldppFsP9op5Ub\n",
"DUUnRgMq/cZoXibYGvrDkK7jk0QLA8Srm0UNaWEaCi/BJNhzwu5A2GGY9M51VYNtyGg7k2TMIzUS\n",
"STSmrdBiRCaj2534L3m7t7qs3WmvYE9MQg0NDQ17HfvaJLRDMZmg6Qplka/khKD5dLYgJydutMe1\n",
"YFjFWttQXDmu9ffqjb1HjLwkM2aKTrtKFr1BW4qmh2guCAy6cOz/ylpPNq9qTZ45mGg3PVtQbMPL\n",
"481IZEYDCppKyUZUtRt1NyeXZ2XpyVJ5pgGlsrSs9jWhRcUWtDBlU9tRzxZUsBtBYjsSjc9RZwsK\n",
"bLWg3XjENhKtJiqb+XVLSmj3vBoVMtpF4wDjPYg2pf1tG6phif0nKzpkCwrv/Qray063H7XtuIaG\n",
"hoaGbUMjJuxQqAgymUDw7Ykru6DxeDab14CinxAFW9Aitxt1i6iOEdezG3nmXFxJllhyVebcvkze\n",
"sef2RU2oE+b2pEBwmmui7RhZn/kWyvY1oElNAyrI6xqQlXutRqXrW0/TGaEBVWxDa4ltqKcBGc1o\n",
"zWgFC12uLVn2XMqoi/aWigYUtBqZSPf1S26bsTac5Cb1nSc6tc9f37cf5Ul9oxIP2oZK2pHruG+j\n",
"bxuSzdCSDEuue/Y3pgFtZd3txkZ67JNynoHL33QQLvpMMbeRiDwVeCHujbtIVV/s5Qfhcoz9JO4r\n",
"Oxj4DVX9gD9/Ei7lRpqy/UHAL6pqGsIpw66fhBoaGhoOBCw2pgmdAqiqHu8TI14qIg9U1SwKjU/Q\n",
"dyou+/SNwAUi8hxVPQ2XwvxXgWNV9RYReSzwThG5v6p+Hpe76rmq+te+rYNw+bWyBH8Wu34S0okg\n",
"c+1Wlx7Rdyis+sg1IZ1MEkZdkCX+QCTsOS+XxE+o5+9Aru1kPkFVP6Gc+RYZcRzUEaY0t+ssjDxq\n",
"RnSaTipzPavbhqwGNMo2JEY2wjZU8h3K5AXbUM0GVJLXNKBNtw2FsivYhjoNKHRihG3IaktjbEOR\n",
"dUfW7qbbhipaUne6wJIz9tVVNBExLNUhe9Fu1nSWYb3EBJ9V+mTgpwBU9WofFPdpwOtM8ZOB89Rn\n",
"TRaRt+ISNJ6GCyr9clW9xbfzfhH5Bs6V5lxVfbtp68eAD6vq14f618IMNDQ0NOwCLHTcp4B747LA\n",
"fjKRfQI4rlD2OFPuKuBoEbmdqt6gqmeHEz5N+8HkKTtSPJUuCk4Vu14TgsQuBCvZhrrF93jbUGS4\n",
"bbFtKMSMW8U2NDFlt8o2tFwD6tuG1oJT1FbZhmoa0IBtqAsgMN42ZGVjbEPRZ2gV21Bm60k7Fwfo\n",
"r5/YhmLZ8bahalSF2FRJ69kP1DmZrGQngmHfnyENKVxpp7PPNrAdd6T/e3Miu5kuYLQtm5a7CfdA\n",
"HUEX2zPgkcBnVPVDthGfzO844CnLOrcnJqGGhoaGvY5NmCJtE7UQI6VLZWU90eHVVMKpAT8NvENH\n",
"UPoO6O24qBWtghYodc+gFD1hGXoRExoa9hM2sB0XyAeHJLJDgC9WytpySrLl5rfh3gS8TlWvqHT3\n",
"KcCo3G+7XxOauG0GSz8N2GyCgizmbiJagaCgOkBaoExQkPDVrEBQ6DbBxhMUattwQwSF1HE1yPKy\n",
"m0NQKG3N5e32ZVtNUFioJxqsQFBIHVdhBEFhomnH88HXaN0L7S0pewSFvudzEtYqcZQl2VYuLIo7\n",
"ivdIgkJh965HNshHsz7iwiaTEXYayWFfZYYZgWtwQaAfAFzsZUcB7y6UvcyXIyl3pap+M5G9FrhU\n",
"Vc8VkYOBI1X12nBSRI70sr8f07k9s6yP0bIbDkisNQ2lYUXILtvV0JGfXj03i74ZlxIHT9E+Fjhb\n",
"RI4QkQ+LyKG++JnAE0TkMM+qOwnnX4Sv+2u49d/bvN3nvqHdBCcCZzMSu14TylM5rEhQSBxXXf0x\n",
"BAWTymEMQaGU3gEGCQqp46rr43KCwtxoS2MICjUNaLMJCpnjajqOWNas7EVZq8wrlqAQrpwSFDDa\n",
"jJWvqXaUZd+Q1ZasFpX6dqolLQStI2yBp+F0EsdVJwvkCKu9B8aFFIgCll6ddRWdMJjmAaikelhC\n",
"3459TrSeVXkJ6Y99bH8D2kV4T/3hkKayLHzPKoFMtxvrV4QA5yd0hohcgvvdP1FVr/MJQ+8P3AG4\n",
"UVU/LiIvBs7HPWUXqerpACJyf+A1vr3nmbZTnAj8zNiO7fpJqKEBMjeZhoY9iY1MQn477RkF+bV0\n",
"7LkgOwc4p1D2U4zYPVPV41fp296YhCZl+mmKom0osQs5mceAbShzXIVRtiFrAxpjG8ocV7PCYci5\n",
"bQjt24ACFkvkKawGFGBtQ0DPNtSVDW9Lx/9dsxsFcdFvbUTueF6RubJk152nykcp0V1B7oKehl72\n",
"7UPQd1ZN6y9NgBd73gU97Rbc1hbUlXVt0KV5WBgNJXaOrA2QHiW7V1by6zrVLneUHWUbigF+/U6C\n",
"tQ1tMoPbbpttdWif9bQ78fKtTLcw36OrrL0xCTU0NDTscezR+KV7YBKauDVaahcCRtmGbFDTzvaT\n",
"24biKlEXvdAoYanY2ZFCvzrbUFQUrA2oIhdSTShcJ/zJE+ClacDHpQBP5BlLrp4C3MrHpQB3Vwht\n",
"jEkB3pOPSAFu5QsrG9CMxqQAT9tEZbkGZJxVJWG6DaYATxtFx6UAzzqXsuHIMWAb6squYBtapvHY\n",
"4KSTRTHRXdrXEksuyraAKbebbEEBG7QJ7Vjs/kmooaGh4QBA04R2KGJqb5PSuOZn0dlhJGHO+HOF\n",
"FOCujrcRyWRcCnAgSwM+IgW4lY9JAe7+JPakAU0H0nA9fQ3Ili1pQEE+Pu1DaENItaJORqJF9W1D\n",
"fT+jum1onisio2xDnU+R+7NVtqFOAxpvGwoKyWq2oXK4nl7Z1DZk3pNNsQ1tU0a8zdJudpp/UECb\n",
"hBoaGhoatg2NmLBTEVeMgfWWMH+g02oIq7VgO+r2tuMKNK4QvU3I2IoUEKsBhb1t4yfURbhMgp7S\n",
"yZyg7j/UZ87VNKDONjTH+RStYhuqaUubbRvqaUCx3bptKMpG2IasbJRtKNh8VrENjdWAEttQL6rC\n",
"KNtQ4jOUXmiTbUO5z1By3SHb0DKNx/oWDZXdCFbwF4pVdqiWMwaqsrzQLsTun4QaGhoaDgC07bgd\n",
"jGgXgm5VacLkS1I2HseAY+NtQ72oCmNsQ7HsCrYhExFhjG3IxqDbKttQF4NuvG2oxJhzZYdsQ9YG\n",
"NGAbKvkMwaBtqBc/boRtqBaDbrNtQ6nPEIy0DZXiydXKhsZtkrwRtqG422A0nmgbKsWKM+zUiAGW\n",
"nHGN64V8Xim+3JLICa4r7go7NaXDLlTeRmFPTEINDQ0Nex1NE9qDUJk4359V6oRI2lsNWaMYfrhh\n",
"07DGOsKgie6fvfkWh6jBYDHfmzah3RVGtgCdSKan60ScbEI+uskEJpPufCgvJZm4z2SS5xwK23OT\n",
"ZMtKJiATdLKWyydr7pM47qmsOZJC+MQ21vJPIhc5qNtmw23DiawhHNSF9PHyCWvZdtpE1pjIGsJa\n",
"FwDVyKOMtewTsOb/Tfy/VBZKd2V9bZ10IX3AX92WlexTkvdkkgQvxU0i4VOSleQT0WxLrSSbSPhY\n",
"ud8uS2Qiivhyk4I8khJw23Ay0Xiuk/uPkbtnWJNtNrrnWjRuNUa5YN4FT1Iw8jDA5Cvq3gGRbg/Q\n",
"DcRv20m3/Rb+HzpuEeRpuW2CyGTTomXXQlrtN+jITwEicnsROUtELhGRy0TkcbXLiMhTReRyX+7U\n",
"RH6QiLxARC4UkQ+KyF+LyA9X2niViPzzmGHtGU0ojabdcOBhPVpNyA/U0LAbsEEN/BRAVfV4n8rh\n",
"UhF5oKpelxYSkWOAU4GjgRuBC0TkOap6GvCdwK8Cx6rqLSLyWOCdIvI9qvq5pI0jgV8E0hxEVex6\n",
"TQizmstWbHQrvKDdDJXttKWCBjSZRFICeA1KJvFcRNCMgtYDnYYTtKNQ1Gs1VgMqyoI2UygrRqsJ\n",
"2lNPAypoO+H/4V+/7CRbAVqtyJV1/6qaUVEDqshVYrqHVIfLtZqanJ62FBSHNdEs51CoZzWdiShr\n",
"uI+Vl8rWNKBUHrWingbkNaWeVlMoGxSUmmaUaf2+TO/5LpSPmo5tI2hL+S4BhXdpqWbkBgsyQSbi\n",
"SApDZTeAoPmsogFtpra0ldDFuI+Fzwt0MvAWAFW9GrgCeFrhMicD56nqDT4191uBZ/tzXwFerqq3\n",
"+HbeD3wDsFGzXw6cRp9LUsTOv/MNDSNQicrX0LB3sP7tuHsDhwOfTGSfAI4rlD3OlLsKOFpEbucn\n",
"ppiszqf5Ppg89fd9ge+gy+C6FHtiO660FTfGeTVzXIVxzqs9OvcI51WT9mGU8+rStA8linY/qCkM\n",
"O68OBTWFsvNqLaTPkPPqUop2bLvvvNqTFZxX5+RbcmOcV2shfYacV0tBTWEJRTtxXIWOoj3kvFoM\n",
"agrDzqsFx1UYdl4dTniXXDd1Xq0lvIsJI0MbyXu1LNjpRqjaiXwnpAXfKuj6iQkhX9DNiexmXOru\n",
"Utm03E24W3sE8G+m7COBz6jqhxLZq4BXAHcb27lt14RE5H4icpuIPCKRPdgb0C4WkXeJyGHb2ceG\n",
"hoaG7cZ6t+PSJsxxbVYr6VNZWRG5PfBq4KREdhxwq6p+fNlYUuwETeg3SQxYInIw8E7g6ap6kYic\n",
"gstxPi1V1olfARmnxzHOq5njKoxyXi0FNYVkFVtwXi0GNYVB59WqQ+uQ82qiFTnZcufVmpPqkPNq\n",
"LaTPuJTgVgOqO68OJbyz8lEJ74DUebUW0meM82opqGnaRqoZDSW8g04zypxXBxLewRLn1VLCO9fp\n",
"etlSwjs3QH99X2yumeOqa31YM/KDJMf2uh+sEux0x2hM6+ddBfLBIcD1yf+/WCl7SHJ8iL9yuuUm\n",
"wJuA16nqFUnZVwHPWbVz2zoJicjDgVtIBgg8Htinqhf54zOBz4jI4ar65f3dx4aGhoYdgfXPgdcA\n",
"NwAPoLPVHAW8u1D2Ml+OpNyVPj14wGuBS1X1XK80HIlj0t0HOMvNURwCfLuIXAj8iaqeUevcdmtC\n",
"pwDPAlJ9/I2UAAAgAElEQVTO+sNxxjAAVPVfReRrwEOB9/Va8PvacQ87rhQTnwa61VpcUeqkH8LD\n",
"pCmOoT6SQKadmSDXlqKGZW1DC+gFNQ1NGHuPGHlJZswUnXal3cmY5nuEbWhMwjvX9Y3ZhoYS3uXy\n",
"0LYutxcltqFSUFNXNkdmG1qS7qFkGxpKeAdl29Bgwrukkcw2NJTwDiq2ISvLx160DQ0kvMvLJrYh\n",
"a48aZRvKNZ9qGJ8t9tAdE75nFezPUD/r9V1X1YWIvBl4JnCxp2gfC/yciBwBvAN4oqreiFv0v8+b\n",
"QG7CbbfFCUREfg33qrxNRO4M3BP4WVV9FcnkJSKPAv5IVR+9rH/bZhMSkcfjZlhr7DoSRwVMcRPw\n",
"bfulYw0NDQ07EYuRnzJOwe2kXQKcA5zofYTuANzf/8Xbc14MnA9cCnxUVU/HVb4/8BrgebgdrK8A\n",
"V2I2CkXkBf56R4jIn4rIsUPD2hZNyPPWXwr8VOF0jWhYNqLlC+tuNWkDL4biSRrwMSnAgSwNeKeZ\n",
"GFvQorMbuePQwSSVg7UbJSnAAWQES66T78vkjhm0z/8/+CeFP7ltKCxcVdPoDAVZAqsZQafxWMyN\n",
"fM6CUlBTWKIZxUX/GNuQleXjKtmGhhLeQdk2NCbhXdqGiA4GNXXNl2xDuY1mlG2oplSsxzZkX8GJ\n",
"qZvK5pWy84IsaD41+0pkohZ+BNbBfOs1sZvZcxvoht9Oe0ZBfi0dey7IzsFNVLbspxihuKjq64HX\n",
"j+3bdmlCTwH+UlVvSmTh6beGMfzxdTQ0NDQcqFi/n9COxnbZhH4QOEZEftQffzvwehH5DHAWcGIo\n",
"KCL3AO4IfKTY0gTQxN8hyMPK0TLg0jTghgEUr+n/hjAZaRpw1fw6cZFk7D5hIaa61vN9kF6s+75P\n",
"kC71E8q1G2XeaTySlw1azMLIJdFPgs0n2pNiz0qMuoWX5WuYni3IY41JMvggizcu1nbyvsI7xjY0\n",
"lPCuJF9Q14CGbUNWttw2NJTwDsq2oeGEd10/MtvQiIR3kNiGipqK6aPZWXA+eXkfgj9dp+mHbgT7\n",
"6KKf3iHuJHiW6Qh7hySMU+hvjSRWsQ1pLzs2pcMOUcg2G9syCanqs9NjH+ju+ar6IRG5HfAGEXmE\n",
"d4J6JnBuY8Y1NDQcyJB9292DrcG2OquKyLEi8v/jvHF/U0Se7/cunwT8dxH5MI4V9+yhdoCeNrNV\n",
"yGJmja0zKdtZBmFtQGOqrKPOerCeaMJWO9pslDSopXVk/6x0rQ1oXKX9tArfX78AuyA2245H247b\n",
"fKjq3wFPLsg/Bpwwqo2JN9oqWRiSSM22zqopZbsQyierMxTOJw3lk7XhkIXz0YXLQ2QICZitvM75\n",
"D7c/UZpUjCxz1wxOroFNG7f5/GXi1t1avBVzs2XXGfJr9Ou+Y2ptey51Vg3khL4D68KXpSeP227B\n",
"Mzdu5VnjuSSU7vwtHCIoLHAT0SoEhaGsqyX5egkKuhDfjxUICkNZV5M2OhIC/r0h2b72pxbmHg+F\n",
"7ekG68vm7022PWdlkNcJmNAReBoc9miWgO32E9o89H9/Gg4grEmfpLUMyW9vQ8OORy/23x7B7p+E\n",
"JnS/JmLkLKFsG8OtxKR1lnwQjKjazXNeA4p1LCEh1iFZeRptKVw/VE7C+VTD9gyE8ymG8kkKl8L5\n",
"WCJCHN4SeYoxBIWoAcWltpEXNKMxwU0zuRCJAV3ZHGtZPTe+VQgKpaCmMExQqGpAAwSFUlBTd9UB\n",
"gkIhWk5WNoal6hxUI2Gg465n1yu9RzVH1iGCgobdBENa6NBnJlgiQsQmU6YtnXsMqWFb6NttEtrh\n",
"aMvaAxprfkuuoWGvQlZV9XcJ9sQkFO1Cnq4NjKNsT8zqcgRlO6VrO9lyynYMduplcQUZadhmJSbd\n",
"Wl8SWbFPKYU7qli5BjQUzmcozUNRXgh+au1GJeLCwsoCpVpyDSgsvV1SuaAVlW1DqbYTNKeaBhRt\n",
"Tompo5MFVTFQcz2sXDT2exXbUCmoaUke4Cja/tlMg5rCoG0othI1LfNch7GntqEoIys7aBuqhe3p\n",
"BuzLJrahnga0JIzPgIYxRNUeHZSUSeLKsDtWr6MzR+8yDsiemIQaGhoa9jzaJLRDIZD5QvZC0A+w\n",
"5QaCm7rjQoDTgeCmrv0+W07CSsusMsNqr68MrOVMOahTtrMQP/1QPsBwOB+zmO2F7anJ6YfyqQU0\n",
"nTMfleYhb8N/sQStKEHUoiSpl9iFYKltaF6RQcqGyzWjLL23YegN2YaWakAl21ApqKm7UCbPbEOd\n",
"KmQGVB0oNnDpGNtQ6rgKA7ah9HuzGtCyMD4ySW7YCppKcIJdUmy3aD8pRmtCuwy7bM5saGhoOECx\n",
"gQCmInJ7ETnLJwu9TEQeVy4JIvJUEbnclzs1kR8kIi8QkQtF5IMi8tci8sOmrojIi0Xkm/2Wy9j9\n",
"mtBBwD666bSnVeR76XFludCcKUeiES3C6tP4D01kMLhpVidzE0qYcmnfhthyZoE4hi3XC+UTNB7N\n",
"w/lE7Yc1elv+hhVXCudTCuUDaSieHKVAp6UU4E7eGSlWSQG+lnyL7k+wI+XXzWxDamVLNCCVKIvX\n",
"k7JtKEDXaRsaTHgHRdtQ9BmKRUfYhnoysrJF29DYIKeZtlNJ/U1uFy2F8dlfLLkxCBp9LXjvll57\n",
"Y35TpwCqqsf7VA6XisgDfSTtCBE5BjgVOBqXI+gCEXmOqp4GfCfwq8CxqnqLiDwWeKeIfI+qfs43\n",
"8UfA54BvGT2ujYyqoaGhoWE/YbEY9zHwWQtOBt4CoKpXA1cATytc5WTgPFW9QR3r6q10EWu+Arxc\n",
"VW/x7bwf+AZwfFL/5cAfrjKs3a8JQdnjO5yryDVJFdBb1w34D6VMOVd3BFtuILgpsCTAaSWQaYEt\n",
"NyrxXVJAWFua+M5GUlho3+YTMDGMp1IkhZotqB/WZ9FL89ALchq1qDQBnvf1ie3m8qzn/lRNWyrZ\n",
"hmoREWosudRPaLGCbWgw4Z27QCbPXBRs6oYh21Alqd2gbahnd/UlrS9etGsqPRuQtQ0NRlAoawA9\n",
"DWmLNaNVUoJvyfXXbxO6N3A48MlE9gnguELZ44B3JsdXAUeLyO1U9Qbg7Ngfl0L1YJLM2Kp6rYjc\n",
"a5XONU2ooaGhYRdAFjrqU0DIF3RzIruZcqLQI025m3ArkSMKZR8JfMYHml43dr0mJJKwzaCzh4Tz\n",
"UV7wHyr5DsE4/6ERqcBjm6ukAgenORl/o1FsuaG4culxypZLmXJJJ4fYcla2qNiISv5CtUCmpUgK\n",
"9fhyuY2onP4h9NdrRmrkWXs5FkYDItVgjL3I2oY6c0jfFmSZc5ttG7L2m1G2oVI8ubRTdidhXmfF\n",
"ZfHlILcN1VJ/j4mgkCS6gzqzLY18sJna0Y5JbrdxdpxtoEYiLF0oKysitwdejUv/vSHs+kmooaGh\n",
"4UDAZD4i6VIZgXxwCHB98v8vVsqmSUUPwU1KccvNb8O9CXidql6x3k4F7P5JaJKvAm2utEH/oTGR\n",
"tklWozrJmHLQMd9idGDDlmMyiatAsX0a8B+qxZWrsuUqMncdLy+x5YztZxlbznU3l5VSf0OZLWdZ\n",
"Rb04c9EHKDM++DIaa4GJpBA1KGv7EXPc3Z0xKcCdPGnDaC8BQ7Yhq/HUfIpS29BQwjsnj4118t47\n",
"MMI2VIgnBwXbUNfJvva0xDaURZa1NqBKxO2UiVr0ISrJC6il+c47XD498e3vmOR2BdLBSFwD3AA8\n",
"ALjYy44C3l0oe5kvR1LuSp9iJ+C1wKWqeq6IHAwc6dOErwvNJtTQ0NCwCyCLxaiPhbp9xDfjEoTi\n",
"KdrHAmeLyBEi8mEROdQXPxN4gogc5ll1JwFnxD6I/BpuPfY2EbkzcN/Q7nqxZyahdeSa20Ojb4hx\n",
"5VapswX9aGjYKojqqE8Fp+B20i4BzgFO9D5CdwDu7/+iqh8HXgycD1wKfFRVT8dVvj/wGuB5wC04\n",
"yvaVpOZHkecBfwCoiPypiDxl2bj2xHYc4AKFSn1brujEulC39UG+LdfbXohJ7mRUuoe0LovFuHQP\n",
"0IUnkUmFrp2MI9C7wyH0adzRoTUnHWSU7YSuDclOS7QxuzbSQKaWxj2Gsr0s8Z1FngbCOq3a7Tm3\n",
"DbemkoXycfDbcx23PZZ3baTXLCMlOdTSPKyFl984co4hKFCQDyW8gzJBQcy27iiCQuq4mnbK3sZC\n",
"+oelBAXS98aLzDZcNcVDutXW8/LO5d3zvnNC8cjSwEHraHPf+vN7++20ZxTk19Kx54LsHNxEZct+\n",
"iiVLd1V9I/DGVfq2+yehgNSE0HDAoaVyaNjrkB00yW4mdv8kJHQT0IS+VX7AibXu2DpA2S7RtUlX\n",
"Y6FuR8sek+4Bcsq2VMoEWMdUSWRdp0ZQtiMBYXkqcDe+pM0ajTt0PdEtSnRtWELZDs6kPbXWhvgh\n",
"pvdeGuKnEM6nlOYhl3eU7U4ByUkLaQpw18M+UcE6qw4GPTWayRiCQi+o6QYICjZwaTcIEip2maBQ\n",
"TPFg0ztY71Qrl0k/hM8yqrZMdqRWtGlYPzFhR2P3T0INDQ0NBwBksW6K9o7Grp+EXIBQ4orN7otH\n",
"lJxYjQPrGMr2qHQPkDmxjkn34NrsKNtj0j0kpxGd9zWfEZTtTpNango8lCsF2IHE6bLgvDo2uGlG\n",
"2c7o2u4syZVSynYt8Z0N25NTtvt2oayuuXEuyk3uyGo1o067CSe0qvEM2YYWsXbuwDpoG7JBTcfY\n",
"hio2oJ4DdUAh/UPneJq/N6mtqJfeYVkYn8WEghE178sAVXt/h/TZH2jbcQ0NDQ0N24f5+okJOxm7\n",
"fxIS9Y5t/thqRLFcfj5nBuV73ENsuTHpHtz5wNyZj0r3kNVJSRa9VMZ5QNOULWfX/D22XBAn2k1n\n",
"H8rTPQyx5ay5YAxbzgY37eTL2XLDie9cK2MS37m6CVsuMt3UlAnXzeHSe+caUE8zMrYhREalAHdN\n",
"JJqR5hrPKNtQLeHdkG2ot2NgypRsQ6njqr9idlwM2zOsAfXC+MRBdWU6x9Ygzl8goW4Lsk6ru9Fm\n",
"1LbjGhoaGhq2D7tw4hyDXT8JOZtQ4vBSsQVFJPvlfVacbzNWDfSvTh6bM6vMWMcw61SFUekeIGPC\n",
"BR6WTvJ0D0NsuZQpF2R5oX5aCBvcdAxbrovGYmw/A2y5sanAS2w5G9y0xJYbk/jOHScr/UriOxvO\n",
"J2PLBVZcKFMKcpr2Q/uaUM0WlIb3UVPG2oaixq+pbSj/skbZhmoJ78J1S7ahCoMu9tHahubddbL0\n",
"DmlbpTA+SQgfIEt0B4ndp+RTtOQHO9OM4rO98R/5iW93sQUThtgbsEew6yehhoaGhgMCbTtuh0L8\n",
"4iesZmq2oYKGZHa/e2w5MatDnUhiJwrNjGDLLUv3UGDLDQU3hTJbLmPKwSi2XC+4aRzyAFtO8zJj\n",
"2HLrSQU+HNzUlcBfyQY3tYnvSmy5UYnvKNuI+qkipChfU422klKaB+i0nni9QtBTtbagUkI8zTWU\n",
"nm3IUkSRvs9QzX5USGpXTP3tOtU/LtmJYDiCgtVqesFI6+y59diAdrzdaHHbdvdgSzBqEppOp/cH\n",
"fgF4FC5g3bfinoAv4zLvvQ84azabfanWRkNDQ0PDBjBaE5LlRXYQBieh6XR6MPB7dDnG/w24Fggx\n",
"hO6Ei8b6WOCU6XT6itls9tqt624f0U/IrwjF7D1XNaLEt6if0GuALWfjyo1gy6VMOdeH5Wy5Xly5\n",
"MWy5hCmXDmeILWf9gMaw5YbiysH62HJDqcAtSmy5obhyrkzfRmS1lhpbzkZSAHqJ76y/UMaWMwzD\n",
"2EbBFhTqRC2ykgI8BqoMSsC8s3FVbUPRrpQMx8aTs8+bR2Ybire/wqAzERQU6ukdKqkcnNyw4izD\n",
"MrxHCUuus8WO9ymqIWhGMaVDoin1WKP7Ie3DeJtQ/2fdJ6E7A/geX+Blqvq+4nVEngq8EPfVXaSq\n",
"LzbnHwb8CfBbqvo2c+5k4FdwwU0PBl6sqn+1Wm89ptPpBHgXcHfg54D/M5vNvlIpeyTw08DLptPp\n",
"kbPZ7KVDF21oaGhoWBEbmIRwUbRVVY/3qRwuFZEH+kjaESJyDHAqcDRwI3CBiDxHVU/z558IPAWX\n",
"AlwLdc8AjlbVT4nIM4B3UE4jHtGnInV4IfAF4NiZQ3ECApjNZl+czWanAw8EfnA6nT5q6KJbgaHc\n",
"VfVKq1fRyToqNWw5ajHohus0NOweqM5HfSx8XqCTgbe4dvRq4ArgaYXLnAycp6o3qKPxvpVuJwzg\n",
"ClU9EafpWDwQuMlH2waXCuIIERmchIa2494J/N5sNhutX85ms69Op9MfAe46przPPfGTuOngbsCb\n",
"VPX3/bkHA6fjdPAbgJNU9YZ+G26LTOckdG2gZ5T1SA2x3mG1Hsg0lyuCzNURFGLZnI5aomyndG13\n",
"bgRl2wY3XYGyXQpumsq7gl2oH2uuH6RshxQRQ8FNk0ZS+aIQ0scdF6jZFUfWEmXbBTDtrOY9arZl\n",
"XhCCnnbCUnqH7FgGsq+G45ozK5BsFvXOQR4MtfMHzbfqettzabm4HbsKQcEQBMYQFEyZ+HUMpXgo\n",
"pHeAwk5bKjDkhV4240DSWYGqnZIPdhzxYBkWt44seEcruDdwOPDJRPYJ4LhC5eNwv/0BVwFHi8jt\n",
"VPWbSzKoXgysicjxqnoJ8BPAR6y2ZVGdhGaz2aeHKlpMp9PvmM1mn5/NZl8Fvjqy2snAY1X1ehG5\n",
"L3CViFwK/D3uRjxdVS8SkVNwat602lLmxNNwoKGlcmhYFSKT3TURrd9PKOQLujmR3YxL3V0qm5a7\n",
"CffregSOE1DvnurnReTJwLki8hXgm8CPL+vcZlK0zwMeumKdp6nq9QCq+mkRuRG4F/DvgH2qepEv\n",
"dybwGRE5XFW/nLWQTj6SLKRWoWwb5uogZTuha7t2g3ZjSAwlynYhuKnrY4GyPRDcNOt/Stk22tEY\n",
"yrb92baU7aj1JInwghbTY+YaavaCTmMKMqsBBVjKtiMb5GUXVltKKNtz77Da0anz5bqlbM9F+9Ts\n",
"TmX0fehrRr1QPlZunVaT9A/xbkun8WRtJXVrQU8tQUHnQaPQ+P8eSUeT55ecoJA5rroLZu2XCApi\n",
"taaoJQW5mmMpp3eAaooHnUg/0V1XycgzRk9WsqglmfO7ZSIqbbWt2oQ5rtkWSqu5pXYIEXkA8Hbg\n",
"Map6pYj8IvCXIvJgVa2qcaMnIU9UeCrwOODbTV0B7je2rQBVvTI2IPIzuJSx5wMvxamBody/isjX\n",
"cJNckdHRcGBjzUW12+5uNDRsHdY/CYXtsEOA65P/f7FS9pDk+BDcpDTG/eYZwGXhd11VzxSR1+Lm\n",
"jPNqlVbRhF4NvAT4R5x/0Ka88Z5R8Se4jcypqn5FRI6kb/i6iQLLwjmq9sOCjKFspz576bkhyvbS\n",
"VOAlynbg9xeCm0KZsr1KKvBw/aHgpulwUsp2Z68akQrcN2KDm87Z54eXU7PTcD41unZnI5oYef/x\n",
"spNMtBERNKG+02rfIbV87GRhjMHRtFekmvhuyGm1t46vBDBNtZ4xKcAhtw1VtaSQBsLajFKP7VVs\n",
"QxVKdtU25O2oYOxEUE/xkDqrGvp2L4xPStVeYgsqaT3LtKWdAl18c71Vr8HZ1R+As9uA24p7d6Hs\n",
"Zb4cSbkrfXrwZfgWwIb63oejalexyiR0IvD9s9ns8tLJ6XT6sRXaivCz5tGeiPBeTwFU1qkSNjQ0\n",
"NOxFdDE+VqynuhCRNwPPBC72FO1jgZ8TkSNwNOonquqNONPH+0TkMNzC/yScPd5C6P8evx84W0S+\n",
"XVW/ICJPwE1Alwz1b5VJ6ObaBOTx+BXa6kFVPyYi5wG/CnwWeIgpcgidWplDCjaZuCAaYMvZaa7C\n",
"ihsOeiqmbIEtt0IqcCfXrr15zoorpQIHv8IcmQrcBjLt/T/to2HLqe7rQvpEwlmuPZXYcjVW3KJi\n",
"I0o1o174Ho80nM9cLCvOOq3mN2UtyejXt/0YbYaOAWeDm9aDn3aaUTGUD0TbUCmcjw3lYwOallI9\n",
"9NI82N2A8LUnCfKi42otVXfBNmQ1nv4OQrdz4ORSD9szGMankgK8kuwute9Uk9oNJcIjf/F3nGa0\n",
"MZvQKcAZInIJ7nf/RFW9TkTuAdwfuANwo6p+XERejDOLLHDOqqeHRvzO1StxGtJzReQEVf1PAKr6\n",
"HhF5FfAeEbkFuD3wM6r6haGOrTIJfWo6nR42m816NGmPnwDeNLYxP9M+SlX/LBF/DTgMpxKemJS9\n",
"B2677iMr9LehoaFhz2AjxAS/nfaMgvxaOvZckJ0DnFNp50rgZweu83rg9av0bShiwj2N6A3A/5pO\n",
"p28DPk5Owxbg+awwCeF8iV4uIu9V1a+LyN1wPkO/A7wXeIOIPEJVP4RTI8/tMeOg8xOqLHSG2HLF\n",
"UD7+XFIl8cnpUhuPSQXu2pAeUy7WMXvesT8i3T63sQXV2HIw7zQeazeybLlSIFMrM75AXcaCg3rG\n",
"JcuWU+tHRMlelNcNCBpTajuqJb5LNaNS0nEnzzWjLCFeZNflmmqW+A4ohfMZTHyX1FlAz17UJcIL\n",
"dUwbhQCmC3NcSvWwLAV4p/Wk44p0QbILDNqGrBYTzhk2aWIbKqZ3SOqUwvjEnQKbAryW7I7xNqEx\n",
"/kI7zVakPXPL3sCQJvSZivwJFfmqThqfB/4CFxbiVtx221uS8BBPAk4TkeisumL7DQ0NDXsGizrL\n",
"eVdjaBK6FngF48kAr1rlwl49fIX/lM5/DDhhWTtudZew1oy/Q9RgS/vjNd8hoxEVfSaiZlJe2XVa\n",
"lCbe4cGO4/sSxhq1HY11ZR4YdcGXyLcXIieI0ZB0rZfSXMSq74WkdoVEd659c5z6C8WFfO5DlCa+\n",
"S26Fd7LP7UghIZ5ly6Wa0UYS3/W1pX4khVGJ7yjZjPq2ocFICpUgp50vUDjhtZ3EHmS1JxsxISo1\n",
"E03cZYxtyNpJE9uQZa1tiW2oEDHBpgLv2KRh4PYg6csQS86wRsew5Mb4EuUX2h5sgp/QjsTQJPSx\n",
"2Wz2trENTafTR25CfxoaGhoaCliskx230zEUtucnV2loNpv1jF77AyFSQM8WVIlRFdlyWaIt/7fi\n",
"mJ0tsm0UhVC2xpZLfYvMvngcg7ErCZN+FIVQONqXfN1k5detTHNbUDWSAvRYcUXmHGSRFEJfBhPf\n",
"pZ0sJLUrJb5zx125MYnvwvGyxHelSAqlKAquTL5qT21DS+PLZVf18kIUBUjtR6GOsRnRtwXZ40wz\n",
"Ct+/SfOQpwA3tqFoC1rBNmS1JTFlrG1IE20svj++6qBtqIue4MqSnEsH1B3XbD/F1A4rRleYJBE5\n",
"tgOqB55NqAcfNeGngUfjYgldD1wAvGOVQKcNDQ0NDathvX5COx2rhO35VlzoBWun+WXgr6bT6X8Y\n",
"Svew1RDRyKYZX4nV6RTrqeO1tYatw3rC9qwn6OkaPe+VhgMI2xlrbn4AEhMsXosLLvoi4G9woXsO\n",
"x01KL/Lnf2mT+7cUaRDGQNeGZFep4pvWZWRlPEEh3X4wznZqKNsBOhGfkRWGsq+6Poatt3k/lE9o\n",
"zxISUgp37J/ZsquF85nP+9TsKiGh22Ibk30VyDKwrpp9dcG8GsqnRNkOW3Il+rY7zp1WS+F8LEGh\n",
"R1wQZa751ltsx9jd03A+NpRPZ/PPn5U01UMtgGktbM+EPmlB7bEhKLDQuFW4EkHByCwlu0xQMISE\n",
"ynEWxqfowJoIisdLfDU2YQLZronogLMJFfCTwAmz2exqI79kOp2+CxeaYb9PQgEioKtqKA17BhMm\n",
"xXhzDQ017KYI2tAmIYAvFCYgAGaz2dXT6fTzm9SnlSCiiHdWDXRtoBcYsRzOxxhhAyorunLYHt+P\n",
"njwhKIxIfJcdqxRD+WTXKYTzWSXxHTgadkd4KBMSSuF8SqF83HFOw04dXmuJ78aE86lpQGk4nzkL\n",
"Jkx6BIXhcD5BK/Iaj0l8ZwkK84LMEhJKTqxDaR6g+1rSVA+9NA9WMypoSjaA6TKCAkiBkDCCoJA5\n",
"rvp2XKeyPhuOPtlga2F70uOaBjRE1U6DmaZ1K0SF0kTUvXs7a4Ja7FGKdt/hoo4vTafTQ0onptPp\n",
"oZiER9Pp9OSNdKyhoaGhoYMyH/UpQURuLyJnicglInKZiDyudh0ReaqIXO7LnVo4/zAR+bSI/ELh\n",
"3BdE5MLk81vLxrWKJvRyYDadTn8L50N0y3Q6vSvwYFzInpeY8r+Cz2m+1ZDErlOzBRWPNZeNsQ11\n",
"TqhBlu+LF8P5mFA+XXvGNpSG8ymF8knq2HA+LObJOS8LFNeBcD7VUD4mmV0qL4bycQN0cs21m4VQ\n",
"DOUDfdsQBc1oUnmxUs2obguy9qMkbE8vyGkoU3NihS4ETyJLyxRsQ2sDaR6gvxKcqwxqPKVjkjrL\n",
"bENJ7J/uxq9iGyo5b5PsCmhfLt5AqL13LL9empJFjAY0iqo9oPHY42U07jHpHzbDxjQWtzGOmFAO\n",
"CcwpgKrq8T6K9qUi8kCbetsHKD0VOBq4ERfR5jlJJJsnAk/BZV8tGUDeq6orueusMgl92P99LMB0\n",
"2su0/VNG1iw0DfsNLaldw17HWJuQnYTEzZgnAz8FoKpXi8gVwNOA15niJwPnqeoNvu5bcUlGT/Pn\n",
"r1DVd4nIhesaRAGrTEJfxOWVGMuD/k+rd2d1xLA9cbUXzhhbUCmcTyXU/aBGNMnL9H73okkicVA1\n",
"oXxs3WI4H5Owq3PuK4fzYbLWJwapsQ1ZNZHkQpWwPRGJzajPnMudVjs2Xlc3akDRJ9FoPAWbUc1e\n",
"VArn49KBFzQjY7RbS9TaXCvqYG1DuYOq0WYSe1EqH2MbqmlGa6rxQV5mG4q9kM6Cscw2lKcBr9mC\n",
"BmxDvR2CivNqQJbe29YdsA1FQ2KQWVWrwJIrpfyG8pbIEntRwJBGFLTx/bEA2gAx4d44JvMnE9kn\n",
"gOMKZY8D3pkcX4XL93Y7Vf2mj7o9hKNE5D3AXfw1XlYKPJ1ilUnoX2ez2avGFp5Opz++QtsNDQ0N\n",
"DQZOmXAAACAASURBVAPYwCQUUjXcnMhuxuUEKpVNy92EW1YcgbH7V3Al8HxV/aqI/A9cRoTvHaow\n",
"ehKazWYPHzo/nU4ljZowm81Ks+ymYyKBGRQE/m/NPSAh9PTiElbaoKA49FhxNpxPsirs+RCVtCXI\n",
"w/mEVV+wDQXGjg3nkxx32lI++C50/yIVO9uQDeVTC9tTSv+QhPJx/ylrRhPWBtM8QPeCZekfRqR5\n",
"gKAFrfn/l21DpYCmg2kekv6nqR5smocxtqHOlygYIXNWXCnVw5gU4GlbIhr/X7MNxculmpIaTWSM\n",
"bagW1LQQrid2M+4uGNvQUIoHm97B2oZC3xOWXDHld9rGCJvQTkvhEHAbt40qd4f6KWsiqe1qlUwp\n",
"o3bAVDUlpJ0CfEVEHq6ql9XqrMKOW4aWcK6hoaFhi7AY+a+AQD5I2c2H4EwspbK2nAJfWrW/qvo1\n",
"XBqe7xoqt2rsuBOAHwW+w9SVZRfaKgRn7G61l9tfrC0oerQmifBG24a65stRFNLrxLQMkmhApm+R\n",
"DedXdpHdNk9YQl7jiavAUGaR1dHUT8hEVbCrwe7epOp9xTZUSPVQiqKQlimleqileajZhuY672k8\n",
"Y2xDVQ2okOphraYBxbo5S24ee981tMw2hPRTgMfHa8A2tCwFeMk2ZBPdWdtQsAXFlCFzSdSVFWxD\n",
"gwnvKNuGerafUNdqVZ0KaX39qhEUooa53M6zES1HZEItncP+SPcwX/923DW4yeABwMVedhTw7kLZ\n",
"y3w5knJX+tQ7gxCRRwNfUdWP+OODgUOBzw3VWyV23AtwTIqv4UL2pN+iAHca21ZDQ0NDw2pYb0QQ\n",
"VV2IyJtxGaov9hTtY4GfE5EjgHcAT1TVG4EzgfeJyGE4e9BJOEKahdDforsn8EPAL/rj5wGfxoV5\n",
"q2IVTeh5uNzif1aKmD2dTq9Yoa1NRbALQafoWJKMmgWfpInw4uIpX32uyzZUkpeiKECmLUHZNlRK\n",
"8+D6FKIjBE2pu+gy21CW6sFEVcjSPEDPNoTOC7J9WdlQN031UEvzMGQb6iwBZZZcmuqhlgBvMWgb\n",
"KkdTWI9tqJQC3JUrMOZG2IasljTGNiSm3VG2odRnCMbZhiyzbT22ofielJ/9aO9J+7YsgoImatoy\n",
"lhzLtaRS5IRJ1Ir3PzagCYGzz5whIpfgfvdPVNXrROQewP1xpqQbVfXjIvJi4HzczbtIVU8PjXg/\n",
"olfiNKTnisgJqhqY0B8AHiEiH8b9DHwF+Aldko1vlUnoq7PZ7NyB8/uFkt3Q0NBwIOI21p9PyG+n\n",
"PaMgv5aOPRdk5wDnVNq5EqeMlM5di/MzWgmrEBM+5dM51LBf2HA1mDxxoyCrjD5eaPUqNondVkHX\n",
"NaDVIVYT2qI6q8BqQFt3ndW/y7V1fP1W89mqOuvC/nnMGgzmLEZ9dhtW0YReBPz+dDr9c5wD01eT\n",
"cwI8l86rdr8hZon0vm06kqCQ+m2W0jwAxXA+qv7YbtHVCArJdlopzYOr4sukqR4W+VbDUoJCaFMm\n",
"5TQPUCQoDKV5gG7yyAgK6lJA1AgKNtVDiaAw96s6G7YnJSgE0kIpzYMrO0lki57zqi0DOUGhRNt2\n",
"x3WCgiMnSLLtJqZMqEM8dqF7lhMU0u25uZevQlBY+G28VQgKueNqN56tJyjYsgmnPY7XEHdGhPER\n",
"79KwlKqdvvhbSGbYTOzGCWYMVpmEDgN+EHhq5fx+WoY1NPSxnlQOLdTPgY3dlsphLrunr6tglUno\n",
"jTgO+R/g2HF20nnVZnVqFQRSglkwEronZuWVEhSG0zz4QlAkJJTSPCQ1MoJClNUICqVUD2YLbwxB\n",
"obt2kOUpHKREULBpHtQEPY0dSLSRWtqHIYJCj7Zt6N0F8sHCkBZqBIU01UOdoJBrPSWCQqe91AkK\n",
"c9FMNoagUArhA7CoEBRKzqqjCAqWgLCEoKApmWEFgsJgwruSvJDKoUpQSK9fC+kzRFQo0bapazNj\n",
"ApmWUCsb3sKtmC726oJplUnoPsB9ZrPZN0onp9PpIzenS+tDS2p3YGONtY2yhxoadjRu3aPP9yqT\n",
"0DW1CchjW/IH2czBcU1kNaO4QEqcVStpH2LbBXm0QVXo250japBLT9NZZhtymlBODy2lAHd1O40o\n",
"tQul/e/sS6FsN8BO5xtpG0rKxOuNsQ1FanyeArxmG4J5lI2xDQWbkN2SG7QNxQX1eNtQP73DcttQ\n",
"6rjqxoPvU1cGctuQpW2PsQ2Vgpp2o+nbhkQ7DWUl29BQwrv0gsEFYZK+D6ZMJd23s6GGOubGWW0n\n",
"pWrb3+kKVXs9qRy2e/tur2pCq/Bczp5Op08fOH/5RjvT0NDQ0FDGuJR2u287aBVN6PuBx02n05dQ\n",
"Zsd912Z2bBUUnVWNbaiU6mFcCnBXwl0ouWht9UdfPioFOOQak3EaLaUAd8edJpQ5rkLdNkQnzxxX\n",
"03HUbEMlZ9URtiGbAnyzbUM2BfgY21BNAxq0DUXFYOO2oSHn1ZotaMg2lDquwjjbkE1413sHgoKc\n",
"nI+7AeF6Pe3JDLCUyqHm0FrYQegYombHIMjDGOZdx5ey5LJBDmtAOwV7VRNaZRJ6Ci4G0F1wobnT\n",
"t0BoYXsaGhoatgy7UcsZg1UmoU/MZrOH1E5uV9ieqDQYDWiZbUjn/YXQltmGCj5D7tyAbSi2Z2xD\n",
"aQpwjG2o4DOU9r9oG7JBTUfYhmKvbQrwAdtQ8BlazTY0z2RDtiFr8xljGyoFNXXt121D9dTfddtQ\n",
"Kaipu37WRGYbGkx4B0XbUC8V+LLwPRN6Ce/CcZf23f3t0oEkRthOFTLHHslXYNlwNdtQmuKhmN4h\n",
"9oFKINN5QdYNKGo5oxwEw+i6d9AGKt2fab5vbZoQL1tyvoXtaWhoaNgizGVvakKjiQmz2ey9S87/\n",
"7ca7szqi5iPuI5LafbpjEc0COJZl5njiF0ei3YpqYj4B4VggWyQnMp34lZ6pqxNxq77wSWQqEm1X\n",
"pbLd8SQpM3GaUfhE+Zr7yCRqSUEWBztZc584eF9X1px9KLX/hOPaJykn/hNFPu5Ad+8Pcp9EPmEt\n",
"+4Rz4bi7xSXZJNd64r9J1HTS/6fH4UpW7v7ve6HSpQFP5PFY/MePJv10dcyxqPsk5yaimf1n6Dj8\n",
"PzzH9tjK3X0f+Q4k57v/5+9a9p5A8p5o8oJi3o/umc/qSPI+mAvFd8I+o+Ydcv0Xxyy1A8FpMeln\n",
"mXy7sRFigojcXkTOEpFLROQyEXlc7Toi8lQRudyXO7Vw/mEi8mkR+QUjv5eIvFVELvDXOcdH6R5E\n",
"9Q5Pp9OXTqfTRy1roFDvl6bT6ZNWrdfQ0NDQUMcG2XGnAKqqx+Ps+38sIt9mC/ko2acCP4Kz/T9U\n",
"RJ6TnH8i8BJcCnB7sZOAG1T1Mf46c+BNy8Y1tB13LnDRdDp9NfDm2Ww2GMJ1Op3eFfg14EnAYCrw\n",
"zUbKjlvFNlTzExqyDfVkI2xDPX+HEbahzGcIRtmGhuLJpePKbENxPJYNV7cN9VJ9j7AN2XhyY2xD\n",
"qc9QkEHZNlSKJ+fk+8c2ZP2DUttQKZ5cer2SbaiW8C6MZs086Kl2tIptKLLheuw4fzwvnK/4A/UT\n",
"1OXjhMTmY8ZciqBQTe8wFEGhYguKCO+I9l/mZay41E9oWUqHyRZoUOslJogb2MnATwGo6tUicgXw\n",
"NFyOuBQnA+ep6g2+7luBl9LFBb1CVd8lIhcWLvUR4Mrk+E+Aty/rX3USms1m10yn0ynwTuDl0+n0\n",
"QuAfcaF7voF7tO6Iy7J6DPAo4N+Ax89ms68tu7C4X6hfAX7St3Uw8Buq+gF//sHA6bjv+QbgpHBj\n",
"GhoaGg403Lp+dty9gcOBTyayT1DOfHAc7jc/4CrgaBG5nap+06drKEJV/8KIbs+ItOCDxITZbPbX\n",
"0+n0gcB/AX4e+LlK0WuAVwP/czabLU0D6/GdwK8Cx6rqLSLyWOCdInJ/XGy6dwJPV9WLROQUXHa/\n",
"qW3EsuNskN4hv6GerLoq9KUWSlzW2YXOkN+QkY3yGyrFk0vqlPyGBhPeQdFvKPUZAkb5DXW3zWhL\n",
"QzHlelEWxvgN1WLF9f2GhuLJhTJgtaeheHLd4FO/oTEJ71ydzm8oaOE2dtyQ31CNHdd77EwU7fB/\n",
"ey49lqQtGwdOzHGIuhAjdqgmCe58nUXObCu+C0ORtV0n8/No8tLa7Y2g+QR5gR0XMKQZpRG17Tk6\n",
"zWiM39DEPENbgfm656CYL+jmRHYzLjFdqWxa7ibct3UETslYBT+OizU6iKXsuNlsdj3wYu+k+j3A\n",
"fYFvxT0uNwBXzWazf1mxc+Cy7r1cVW8BUNX3i8g3gB8AbgP2qepFvuyZwGdE5HBV/fI6rtXQ0NCw\n",
"q7EJfkK2AbsUqJUbKluEiJwAfDcjwrmNpmj7lN6fJFfp1g2/tXZ2OBaRsCV3Hc4o9smk7L+KyNeA\n",
"hwLvK7WXOliPRcnG07A7MWEt2oXG11k9/UPDgY3tjB+3gfCl1/m/hwDXJ///YqXsIcnxIbhJaem2\n",
"WoCIfBfwGuDJy1J7w2p+QluNRwKfAT6M2/q72Zy/CeixOWx0j1UICqp+IrIEhYHAplXSwgBBIXNc\n",
"DR2FQYJCMagpDBIUxiS8S8fAYk4pqKlr3x+WCAqloKZJmW7LLdliSxxXXZl+wjvICQqloKZQJiiE\n",
"LTlLUKiF70kJCnEbbglBYc68GNTUlakTFLr/azKKcgI87P9HEhQW9AOYLswWnj2eJ6kcViEoqN2S\n",
"7hEUwnFSbm5kNoyP3eLTQjuWwGPCbrn6hsQQ3+WwndhtsVUDli5xQN2uiejW9Ve9Brdr9QDgYi87\n",
"Cnh3oexlvhxJuSt9evCl8JTs/wX8gqp+SUTuAXxRVavd3xEkeBG5Pc6mdJKqKu5tXUkllJWUxYaG\n",
"hobdhfnIj4W6GfPNwDMBROR+wLHA2SJyhIh8WEQO9cXPBJ4gIod5Vt1JOHu8hfX4QkTujGNVvwK4\n",
"3h//Mo68VsW2a0J+G+5NwOtUNYT+uQ6wIYIOoVMrMwRtKE3rMIagYIOapgnv3J8+QWEwqCmMIyhE\n",
"GmrWUkZQ6GQjCQqLJMzJKgSFQlBTGCYoFIOawjBBYSjhHfQ1pUR7sgSFoO1IQlCwQU0XSzQgpz1Z\n",
"IkIo09eA3LFAUVYP5zNHY1BTS1DoiAhk92BOF9S0pgH1ApjSPfwrERTC/63GY46zIKVjUn+nA5zQ\n",
"05bGhPFJQ/i4OiOo2sVQPpSp2olWBMPUbNfUBKWiPe0HzWgDxARwfkJniMgluN/9E1X1Oq+p3B+4\n",
"A3Cjqn5cRF4MnI/7Ni5S1dNDI96P6JU4Dem5IvIDqvosf/q/Aj8EpPRtpTyJRWz7JAS8FrhUVc8V\n",
"kYNx7IzLgBNDAX+j7ojjoTc09NCS2jXsdWzk6fbbac8oyK+lY88F2TnAOZV2rgR+tnLu14FfX7Vv\n",
"o7fjptPp61dtfBlE5Ndwi7y3edXtvjiV8b3AQSLyCF/0mcC5JWacYKJ+SDHSRzxOo3r0y/jQJjZU\n",
"STiWiqxwnSw0T/p/kjomxEgMwZO014XlMW2YkDxMJAvH4+q68D290D+hXFbfHPsBxnA9Ue7D+kwS\n",
"q4cJyRPC9qTyUlgedysO6hxYk7ayEDyyFu1AQBemx8hTracL5VMO3zPplcidV9e0HDCoa8eE6fFh\n",
"fPohf+jVsWF8SuF80v+nxyHkTr+O9soEDIXmWaUsmPeiGAor6VQp3FXXSZaG8Ql1xrzMoXOF8Dy9\n",
"czVHUn9u54btWd923E7HKprQ06fT6VXA2bPZ7KtLSy+B9wd6jT98XnLqFFW9VUSeBJwmItFZdaPX\n",
"bGhoaNituE1leaFdiFUmoVuAhwGnTKfTdwFvms1m694eU9VPMaCJqerHgBOWtTMRt1da8nUL51N5\n",
"ahuK/7cMnQHb0KiEd5Dvh9dsQAO2ocGEd/T3yYGOKbeCbahnA4qBTcnlqW3IJryrOqkutw3ZkD9Z\n",
"+gfvuBpZdsZJNbUNdUpnP80DlG1DQbaKbaizAeHPLWfLxTLRJhPqhjr+OLEdxZ6EOuE4SXwHeWie\n",
"HvGsErana0oTE0zOVhtK8VBO7wDJl+c7krwbFSfVXhifhCWXhvBxZcq2oCzZnU10N8CSG8uGCxCZ\n",
"xHfKpnRIy5Tkm4HdqOWMwSq65mw2mz0LZ8S6AnjzdDr9yHQ6fdZ0Om0J7RoaGhq2EAuVUZ/dhtGT\n",
"0Gw2e4n/e8tsNjtjNps9FEe/OwH43HQ6/cMt6uNSFMwrS21D7v8hvP1421DPBrTMNkRybgXbkG0v\n",
"2obCcbDzpLackN5hBdtQ+v9oJ1pmG6qkdBiyDXX35aAuNA+dDahkG+puk7fIDNiGAoLNp1emYBvq\n",
"/j9sG+rKpYkkKmkfiikeyrah7rhsBxpjG0qP7Tn7Kdl7au0N2YZK6R2KtqEhu+hEO1YdlN+Rystr\n",
"34X4/qSyWGcDtiAj327sVZvQKsSEO5njewJPxEU3uAsuEGlDQ0NDwxZgr2pCq9iEPjydTo/DTTzP\n",
"wk0+Exwn/AXAOza/e8sx8c5BNgOw5dRbeRpdYRXbUC+oqV16WH+HdJpfxTZkg5oaD/PYjzG2IbPV\n",
"HW1DiwVdBAZfd4xtyNiJYgqMIduQtQH5dX1nTjBRGLQLShpMC0O2oY5xVwt6mtuGFiywaR/6NqC+\n",
"rajvH4Q57vsLLYuqUErxMM+/7iTgQNl/CNHB1N/uWHrHteCmQxEUiukd0s6WIihMTJllERSyUCim\n",
"3ThmY4tyHfR/g2wgoGnQ+OfDtqAx2B9pvhsxwYUD/xfg7rgo17+HIydcvRUda2hoaGjosBu1nDFY\n",
"ZRK6K/AxXFa9c2ez2QZCGW0eYrQEI1+a1A7iAqrbz5ZUnDCGvHxe0IBsqHvjAS6LZJ0m+blqWHv6\n",
"LLjYh1AlxowLK9RJkqwuZwT1tZ15lEctJkRMiEnybHC8ZOC9/fFKxIRYd63HlItr2J4GlKR4sFpS\n",
"0ICiVttpPYslGtAk0YDc8SQpk2s89jhGMpBFQQPqmHPuWEwd7WtAcVxBm8nV9jnl9A5uzH0NKMhr\n",
"ERKGIihYDWhUBAXzvoyKoDAUSYRyBIVSojs35DpLTsy9rMWS84PyYzcsOSNPj8VEWZhonu5jKzGv\n",
"Ry3b1VhlEvr72Wz2qK3qSENDQ0NDHatmCdgtWGUS+v4t68U2YT3pHxp2Jlw68N3IDWpoGIe9uh23\n",
"CkX7G1vZkY1gkuyxpWzNLKKHZ3uWqNw9JmiNzSnm/1IoW6KaTszHQ5Mth7TzWmivT0vFb1l08j6N\n",
"29Gua/RuHQrbE47NALUQtqcaxiehcpdkbnjlMD5SoGJb2LoTWevLTN1Aw14ryPplJiZUT/7/Uhif\n",
"Pv26T8UeQ9EeOrdGn1JtQ/iMDeNj6dVjqNrx/2Op2kAWwgeS96FC1U4xlqrtBjCKsi2lczsc+xaT\n",
"UZ8SROT2InKWiFwiIpeJyONq1xGRp4rI5b7cqau0IyI/JiIX+/MfFJH7LhvXTghguiloWk1DQ8NK\n",
"2GVZLTeoCZ0CqKoe71M5XCoiD1TVLDOBj5J9KnA0cCNwgYg8R1VPW9aOn3BmwENU9RoReSHwDhH5\n",
"9z5FTxHb74G1QaRfSykeYjXWYWmRVFnJlVaOfQ3IrfR68ppGJaaMdXgtaDpxkIVVXOaEGoOeWrXP\n",
"aTfBeTWVdXXX3Kfn8Ork5ciuoQ/eobXizFqUmeMs6KnVkipBUEtaUl8Dyl1Mnawc3LSkAYXjugZU\n",
"ludljAYkCUmBXOsJ56Lm4z9ropGkYOv0HVit1r/cwVXsZ+I/9nhS15aKjqm1c9XO1mSFuulLHmWT\n",
"8qdXd9LT+pc5rW6X46qqjPpY+LxAJwNvce3o1bioN08rXOZk4DxVvcFPHG8Fnj2ynROBv1HVa/zx\n",
"mbjJ7Pihce36SaihAcoTUUPDXsIGnFXvDRwOfDKRfQI4rlD2OFPuKuBoEbndiHb+Hc59BwBVvQX4\n",
"Oi6BXhV7YjvOLYDczV94ra+SETjzb1tK4zZ07Ex7NyzVHgpaVyn1d9ZUms641/H8sJj+eyD1tyub\n",
"G+5VuuR5xdTfdG2mDn1dUjtDzY43su94GodhZDGETxK41LV90GDqbwCb5nuS0LrT1N/u9uRjd2m9\n",
"+w6stoy7Xpf+ey6Bvp0/JNUkdypdn6pOqlI8n5fB97F8nNKtx1K1F+b/QKRoV51Y05Tgq1C147MR\n",
"OmwGEC6f0rInBRnDVO0smGnWrqmjKW98dcREd/HdMsFbtwDzir1nBEK+oJsT2c24xHSlsmm5m3Bf\n",
"8BEj2rkWOCacEJG74pLlHcoAmibU0NDQsAuwCWF77BRZK1yaSmXgfDj3duBhIvJ9/viFwD5gkNS2\n",
"6zWhiQjzxOZVCs8D5RQPQ6m/nVwyeWpa64cz8WWGUjwsWQWmq8Wh1N+5vAvRI/PK6m8gxcNg6m8o\n",
"hvEZSv0NiaZUTOVQC+3TT/FQSu8AnfaUhvHpbmnu2DoUxsdqQGPC+PTTPQyH8XGyWtieERqQfZ7N\n",
"+VRj6WtL+YNtUzmkYXtqDq2lMD5pCB930mgkvTqpw3bl/Qgv14ADdy9Mj3SsUCB//tOU31BUetK0\n",
"Dq45/4wvcVodQniGbBrwzUDJ3jMSgXxwCHB98v8vVsoekhwfgrvhX8JpNdV2VPWfReQJwG+LyO2B\n",
"dwGfAj471LmmCTU0NDTsAmxAE7oGlxj0AYnsKOCyQtnLCuWu9OnBl7ajqh9U1cep6g8Bv4+zE31o\n",
"aFx7YhISCuy3ijxN8RDLmuOafGnq7wI5x9qGStfpsX4KHc9SfydtZH5CpfQOyaBLKR4semVKKR6q\n",
"TKM6461aJohLKR4qqb/jceClDfgUheNSiofu/53vUCnFQ3ZcTO8g1TpQS+9QZ8m5Mkv8hQosuTge\n",
"k8LBytPj2rl+Wu+OEWfZcX0fIO/7kz7X1h+o9Mzb5z76Eq3Akhub0mGdDLfIkKNLcLe/sF52nKou\n",
"gDcDz3RjkPvhyAJni8gRIvJhEQl2mzOBJ4jIYZ4NdxJwxrJ2/PF3i8hpdHgxcLaqfmloXLt+O66h\n",
"oaHhQMBivqFJ7xTgDBG5BPe7f6L37bkHLlHpHYAbVfXjIvJi4Hzc5uhFqnr6snb8ua8Cx4jIZcBt\n",
"wCW4DAuD2PWTUGDGzSMrLmfJ5eXyFA/L7EelFA82vYM10RVTPFRYcRGlVA5W1mMR5fYeJskevEnv\n",
"IIZpl6Z4iP5FaXqH5DgON7MNGdtPNFQMpHiopPOOPbMpHpJNfMuoK6V46NmAfJ1u6DaQ6bxnA7K2\n",
"oXms05Vblu6hF8iUVVI5dA+VZcWNYclFTcakAA+2oQV9tlwvcKk5LtmItGYvKj374DSgNJhpOqAB\n",
"lpyYMqNZcnTPdefTUzQK+b/hHcovZG1BWtCetjKdt8UGbEL47bRnFOTX0rHeguwc4JxV2vHnvgQ8\n",
"YtW+7fpJqKGhoeFAQKCf7zXsmUnIfj3L/IQmVNI7QEK+yVlymQyzcrSpHJIUD6X0DsllKkwgO6BQ\n",
"xzCBkhQPvfQOYTiWUZekeOg0nbpfkGszsOgWHXPOsOKqS+FCKoeuYZsGomPJxf6n6R3ItaRO3td0\n",
"UgT5PClX8h1Ky6zFOt29GK8BdfLlrDhfJ2odpVQOxHNd64YlZzSgPvMNf9ydLyW6S49LLLlSojug\n",
"l+yOrA7+/2TnBllytd/cUjI7PEO0oBWlg49anMq63ISsf1Bk25Lf+63ARjShnYw9Mwk1NDQ07GU0\n",
"TWgPQiT3/dm6C7GlntQNrCuVQ0kbajiAsMsCmAZNc69h109CEwREbZJFgi4/KoxPWXMvhvHp7Tyt\n",
"I4xP0nnfqfzYba2ZziwJ41NuL3ecGwrj09vCGwrj09uqWx7GpxTCB/qkgzSMTxrCx4mWh/GZG/LC\n",
"sjA+KVW7FsZnrURM8MbrMWF8shA+uGyreR2/9eWPsjrxupSPEyKB3YYbE8ZnWdlSxtVitlUobLEl\n",
"dWPnwnab2YbrOW5Lsm+YyFzHTd3C9twIZ1U/CGTiHbSXOK0Wq5tzdptuM9G243Y4JiI9RlxDQ0PD\n",
"nkHbjtuZSCcfG8LHydzfIg17GTU7fucdQaEUwseVrdBVF0rP+BpgaaolWneJvp10NiMd2PA8I8L4\n",
"pCF8nMxQtWPdJIxPEsInyiBSs0uaUjGEDxSICclNqNG6e9rUQVkIH0gX2MvD+CwqGlApjE+dmJCP\n",
"OQ3js4yY0K+jyzWg2PcOVouphfFJqdqLmsYzQNW2hJ1RVO1ecFMzoIJGZCnZy+ROe1riO5GRJlbb\n",
"io1pHbYJTRNqaNjBENZ6rLmGhj2F3WO+Wgl7YhKaOOIw0GkKdvu4JC85sEKVAZoFYqxRs3u2Iior\n",
"w+Q6Par2hH4gRzOgkrbT03SCnSfQrqNdp79PHjUgQ9WODqe+QyqTvnOqHWBvtdh3Vo2w8mhfSu1E\n",
"QQPKqdqdSSXVdsrUbGsDmiflJhUNaG7aWmPSsxNZjSh8cfOkTjXNgzHsRbl0z+RYqjZJKodSmgco\n",
"U7WraR6GnFWNe8IYqnYazNTV6frt/5PdCyZaeAfMwGIAU39Y8ESvpXbIsMRptVjF2H6s0+pWJL5r\n",
"xISGhoaGhu1DswntTASbUI1ENhTGx7Lf4gJ/wFZUjdIxsBXdObCGi5M3XFAc6uF6jDysLCeShfDJ\n",
"2qcr4+omLLlJYM75MkvC+ERtKgwudBiGw/jYRHexU3V7j3VOjcFMo12u025sorteuB6TAG+tYBOq\n",
"Oat2Lc0pJbqDjiUXkDqxFhPd0bHkujpdj5eH+MmPU9kqLLmaxjPEkutpPMtYclB45issufSRqr/U\n",
"ZblI1R5q2XKqiWPrkl3c/9fe2UffVpR1/POci1x8SRBQ0iTNd14MRdBMKjWxUFNX2dYEefFaC7Fc\n",
"pWKWLbmWLy1F01qJJCjhUlk7KynJkhdFMNCL4GqBaIqWrwFy4ZJv5P2d6Y+Zvffs2TN77/P7nf3b\n",
"55z7fNY665w9e2b2zHmb/cx853l89z2h655NdWK6otNx462yKYqiKP0x0u8RQUT2EZHzROQqT+OU\n",
"0QAAIABJREFUEdkhIsemLiMix4vINS7fmbPUIyL7isi5IvJpEblORD4mEqqR6ow6CInI40TkKyJy\n",
"UpD+GNfJK0Xkn0Rk/7Z6Jp7fnYkIExe2Wmp57MNP7xvCoUo3jTwNl/fuuCakCV3dl40i/glIJE+j\n",
"Q5GyifAPZYiHIL3uHn8CkdAOYbqZbImEhNhSqufq/ajeBCNbSkupnice/iGaFhZ1QRGARiiIsvlB\n",
"aAe/TCpPGNLBT69eR8I8eI8qXWqvY8ex9Op1R0gH9/A/uTAsQ7OMKZVynSEcguNZ8pYffy2Eg6E1\n",
"pENZuHppJm4GIMibSneNoTW0Q+ySE7GB7nqo4IqQDuFxmD5Xpj0fcbYDxhjzROCFwAUicr8wk4gc\n",
"DpwJPB14PHCkiJw2Qz0fBC42xjwJOBL4Po0FvzqjDUIi8mzgdGyMcuOl7w1cCLzGGHMMcC0unoWi\n",
"KMoey+6ejwAXF2gbcC6AMebLwHXACZGrbAMuMsbsNMYY4L3AqX3qEZHHAQ83xlzgzhtjzPNMoShK\n",
"MKYldJ0x5gXAnUH6ccBuY8zl7vgc4NdF5IBYJdVUspROBGvnEzdCqUB3sWB3fh1JKyl18ySRPIFV\n",
"E60zdYfYFuwuTGuUbVo7sUB3ZjJpWk+R+fBGfW3B7vzXLjhdV7C7aKC7RLC78jVVoLuuYHexQHdd\n",
"we5SFlCIb93ErKNQGdcoI/VAd82gdpFAeJFAd7Fgd9Vx27n0cbcFFAS9i5YJvvPFbIH/vQ+tprIx\n",
"RH8bpXXfZvnEgtslvrelVdMnz2Zgej6aPAQ4APiil/YF4KhI3qOCfDcCh4nI1h71/DJwk4i81U3H\n",
"/bOIHNbVrdEGIRfHIsbR2I4X+b4J/ABr2imKouyZTKXfo0kRL2iXl7YLaEzHubx+vjuwt7gH9qjn\n",
"wdiB6AY3Hfd+4BMicq+2bi2iMOEgmtbRHcTfMCBcEyoeEl0rqkUPbrGSohGCI1ZN847OPvw7v6SV\n",
"FN7RzTDvXabjpwUWkCsTpscspZT1VFhGVZ0xS6fewXL9Z+IeUH9dK1u3gEoLKWLdNIp61lRoxaTK\n",
"hhZSrGxhAYX4aWGeIux3zFKqXgdrPy7sd3NNiHSZxvnudaM2q2YWCwhotXhiFlBYpvmdb7F2GhUR\n",
"X12ITCXU1j0j66LrpcvySc3IzIWp6fdIE55MNTRWiXScB9gK/K8x5jwAY0wO3AX8WlujFlGinTIq\n",
"B/pkFUVRFp+Gm6L+FOG39wO+672+OZF3P+94P+z/8a3YEOCxeor6b/fSC74N/FRb4xbREgrfBKh3\n",
"NMpkDnc5fajd1Q16oc25TF/l0CoSs5y6iKnmusvM/h6vp8wi/pgXmei66SKzfnXcTcBO4FFe2qHA\n",
"jkjeHZF817uw3l31fB47bedzIHYgSrKI39sdwCOLAxE5GLgH8LlYZgmm3BrnSVjwtTrias5Khh0K\n",
"FuJTDtF0/x12i6+N6bk+Uu1Uh7wptyotPp8Yk2qX+cuy3VLtQpI9k1S7bIObqusjw55Bql2+dmKG\n",
"xjRcRKpdpKfyrEeqnUpvk2qn0meRald98F73kGqn8raJD3rn9bcmNKTZPUQHxXe9sZ2geES+5ynl\n",
"UNXRWrmamCEUIRTf+ch8ekqiXTUxPqW7IdY5CBljpsB7gBfbtsrDgSOAD4jIgSJyhYjcx2U/B3iG\n",
"iOzv1HAn49TJbfW4sh8BdovIc9z5XwR+AvhoW7cWYRAK/1Y/BuzlOgC2w39vjLlt01umLA0d++EU\n",
"ZfnZ2JrQdkBE5CrsXp4XGGNuwU6xPcI9Y4y5AXgV8HHgauBaY8xZPerBGPM94FnAH4vIlcDrgeOM\n",
"MeEaf43R1oTcpqgzsObcy0TkScaY3zHG/J+IPBd4l4isYc2/k9vqmuB7wCm01PbDCIPd+Z9Ra6A7\n",
"aKxMxVz9FMN4w4GpP3+bujGL3MSBExyELn1iQb/Cxgf1VWEeChcpRVlXqeeCJwx0V4oRnNse8fJJ\n",
"2Jgib+DQtHLe4jkwDXykNNz1RBydFjLt0KFpcYFasLvAn1Lorid0aArpQHexkA6hA9OUQ9OqbvFc\n",
"7RR54g5N62Wcux73xvd1aGrT6i532hyaNuLE9XBoGrrySTk0LfBdVzXL2PRSZOOHdki5rEqFdjDN\n",
"MuUFUt6KZ8AP5WD8ND9hQEIXXrPgptNOiaR/g0r1VqR9EDvA9K7HO78DeMIsbRttEDLGXA/8ZuLc\n",
"54Gfn6U+DWqnKMpKs4EBdJFZRHXcuokNRMU60ZoX+M5iko5Kw2B3/mefDmoXpBcGGZ4b++KmaRpk\n",
"ilk7wZ1hI6hdLD0MdBeYeKFzR38dyQ90Z48JyjpLaTqt1oaKsmFIh9Aj5GRL3ZkpVCEiEhaQiLdi\n",
"EQSsC+MGiWypOTO1Lahv0m4EtfNCgZfNbFhEdcsllpZyaFp8UPVAeImQDkaCMrGgdkGZxvmmH84+\n",
"Dk1jIb9TeYvjWMhvoBFqwE9vWEkpC993flrkKX5UXaEd8L6/xXuamp6SCfUAd9VzMtx3WD5W7YAb\n",
"V8Pf86qwUoOQsueiQe2UlWdFZ3qWfhCaIHaTcGjplDdP7u4mUja1VBNaQDWxTZlWt2aqOeLmvHgy\n",
"qF2YXtxET31neolGRtIbbuwTi15lsDs/MQx017B2vGB3jfDdxXpSESJ8S9kPm7DW8iYEa0CtYb53\n",
"p/NQH4hKdz9l15trQmEo8LVIHmiG/fbTusN8x4LaERy3rAn1sICK49DyKWgL+x0L+Q2VRRQN+52w\n",
"lgrioR0KC8jVFwZoi4V26LMOWkuHcHEmtIx6BblLkXDbY+trqufmjlpCiqIoyljodNyC01dEVi2D\n",
"VOtHs6jkkiHBUyq5qVd+Bqum0aFAhCPVRH+z8bOo5KZ1ayJUyVXplfUUWj5VmwOVXFnW04jNoJIr\n",
"335Tt25iKrmaQg56qeSmQRuGUsk1LaBulVzDAuqhkgtDfvdZI4qF/C7O2eOmhZS0gIpZgIh6rvH7\n",
"KM6VjQ3WivqoS9vWUqeRH2+jvNTbW/SnJrcL2CxnpTHWwn+31WBlBiFFUZSVRi2hxWQiAqa6Y0yH\n",
"866r5GwaLq1+nFLJTanWBnur5IjsjdiASi6VbibeDeMMKrnQdUkflVwY8ruXSq7wpDCjSi6WFlPJ\n",
"hZZUH5VcaB2VTZ2jSs4P+91XJeeH/Z5FJde0juIWS62vCRVcSiUXo2u/kExMFQo8tZeovpRrrZDw\n",
"BzmpW0+x/UJJ/2rhfiEjnWG9SwoL3/MYHCrmyrWh4HmeiAoTFEVRlNFQS2ixkcAEGUIlZ69Drd6h\n",
"VHLFDfQ8VXLVepJrmwuDXqOPSi659tOikmt1KUFcJZfMk1bJhVZSm0qusHTCPUQFMZXcNFgH61LJ\n",
"+f7j+qrkfHUcyTz1Fm8JXtt2x4/7WDcpYh4TUvuFasfB76SqMGxcoJLz84SWyzz2C81IygIKGSSc\n",
"g9E1IUVRFGUs1vrOHy4Xi+DAdK6kOhQ62y0C38XyxBzzdtVX5p1EjsO861HJBcdhQLt4FL7Edfyy\n",
"MY/Z0AzlXaZLZ55YnSbmITtCI+R3mIZn3XhlhGZatGlByO/aOXcces4OA97FCD1pp0J+t5eRZN4y\n",
"PUje0nKu+RUJrSv/dbiGWj8OvWT3zRs7toXCvT6xPM0kd8HECeI/1khZI1L9WBPPUlynZX/QZiJT\n",
"0+sRLSuyj4icJyJXicgOETk2eR2R40XkGpfvzL71iMhPisjZInKJC+/9KRE5oqtfSz8IxczeWb67\n",
"mzEQ2cSO47b0vgNR3/rCOmcciHrl2eSBCNCBKHKuayCq1zPcQBRl3gNROWj0G4hs3iUaiMy03yPO\n",
"dsAYY54IvBC4QEQa0aqdY+kzgacDjweOFJHTetZzOLCPMeZpLrz3ZSQcofos/SBUMFhIXUVRlIIx\n",
"9wmtM5SDiwu0DTgXwBjzZeA64ITIVbYBFxljdhpjDPBe4NSe9VwL/JFX1w7gQV3dWok1ocJx6cRb\n",
"0N2IVLsgFCFMpFr7HFyq3dh1655bpNoNJ6c9pNoxZ6Z+3phUu+bMFIaTaneIFzYu1a6cmUI/qfaa\n",
"J1LwaZNqr0koZuiWahcy7TbXPrUyNIUJKam2L1RoSrLduZZNqyn5dp/QDslNqzHBQuDOKhnawUTO\n",
"B5tWG2KddSAywZhpfSBKSLOHcduzuztPnIcABwBf9NK+ABwVyXsUcKF3fCNwmIhsBQ5uq8cYs7NI\n",
"FJG7AycCf9PVuJWxhBRFUVYZMzW9HhGKeEG7vLRdQGM6zuX1892Bva89sG89IvJq4OvY+53XdvVr\n",
"6QehSSPsrlRybazlMxFhgtSm7FJS7eiSjMTXiMKQ3/bRTG9cZxK36mvpE/toRCtONTIh1Y6F/fbT\n",
"w5DfYd7qeAKTSRXszktrXDcS9rvZ3tSbsKWyeBLhvKNhv8Ms1MN7F2G/fWIhv4VmXX6+tpDfqbDf\n",
"6RDgzXDexXHzXDyct58e5uk69tdxwpDdbced4b0nppRnp9LCMtHvux8e3KclNLiZdKyVFheYNB/l\n",
"7yIV7rveodapuTDc91zY2JoQNH0YpdYwYiOZtJyv1WOMeYsx5r7A9cBl0vFGLP0gpCiQFiEoysqw\n",
"/kHoFve8n5e2H3BzIm+YzwC3zlgPwJ8CjwWe3NatlRiEUgq5WOf8G/zqpkdqKrnwJqmgj8FQ5vVu\n",
"lMrXoRUzg1XT6JA7rt35JRvTUmf5up9VY9OkIdWOquT8Ot2bYGRLXSWXsHYaCrlonrp1U0i1fZWc\n",
"yJZaXYW1Eyrk6uo3exxaN6FCLqaSC62e8HXcIgrLxK2j2HGV7r2Wukqu8dXxLJmmtWRqKrmY1eO/\n",
"lhbrqZwd8K2W0LpJWjUJpVxKeZq4p29YNxshoSKFyvIp51yGsISm036PJjcBO4FHeWmHYoUDITsi\n",
"+a53Yb1b6xGRZ4rI/YsTxpgpcBdwz7ZurcQgpCiKsuqY6e5ej0Y5Oxi8B3gxgIg8HDgC+ICIHCgi\n",
"V4jIfVz2c4BniMj+bhrtZODdXfW4ske5/Ljzz3cvP9PWr6VXx02ck5jQTU8fldxaI82VbVHJhT4V\n",
"+6jkmmk9VHIJRVCrSi4VCrxFJdcW8tvPu3GVXCTkNzRUcrVd4ZGQ3379vkouFvIb2lVysZDf0K6S\n",
"Cx2XxgLe2TKVSi50Zhq67SnwFW+hM9Ou0A42j9+bfiq50JVPH5Vcl5PTqCou9Z1vfI+LH7D3GwjV\n",
"b0W1sTAmKS/EIROpAtyVUxYm/ux/Uo016Pp3vqp+gL/Wjbnt2Q68W0Suwv7vv8AYc4uIHAw8Arg7\n",
"cLsx5gYReRXwcey7eLkx5qyuety5DwNniMjl2E9PgOd456Ms/SCkKIqyJ2A2MAi56bRTIunfoFK9\n",
"FWkfJLHJNFWPO3cDkM3atpUYhCYyYRqGeu7h0DQ1Q5xKn0j3XqKYQ9Ok+/rE3aBMiIb89i7Tfy2J\n",
"yB4Jz6Gp78w0WkXEoWll6Xghv6HVoWnpzDQMzBW8CcV6kXhpqXDeNYemLSG/bV/jgfGguR+ozaFp\n",
"KuR3uD7kOzDtdm46rV1ttvDe9fPxPP5V4vuEwlAObaEdUnlDh6YFIqYW1gGoQn97ecDfYxSxYfrs\n",
"F0rNHGxgv1DhNcH0COVQXnaycJbQwrISg5CiKMqqsxFLaJFZ+kFI3KJLoZCbBvdPyaUViYf8hmr9\n",
"KBTTrJmm5ePXV+Sptc+zamohv/06ZrBqmq7vq6paQ36n6mzU5xJ6hP0OrafWsN+BF4Wq6+mw37GQ\n",
"3zYhtHr3iob8tk/xsN9WIeeuGKwbheG968Hs4hbQWmA1+WG/UxZQqswaaxELqOs4ssYUfCfD5UTf\n",
"qmlaS84ComkRNUI4pCwgz+ppWECuTDVBUUhWg4Z4eRtf4DLsg5/m6iusozbH036AO++5DPcdKZuy\n",
"gMLjyZa9Wy68PmKig1Vg6QchRVGUPQG1hBaYiXfvPYtKLuZHzqa7srH1n8B66qOSSyvn2lRyLXeG\n",
"fvpaOm0olVws5LfN06KSi/mRg7mp5GJ+5Gp5N6CS8/cKpUJ+t6nkUiG/21RyqZDfbSq5+EpWu0ou\n",
"tID6qORCVdw0sIBia0WmRTlnC3mquOK4VMUFv4WUQrRlG1AzyJ33Y05RWkqRC3bsAZrstU973etA\n",
"ByFFURRlNHQQWnCsQm41PyRlcfAVcsoeyGSS8kowODoILQGzSrULEUNq71t0Hb9tqs7Va4+rxdRq\n",
"0TKQSIeChYi1P7RUu2zbLFLtxnRbt1S7mDAtpdrlPGb8TTDiOafpI9U2u1vzxKTapmVTql9mPVJt\n",
"X3SQFiakhQp9hAh+y+qy7lir41LtcBquj1Q7PEeiDn/qre82hWi+rum3jWxanQWZWFXRZOJtnJ3U\n",
"nstQDnvfY+PXC5gaFSYoysJiB5XV/JEqA+FLZJcAtYQWlAkTpjKtLJ3i9qi8OSrS3bFfNqFkDkUM\n",
"/hr9WmnpFNevp8cspaSLklDuLVX+Mu8MUu3Kqik74qo39fQZpNrGJVSW0qQm07ZpCal2Wedai5VU\n",
"bE7dfKl2YekMIdWuixn6WUCV7TdppPWRane57anaXqWH1ksfqXYfIQJQSbgnppRmF1JtMXXJdsMC\n",
"msSs/oQqyGEmxAPc+Z32Nq3Kmpmb+54G++7bfn4drOogtLAOTEVkHxE5T0SuEpEdInLs2G1SFEUZ\n",
"iymm1yPGLP+nInK8iFzj8p05Sz0i8hh37koR+ScR2b+rX4tsCW0HjDHmic5b69UickjKGV61WTWe\n",
"Hkq1wbd0mvJtm26fa1ZNYrNqKMOuLuPLU+t5qrKRefBwg154uxCGPm6RajfSPal2Jd+uS7NDtya+\n",
"rLt8XVQ4LdZ7Aql2WXZLe8hviEq1oyG/IS7Vbgn5XcvrWTvxkN9V2+Yl1Y6F/IZ2qXYs5Lctk5Zq\n",
"Fx9ZU5LtXpgwPb1ZtUuqDRGLJzj2LaTyOx5YUc3veSDV9tOmQdnSwekGN632RCZSc90D3hpQ8Pzj\n",
"B89/TWiDltB2evyfisjhwJnAYcDtwKUicpox5l1d9YjI3tjQ4CcaYy4Xke1YD9yt/uQW0hJyLsS3\n",
"AecCGGO+DFwHnDBmuxRFUcZizezu9QiZ8f90G3CRMWanMcYA7wVO7VnPccBuY8zl7vgc4NdF5IC2\n",
"fi2qJfQQ4ADgi17aF7DxKmpMRMBUc+iTascn4CngImZI0tVPQiUXc/UTim9illKwf7ZbJefd8CSd\n",
"nAZ1IjTXdRLqoZi1E25gLdd5Qjc+E2kYMaUFFKjkSkuGacOlT1+VnE0L1oBia0K+M1PvuPw4XLpv\n",
"KZUB8cq3Ib4mFFPJhXnaHJp2q+KaDk1TDkrbVHJdCroC3+pJWUBtKrlUKIe20A6VY1JTe24N7dDY\n",
"wOrSk+s+LUIDKdaC3KERbxNs8WOqP5czGCb8sVcWT0rWsO2obyfOrJ8NWEK9/09d2oXe8Y3AYSKy\n",
"FTi4o56jXX7XXvNNEfkBcCRwcapxC2kJUbkW3+Wl7QLuN0JbFEVRRmcDa0Kz/J8eFOS7A3svd2CP\n",
"eg4C7gzquyNxnZJFtYQKwnc0pRWLWjoQUbpVBSLqN3cnGe4porizS+8TCtN9S6lcJwrXZILp8Jr1\n",
"FN4ZVsKpWh2124jQXU/K5b1nKTVc+CTcBNXWhIrGTQrlnMuT3GM08SypwrKqNzqqkiuXj4I1oMAi\n",
"Ei8tFqrB5nHpvkou3CcUrAGlVHKxtFRIB9+BaZdKzg/pkFLOtankQguooDwOv6PUnZnac90quSJv\n",
"l9XkrxElQzgE6eIp0vywDhCxOlIurfxzKQnsehBffuel0VwTevMbbOTsSy+9dOPXdcxhM37f/9PY\n",
"myUd5/uWbbCollCxWLafl7YfcPMIbVEURRmdNbPW6xFhlv/TWyL5DHBrSz1Felg2PB9lUS2hm4Cd\n",
"wKOAK13aocBHw4zzvNPYowm/CfP3RL8B9gqet47VkJK7Bcfjt0hZJIb4X7rkkkvWW7T3/ymww+XD\n",
"y3e9MeYuEemq57PA84uCLnT4PYDPtTVOTKgXXhBE5M3AQcaYFzsp4FXAoV3xyhVFUZQ6qf9T7GTm\n",
"PwLPNsbcLiKHYUUEh2PXcy4GPmyMOautHifR3gp8CSvR/pSInOHOPZ8WFnkQ2orVmD8Kewv8R8aY\n",
"dd8KKIqi7Kmk/k+dtXIN8FhjzLdd3hcCr8AOUJcbY07vqsc7/xjgXdjlxp3AycaY21vbtqiDkKIo\n",
"irL6LKowoWRe7iaWgb59FZFjROSfReQS1983iEirAmXRWI9bJhH5pIi8bzPaN29m/B4fIiL/IiJX\n",
"iMj1IvLGzWzrRpnhe7yXiLxTRD4nIv/u3Lw8YLPbu1FE5HEi8hUROakj31L/Pw2GMWahH8CfA+9z\n",
"rx8O3AbcL5LvcOA7wP5YSeBlwGljt3+gvn4CeJZ7fU/sBrGXj93+Ifrq5X8mdo76vWO3feDPdl/g\n",
"BuCR7nh/4Pyx2z9QX08Dvgxsdcd/i11/GL0PM/T12cAF2MX3E1vyLf3/01CPhbaE5uVuYhmYsa//\n",
"aIz5qMv3faw65emb1NQNM6tbJpf/ldjPdKksPpi5vy8BrjTGfMnl3WmMOXGz2rpRZuzrocDnjTF3\n",
"ueOrsT7LlonrjDEvoLlJM2Sp/5+GZKEHIWZ3N+Hn891NLAO9+2qM+csg6e50aPEXjFk+V4AXYQfa\n",
"XYnzi84s/X0qcKeIXOA8EZ8jIuHei0Vmlr5+DHiCiBwgdpfxccBFwzdxfhhjvtEz67L/Pw3Gog9C\n",
"83I3sQysy1WRWDcCTwfOGqhdQ9C7r+5H+hLgr1lCK8gxy2f7M8DvAK83xhwD3AV8aNjmzZXefTXG\n",
"XAS8A/vn/FXgR8AfDt3AkVj2/6fBWPRBqGBe7iaWgd6uihyvwk7P7RioPUPSp6+/C5zrpmyWXcrZ\n",
"p79bgcuMMYUjyLcDvyIiPzloy+ZPZ19F5ATgZOBhwIOAHwLvGbxl47EK/09zZ9EHoXm5m1gGZnZV\n",
"JCLPwHqofc2A7RqCXn0VkX2B5wDnF0nDN20QZvlsdwbp33HPDxygXUMwS19/Dyu62OXWSd4JnCQi\n",
"9xq4jWOw7P9Pg7Hog5DvJqLgUKxriZCku4nhmjdXZukrIvJzwMuBFxljjIg8bPgmzo2+fT0auDc2\n",
"sNYngJOAXxWRy1z/l4VZPtvPU5+6KqZr5h8bYBhm6evdoBZZcDf2RiP0irQKLPv/03CMLc/regBv\n",
"xslysXLP72J/pAcCVwD3cecOw/5Q98cOrpcCLx27/QP19RDgM9i743u5x/vHbv8QfQ3KbMdJf5ft\n",
"McNn+3PY9YKfcsevAy4eu/1z7ut+7txb3PFe3vHnxm7/Ovv8SeAk73jl/p8Ge+/GbkCPD3cr8D6s\n",
"j6IdwNNc+sFYE/8BXt4XYl1QfBZ469htH6qvWOeBa1i3GsXjq2O3f6jP1aW/B7t/5mvA3xV/XMvy\n",
"mPF7fDLwH8Dlrq/3Hbv9Q/QV69zy3S7Pp7FquYeO3f4Z+3q4+4xucf87f9PyuS71/9NQD3XboyiK\n",
"oozGoq8JKYqiKCuMDkKKoijKaOggpCiKooyGDkKKoijKaOggpCiKooyGDkKKoijKaOggpCiKooyG\n",
"DkKKoijKaOw1dgMUZR5kWfZQ4DfyPH/LgNfYhvXqfQTw+jzPXz/ANR6Gddr6F3meT+ddv6IsGmoJ\n",
"KUtPlmU/j3Vx89k51/t3WZZ9pjjO8/zcPM8f6w6HcjXyVeDRwL9kWbb3QNdQlIVBByFlqcmy7EDg\n",
"QuCNeZ5/cs7Vfwv4rznX2Yqzfk4BHgD8xWZeW1HGQAchZdl5DfBjBgiGluf57+d5/vx519vjugZ4\n",
"A3BqlmWP2OzrK8pmomtCymhkWXY+8Ayse/v/BJ6HdYF/FvAQrNfsZ+d5/s1EeQFeBFyS5/luL/04\n",
"4E3YmC0fBK4Gjgceio1383vYmDdvBx6BDYXxV3mev8Or4zKX/2DgwXmef72jL1uA07Exj8B6kr4Y\n",
"+OM8z2/z8j3Wte0B2Pg5U+AS4G15nn/Xq/Jf3fNJwGvbrq0oy4xaQspo5Hl+IvBMbFiKT+d5fr2b\n",
"UrsM+JM8z49MDUCORwL3Bb4U1Psxt3bzbeBY4Id5nv8idlDZCnwEOxC8JM/zI4BXA293a0tFHU/F\n",
"xvLpy/lYq+z4PM8PAR6DHQQ/mWXZVoAsy+6FHZiuzvP8iDzPHwecALzE5fX7cCd2OvDJM7RBUZYO\n",
"HYSUUcnz/DPAG4FTsiz7jSzLngUckuf5W3sUL6LJ3pI4L8DteZ6f7671I2zslwcBH8nzfJfL9w/A\n",
"D4BfiZTvJMuyXwB+C3h3nufXumvdibVgDsPGBwIbWXN/4CtF2TzPv4S1oL4VqfoWr4+KspLodJyy\n",
"CPwZ8KvA2cBtwC/3LLeve/5RS54bg+PbwvQ8z02WZbcB9+953ZDj3POngvT/cM/HYvv2BeCbwNlZ\n",
"lj0eyLFW0XmJen9I1UdFWUnUElJGJ8/zNeDFWCvhlo4pOJ9iH03KYjHA9yJpRNKnrP+m7L7u+e1Z\n",
"ll1XPIBPAP+DnQIkz/MfAEdj17yehw3//K0sy87Isix27QlVHxVlJVFLSFkUTsGGLT8my7I/yPO8\n",
"jzx5p3u+x3DN6sWt7vm38zy/oi1jnuc3A6dnWfZq4Jew61FnYAfS7UH2e1JZboqykqglpIxOlmVP\n",
"AY4BngJ8GHhTlmWP7lG0mFJb7zTavLjIPR8Vnsiy7LVZlp3gXj86y7LXgp0CdCKMZwG7sF4YQu5P\n",
"czpRUVYKHYSUUcmy7D7Y6akXuWm5U7EWzgcKVVkKJ5u+CTg8kUVIT9WF6bPkraXlef5p4P1YC6cc\n",
"TLIsex7wcuDfXdIBwCuzLPPb+3jgJ4B/8yvPsuwg7DTfxYk2KcpKIMYM5X1EUdrJsuxPgROxi+8X\n",
"5nl+cpZlrwNeht0v9DXs/pmzWuo4HatC+2mnSMMt+p8NHAJ8H/hvrJVyLvBU4IHAF4G/Bq5J5L0U\n",
"u1epyPsB4Gas77ifxSrX/ivP8ye6awrwCmAbsAW75vQ14Iw8z29weQ4AXoVV4RmXz2AefC8aAAAA\n",
"oklEQVRVdWcH/XopcCZ2j9KtKMqKooOQstQ4a+larOR6JTZ1uv1ENwLvyPP8bWO3R1GGRKfjlKUm\n",
"z/O7sBLp52ZZ9ttjt2ejuAHo48CHdABS9gR0EFKWHrc2dDSVSm2ZuTfW1c+rx26IomwGOh2nKIqi\n",
"jIZaQoqiKMpo6CCkKIqijIYOQoqiKMpo6CCkKIqijIYOQoqiKMpo6CCkKIqijMb/A1I3CzrKizKz\n",
"AAAAAElFTkSuQmCC\n"
],
"text/plain": [
"<matplotlib.figure.Figure at 0x7f0ba4b95990>"
]
},
"metadata": {},
"output_type": "display_data"
}
],
"source": [
"xgrid,vgrid = np.mgrid[0:dx*XNUM:XNUM*1j,0:VMAX:VNUM*1j]\n",
"galloncosts = costs*gallonperj\n",
"plt.pcolor( xgrid/1609.0, vgrid*mphfact, galloncosts, vmin=0, vmax=galloncosts[np.isfinite(costs)].max(), cmap='cubehelix' );\n",
"plt.xlim((0,dx*XNUM/1609));\n",
"plt.ylim((0,VMAX*mphfact));\n",
"plt.ylabel(\"v (mph)\");\n",
"plt.xlabel(\"x (miles)\");\n",
"plt.title(\"Fuel cost for optimal path (gallons)\");\n",
"plt.colorbar();\n",
"plt.tight_layout();"
]
},
{
"cell_type": "markdown",
"metadata": {},
"source": [
"Now we want to extract the path we are interested in, namely $C(0,0)$. For this we need to trace through all of the optimal arguments at each step"
]
},
{
"cell_type": "code",
"execution_count": 232,
"metadata": {
"collapsed": true
},
"outputs": [],
"source": [
"traceargs = np.zeros(XNUM)\n",
"traceargs[0] = 0\n",
"for i in xrange(1,XNUM):\n",
" traceargs[i] = args[i-1,traceargs[i-1]]\n",
" \n",
"traceu = np.array([ uvals[i,arg] for i,arg in enumerate(traceargs) ])\n",
"tracev = np.array([ vs[arg] for i,arg in enumerate(traceargs) ])\n",
"tracecost = np.array([ costs[i,arg] for i,arg in enumerate(traceargs) ])"
]
},
{
"cell_type": "code",
"execution_count": 233,
"metadata": {
"collapsed": false
},
"outputs": [
{
"name": "stdout",
"output_type": "stream",
"text": [
"108.905442186\n"
]
}
],
"source": [
"v = 0.5*(tracev[:-1] + tracev[1:])\n",
"dv = tracev[1:] - tracev[:-1]\n",
"vdot = f(v, traceu[:-1])\n",
"dt = dx / v\n",
"ts = dt.cumsum()\n",
"print ts.max()\n",
"xs = np.linspace(0, XNUM*dx, XNUM)"
]
},
{
"cell_type": "code",
"execution_count": 234,
"metadata": {
"collapsed": true
},
"outputs": [],
"source": [
"# Convience routines for a nice plot \n",
"# FROM: http://stackoverflow.com/questions/10481990/matplotlib-axis-with-two-scales-shared-origin\n",
"def adjust_yaxis(ax,ydif,v):\n",
" \"\"\"shift axis ax by ydiff, maintaining point v at the same location\"\"\"\n",
" inv = ax.transData.inverted()\n",
" _, dy = inv.transform((0, 0)) - inv.transform((0, ydif))\n",
" miny, maxy = ax.get_ylim()\n",
" miny, maxy = miny - v, maxy - v\n",
" if -miny>maxy or (-miny==maxy and dy > 0):\n",
" nminy = miny\n",
" nmaxy = miny*(maxy+dy)/(miny+dy)\n",
" else:\n",
" nmaxy = maxy\n",
" nminy = maxy*(miny+dy)/(maxy+dy)\n",
" ax.set_ylim(nminy+v, nmaxy+v)\n",
" \n",
"def align_yaxis(ax1, v1, ax2, v2):\n",
" \"\"\"adjust ax2 ylimit so that v2 in ax2 is aligned to v1 in ax1\"\"\"\n",
" _, y1 = ax1.transData.transform((0, v1))\n",
" _, y2 = ax2.transData.transform((0, v2))\n",
" adjust_yaxis(ax2,(y1-y2)/2,v2)\n",
" adjust_yaxis(ax1,(y2-y1)/2,v1)"
]
},
{
"cell_type": "code",
"execution_count": 235,
"metadata": {
"collapsed": false
},
"outputs": [
{
"data": {
"image/png": [
"iVBORw0KGgoAAAANSUhEUgAAAb0AAAEfCAYAAADV+hihAAAABHNCSVQICAgIfAhkiAAAAAlwSFlz\n",
"AAALEgAACxIB0t1+/AAAIABJREFUeJzsvXm8HEW5///uBBLClpAQAqdOS5RNIVeQgGk3GIzaKgYV\n",
"NfITLyK4oF4uLuBVlPRpN5DFlQuKqIiCGkUuIpKWBDqA0Mp67zfsIME+JRAghCUBQpL5/VE9YTKZ\n",
"Oadn7e45z/v1mldPV1dXf6bnnH6mqp7nKatcLiMIgiAIY4FxWQsQBEEQhF4hRk8QBEEYM4jREwRB\n",
"EMYMYvQEQRCEMYMYPUEQBGHMIEZPEARBGDNskbWAemhl7wHcCcxVOr42KdsPOBdYD6wEjlY6Xpmd\n",
"SkEQhP7G8YLjgXcDFjAdOC/y3bPr1Cthns+PVBWfHfnuJb3Q2Qy5NHrA14AXKjta2ROAy4CjlI6X\n",
"amUPAT8C5mcjTxAEYUxwLPCWyHcfd7xgd+Auxwv+FvnuTTX1ysCpke9e2HuJzZG74U2t7AOBZ4DH\n",
"qorfAaxTOl6a7J8PHK6VPa3X+gRBEMYQH45893GAyHfvB54Edm1Q1+qZqjbIY09vCPgE8NaqsgOB\n",
"uyo7SsfDWtlrgP2Bq3qqThAEYYwQ+e6yynvHC96H6ZD8pUH19zhe8BGMXQmA0yLfXd99lc2RK6On\n",
"lf0OYJnSsdbKrj40A3i6pvoqYKdeaRMEQRiLOF4wC/gtsDXwwch3a5/FAE8BNwBnAZOAPwE7ACf2\n",
"SmdacmP0tLLHAV8E3lvncDl51VKI7rQgCEJRSXp7+zhesB9wpeMFh9XO6UW+eztwe7K72vGCbwO/\n",
"J4dGL09zeh8CAqXjVVVlFaO2AphSU39KUi4IgiB0mcSwXQH8Z4rqMbC14wU7dldV8+Smpwe8EZil\n",
"le0m+zsD39PKXg5cABxRqaiVbWO62rfUNrJkyRJZNkIQBKEF5s6du3H0zPGCqUAp8t0/VFVZgxm2\n",
"3IQktOG8yHcrXvczgLXAE12U2xK5MXpKx8dV72tlPwicoHR8rVb2ROD7WtkHJXF7xwCXKB3XvaHV\n",
"X1xe2WuvvS645557js5ax2gUQWcRNEIxdBZBI4jOTlLRWKfDsD1wiuMFV0a++5zjBdMxMXvfdrxg\n",
"GvA/wLzId1dhnArnA790vGA8cDxwUeS7ueuE5Gl4EwCt7H21sn8P7Ah8TSv7BKXjF4D3AKdpZV+H\n",
"ucHHjdRO3tlll11mZq0hDUXQWQSNUAydRdAIorOTjKDxYeByYInjBSHGU/6nke+egxlp2zPZAvwE\n",
"OMLxgquBG4FHgRO6KLtlctPTq6B0/L/A++uU3w68vveKBEEQxh7JUOWC5FV7LMYMYVb2bwAO7Z26\n",
"1sldT2+sMHv27OVZa0hDEXQWQSMUQ2cRNILo7CRF0NhJxOhlxLx585ZnrSENRdBZBI1QDJ1F0Aii\n",
"s5MUQWMnEaMnCIIgjBnE6GVHmLWAlIRZC0hBmLWAlIRZC0hBmLWAlIRZC0hJmLWAFIRZC+glVrmc\n",
"O4/StliyZEm5CCELgiAIeWKsPDulpycIgiCMGcToCYIgCGMGMXqCIAjCmEGMniAIgjBmEKOXEWEY\n",
"lrLWkIYi6CyCRiiGziJoBNHZSYqgsZPkLg1ZEdHKHk+Ta/tZP//pm7Wyr++SpI5RBJ1F0AjF0FkE\n",
"jdBVnWWl406u9l0i/yEBJfKvsWOI0WsTrew3AT4mI3lqdv3CSQPAO7siqoMUQWcRNEIxdBZBI3RV\n",
"52qt7G8qHf+lC20LOUCMXhtoZc8EvgFsA6yn/uru9Rk3bgOwrivCOkkRdBZBIxRDZxE0Qrd0Wpj/\n",
"ZU8r+59Kx3d3uH0hB4jRaxGt7G2AMzH/JFcBJysdpzZ6YRgOlUqloS7J6xhF0FkEjVAMnUXQCN3T\n",
"qZX9VcwyZmdqZX9Y6XhVp68hZIs4srSAVvY4YAiYCdwPfL0ZgycIQm45HVgG7Ax8K5mvF/qI3PX0\n",
"tLKPx6zOawHTgfOUjs9Ojj0C3FVV/Xql41N6r5KjgUOAZ4GTlI7XtNBG2ElBXSTMWkAKwqwFpCTM\n",
"WkAKwqwFpCTsRqNKx2u1sv8L+CXwWuAzwA/aaDLshK4uE2YtoJfkLvemVvbtwFuUjh/Xyt4dY+Re\n",
"p3R8s1b2z5WOPzrS+d3OH6eV/Xrg+8nu55SOc+/pJghCc2hlzwbOAcYDX1Y6vipjSV1Hcm9mx4eV\n",
"jh8HUDq+H3gSM4yYOVrZCuO4YmF6oGLwBKEPUTq+BfhesrtAK3u3LPUInSN3Rk/peFnlvVb2+4Bn\n",
"gIr78N5a2X/Wyr5OK/vHWtnTeqVLK3sr4AxMaMJS4Ke9urYgCJnwG+BKYBLGsWW7jPUIHSB3c3oA\n",
"WtmzgN8CWwMfVDp+Ojm0DDhB6fhZrezTMX+Qr+2BHgv4KrAn8BDgKR1v6PZ1BUHIDqXjslb2N4FX\n",
"AHsBX9fK/vxY+t93vGAzH4vId89uUPdI4HOY0K2lke+e2DOhTZC7nh6Y3p7S8T7Ae4HLtLIPTMqP\n",
"VTp+Nqk2BOxfOdZl/j/g7cAajOPKs6PUFwShD1A6fh74IvA08EbgY9kq6jnHAkdEvjsXOBz4ruMF\n",
"mz1zHS+YhQnhehumI7K/4wWf7qnSlOTS6FVQOr4duAL4zzrH1gArgV27qSGZ0D4h2R1SOv5HJ9ot\n",
"Sr67IugsgkYohs4iaITe6lQ61sBXMD2YT2hlH5T23CLcz1E0fjjy3ccBIt+t+FjUe+YeC1wR+e7K\n",
"yHfLwM+A4zqttRPkanhTK3sqUFI6/kNV8Rpgqlb2IcDTyQQzWtkTgB2Af9W2c9A7Dr9x3HMrXwCY\n",
"PXv28nnz5i0HwlKpFNbWTb7wUh054R5H/vtS4MsYD64LlI6vHql+k+3PpI6rcAfb70h94OgG/xSZ\n",
"6GlQv5QEK+dFT6P6pTAMyZGeevVLoRGZFz2N6pdM9R7pueiX7HjRxXrb6G+vLW+55YW3fPNb/zX7\n",
"Kyf/NkX7RbifJRqELUS+u9HHwvGCWh+Lag4ALqvavwvYx/GCiZHvvlCv7cwol8u5eQ0PDM4cHhi8\n",
"bXhgcFKyP314YPCh4YHBTw8PDH5keGDw/Kq6XxgeGLxreGBwfHUbixcvLs9ZsOj1HdKz//DA4M3D\n",
"A4OLhgcGt+jkZ73mmmuGsr7f/aKzCBqLorMIGrPSOTwwaA0PDJ6ePBMWDg8MbpNHna3ey8WLF5fr\n",
"HZ+zYNGsOQsW3TFnwaIH5yxY9NoGde6ds2DRx6v2d5uzYNGGOQsWqaw/X+0rb8ObDwOXA0u0skNM\n",
"eq+fAucCVwNW4rl5PTAXmNcgI/rWHdLznmR7mdJx/vMRCoLQNZKsSz7wIMa5ZUHi5NbXRL67LPLd\n",
"jT4W9eb0EuoFfefu/uRqeFPp+AVgQfKqJcaMG6ehbaOnlT0ZeEuy+8d22xMEofgoHa/Wyj4R+AXm\n",
"h/dRyfvCYVlWCSgdfPDBpaVLlw4tXrx4xPqR797ueEHFx+Lfaw6vAKZU7U/BGMHHOqe4M+Stp9cp\n",
"OtHTewcwAYiSiWxBEASUjh/ipR/mn9HKnpOlnlYpl8thuVweGhoaCsvl8lDtcccLpjpecHhN8Rrq\n",
"P19vAl5Ztb83sCx383mI0atLMmRRGdq8tH05dQm71G6nCbMWkIIwawEpCbMWkIIwawEpCbO8uNLx\n",
"tcD5mGfot7SyBxpUDXsmqnXCBuXbA6c4XjAJwPGC6ZiYvSWOF0xzvOA6xwt2SOqeD7wzMZTjMPmJ\n",
"f9Rd2a0hRq8+/wbsjnHPvbZ9OZvTwFMydxRBZxE0QjF0FkEj5EbnecBfgcnA6VrZE2sr5ETniIyg\n",
"caOPheMFIYmPReS752CesXtistUQ+e4dwIkYz84IuDXy3XO7q7w1cjWn10HaNXqVXt4flY5fbFeM\n",
"IAj9h9LxBq3sU4ALMUN7J2tlD/XLMmPJ0GRdH4vId2NgRk3ZxcDFvVHXOv3a09um1RO1srfFZBWA\n",
"TeNOBEEQNiFJkXgi8DxwKDA/W0XCaPSr0Wunp/d2YCvgZqXjf3ZIjyAIfUqyGszXkt3PaWW/Jks9\n",
"wsiI0ducbjuwCILQZygd/wX4FWbK6Nta2TtlLElogBi9KrSyZ2DG5lfTZa+rIuTkg2LoLIJGKIbO\n",
"ImiE3Or8IcZ1fyrGsWVCTnVuQhE0dhIxeptSyTRwSxIo301KXW6/U5SyFpCCUtYCUlLKWkAKSlkL\n",
"SEkpawG1JNmhTgYeAWYBXyCHOutQylpALxGjtymVtflu6pQQQRDGDkrHTwInAWuB9+3wx8v3yliS\n",
"UIMYvYQkIL1i9P7eWTmCIIwVlI7vAk4FmLzk6jdqZe+TsSShin41eq2ELLwc2BF4HOjImnmCIIxN\n",
"lI4vB37Hhg3jMfN7U7PWJBj61eht4XjBlk2es3Fos1+CSwVByJTvrJs+/RFMEPepWtn9mgykUPSr\n",
"0YPmhzgrTiy9GtoMe3SddgmzFpCCMGsBKQmzFpCCMGsBKQmzFjAaSscvPnH4e33M6NFszOoEeSTM\n",
"WkAvscrl/urULFmypPyVa9fdAhwW+e5mq6rXQyt7PGa9vm2AQ5WOH+2mRkEQxg5a2fsCP8bE8H1V\n",
"6XhRxpLqsmTJkvLcuXNzt/5dp+nnnt6kJurugzF4D4nBEwShkygd/y9wVrL7Va1s8ejMkNyNMWtl\n",
"H49ZvsICpgPnKR2fnRzbD7OK+npgJXC00vHKBk0148zS66FNQRDGFr/HrDE3D+PYcpTS8VMZaxqT\n",
"5LGndyxwhNLxXOBw4Lta2QdoZU/AJID+ktLxG4FbGXm9pmbm9CQ+TxCErpE4x50G3AUo4BvJtIrQ\n",
"Y/Jo9D6sdPw4bEzk+iQwE7OS+Tql46VJvfOBw7WypzVoJ5XR08qeBOyLWdr+5jZ0C4IgNCTJ8nQS\n",
"sAp4HfDJbBWNTXJn9JSOl1Xea2W/D3gGszDhgZhfSZV6w5il6/dv0FTant5+mGHeu5JlQnpCUfLd\n",
"FUFnETRCMXQWQSMUV6fS8SPAl4ENwDFa2YdkoauaotzLTpE7oweglT1LK/sO4EzMUOfTmFiXWqO0\n",
"CmiUzTyt0csqC0upx9drlVLWAlJQylpASkpZC0hBKWsBKSllLSAlpdoCpeObMMmpAXyt7Jf3VNHm\n",
"lDK+fk/JpdFTOl6mdLwP8F7gj1rZB2KGH+vFVzRysU1r9MSJRRCEXvMrzAjW1sAZWtktL3wtNEfu\n",
"vDerUTq+XSv7CkxQ50NA7eKMU4AVtedZqx/fa/yKO04ulU49evbs2cvnzZu3HAhLpVJYXU8re/L6\n",
"SZMO3LDtNls/dOq35t0Xhu+sOrxZfdg4FFCqI7fZ+jPrlHWy/Y7UB2aGYTiUFz0N6pfCMCzlSE+R\n",
"65eS7zsveopev+79VDoua2V//cVpUx3KvH6tbV8VLllyFePHV37YZ/X/3vfkKjg9yU9XUjr+Q1XZ\n",
"D4BB4BfAWUrHuyflNvAgMEPp+IlK/arg9N9GvnvGKNd7PfAD4Dal4493/AONQBiGQ6VSaaiX12yF\n",
"IugsgkYohs4iaIT+0Zk8xy4EtgPOUTr+Wa+0VahorA1Od7xgC+A/eCmEbALw1ch3r65tw/GCEiac\n",
"7JGq4rMj372kq+JbIG/Dm9sDpyQelWhlT8fc8MXAlcAWWtkHJXWPAS6pNng1pAlOf1WyvWvEWoIg\n",
"CF1A6TgGvoqZuvlU8kM8LwxiRtkOi3y3BCwALnO8YKBO3TJwauS7h1S9cmfwIH9G72HgcmCJVnYI\n",
"XAX8VOn4HKXjtcB7gNO0sq/DeG0eN0JbacbIK0bv7tYlt0yYwTVbIcxaQArCrAWkJMxaQArCrAWk\n",
"JMxaQErC0SooHf8Vk6bMAr6plT3YbVE1hA3KnwZOiXz3GYDIdxcDz2PCLepRiBRmuRre7ARVw5s3\n",
"Rr57/Eh1tbL/BOwMfEDp+MGeCBQEQahBK3sccAZwMHA/8FGl4+d6qWG03JuOF1gYj/l5ke9eW3Ps\n",
"YOCzwGSMr0gAnBb57vouSm6JvPX0OsmIw5ta2VMwBu854J89USQIglAHpeMNgIdx2Nsdk6Mzbz2n\n",
"g4HltQYv4SngBuAtmEQibwG+3UNtqcm192abjDa8WRnavEfpOHe/RgRBGFsoHT+rlX0ScAHgAncC\n",
"F2UqKsHxgq2AbwJH1zse+e7twO3J7mrHC76NyTd6Yk8ENkE/9/RGi9PLcj5PEARhM5SO/wEMJbv/\n",
"qZV9QLeuZVlWybKsocqrUb1kWPM84DuR796WsvkY2Nrxgh07ILWj9LPRG62n98pkK56bgiDkBqXj\n",
"qzG9vfEYx72du3GdcrkclsvlocprhKpnAVHku5c4XjDB8QK7toLjBcc7XjCxqmgGsBZo5F2fGf1s\n",
"9EYLWagYvUx6ekXJd1cEnUXQCMXQWQSNMCZ0ngtEmAQcp2tlTxylfsuMpNHxgv/CGN9fOF6wLWa+\n",
"8RjHC6Y5XnCd4wVTkqr7A/OTc8YDxwMXRb6bO0/JfjV6G4CJyc3fDK3sycAAxv12eQ91VVPK6LrN\n",
"UspaQApKWQtISSlrASkoZS0gJaWsBaSk1MpJiZ/BV4B/Ydbh+2IXHVtK9QodL9gTOBVjwJ7BhDAs\n",
"w8TkTQL25KVppJ8ARzhecDVwI/AocEKX9LZFvzqyrAG2xQxx1ls5obJy8X3ixCIIQh5ROn4qcWz5\n",
"KSZJx51AzwK+I9+9l5E7RjOq6t4AHNp1UR2gX3t6a5JtoyHOmcn2ge5LEQRBaA2l43swXpMAJ2ll\n",
"vzpLPf1Avxu9Rs4sM5OtBKQLgpBrlI6vBH6NGZk7XSs7dx6RRaKfhzehcdhCZf2q5d2XIgiC0Dbf\n",
"x0zL7I/x6PyU0vGLGWvqGo4X7IFxmpmM8dF4Argr8t1/tdu2GL3sCDO8djOEWQtIQZi1gJSEWQtI\n",
"QZi1gJSEWQtISdiJRpSO12llfxn4JbAf8Dng9E60TU7upeMFk4GTMAHw9ZJa43jBXRjP1nNbTXHW\n",
"r7k3vwscBJwY+W5YfVwre1vMl/wCcJA4sgiCUBS0smdhPCW3BIaUjv/UqbZHy73ZTRwvmA38EZiI\n",
"eT7fhfEAfQ4zDbcNsAvwauBNwD3AoZHvPlKvvZHo955evTm9mcn2ITF4giAUCaXjZVrZp2PCGb6s\n",
"lX2/0nGhs0olwe6XAd/FrMH3/Cj1pwGnAFc6XjAn8t21zVyvXx1ZViXbKXWOzUy2y3uiRBAEoYMo\n",
"HV8KXIrpFZ2ZJM8vMkcDx0W+e+ZoBg8g8t0nIt/9LPAr4LBmL9avPb2VyXZqnWN5mM8TBEFohzOA\n",
"PYBZwLe0so8v6shV5Ltfb/G8s1o5r2Wjp5Vd17tG6bhl7xqt7LrL0ye56NDKfoRNc2Ver3R8Sp2m\n",
"KvneptU5NjPZLm9VpyAIQpYoHa/Vyv4vjGPLa4HPAD/IVlUxaGp4Uyt7slb2N7SyhzETiVcAFwO/\n",
"waxyPqyVfYdW9n9oZddNATYKG5enVzoukSxPr5W9S3L8SqXjQ6pe9QwejNzTm5lsM43RGwO5A3tG\n",
"ETRCMXQWQSOITgCl40eBLwHrgKO0st/aSjtFuZedIrXR08qejUmDcxwmEeo3MQbq48Angc9jutz/\n",
"BE4D/t5CdvCngVOUjp8BUDquLE//+ibbqWv0tLK3xBjWMtkvHFvK+PppKWUtIAWlrAWkpJS1gBSU\n",
"shaQklLWAlJS6mbjSse3At9LdhdoZe/WQjOlzinKP6mGN7WyN/GuUToecbJRK3ujd41W9hyl41Te\n",
"NUrHK6laNDFJsDoBeCwp2lsr+8/AdhgDfLLScb2lKxoNb9qYjOHDSscvpNEkCIKQc34L7INZsfxM\n",
"reyjKh0HYXPS9vSOBo5TOj5zNIMHoHT8hNJxy941VRwMLFc6rixPvwyYr3T8Jszy9Fc2OO/JZLuD\n",
"4wXVn3Fmsn2oDU2CIAi5Qem4jBl5uwfzw/7rWtn96pnfNql6ekrHLXnXKB235F0DoJW92fL0SsfH\n",
"VlUZAp7Wyj5Q6fim6nMj333B8YJnMSstbIcxkGCGNiH7oU1BEISOoXT8vFb2F4ELgTcCH8Osdp5L\n",
"HC94PfAGTGqxPyVlM4HXAf+MfPev3bp2LkMWkmHN84DvKB3XXZ5e6XiNVvZKYFdgE6NXKpXCca8+\n",
"Yp/yhO22erV130/DcOIyINwDVOUSlbrJJG6pziXCUqkUblbYufoz65RlqadufWBmGIZDedHToH4p\n",
"DMNSjvQUuX4p+b7zoqfo9Xt3P3UcamV/BePF+Qmt7Lvvu+iX45ppvxc4XvAZ4ETMc/s9jhd8GXh3\n",
"5LvLHS8As/pNK46Q6SiXy02/hgcG9x0eGPSGBwbfluxPHx4YDIcHBp8cHhj8zfDA4NattFvV/neG\n",
"BwY/nbyfMDwwaA8PDB4yPDA4u6rOhOGBwReHBwZfX33u4sWLy+VymTkLFv1kzoJFN89ZsOiAqnN+\n",
"ODwwePPwwOCb2tHXidc111xTylpDv+gsgsai6CyCRtE58mt4YPDo5DkXDg8M7ppWY+XZ2e3XnAWL\n",
"fjFnwaKJVfv7zVmw6NdzFizaac6CRTvPWbBoQzev3+q4738Ab8XkRQM4E5MP7XrgQMBr1QgnsSfj\n",
"gV8keTJ3B44BXgZ8qqrq8cD9wN8aNFXPg7PS02s7U3e7ZPUrq1mKoLMIGqEYOougEUTnKPwCWIKZ\n",
"3jlDK7vREmtAJhr/N/LdjY6Eke/ejnnGfxrznO8qrRq9A4FDlY6vS27o+zFenfMw4QXvaqVRreyR\n",
"lqe/GrC0sq/Tyr4emAvMGyELQcXoTUvaHs9Lmbt13TMEQRAKTuLY8jXgH8ArMKEMmSSSbkDseMFH\n",
"HS+IHS+YBRD57nOR7w5hlk7a0M2Ltzqnt0HpuOIc8hbMCuU/BhMwqZXdVALQCkrHoy1Pf+wIx2qp\n",
"7elNx3zex9N4oAqCIBQVpePVWtknYXp9c4GjkveZE/nu7xwv2A3Tubmn5tiPHC9Y1s3rt2r0ttTK\n",
"3kLpeB3w78AypeM7q47n4VdFJVavYvQ2c2IRBEHoV5SOH9LKXgB8B/iMVvbdSseNpoM2w/GCumkh\n",
"I9+9ukH9IzHr/JWBpZHvntio7ch3H8A4rNQ7dn1aja3Q6vDm34ArtLJ/ArwX+FnlgFb2kZhhyazZ\n",
"ZHgTMXqCIIwxkhjn8zHP+m9pZdddnLUBG9NCRr5bIkkL6XjBZm0kw5RnAm/D5ALd3/GCT7ei2fGC\n",
"fR0v8BwveFuyP93xgtDxgicdL/iN4wWNFgdPRatG78vJ9gjgf4D/BtDK/g0mAeql7YjqELXDm7lx\n",
"YoHi5Lsrgs4iaIRi6CyCRhCdTXIe8FfM4gCna2VPrD44gsangVMi330GIPLdSlrI19WpeyxwReS7\n",
"KyPfLWM6Qse1qLdrjpLQXO7Ni7Wy36eVPUnp+DGlY1fpeDul4/cpHb8IoHR8hNLxOKXj77YjqkPk\n",
"fXizlLWAlJSyFpCCUtYCUlLKWkAKSlkLSEkpawEpKWUtQOl4AyYt5DDwSuDkGseWUr3zEgO2MS2k\n",
"4wW1aSGrOQCoXsz2LmAfxwsm1qk7GgdiVkW/zvGCjY6Ske+25ShZoZme3jaYXtwTWtmXamUflfPF\n",
"CzcObyZfVt6MniAIQk9QOn4a+AKmp3YoML+FZg4Glke+e22dYzN4KfMVmIW8LWDHFq6zIfLduo6S\n",
"ke8+CrTkKFkhtdFTOn43xgPyI5hu5w+AFVrZf9HK/lQLKyp0lch3n8Po3BJjsMXoCYIwZlE6fgDw\n",
"k93PaWW/Ju25jhdslhayDuU6Za04NW6ZONFA4igZ+W7HHCWbmtNTOl6tdPw7peMPATsB78Ekbx4C\n",
"tFb2X7Wyv6CV/Yp2RHWQJwD2fPQBhRnmfJH6XXNBEIS+R+n4KsyI3RaLnnvuvGnjx58xNDRUsixr\n",
"qNE5yUjZecB3It+tmxYSWAFUj/xNwRjBVp63fwOucLxgM0fJxEO0LUfJljNxKx2vVTr+s9Lxx4Fd\n",
"gDcDN2O8fe7Xyr49cZfNkpUAuzz96J7Jvk7GtwVBEMYqZwM3vX3SJOv/dh6Y7n/pS9eVy+WhEeqf\n",
"BUSR717ieMEExwvsOnVuwswXVtgb00NrZQm3uo6Sjhd0xFGyIwmnE0OyNHmdoJV9AHA4RvTXOnGN\n",
"FlkJMHHd2srCirnw3EwIsxaQkjBrASkIsxaQkjBrASkIsxaQkjBrASkJsxZQi9Lxeq3skzEGZNbg\n",
"t057kbe/vW5dxws2poV0vGBbTJqwDzhecDbGIB0W+e6TmLCIqxwvmIqZzzsa+FEr+iLffQxw65Qf\n",
"gbEpbWGVy/WGYdtDK9tKUuH0nCVLlpTnzp1rASTZu9/3trvCv37yr796A/B7pePTstAlCIKQJ7Sy\n",
"XwX8FOOR+e67L7xguPLsBHC8YE829cisMIQZcrwFeE3ku/9K6n8I+DwmjdjSyHdPSqvF8YKLgUuA\n",
"Pyf+GF2jW0sL3YLJoZY1jwG8sMWEihPLigy1CIIg5Aal47u0su/HDEVu5okf+e5oaSFn1NS/GLi4\n",
"RTmV6AAcLwgwQ5h/jHx3VYvtNaRlo6eV/XpMF3SXmnYszBp3eSAGWD1h612SfXFiEQRBeIlKwv5M\n",
"11aNfPfdSUzeOzHOKz8Azne8IMQYwEsj332kE9dq6YNqZX8Wk89tDcZDsto5xMJY7TwwDLB6wtbT\n",
"MR4/j2crRxAEIVesS7bdW7Q1JZHvrgZ+B/zO8YIJmBi992KGU892vCAC/oAxgP9o9TqtWvfjgQ8A\n",
"f6g3d6eV3cittdcMAzw7cZsdyvC0JUZPEAShmorRy7SnV0vku2uBPwN/drzgk5g0ZIdjogPOcLzg\n",
"/4A/RL7btKNkqyELzyodXzKCs8onW2y3oyRR/U9vGDdu4lOTth9PjoY3c5KTb1SKoLMIGqEYOoug\n",
"EURnB1m/ftKk7ciZ0asm8t0Nke8ujXz3hMh3d8Xk/vwzLXpytvpB79XKnly1pl4tBwB/b7ZRrey6\n",
"S1koHV+dHN8POBczDr0SOFrpeGWD5gAYt2G9HlfeMOfR7XYcP+W5pxvpzYISOXRnrkOJ/OsskX+N\n",
"UAydJfKvEURnp1i3YdKk7cY/91zmw5tNcG7ku/sDJ7dycqtG7/PA2VrZ/4NJLPps1TEL+AxwTgvt\n",
"Vpay2FcDtpcZAAAgAElEQVTp+Bmt7LcAlyUrqj8BXAYcpXS8VCt7CBMHMmIOue1eWL0S4F+Td37+\n",
"zbctzSSMQhAEIafkwpGlFscLuuYo2eoHnQq8ETiywfFWjcvTwClKx88AKB0v1sp+HngDJoXYOqXj\n",
"pUnd84HlWtnTlI6fqN8cTH/2iWde2GIC/5o8Y32jOoIgCGOU3DiyVHC8oKuOkq0avR9iYt7+OxFV\n",
"a+T8zc5IQTJUuXEpi2T5iwnJtd5GVaCk0vGwVvYaTDzgVY3afNnK4efu2+kVPLL9TnlYzV0QBCFP\n",
"5NGRZaOjZLI23yY4XtCWo2SrH3Q3YDel4+frHdTKPrh1SZtwMLAcuA6Tbbt2Tm4VJvF1Q/Z47MF1\n",
"9+30Ch7bdtqWHdIkCILQL+TR6D0b+e4lIxxvy1GyVe/NBxoZvIRjW2x3I1rZG5eySLxEK69aRuzB\n",
"7f3wvWWAJydNntSupg4TZi0gJWHWAlIQZi0gJWHWAlIQZi0gJWHWAlISZi1gFNaNe+65Z8jR8CZw\n",
"r+MFk0c4fkA7jbdq3S/Syj5K6fjCBsdvpo00ZMmw5nnAd5SOK13ZFUDt+k9TqJNarFQqhZX3X5q6\n",
"1ZSJ5XVbPLvVVts7XrB95LubLEuRuBSX6sgIq9vpRv06ZZnqqVc/OWcoL3oa1Q/DkDzpqVe/VCqF\n",
"edLTqH5Slhs99epX3udFz0j1c34/1483Ri9PPb3PY4LRO+0oaRpoJeG0VvbPgbcCTzYQdZjS8bRW\n",
"RWllfwe4X+n4HK3sCZgcb/sDZykd757UsYEHgRnVjizVCaeTev/9rbcdf8SdO++54rkJk95Xsxih\n",
"IAjCmEUr+4sYD/jT777wgt9WPzuzwvGCfTErODTy0ixHvttyz7RV6/4hzDI92wGvZdNhx7a8a7Sy\n",
"Ny5loZW9cSkL4FTg+1rZBykdXwscA1wykudmwrRpq598cf248eswIRFi9ARBEAx5nNPriqNkhVY/\n",
"6J1Kxw2Xmm81DVkSj3dqsnt81aEhpeO1WtnvAc7Ryt4YnJ6i2enTVj+5NjF6L2tFlyAIQp+Sxzi9\n",
"3YDdIt+t6zfieEFbjpKtftDRIuFb8q5ROh5xKQul49uB16dtLxkanbzz0yvWrLfGrQf2a0WXIAhC\n",
"n5LHnt4DjQxeQluOkqm8N7WyP6iVPVDZVzq+cqT6Ssd/T86brZV9UDsC22QawKyH7/knllUGZifL\n",
"V2ROAXLyAcXQWQSNUAydRdAIorOD5DH35kWOFxw1wvGb22k87Qd9DFiilf1+peM70pyglf1WjAdm\n",
"lkZvOsCU555+GPgnpqc3B7g6Q00VSuTfnRmKobNE/jVCMXSWyL9GEJ2dIo+5Nx3grY4XnER9R8nu\n",
"pyFTOr5aK/sc4Hat7MuAALgHM9n4fCJka0yetFnAezCxFO9VOo7bEdgmOybbx4H/wxi9g8iH0RME\n",
"QciaPA5vds1REpr4oErHP9TKXgacAfx4hKplYBEwW+n4nnbEdYBqo3cdxjnmjY4XjI98V3JxCoIw\n",
"1smjI8udke82dJTsaRoypeNrgAO0svfBLOq3OzAZkxB0JaYrek3GvbtqKrGCj2Ni+oYxYQuzgP/N\n",
"SpQgCEJOGDHhtOMFs4HfAl+PfPcXDeqUMEu+PVJVfPYoqcRGoiuOkhVasu7JvF6qub2MmZJsV0W+\n",
"W3a84FpM1/kgxOgJgiA0HN50vOAwzPPyKUZeOacMnBr5bqMMXU0R+e6IjpKR7za9Vms1rebeLArb\n",
"Jdtnkm2UbP8tAy21hFkLSEmYtYAUhFkLSEmYtYAUhFkLSEmYtYCUhFkLGIX14xqnIbst8t0jMEu+\n",
"jUbLmVwcL/ig4wUDo9fc7LzZjhc07SjZ70Zv+2Rb+dLuTbZ7Ol6QabqdRnku80YRdBZBIxRDZxE0\n",
"gujsIOvGN0g4HfluM9NU73G84GrHC651vOArjhc04w36GLDE8YJ90p7geMFbgd9jpq2aIk+Tl92g\n",
"tqf3BGbucSrG0/RfWYgSBEHICZ3w3nwKuAE4C5gE/AnYATgxzcmR717teME5wO2OFzQVHdCkYQb6\n",
"3+ht0tNL5vXuw8Tq7YkYPUEQxjZtr5we+e7twO3J7mrHC76N6YWlMnpJGz90vKCp6IDId1uKDuh3\n",
"o1fb0wPzC6Ji9MJeCxIEQcgDlmWVXr3llkfOmTBxYMWG9QcfP/opaYmBrR0v2DHy3cfTnhT57jXA\n",
"AckwZ8PogFZ6d9X0rdFL1uSrGL3qidj7ku0evVUkCIKQH8rlcqiVXQb2Af52dxN5jatxvOB44LzI\n",
"d19IimYAazHTSU0T+W5XowP62ZFla0yX/Tml43VV5RudWXov6SUKkJMPKIbOImiEYugsgkYQnR1k\n",
"XZJ7c6ThTYsq70zHC3Z0vOA6xwsqIWH7Y9bkI3FgOR64KPLd5hdr7QFtGT2t7C21sqdrZe84eu2e\n",
"U+u5WWE55leIcrxg254q2pRShtduhlLWAlJQylpASkpZC0hBKWsBKSllLSAlpawFjML6DQ0STjte\n",
"MMvxgt8BewOfcbzgvOTQJEynYetk/yfAEY4XXA3cCDwKnNB15S3S1PBmslTPezHeM3MwiT+t5Nh6\n",
"zIe9GbgC+I3S8TMNmuoFFaO3iYbId9c7XvAA8CrMEGdbKW0EQRAKTEPvzch3l2EW8K4tjzFDmJX9\n",
"G4BDuyWw06Q2elrZxwCfxxi1q4HTMfEVKzH526ZhQgFeg5mEPFEr+1fAaUrHLzZxnY1pb5SOf1FV\n",
"/ghmIrPC9UrHp4zQVKUXV8/w3osYPUEQhLa9N4vGqEYv6d39BLNKwetG6L39K3ktA36plT0ROAr4\n",
"nVb2J5SOV6S41khpb65UOv7oaG1U0Wh4E16a19urifYEQRD6jTwmnO4qaeb0TgK+qXR8VjPDlUrH\n",
"Lygd/wSTHPTLKU+7Tek4bdqb0agXrlDh/mS7WweuIwiCUFTyuLRQVxn1gyodf7OdCygdPwp8LmXd\n",
"keIv9tbK/jPGmN0JnKx0PJJL7Eg9vYeS7WAaXV0izPDazRBmLSAFYdYCUhJmLSAFYdYCUhJmLSAl\n",
"YdYCRmGk3JuZ4njBlphFA8rNxPuNRssfVCv7LODDwJuUju9Nyj4M/EnpeFWH9FWzDDhB6fhZrezT\n",
"gSsxCww2YqSe3hOY9DZTHC/YPvLdTvQsm6IAOfmAYugsgkYohs4iaATR2UEquTczN3qOFzR0lHS8\n",
"YDNHych3W3KUbCdk4UngPGBNVVkAfFYr+9VttFsXpeNjlY4ry8YPAftrZR84wikNe3pJ/EilV5ll\n",
"b08QBCFLcuHI4njBMcCtwDswjpLvwxi9bTEhErsCbwcuAQ4Ebna84JSkN9gU7Vj3MnCu0vHG/JVK\n",
"x48BQ1rZ38A4vnQFpeM1WtkrMTfiptrjpVIp/Ojk7V/+qjXPTb1jv30/9vIwfBUQ1vzq+iewx4G7\n",
"jDs0DMP5dS5TW98UmmDTktSX+lJf6vdB/UwdWZLe3UZHyRF6b5s4SjpesNFR0vGCT0S+O6qjZAWr\n",
"XG4taF4r+1WY8erbku1S4Cal43Va2WcpHX+hpYZN29cAP1c6vjDZPwR4Wun4lmR/ArAaOFjp+Ibq\n",
"c5csWVKeO3eupZX9A0xanc8qHV9fe40kdc5HgB9Fvnt+q1oFQRCKilb2DMxw4Yq7L7zgHXPnzu3p\n",
"kmuOF3wF+F3ku/eOWrn++TOAL0W+m8pvBNqz7l8DvooZHnSBBYCllf048PM22oWatDfAyzCxfx9L\n",
"9o/HeGD+bYQ26ganV1EZ3rRb1CgIglB0MvXejHy3LUfJyHdTO0pWaGdO71al458oHXtKx4dg1k96\n",
"O7AQM7fXNFrZs7SyN6a90cquLDFxNcagXqeVfT0wF5indLy+UVuM7MgCGRu9AuTkA4qhswgaoRg6\n",
"i6ARRGcHqeTezNyRpVe080HXaGXvpXR8D5i4PGCpVvbfgZOBvzbboNJx3bQ3SSjDsU02N1LIAmTf\n",
"0yuRf3dmKIbOEvnXCMXQWSL/GkF0dop1GyZN2m78c8/lKiOL4wUbowMqQ5+OF3wY+FPku21FB7Tc\n",
"01M6/j7gamV/vlKmlf1mjFdnpkHfNcsKNerpPY4JW9gh48TTgiAIWZHXjCwNowMcL2grOqCtVRaU\n",
"jn+gdPydqqLrgC8A32qn3Q4wCfMlvpD0QDcj8t0NwHCyazte0M/LLAmCINQjrxlZysC5ke9WntFE\n",
"vvtY5LtDJMsYtUpHP2iSWPq/O9lmi4zWy6sQY1bnnQV83fECHflubpfEEARB6DCVnl6uhjeBPwC3\n",
"OV6wSXRA5LvrMJ2alhm1d6OV/RGt7JZviFb2OK3sDq5En4p6K6bXozKv95/ATOD1jhfk7RePIAhC\n",
"V1A6LmNZG7LWUYdKdMDfMNEBS4BnHC+IMeFqLZPmAX8tcKFW9hlKx7c307hW9p7A14EzWxHXBqM5\n",
"sVSodJ0rvxwsYDrwcDdE1RD24BqdIMxaQArCrAWkJMxaQArCrAWkJMxaQErCrAWMxrjnn38yaw11\n",
"uDXy3Z9UdpJgdAc4jBajAyqM2tNTOn4Q0xP6jFb2VVrZR2plv6JRfa3sAa3sI7Sy/wB8H/ii0vFm\n",
"WVO6TNrhzX9Wva+Mbe/ceTmbU4CcfEAxdBZBIxRDZxE0gujsJONXr86j0VvjeMHGpd8i330h8t2l\n",
"mN7f29tpOJXzhtLxE0rHH8cYvzcAV2llr9TKvlsrO9LKviF5/xgmLZgL/EDp+B1Kxw+N1HaXSNvT\n",
"uxuTfPoGzJgxVK0ILAiCMAZYN3qV3hL57vcB1/GCjdEBjhd0JDqgqfkrpeO7gE8DaGXPxGRK2Qlj\n",
"PB8DdGXFhYxJ1dOLfPdZxwveifEUqjiwiNETBGEskTujBxD57g9qiirRAUvrVE9NO04bDwHPKB1f\n",
"246ALjFaCrKNRL67HsDxgkeTop4MbwqCIOSEhpmtHC+YDfwW+Hrku78Yod6RmHRgZWBp5Lsndlpk\n",
"5LsdiQ5oJzZtOfCwVvYlWtknaWV/Qit7WruCOkTaOb1qHkm20tMTBGEsUben53jBYcBJwFMYY1YX\n",
"xwtmYZwV34ZZ43R/xws+nebCjhd8xPGClqMDHC8YlywekJp2jN5nAUfp+H1Kx2cAvwPer5X9yTba\n",
"7BSVDCvNGL2e9vQKkJMPKIbOImiEYugsgkYQnZ1k3ZQpOzQ4dFvku0cwum/EscAVke+uTNYq/Rlw\n",
"XMrLXwtc6HjBfinrb8Txgj2BXwNRM+e1k4bsUqXjW6v2n1Q6/jGwSCu7qazXXWCrZPt8E+f0uqdX\n",
"6tF12qWUtYAUlLIWkJJS1gJSUMpaQEpKWQtISSlrAaOxbsrkqfXKI9+N65XX4QCMU2CFu4B9kjCD\n",
"EYl8d2N0gOMFVzlecKTjBQ2jAxwvGHC84AjHCzZGB0S+21R0QMtzelrZEzGrIdyhdLy2Uq50/JBW\n",
"9tatttshKje7bgqyBjwJvAhMdrxgUuS7z3VeliAIQs4YN67d4PQZmCHQCqswMc87Anq0kyPffQL4\n",
"uOMFr8IsG/c1xwt2AFYkbW0ApgLTgLXAX4AfRL4btiK2HUeW/8Es0mppZV8GLMKsarsFMLuNdjvB\n",
"hGSb2uhFvrshcWYZxHyJy7ugSxAEIVeU2zd6UH/Or6kFaSPf3Rgd4HjBTDaPDhiOfPe+9mS2Z/Su\n",
"Bw4FDgGOBL6LsezPAx9tV1ibVIze2hFrbY4YPUEQxgSWZZWA0jGD9k7jn1q14YOtN7UCmFK1PwVj\n",
"BB9rtcHId5dT8wx2vOAwxwueinx3RavtQntG7/8Bnwd+rXR8TLKcz3RgVfVwZ7NoZW90kVU6/kVV\n",
"+X7AuRj32pXA0UrHKxs0U5nTa2Z4E16a15OwBUEQ+ppyuRwC4f1vfsthk+65Z8PdsEuLTd0EvLJq\n",
"f29gWeS7zT5/N5J4ZC4Hro98t5Ix5k7gA44XrIt898cNTx6Flo2e0vEftbKnY1Yx/43ScRlj8VtG\n",
"K/sw4EPUuMhqZU8ALgOOUjpeqpU9BPyIxktMND28mdBLZ5awB9foBGHWAlIQZi0gJWHWAlIQZi0g\n",
"JWHWAlISZi1gNMavWjUMDIxQxaJqqNLxgh2BS4HDEoN0PnCV4wVTMXNwR2Oez+0wC/Aw653egQlM\n",
"vzbZfhZo2ei1u57eY0rHv2mnjRpuUzqu5yL7DmCd0nElEv984PAR4gIrjiytDG9CD4xeEXLyQTF0\n",
"FkEjFENnETSC6OwkEx59tK6XpuMFsxwv+B2m5/YZxwvOSw5NAvZMtkS+ewdwIsbBJMIkiz63HU2R\n",
"734SM3K4H8aATgXOAG5h05zJTZOrZXSUjhu5yB6IcYOt1BvWyl4D7A9cVad+K96bUDW86XjBALB9\n",
"5Lt3j3SCIAhCwambkSXy3WXAB+qUx9R0DCLfvRi4uJOikpi//5e8zgFwvODdNOkgU0tRVgufwea9\n",
"v1UYz556tGr0Kj29WZhg+wscL5gyQn1BEISik7vcm44XTHS84DWOF0yoLo989zJg33bazlVPbwTK\n",
"NOcS2+qcXsXobVNVNogxsIIgCP1I7owexofjdcB6xwsWAr/HOMxMxCz43TJF6enVusSS7DdynGlp\n",
"Ti/y3WeBB4BneWmB2enNtCEIglAwGiaczpDrgR2A92M6Z7/CJBCJgRvbabgoPb2bgCMqO1rZNrA1\n",
"ZlJzM/SWW+613rI2fHlX++rZX/jC8nnz5i0HwnqTykluvFJl33vThPuA+372vy/uFD9dHqTG6NXW\n",
"rz6Upv0qVpVKpe81Ub/Z9jtV/7Ns/oMjSz316s8ELsiRnrr1q/Iw5kJPg/ozMa7iedFTt34YhqVS\n",
"qRTmRc8I9WeS8/v5/K67qq0eymLZ0xFZBnwRuCjy3Y8BOF4wDVjTdrascrmcu9fwwGA4PDD4kar9\n",
"CcMDg8uHBwYPSva94YHB39Y7d/HixeXhgcGbhwcGw3Y0zFmw6ONzFiy6ec6CRZ/uxme85pprhrK+\n",
"z/2iswgai6KzCBpFZ2dfd7338HB4YPDmxYsXl7PWUv2as2DRjnMWLHp/p9vNVU9PK7sSm7E38Bmt\n",
"7DcoHX9C6XitVvZ7gHO0sjcGp4/SXMuBkQmVoVMZ3hQEoX/pTBqyjhP57uOYubyOkiujp3Rc10U2\n",
"OXY7JtdnWlrOCpNQSaEjRk8QhL6lQ7k3C0NRHFlaod2e3uPJdsd2hQiCIOQWMXp9Q7tGr9LTaxQL\n",
"KAiCUHjK48aL0esT2h3efAoTv7J9msUQWyDsQpvdIMxaQArCrAWkJMxaQArCrAWkJMxaQErCrAWM\n",
"xpYrVtyTtYZeYpXL9WK+i8uSJUvKrzzq6FuAm5WO0y5ZXxfHC/6EWXHh3ZHvjroYoiAIQtHQyv4U\n",
"cOzdF14we+7cuW2l+CoC/dzTa3d4E8SZRRCE/iePweldQ4zeyEjYgiAI/U4e05B1jX42eu3O6cFL\n",
"Hpxi9ARB6FfE6PUJMrwpCIIwOjK82Sfk2uhV5WHMNUXQWQSNUAydRdAIorOTPHPggXtmraGXiNEb\n",
"mW729EpdaLMblLIWkIJS1gJSUspaQApKWQtISSlrASkpZS1gNNbu+rJXZq2hl/Sz0evknJ5kZREE\n",
"oS8pjxvXX3Fro5Cr3Jsdpqvem44XbA1MA4aTZe0FQRAKR3n82MrIIkZvZFYDzwNbO16wD7AbMAv4\n",
"t+T9OGAB8OcOXEsQBKH3jGD0HC/YCvgRsBfGXpwc+e5VdeqVgHOBR6qKz45895LOim2ffjZ6bQ9v\n",
"Rr5bdrzgMcAGftGg2t6I0RMEoaCMssrCEFCOfPd1jhfsAUSOF7wq8t0VNfXKwKmR717YLZ2dop/n\n",
"9DrR0wO4LdmuAJYA3wOOBb6WlE9rsd2wPVk9I8xaQArCrAWkJMxaQArCrAWkJMxaQErCrAWMxoSH\n",
"H7m1XrnjBeMwz7qfAkS+ex/mefjhBk0VIoVZoXp6WtkXALvWFL9L6Xh1neqdMnpfB74f+e5T1YWO\n",
"F1TuXUtOLqVSKWxTV08ogs4iaIRi6CyCRhCdnWTKVVfdSn1D9grMj/q7q8ruBA5o0NR7HC/4CMau\n",
"BMBpke/mLgawUEYPKCsdH5KybkeMXuKk8lSdQ08k21Z7eoIgCHmgUUaWGcm2+vn3FGZKp5angBuA\n",
"s4BJwJ+AHYATO6SxYxTN6DVDJ0IWRkKMniAI/cBoachqvdM3G8aMfPd24PZkd7XjBd8Gfk8OjV7h\n",
"5vS0ss/Vyr5WK/vPWtmlEap2anizEc9iDOs2jhdM6vK1BEEQOoplWSXLsoYOWfHIsf5TqwbqVKk4\n",
"q0ypKpsCPJqi+Rjj9Z67GOeiGb27gPOVjg/ChApcrpW9b4O6XTV6ybBnJXh9ajevJQiC0GnK5XJY\n",
"LpeHrtlp53O8yVP+VafKA8BKoDpjy97ATbUVHS84vmax7RmYTsETtXWzplBGT+n4dKXjW5L3NwNX\n",
"AJ9sUL3bw5vw0hfa9K+ZIuTkg2LoLIJGKIbOImgE0dlJHjvyQ6+pVx757gbgJ8AxAEnIwr7ARY4X\n",
"7Oh4wXWOF1R6gfsD85N644HjgYvymLijUEavDjGbe3NW6PbwJrQ3r1fqoI5uUspaQApKWQtISSlr\n",
"ASkoZS0gJaWsBaSklLWA0XjhZfbsEQ4PAZbjBTcCFwNHJDF6k4A9ga2Tej8BjnC84GrgRswQ6Ald\n",
"E90GhXJk0cr+otLx6VVFMwC9Wb0tt9zrdLXLrx8vldbOnj17+bx585YDYT334eSXWKnO5Uatv9+M\n",
"cfs9uGrDwAG7jP9UGIYbmmx/Zp2ytvR0oz4wMwzDobzoaVC/FIZhKUd6ily/lHzfedFT9Pq5v5/l\n",
"LbZoGJwe+e4LwEfrlMe85N1J5Ls3AIc2aidXlMvlwryGBwYfHh4YnJ68f/nwwOCq4YHBN1XXWbx4\n",
"cXl4YPDm4YHBHbqtZ86CRZ+Ys2DRzXMWLPpUs+dec801Q1nfz37RWQSNRdFZBI2is7OvG88773vD\n",
"A4M3L168uJy1ll68CtXTA84ELtXKXgdsA3xG6fi6BnV7MadXcWSRsAVBEArJSD29fqRQRk/p+CxM\n",
"8GManu+mlgQxeoIgFJqxtspC0R1ZGrFe6bgX6W9a9t6kADn5EsKsBaQgzFpASsKsBaQgzFpASsKs\n",
"BaQkzFrAaIx/+plGo2V9iVUu586jtC2WLFlSfuVRR1+XxPJ1FccLdsak21kR+e47u309QRCETqOV\n",
"vQNw1d0XXjB77ty5hUga3Q792tPrxXwemMBNgKlJRnJBEISikbuk0N2kXx/UvZjPI/LdtcDTmLnR\n",
"7XtxTUEQhA4zWu7NvqJfjV6venogziyCIBQbMXp9QC+Nnqy2IAhCkZHhzT6gJ8ObCS15cBYhJx8U\n",
"Q2cRNEIxdBZBI4jOTnLfRb98E5svH9S3FCpOrwmy6OkNOF6wTdqTvvqGCW91vGCzbOV5owg6i6AR\n",
"iqGzCBqhKzpfTOboO02J/IctlBhDvb1+NXq9SDZdoWL0jkteqfjhzWsHALcrijpIEXQWQSMUQ2cR\n",
"NEJXdD7veMHHIt+9u4NtFokxM6/Xr0avlz29GzBLamzXzElbjGMtsLorijpIEXQWQSMUQ2cRNELH\n",
"dU4EtgJeA4jR63P61ej1bE4v8t37gXc1e14YhkOlUmmo84o6SxF0FkEjFENnETRCZ3U6XnAM8Gla\n",
"y6zUL4wZo9evjiy97OkJglBsxANbjF7h6eWcXquEWQtISZi1gBSEWQtISZi1gBSEWQtISdjBtroZ\n",
"axt2oc1OEzKGHFnE6GVEgwVbc0cRdBZBIxRDZxE0Qsd1tpM4fkSKcD8TjWOmp9evc3oyvCkIQlrG\n",
"7PCm4wXvB1570RYTx8zyQoUyelrZWwE/AvbCaD9Z6fiqOlVz39MTBCE3rMQEZ+/geMH4yHfHzFAf\n",
"cCww/caX779mlzoHHS/Y7Jkb+W69Zy6OFxwJfA5zL5dGvntilzS3RdGGN4eAstLx64APAb/Ryt6p\n",
"Tj3p6QmCkIrEyD0JWMDUjOX0DMcLpgLTAf5vYO9GvdwhoBz57sZnruMFmz1zHS+YBZwJvA14LbC/\n",
"4wWf7obudimM0dPKHof5VfJTAKXj+4DbgA/Xqd7LNGSCIBSfsTjEuUflzYM7vmyz+cxkubSNz9zI\n",
"d0d65h4LXBH57srId8vAz2giWUcvKYzRA16B+YOsDh69EzigTt3c9/SKkJMPiqGzCBqhGDqLoBG6\n",
"orMrHpw5v597AWw1nu3Wjt9yfJ3jzTxzD6ipdxewj+MFEzuktWMUyejNSLZPVZU9BdQb3izCnF4p\n",
"awEpKWUtIAWlrAWkpJS1gBSUshaQklKH2+uWB2epw+11kr0ApmxllcGql3C6mWfujJp6qzDDxbkL\n",
"+C+S0atQ++XUW96+CEZPEIT8MBaHN/cE2G/GuNvKI6+ykOaZW6/eSHUzo0hGb0WynVJVNgV4tE5d\n",
"MXqCIDTDmFoMOvHK3BVY76jx92/7wup6eUybeeauqFOvDDzWAbkdpUghCw9gXItfCVyflO0N/Km2\n",
"oj848L17S6XVALNnz14+b9685UBYL1A0GXMv1blet+vPrFOWpZ669YGZYRgO5UVPg/qlMAxLOdJT\n",
"5Pql5PvOi56e1H/tLuOe/fvDG6BmOK5f7+c7dxv/0B/veGry80/oF0/7xh/eZE3YYx3sXlst9TMX\n",
"uCmpR1W9ZZHv5q4DYpXLxVk7UCv7VGCG0vExWtl7ADcCeysdV36RsGTJkvLcuXNz16WuZSwm9u0W\n",
"RdAIxdBZBI3QeZ2OF8wGfgzcHvnuxzrVbl7vp+MFhwMnA1ecdsjE8peueeEb3zxoixdrn52OF5wK\n",
"zIh89xjHCzY+c4ENwKXAYZHvPul4wT7AVcAszHzeVcDvI989t4cfKxVFGt4EEzNiaWXfCFwMHFFt\n",
"8ApGmLWAlIRZC0hBmLWAlIRZC0hBmLWAlIQdbq8yvNlpx4uww+11ir2S7b1AGPluozRkQ4DleMHG\n",
"Z27kuyuASZg5wUkAke/eAZwI/AWIgFvzaPCgYD29NBSlpycIQn5wvGAbYCkmxvdNSaxZYXG8YAKw\n",
"Q72fF4oAAAydSURBVM1ravKaBrwueX9c5Ls3w9h5dhZpTk8QBKFbrMEYvK2ArcnZQrqOF2xBfSM2\n",
"JdnWlm2botk1jMFFc8XoCYIw5ol8t+x4wePAIKYn1FWjlxixydQ3ZPXKtmvyEuswc2srMSnWVmHC\n",
"Mp5MtiuB+yLffbbdz1I0xOgJgiAYnsAYvR2Bf6Y9Kck6MhnTw6rejlSWpidWzQaMwar3WsmmBu5J\n",
"4JmiD9F2CzF6giAIhkqA+u6OFzxHY8NVvZ2CGRJtlvXA04xuvCrlz0S+O2aW/+kmYvQyolFcWd4o\n",
"gs4iaIRi6CyCRuiazooH5xebPK8ylLgKk4pr43bvHccN3Pn4hqjOsWfzYsSK8p13CjF62VEiv+7M\n",
"1ZTIv84S+dcIxdBZIv8aoTs6lwJzMUas2kBtYsjqbNc0GkpM4vTqBXPniRLF+M47ghi9jLj88stn\n",
"lkqlrGWMShF0FkEjFENnETRCd3RGvvs3wO1km0W4n0XQ2EmKFpzeN9xyyy0zs9aQhiLoLIJGKIbO\n",
"ImgE0dlJiqCxk4jREwRBEMYMYvQEQRCEMUNfpiHLWoMgCEIRGQtpyPrO6AmCIAhCI2R4UxAEQRgz\n",
"iNETBEEQxgx9Y/Qsy9rKsqwLLMu60bKsmyzLemsGGrawLOuzlmVdY1lWaFnWDZZlvbnq+H6Jvust\n",
"y/qjZVlTa87/gmVZNyevE3ukeQ/Lsl60LOugPOq0LOtjyX281rKs2ys6c6bxVcl3fp1lWbdZlvXV\n",
"qmOZ6bQsa7ZlWfdblvWRmvKWNVmWNTU557qkjdd0Q6dlWTMty/qZZVlLkutcbFnWjjXnZa6z5rhv\n",
"WdaDdcp7pnOE73yyZVk/tSzrr8nf6JWWZY3PQmPmlMvlvngBpwE/T97vgcmjt1OPNcwE/gFsl+y/\n",
"BXgG2AWYADwEHJwcGwIWVp37duCupN7E5P07e6D518CzwEHJfm50Ah8ALgfGJ/tHJWW50Zhc7+/A\n",
"15P3U5O/vUOz1AkcBvwGuAU4qqq8LU1Jm17yvoRJzDyhCzqHgDOr9n8J/CFvOquOzwA08I+a8p7p\n",
"HEkjcAVwRPLeAn4PbJHFvcz6lbmAjnwI02N9DHhjVdli4PM91jEVOLKm7DHg/cC7gQeqygcx6Y6m\n",
"JfuXAl+tOv5V4I9d1nsgcB7wIC8ZvdzoBP4f8Lo65bnRmLT/bM1DIsLkb8xMJ2An22vY1Ji0rCn5\n",
"+14HDFYdvx84vAs65wEvr9p/F/BM1X4udFYdPxv4CvBgTXnPdI5wL2cD945wXk/vZdavfhnefAVm\n",
"DazqBRHvBA7opYhyubyyXC5fVNm3LMvC/HpagTEwd1fVHcYs4rh/UnQAm+q/i+7rHwL8mrIDk2sD\n",
"2em0LGsnYB9gz2SI61rLso7Lk8YqrsD8ysayrN0wIw2Ls9RZLpfjBofa0bQ/8EJyToW2/s8a6SyX\n",
"y5eXy+XqocKtMD8gK+RCJ4BlWbtjRnOur3O4ZzpH0DgXeMCyrDOS4c3LLcvaJwuNeaBfjN6MZPtU\n",
"VdlTwE4ZaKnmYGA5cB1G41M1x1fxksba49XHOo5lWe8AlpXLZV1zaAZmyZNqstA5M9l+EJMP8X3A\n",
"f1mW9cHkennQWOFjGON8P6aX9x/lcvlW8nMvq2lH02h/w93kXcB/V+3nSacPLMAMG9aSB50zMYbv\n",
"jnK5/AbMUPE1lmVtkyONPaNfjF6F2qDDzAItLcvaCvgmcHTZjAlUXptVrXrfk6BJy7LGYYbfTq1z\n",
"OC86JybbH5fL5XXlcvkx4FfAMSNo6Pm9TPgDcEO5XN4deA1wmmUcmPJyL6tpV1PP/8csy3o98HLg\n",
"e6NoGelYV3RalnUAsLZcLt/RhJaRjnVD50TM0PAFAOVyeSHwAmYIuZEORjhW6AD2fjF6K5LtlKqy\n",
"KcCjGWipDGueB3ynXC7flhSvYFN9JPsrGhyfwqbDOZ3kQ0BQLpdXVZVV/pDzovPJZFv9HT6MmYPK\n",
"i0Ysy3oV5lf092DjcOFlwKfypLOKdjStwCye2ujcjmNZ1q6YH2fvL5fL66sO5UWnj5kmaEQedD7J\n",
"S2sFVvgX5n8pLxp7Rr8YvQcwqwu/sqpsb+CmbORwFhCVy+VLLMuaYFmWnWjZq1IhKdsa42lFcrxW\n",
"/9+7pO+NwLsSN/trgJ2B71mWdWmOdN6HmWuqHkbZEeMhlxeNYOZsAV6sKluH+XWdJ50V2tF0K7CV\n",
"ZVmDVcdfRZf+z5IQhQuBj5TL5ccsy7Ity6rc78x1Wpa1HbAbcEHyf/RdYOfk/+qTedEJ3Ib536lm\n",
"R4zhy4vG3pG1J02nXphfgz9L3u+B+WXT05CF5Nr/BXwf2AbYFvMH5GEejst5yUvSA35bdZ4L3IF5\n",
"WG6FmSx+R480V3tvTsyLTuAc4JLk/SSMN+eRebqXyTVi4FPJ/nYYg318HnRiFgf9SNV+W5owIS4L\n",
"kvcHY8If2nZfr6NzW8yirgcn77cFvgXsmiedNcdKbO692XOdDe7lo8C7k/2DMD217bO8l1m9MhfQ\n",
"sQ9ivrCfAzdifoW8JQMNewIb6rwqfzD7ATdgHFsuA3aoOf/zwM3J6ws90LsvJl7nmeQf5YQ86cT0\n",
"QC7E/FK9ATix6lguNCbXmo15QF+L+WV8KjAuS53ALOB3ycPt78B5nbh3wA7JOdcBfwX267DOHyfl\n",
"36zzf7QeeFmedFYd/2zyP/RMUm/fXusc5Ts/EPgbxsP0GmB2Vt951i9JOC0IgiCMGfplTk8QBEEQ\n",
"RkWMniAIgjBmEKMnCIIgjBnE6AmCIAhjBjF6giAIwphBjJ4gCIIwZhCjJwiCIIwZxOgJgiAIYwYx\n",
"eoIgCMKYYYusBQhCEZk/f/6bgekLFy78bY+v+w5MDsq9gYsXLlz40VHqnwJcsHDhwoaLoArCWEJ6\n",
"eoLQJPPnzz8es6Dpdb2+9sKFC69cuHDhazAZ8tPkELwNuHH+/Pmzu6tMEIqBGD1BaIL58+cfAnwH\n",
"eP/ChQv/NVr9rFm4cOGfMGv9/XH+/Pm166IJwphDjJ4gNMcZwGULFy4caaXsvHE2ZhWSk7IWIghZ\n",
"I3N6gpCS+fPn7wP/f3t3EyplFcdx/DuRSF0XgbkwokCNMsRC6xpBtwgEKQjc/ClwERRu7GUh2NsN\n",
"5G6KoItQGyMJghb+BJGsuNVC24UaRhC3NoHUIvOaXLAUqpkW50x3enjmzkxOyNzz+2wOc57z8szA\n",
"8Oe8PM9hE7C/kv+vdTbgS9K5f2tJBxw/SzrkeJp0/NQK4C1J+3L9p4BnSEc9vU4KUBMsnBL/qqQP\n",
"a26pERE7gR3ALaQpz12STncWknQ5Io4DTwKTV/IbmI06j/SsOBHxfES8FBGHImJ9RLwSEZMR8WaP\n",
"qg/k9PvOzMo621bgkqQJUtBbDhwhBcWnJd0F7AGmI+L+XP9Arg8pQJ6QdK+k1cAMcDgiHqncSyP3\n",
"1cx9rSEdFHowIur+17PATRFxW4/vaLakOehZUSLiOWBG0mvAZ6SDP98jBacdPaqvy+kvXa43gAuS\n",
"3oc0wiId6nkrcETSfC53GPiddGJ11YnKjtBJ0ijxjZqy85LezX01SSdcr8v9VZ2tfAezIjnoWWmu\n",
"ldQeqd0MnM4bUvazMJLrpr0R5PIiZWYrn89X8yW1cv7qmvpfd36Q9AdpB+adEbGqUra6rjiX07p2\n",
"L+XUm1msaF7Ts6JImu74OAF8mvN/6qN6M6eNLtdbwMWaPGrym9T//+Zr8tqB80bg3CJ9te+vrt1r\n",
"KmXMiuSRnhUpIpYDW4AvBqj2a06vH/4d/eOGmryVpCA3V3OtX2M5Pb9oKbMlzkHPihERy/KbVADu\n",
"I43YTuZrqyKi15pee4qybvpwWO7u/JCD82bgW0nn6qv0pX3P1elXs6I46FlJdgKfRMR1wGPAnKQ/\n",
"87VdwEc96h8jjbg2dLneoPvUZzW/W9k7IuJxgIhoAFOkdbgXrqAvgI3A7Cg8UG/2f2q0Wv28ychs\n",
"9EXEBuBl0rNzM6Tdk2OkTR6S9E0fbXwMjEl6qCNvnLQRZj3wG3AGuAc4ADxM2jDzHenVZadqym6W\n",
"1IqIJinIAWwj7cI8S3pO72i3viRtiogp4AnSows/AB9I2pvrrMz9vCjp7YF+NLMlxkHPbAARcTvw\n",
"FbBd0udDbrsJ7JU01bPwYO3uI+1MHZf01zDbNhs1nt40G0B+3GE78M4ovMQ5InYDDwKPOuCZOeiZ\n",
"DSyP8CZY2BE5TN3W6f6rM8AWST8PuV2zkeTpTbOrrOPdmxtJb3v5UdL41b0rs6XJQc/MzIrh6U0z\n",
"MyuGg56ZmRXDQc/MzIrhoGdmZsVw0DMzs2I46JmZWTEc9MzMrBgOemZmVoy/AQZjib3bEzqAAAAA\n",
"AElFTkSuQmCC\n"
],
"text/plain": [
"<matplotlib.figure.Figure at 0x7f0b97d50450>"
]
},
"metadata": {},
"output_type": "display_data"
}
],
"source": [
"# plot v(x), u(x)\n",
"fig,ax = plt.subplots()\n",
"c1 = \"#e41a1c\"\n",
"c2 = \"#377eb8\"\n",
"ax.plot( xs, tracev*mphfact, lw=2, alpha=0.8, c=c1 );\n",
"ax.set_xlabel(r\"$x$ (mph)\");\n",
"for tl in ax.get_yticklabels():\n",
" tl.set_color(c1)\n",
"ax.set_ylabel(r\"$v(x)$ (m/s)\", color=c1);\n",
"ax2 = ax.twinx()\n",
"ax2.plot( xs, traceu, lw=2, alpha=0.8, c=c2 );\n",
"for tl in ax2.get_yticklabels():\n",
" tl.set_color(c2)\n",
"ax2.set_ylabel(r\"$u(x)$ (m/s$^2$)\", color=c2);\n",
"ax.set_xlim((0,1609));\n",
"ax2.grid('off');\n",
"align_yaxis(ax,0,ax2,0);"
]
},
{
"cell_type": "code",
"execution_count": 236,
"metadata": {
"collapsed": false
},
"outputs": [
{
"data": {
"image/png": [
"iVBORw0KGgoAAAANSUhEUgAAAcUAAAEdCAYAAAB9rHzGAAAABHNCSVQICAgIfAhkiAAAAAlwSFlz\n",
"AAALEgAACxIB0t1+/AAAIABJREFUeJzsnXe4HUX5+D83pFETaoTJYqhC4CclYKEejCiCYRHpLkiT\n",
"xVBEECkKhI4QQlEDi/AVcCMgdVGaJHAoSg1EDQQEJLgZhISEkEAgIeH+/njnhM3JLafvnnPm8zz3\n",
"OffMzM6+c8t59515S0dnZycWi8VisVigT9oCWCwWi8WSFaxStFgsFovFYJWixWKxWCwGqxQtFovF\n",
"YjFYpWixWCwWi8EqRYvFYrFYDH3TFqArtHI2AV4GRiodP27atgauAZYAc4DDlY7npCelxWKxtDaB\n",
"G44AbgPO9yPvpkT7UcDxwDygP/AzP/L+lug/BTjYvL3Vj7yxib41gBuB1REdNNqPvBfrvJSSyaql\n",
"eB6wsPBGK6c/EAGnKx3vBLwAXJuSbBaLxdLyBG64N3Aq8AHQmWjfEvn8PdCPvF2B64C7E/17AEcD\n",
"OwA7AkcFbrhnYurxwGQ/8nYGzgCiwA3713k5JZM5paiVsz0wH5iVaP4OsFjp+DHz/npgX62cNRst\n",
"n8VisbQJL/qRdxBiDSbZHJjrR96/zfungbUCN1zHvPeBCX7kLfIjbyEwATgWllqJ+wE3APiRlwcW\n",
"Ad+t50LKIXNKERgDnFvUtj0wrfBG6XgGsADYtnFiWSwWS/vgR17cTdeTwAqBG37dvB+FWH4zzfvt\n",
"gFcS46eZNpDP7IV+5M1I9L+c6E+dTClFrZzvAFOVjnVR1xCWf1qZC6yDxWKxWBqGH3n/Q6y9OwM3\n",
"fAX4AbBvYsgQZMu1QPKzurivuD91MqMUtXL6AD8HLu6iu5PEnnaCjroKZbFYLJZlCNxwM+AW4Ft+\n",
"5G0G/Bp4KHDDfolhPSXVLu7L1Od4lrxPDwEeUjqem2gr/LBmAtsUjR9s2pdh0qRJNsO5xWKxVMDI\n",
"kSNLUVBHAM/5kTcVwI+86wM3vBzYHbgf+VwenBg/mM99RGYCg4rmGwz8qxq5a0mWlOJOwJZaOd82\n",
"778AXKmVMx1x3z2oMFArxwFWAiZ3NVGJv9iWoqOjY0xnZ+eYtOVoJO24ZmjPdbfjmqGx6y7DoOgH\n",
"LC5qWwwMMN8/B2yW6BsOPGu+fwEYGLjh0MS54ubA/5UvcX3IzPap0vGxSsc7KR3vpnS8G/AO8BOl\n",
"4+8BDwJ9tXJ2McOPBO5UOp6dlrxZY9NNNx2WtgyNph3XDO257nZcM2Rm3R0su8U5Edg1cMMvAJhw\n",
"i/7AU6b/WuDgwA0HBG44EIlXvBbAj7zZwO3IZziBG+6KKNn7GrCOksiMUiyglbOVVs4dwFrAeVo5\n",
"P1E6XgjsA1yilfME4sF0bJpyZo111113WNoyNJp2XDO057rbcc2Q7roDN9wycMPbEUvvuMANrwPw\n",
"I+9+JELg/sANHwPOAb7vR947pv8hJOTib4in6g1+5D2QmHo0MCJwwyeAiwDXj7xFjVpXb2Rp+xQA\n",
"peN/IJ5Nxe1TkGBQSxeMGDFietoyNJp2XDO057rbcc2Q7rrNmeH+3fRdCVzZw7XjgHHd9L0PuLWQ\n",
"sR5kzlK0VMaoUaOmpy1Do2nHNUN7rrsd1wztu+40sUrRYrFYLBaDVYoWi8VisRisUmwd8mkLkAL5\n",
"tAVIiXzaAqRAPm0BUiKftgDtRkdnZ2vFuk+aNKmzHeMULRaLpRrsZ6dgLUWLxWKxWAxWKVosFovF\n",
"YrBK0WKxWCwWg1WKFovFYrEYrFJsEfL5fC5tGRpNO64Z2nPd7bhmaN91p4lViq1DLm0BUiCXtgAp\n",
"kUtbgBTIpS1ASuTSFqDdyFzu03ZCK0cBK9ZirhV/eeYa+geHblyLuZqFdlwztOe6q1jz/5SOP6q5\n",
"QJaWxSrFlNDK2Qf4Za3mGzL+2vVos4Tp7bhmaM91V7Hmt7VyDlE6/rDWMllaE6sUU0ArZz3gZPP2\n",
"TWBJtXMuGTxoYL85c16vdp5moh3XDO257grXvBZQ+F87r/ZSWVoRqxQbjFZOB3AWsBIwUen49FrM\n",
"m8/nx+RyuTG1mKtZaMc1Q3uuu5I1a+VsAEwA9tbKeVTp+Im6CNfCBG44ArgNON+PvJsS7YOQ0lCb\n",
"IZ9l7wDf9SNviek/BSkuDHCrH3ljE9euAdwIrI7ooNF+5L1Y/9WUhnW0aTz7AdsDc4FLazhvvoZz\n",
"NQv5tAVIiXzaAqRAvtwLlI7fBMabt7/QyhlUU4kaQz6tGwduuDdwKvABUJwP9I/Aw37k7YgUff8I\n",
"6DDX7QEcjWx37wgcFbjhnolrxwOT/cjbGTgDiAI37F/PtZSDVYoNRCtnfeAn5u1FSsdzajV3LpfL\n",
"12quZqEd1wztue4q1nwLMAXZSj21ZgI1iJR/1y/6kXcQMC/ZaKzHTfzIuxXAj7xOP/L28yNvsRni\n",
"AxP8yFvkR95CxFo/1ly7BmIY3GCuzQOLgO82YD0lkbntU62cE5CqzB3A2sB1Sse/MX3vANMSw59U\n",
"Oj6r8VKWj1bOCsAYYCDwgNLxI+lKZLG0PkrHS7RyzkWU4x5aOY/Y/73lCdyw38YnrrtMmx95cTfD\n",
"RwJvBG54GWINzgFO9yPvJdO/HaIIC0wDRpvvtwUW+pE3I9H/srnmrqoWUSMypxSBo4BvKh2/p5Wz\n",
"MTBNK+dppePnEWVyRMryVcphwJeBmcBlKctisbQNSsexVs7VwM+BM7RyXlQ6fj9tudIicMN+wIbA\n",
"cORMcDhQTrjLMEQx3uZH3qmBGx4APBq44QZ+5H0EDEG2XAvMBdYx3xf3FfenThaVoqd0/B6A0vHr\n",
"WjnvI7+E51OVqgq0cjZFthQAzlM6ntfTeIvFUnPuAL6BWCSna+WcrnTcWnXzesCc2Y0E9gH+H1DN\n",
"Gd4AYL4feTcC+JH3p8ANLwdGAbeaMT39bIv7MlWuKnNKUel4auF7rZzvA/OBv5qm4Vo59wOrIib3\n",
"mUrHsxsvZelo5fRH3MH7ArcrHT+dskgWS9uhdPyZVs55yIf2SODbwIPpSlV/AjdUwL7A3oi3Z4H/\n",
"xh+89dFLM/+58n/nTp/179nT3rvnxLtHlDjt+8B7RW1vA0PN9zOBwYm+wcCsRF+xw9Ng4F8l3rvu\n",
"ZNLRRitnS62cl4CxwMEJy2oqcIDS8c6ICf5AWjKWwTHI1kQMXF2vm7RjjsR2XDO057prsWal47eR\n",
"MAIQa/EL1c5ZbypZd+CGfQI33DFwwyuBe4AfIgrxVeBCYDc/8va9IP+LQ+9++bZ9J7/9jD9/4bxf\n",
"lHGLFxHHpSRrIYoR4DlkW7bAcOBZ8/0LwMDADYcm+jc312SCTCpFpeOpSsdbAN8DIq2c7U37UYnM\n",
"FGOAbQt9WUQrZyvkLPEzYIzS8cd1vF2ujnNnlVzaAqRELm0BUiBXo3ki4HFgFeBc4wCXZXKlDgzc\n",
"cGVzvncHcBWwE7AYuA84EvD8yLvbj7z5ZcrQwbJbnBGwOHBD19x3F2T37i+m/1rg4MANBwRuOBCJ\n",
"V7wWwI+82cDtRh4CN9wV6GdkzAQdnZ3Z3lbXyrkeGKB0fGgXfTOB0UrHdxTaJk2a1Hn++ec/Vng/\n",
"YsSI6aNGjZoO5LtybzZPYrkubl3VeK2cvsDtS1Zccfj8Hb7+xqwjj3i2p/E1kCfHsjFNDV1vSuNz\n",
"fL7mLMjTqPE5YEyG5GnE+GG5XO7wWszfd+bM76jLxu7X8cnCFefv8PVnZh980D8aIH9F44uTFnQ1\n",
"/pOZi1admZ874GO9aEtE2YME09++/oFrz15xvQHblSLPpEmTOkeOHLlU+QVuuCVwDrArMB2Y4kfe\n",
"MaZve+A3wKfm62d+5E1OXHsycIh5e4sfeZcn+lZHgvfXQAyz4/zIm9KFjKmQKaWolbMGkFM6vivR\n",
"djWyV/1rYJ7S8WTT3h8JGN1V6fjvhfHFv9i00MrZBdmq+S9wkNLxonrez2Y5aR/acd21XrNWzo6I\n",
"NbUY+KHS8au1mruWdLfuwA07kPCGg4Fd+HzX70Uk/OSxQnaZUsnKZ2faZM3RZjXgLK2cB5SOP9bK\n",
"WRuJWfwVsD6wM5IpAeAE4HXgmVQk7Z1CBod76q0QLRZLeSgd/00r53Zgf+ACrZxDlY4/SVuu3jDh\n",
"FN8CfgBsapo/RfwrbvEj75W0ZGsVsqYU/wf8GZiklbMI8Uq6AbgGsRZ30cp5AtnfngeMUjquOpl2\n",
"rdHKWQV5eusEHkpZHIvF0jVXISkXN0AesjMbPxy44cpIOMUhSKwfSND8HcCd5qzOUgMypRSVjhcC\n",
"Z5uvYmIksL8Z+AYSB/S80vG7DbpnvkH3yRL5tAVIiXzaAqRAvtYTKh1/opXzS+R860CtnL8lj2Ky\n",
"wIIZC18I3HA0YtGuapr/A4TAg37k2V2oGpOpM8VakIV9ca2cABiBBOrfm6YsFoulZ7RyfohYirOR\n",
"ELCa5SSulMANHcBDAuILgfZTgJuBJ/3I+6zW98zCZ2cWyJSl2AqY2KcRwELA5li0WLJPiOTwHAGc\n",
"rZXz07Sy3QRuuAkSrvBNPg+DeAy42Y+8f6QhU7thlWLt+bZ5fdxW+7ZYso9JGn4OUg5pJ8Sj84+N\n",
"lCFww80QJ8KcaSrEF4Z+5L3ZSFnaHasUa4gpIFzwOm2GbDsWiwVQOn7HpIEbC5yolTNF6fjlet/X\n",
"xAIejShjkB2mexDLsFH+CJYEVinWlk2BjZCs70+lLIvFYikDpeO8Vs5twIHARVo5Xr12ewI33BpR\n",
"hl8zTZ8gnqR/sJ6k6ZLJNG9NTMFK/KvS8aeNvLHNh9k+tOO6G7jmq5EcoUOBM83uT80I3HDLwA1/\n",
"C1yPKMQFiPfrKD/yrixWiO34u04baynWCJNDsXCemMbWaY72c9XP0X5rhvZcd44GrFnpeKFWzpmI\n",
"8823kETVd1c7b+CGmyLV53cxTR8hmWdu8SOvuL5gkhzt97tOFasUa8eWSKb4GUg1D4vF0oQoHb+l\n",
"lXMxUvLtZ1o5/1Q6fqOSuQI3HIZUyvmWafoEKV/1h16UoSUlrFKsHYVqHU+1U/FSi6UVUTq+31Tg\n",
"GQVcrJVzWDlp4AI3XA/4EbAXckz1KXJmeKM9M8w2VinWjoJSzExdMIvFUhWXIlXqNwROBc7v7YLA\n",
"DVdD4gwPREoiLUa8SW+w3qTNgVWKNUArZyDwZSTX6eRehlsslibAFCU4A3GEcbVy/tFdhqrADfsj\n",
"qdiOQgobADwIXOtH3oxGyGupDVYp1oatkKfCV5SO0zonyKd03zTJpy1ASuTTFiAF8mncVOn4Na2c\n",
"XyH5mE/Tynk1WWYqcMM+wO7AccB6pvl54Co/8qbVQIR8DeaomMANRwC3Aef7kXdTF/3nAof5kbdB\n",
"UfspSBIEgFv9yBub6FsDedBYHdFBo/3Ie7E+KygfG5JRG1LfOu2qgGmr045rhvZcd5prNtZhBAwA\n",
"fqWVsyosVRg3AhciCvE/wEnAj2ukEFNdd+CGeyPbxh8gu2DF/UOQWMvOovY9TPsOwI7AUYEb7pkY\n",
"Mh6Y7EfezsAZQGQs7UxglWJtSF0pWiyWunIp8Aow9P2Vhl4R7P2HK4AAGA68B1wAHOxH3pN+5LWK\n",
"o92LfuQdhJTp64qzEAVXHMvpAxP8yFvkR95CYAISjlKwEvdDSgLiR14eWAR8t+bSV4jdPq0S89S4\n",
"OXKgPiVlcSwWSx1QOl74/BbfPveDldT97w7e/KD+iz+atajfKjOAmxAF8HHaMtYaP/Li7voCN9wY\n",
"WBe4nc8LvxfYDlGEBaYBo8332wILi85ZXzbX3FWtzLXAKsXq2RaxuP+pdLwgbWEsFkttMeeG32Xj\n",
"Q4/vu+STJSsumtsxdPbkvmvNe+PcbaY9MjFt+VLiXOScde0u+oYgW64F5gLrdNNX3J86dvu0euzW\n",
"qcXSopiE3b9HFMAai1cY+OSX9F8vGz7jwXfXmffaaVo5mfkwbxSBG24HLPIj76UehvW0hVzcl6ka\n",
"jplTilo5J2jlTNTKmaSV80+tnOMTfVtr5TyllfOkVs69WjlrpCmrIRNKsR1zJLbjmqE9193oNQdu\n",
"uGbghucgjjRbIOeGZwE/+sLcaecDzyDek5do5fSrlxz1XHdHR0euo6NjTOGrjEvPBXoaPxMYnHg/\n",
"GJiV6BtUNH6wac8EmVOKSJzPQUrHI4F9gSu0crbTyumPeICdrnS8E/ACcG2KcqKVsyZSFeMT0k/t\n",
"lkv5/mmQS1uAlMilLUAK5Bpxk8AN+wZu+APkfGsUkonmRuD7fuQ94Edep9LxEuCXwLtIfPLJdRQp\n",
"V6+JOzs7852dnWMKX6VcE7jhqshn3o2BGz4KXAF8IXDDRwM39M2w54DNEpcNB541378ADAzccGii\n",
"f3MytNOWxTNFT+n4PQCl49e1ct4HhgEKWKx0/JgZdz0wXStnTaXjtNImbWdepygdL0pJBovFUgMC\n",
"N/wycCawsWl6ArjCj7z/Fo9VOn5fK+c04HfA/lo5/1Y6rjpxeEbpMF/4kTefhMIL3DAH/N6PvN0S\n",
"468FxgVueJm57mDgFHP97MANb0ey/pwXuOGuSIz3fQ1YR0lkzlJUOl5qcWnlfB+YD/wV2aaclhg3\n",
"Aym7sm2jZUzwFfOamacci8VSHoEbrha44RlImMDGgAZ+4kfeT7tSiAXMZ9WF5u1pWjlb1V/axmHK\n",
"XN2OWHrHBW4YFPWfhGyjrhW44e2BG24F4EfeQ8jP8m/Ak0iKu2TloNHAiMANnwAuAlw/8jJjVGTR\n",
"UkQrZ0ski8JKwAFKx/O0coawfLxM2l5LBUvRKkWLpckI3LADqV5xMrAmElb1B+RDvKTk30rHf9HK\n",
"2RQ4BLjUJA5viRynfuRNRVLXddd/JXBlN33jgHHd9L0PuLWQsR5kUimaJ7AttHK2Bh7Qytkb8Vjq\n",
"yqMpFc8lrRyFbOnOR4qSWiyWJsGcaZ0BfNU0TQEu9iOvkhJRVyEW5leAsVo5RysdL6yNpJZGk0ml\n",
"WEDpeIpWzn3AicBbwDZFQ7r0WkqmRhoxYsT0UaNGTQfyXaVMMt5duS5u3+P41Q884EuDHp643qL1\n",
"nelvn/qzs17L52s6fwXjh+Xz+TF1nD+L45NrzoI8jRo/TP7cMiNPI8bP7aKt7PknPfTIyNlPzzux\n",
"32orbNP5GX07+nYsHLTFSs+svu2qE77xzd2WU4ilzK90vMQkDr95yYorfuXjLbf4a/6RRx6lT58u\n",
"x5c5/zLX1Pvnb4GOzs7sZCQyIRY5peO7Em1XA0ORzBGXKx1vbNod4E1gSNLRZtKkSZ0jR46su/Wo\n",
"lTMGSU00Vun41nrfz2KxVEfghpsC5wBfMk33A1f6kTenFvNr5WyMxDSuCFyldPyHWszbKBr12Zl1\n",
"suZosxpwllbOigBaOWsje88TgQeAvlo5u5ixRwJ3puh5uoV5/VdK97dYLCVgwix+BNyMKMS3kcoM\n",
"Z9dKIYJ4yyNKF+AErZyv12puS+PImlL8H/BnYJJWTh54GLhB6Xi8CXnYBwmWfQLxOj02DSG1clZB\n",
"wkQ+BV5LQwaLxdI7gRtugsQZ+shx0R3AQX7kPdvTdZWidPwIcB3y2XqRVs4X63EfS/3I1JmiOZw+\n",
"23x11T8FKUeSNsMRB59XbXyixZI9AjfsCxyOJKvuizxwn+dHXiM8xa8HNgF2A67Syjlc6bjLM1FL\n",
"9siapdgsFLZOe8r9Z7FYUsBUcLgR2UnqC9yJWIcNCZ1SOv4MebB/BfGHGGsyclmaAKsUKyNzStHm\n",
"w2wf2nHdpaw5cMMVAjc8EgiRrCvvIGeHF/uR91GdRVwGpeOPgZ8iqeC2Bs7WyinbiaUdf9dpY5Vi\n",
"ZWROKWLzYbYTubQFSIFcT52BG26EWIej+dw6PLBeZ4eloHQ8C1GMC4A9gB9VME2uljJZeidTZ4rN\n",
"gMmsszYStN9tEU6LxVJ/AjdcATgMOAbJofkOcL4fec+kKphB6fjfWjlnItldjtHKmaF0fH/aclm6\n",
"x1qK5VOwEl82ZwcWiyUFAjfcEPg/4DhEId6NWIeZUIgFlI6fBC43b8/SyilOQmLJENZSLJ+CUky7\n",
"VJTF0pYY6/BQJMyiYB1e4Efe06kK1gNKx7dp5awPHIg43hyhdNxtsnFLelhLsXyyeJ5osbQFxjq8\n",
"ATgeUYj3IJ6lmVWICcYhVSMGAVdq5RQX27VkAGsploFWTh+kICZkTynm0xYgBfJpC5AS+bQFaDSf\n",
"Lfrs8cANf4hYh/0Rr84L/Mh7Kl3JSsfkSP0FUoNxU6SA+mil454qcuQbIpxlKZnKfVoL6pm/Tytn\n",
"KPJk+p7S8R71uIfFYlmWwA2HIXX7tjRNEVL898O0ZKoGrZx1kLPQLyCFjH+mdLwkXam6/uwM3HAE\n",
"UsbvfD/ybjJtw5A4zC8i5f3eBE70I++9xHWnIMWFAW71I29som8NxFN4dcQwG+1H3ot1WlbZ2O3T\n",
"8tjUvP47VSksljbAxB0eBvwRUYgzkQ/f85tVIQIoHc8ETgA+AHYGzqwkhrHeBG64N3AqImfSejoc\n",
"mONH3kg/8r4OLEFS2xWu2wPJJLQDsCNwVOCGeyauHw9M9iNvZ6R8VxS4YWaSG1ilWB6bmFeb79Ri\n",
"qSPGGrkeKRvXH7gX8Sz9e5py1Qql4zeRGMZPkKIHqeRx7oUX/cg7iOWLu08Gfpt4fxuwe+K9D0zw\n",
"I2+RH3kLgQmY9RkrcT/kXBg/8vLAIqTiUCawZ4rlYS1Fi6WOGM/SQ4AfI8pwFnJ2+LdUBasDSsf/\n",
"1Mo5HQnXOEorZ47S8W1py1XAj7wu47D9yPtzUdNA5PdUYDtEERaYhiRVACnksNCPvBmJ/pfNNXeR\n",
"AaylWB7WUrRY6kTghl9EnFB+gijEvyDWYcspxAImhvFC8/ZnWjm79zQ+o3yXZS3HIciWa4G5wDrd\n",
"9BX3p461FEtEK2dVYD3E1H8rZXGWo7tK7K1MO64ZWm/dxjo8CLEmBgDvIdbhk4UxrbbmJErH95oC\n",
"68cD52nlvK90/Dxkf92BG+4AbAAcVdTVkwdncV9F56mmLNjGSIjLZ8BsYJofeW9XMl8BqxRLp2Al\n",
"vpEFT7EuyNF+7ts52m/N0ELrNtbhWUjSbID7gMv9yCs+x8rRImvuhpuAtZCHg8u1co5ROn6VOq67\n",
"o6MjRyK36sSJE8u63vzuLgb28yMv+Zk4ExiceD+Yz7dXZyJKjKL+koq1B244CHH+ORwxUroaMw24\n",
"BrimSK6SsEqxdApK0Z4nWixVYqzDA5EUbQXr8CI/8h5PVbCUUDru1MoZB6wBfAu4WivnaCb8oW73\n",
"7OzszJNQuJMmTTqn1GsDN1wLuBn4oR95swI3dIB3/chbBDyHVCkpMBwoJGZ/ARgYuOHQxLni5kiI\n",
"Sm/3HIE4XA0wck9D4lU/Ro4CVwbWBb4MXAIcEbjhXn7kvVPqusAqxXKwTjYWSw0wH6DnsKx1OM6P\n",
"vOKzprZC6fgzrZwxiCX1VeCa/jP0tHSlAmR7c+kWZ+CGqyBVSM4G3jPvfwwEyNHStcC4wA0vM9cd\n",
"DJwC4Efe7MANbweOBM4L3HBXJDPRfT0JYP5mIuAK4Dd+5PWU8IDADddEdiAeCNzwq0ZZl0SmlKJW\n",
"Tl9kX91Ffpj9gV8qHT9i+t9Bng4KPKl0fFaDxLNONhZLFQRu2IfPrcOBtLl12BVKx4u0ck4FfgN8\n",
"ecj4azbUp51+idLx7EbLErjhlsjDy3DguMANd/Ajz0diC3cGHk0M70SUIX7kPRS44Q1AwUHqBj/y\n",
"HkiMHQ3cGLjhE4iF55agtA4HjvUj7y+lyO5H3mzgJJNEYG/gjlKug4wpRaRK9YnAVkrH87VyvglE\n",
"WjmbKh3/D3hA6fiIRgullbMCsJF5a5WixVIm5kn/bKBQIeIBYGy7W4ddoXS8QCvnJ8C1K8yfvxvw\n",
"W60cX+m4oT8rP/KmAvt30f4L4Be9XDsOyfXaVd/7iOFTjiznlzM+cd3lvY9alqwpxXnAWUrH8wGU\n",
"jidq5XyCZEa4M0W51kf2sf9XkC2D5NMWIAXyaQuQEvm0BSgVYx0egOwADUQ8BC82QdvlUO74psYY\n",
"BSfQ2Xk34mF5lVbOcUrHH6UtW6uT6dynJvXRXGCU0vHjWjnPIP9UqyIBn2cWbyvUI/epVs63gIuA\n",
"x5SOT6nl3BZLqxK44VDEOtzWND0EXGqtw9IxRc1/h3haTgZOVDpeWI971TNvdK0J3HArYB/gKT/y\n",
"/hq44drA7cBWyN/ZkX7kLahk7ootRa2cLmNElI6rihEpYldgutJx4cxhKvATpeMPtXIuRbZgvlLD\n",
"+3VHwcnGbp1aLL1grMP9kfyeA4E5iHX4aI8XWpZD6fhdrZzRSMq7EUi4xin1UoxNxPGI1+oj5v1Y\n",
"5JzzfmB75Cz0tEomLiujjVbOIK2cC7RyZgCvIh5DfwRuBR4GZmjlvKSVc7w5h6sYrZyBSKaHwwtt\n",
"SsdHKR0XEgGPAbbVytm+mvuUSOE88fUG3MtiaVoCN1RIjNipiEJ8CDjAKsTKUTqegTinvA98DbhU\n",
"KyczCbRTYntgLz/yngjccGUkn+pv/MgbhRy3VZxLtWRLUSun7BgRrZy9lI7LihEx9+pAsq6PUzru\n",
"sqSIOYyeg5QveS7Zl8wAMWLEiOmjRo2aDuS7ygyRz+dzJAJYk12J8QWl+EaJ48ud346345t6/GeL\n",
"Ox977dd6bcQ6XBH5AL/4Sz8d+hlwUj6//FRZkj/r45WO/6OVc+ziVVf9U2e/fvsvGjr0K489+ODE\n",
"zoEDl3Q1vlJ5mojPEtvw30T+5gIAP/LeDdyw5BCMYko6U9TKcYCngCuB3/RSFBOtnEKMyK7AV5WO\n",
"yxLQBLG+rnQ83jwRDUG2aucpHU82Y/oDHwG7Kh0vzZxf631xrZyVgMeBT4GdlY4X12pui6UVMNbh\n",
"WUhSZ4C/ImeHc9OTqjXRytkUscQHAY8Bpysdf1qLuZvsTPFfwDZ+5C0O3PAOYFM/8r6c6J/iR97W\n",
"3c/QPaVunx4OHKt0PLY3hQigdDxb6fgkIERiREpGK+c0YAXgJq2cVRBleCTiAfrjxNATkO3MZ8qZ\n",
"vwI2MK/Ts6wQzRNhW9GOa4bsrDtwwz6BG+6HHJ9sh1iHp/mRd2atFWJW1txoitetdPxvZCt1HmJ0\n",
"XGTiu9vmdh6GAAAgAElEQVSNZ4D7Ajf8HfA9EhlxAjf8AcuXuyqZkpSi0vH5SsclBU0CaOX0Mddd\n",
"rnRcctCkeQq6GFF485GFTUUCQx8BOrRyntDKeRIYiXil1jsP6dKt0zrfp1pyaQuQArm0BUiJXNoC\n",
"BG64HlIZ4XRk62oicnY4qU63zNVp3qyTK24wOVGPAz4EdgMubEPFeIZ5PQi4B1OlI3DDW4E/AHdX\n",
"OnG9fpDP87kbdsmYp6CeFHVxJvZG0CxK0WKpO4EbdgD7IuWdVkJCpi7xI6+8bNKWqlA6nqaVcxyi\n",
"DEYCK2jlnFnuUVUzEbjhH5F49fv9yJsFfLt4jCmKfFA196kmJGMHI9S6RfN0IM4vrcLG5tUqRUtb\n",
"E7jhusjZYSEMahLwKz/y5qQnVfuidPySUYy/QSzKy7Ryft7C4RorI1YggRs+hFiD99Z6q74ipaiV\n",
"cxKSwmcBEp/4WaK7AxG+VbCWoqWtMdbh94CT+Nw6vBR42I+87Gb/aAOMYjwWsRh3BMaZOMZefT+a\n",
"DT/yXBN+sSfy93g1cH3ghnlEQd5dbkWMrqjUUjwBCc69S+l4uX8KrZwuwyiaDa2cQUiNs0+A/6Us\n",
"jsXScAI3/AJiHX7VND2CWIcNT1Bt6Rql41eNYhyP/J6u0sr5qdJxRRldsowfeR8hmWtuD9ywPxKO\n",
"8T0kbv03gRs+DdyFKMj/VHKPSpXih0rHPeUi9SucN2sstRKVjj/rcWT65NMWIAXyaQuQEvl638BY\n",
"h/sg1uHKwAfAr0jPOsyncM8skC9lkNLx61o5xyCVKkYg9Rh/0sq5Uk1ljfuB+wM39JGMNvsiRSUu\n",
"C9zwn8BdfuSdV868FeU+1cq5HTi6u6ztWjmjlY7Hlz1xDahlrI1Wzn6Id929Ssdl/WAtlmbFWIe/\n",
"RLKngJQIusRah9nHxJRfA3wByQ99otJxSWduXX12msK+twHn+5F3U6J9a3OfJUgav8OTZ8umZNPB\n",
"5u2tfuSNTfStAdwIrI4YZqP9yKvp7mLghtshCnIfP/KGl3NtWWneEpwM/EYr5/taOcO1ctZPfH0R\n",
"cRduBex5oqVtCNywI3BDF/kQ/BpiHf4S+LlViM2B0nEMHANopA7idVo561QyV+CGeyPp+j5AwuIK\n",
"7f2Rgr+n+5G3E/ACppai6d8DOBpJt7YjcFTghnsmph4PTPYjb2cktCIyc9aSySZetiyFCJUrxTWA\n",
"nZC93anA9MTXm8BmFc6bNaxStLQFgRsOAa5Czg9XRrKlHOBH3oPWmaa5MEUZjkY+tzYErjcWZLm8\n",
"aEIcigPhvwMs9iPvMfP+emBfU+0e5Phsgh95i/zIWwhMAI6FpVbifsANAKaE2CKqyFXaDZMrvbDS\n",
"M8VfAzMRj6fZJJ4iDOdWKlBWMPlXrVK0tDTm7HAUsvuzCvIBeBlglWETo3Q8y5wxXgVsiSjG45WO\n",
"S67040de3E3X9kju68K4GYEbLkBi0x9GshtNSIyfhmThwYxZ6EfejET/y+aau0qVDSBww7qEBVaq\n",
"FDcCNurO7VcrZ9dKBcoQayL5BecD76Usi8VSc4x1eCayxQViHV7sR579e28BlI4/MGWnxiKxpddp\n",
"5ZykdPyPKqcewvLW41xgnUT/ByX2FfeXROCGdQsLrHT79I1e4mDSyDxTa5Kep5l/Ym7H3JDtuGao\n",
"ft3m7HBv5OxwR+QD7mzgZ1lViPZ3XRkmLOMkxFlqVeC3WjnV1qDtZPndQRBllBzT0/XdXVcqhbDA\n",
"Vf3I+6IfeRskvoaRsGTLpVKlOEEr57Ae+p+vcN4ssaF5bZat01zaAqRALm0BUiJX6YWBG66DVLs5\n",
"G9kufRw5O7w/49ulubQFSIlctROY1G+nA39GalyeWuWUM4HBRW2DTXtX/YOBWYm+QT1cWyof+pF3\n",
"Zw9/sxWHBVa6ffo1YHetnFMRjfxhoq9V0rwNM6/TU5TBYqkJ5uxwL+BniDKcj2yrZV0ZWmqA0vES\n",
"rZwrkfPjtQE6OjpyJJTuxIklp699jkR+0cANHSTT0eREf9LZcjjwrPn+BWBg4IZDE+eKm5OoclEi\n",
"/w7ccFCipmIx2yXuWRaVKsVDgLcRc/wrLGsOt0qat/XN61upSmGxVEnghmsDv0A8xgGeBC7yI6/c\n",
"p3NLc1MI5F9RK6ejs7MzTyI5wKRJk87p5roOlt3ifAC4KnDDXfzIexwp7XdnImznWmBc4IaXmesO\n",
"Bk4B8CNvduCGt5trzgvccFegH3BfmWs5Gclgcw9dG2bHIaEfZVOpUnxZ6Xib7jpbJM1bwdq1StHS\n",
"lBjrcE/EOlwV+eC4HPiLtQ7bD6XjxVo5i4D+5qvHxOGBG24JnINYescFbrijH3nH+JG3KHDDfYDx\n",
"gRsuDd4vXOdH3kOBG94A/M003eBH3gOJqUcDNwZu+ARyhOea7DTlUAgL/EE3/RX/fVeqFM/spb+p\n",
"07xp5ayEeEN9ClSdYNZiaTTGOjwTSX0F8gF1obUO254FiEJcmV6Uoh95UxFnlq76piDB+d1dOw7x\n",
"Du2q733ALVHe7qhbWGBFSlHp+IFe+ivay80QhUDXuAFFjGtFPm0BUiCftgApke+uw1iH30Gsw9UQ\n",
"63Ac8Ocmtw7zaQuQEvkaz/cR4tiyEmLhNSsbARv5kddlFITZlq2IkpSiVs6BwBMmU0LJaOWMAFZW\n",
"On68EuFSpOm2TnO5XD5tGRpNO64Zul+3yShyJlD4QPg7Yh2+2yDR6ob9XdeMj81rs/t9vNGdQjRU\n",
"HBZYqqU4C5iklbOf0vFLpVyglbM7cB2wS6nCaOX0BY5HTOsOxMz/pdLxI6Z/uSS0Ssf1eNoZZl6b\n",
"Rila2hdjHe6BuNq3knVoqT1LnW1SlaJ6JgRueJgfeTd30/88kj2nbEpSikrHj2jljAemaOVEwEPA\n",
"q8ie7ieIAlsJSbezJVJyZjvgeyZBbakMRcp+bKV0PF8r55tApJWzKbJvHAGHKR0/ppUzBvFyOqCM\n",
"+Uul4Hn63zrMbbHUDGMdnsHnrvVPARe0gnVoqQuFGosrpSpF9XwN2D1ww5qHBZZ8pqh0/GutnKlI\n",
"XsSgh6GdwIPACKXjV8uUZx5wltLxfHPPiVo5nyBZNz4FFisdJ5PQTtfKWVPpuNYZ/Jtu+9TSXhjr\n",
"8FvAz5Fg6I+AK4DIWoeWHigoxWbfPq1bWGBZjjZKx48C22nlbIF4tW2M/EN+hmxnTgMeLdM6TM4/\n",
"h0QiWZOUuz9ikX4LeCUxdoZWTjIJbU0w9ywoxem1mtdiqRXGOjwd2M00PYPUu7Oe0pbeaBVL8WU/\n",
"8roNCwzcsOKwwEq9T18CSjpbrJJdEcX0BHAoNUgkWwJrIn8wH3RXRDmL5PP5XLs5I7Tbmo11uPsK\n",
"K/a5eMnHn3Ui1uGVwD2tbh222++6QB3W3SpKsW5hgZXmPq07WjkDgQsRZ5pCAtrektDWgmbdOs2l\n",
"LUAK5NIWoFGYOnS/Ai7qWKFjXcQ6PNCPvLtbXSEacmkLkBK5Gs9XcLRpaqVYlAygq/6KwwIrDd6v\n",
"K2YL8zpgnNJxwQyeCRSby10mkk0+WY0YMWL6qFGjpgP5rp64TBb6XOH96vt9f/NBjzy63pJBg/7V\n",
"lWzF45Ndpcxfx/G5fD4/JkPyNGJ8cs1ZkKcu49+f8uGR/Vbvu1Pnp50DO1bg0zW+stqcQcNXun23\n",
"b+y23HZpFuWv0fhhXbQ1k/yZGL/WXntuvcpTT6/36ZAhX+5ijswSuOGBwBN+5JUVFhi44QhgZZOO\n",
"riQ6Ojuz95CplTMOeF3peLxWTn+kBte2wOVKxxubMQ7wJjAk6WgzadKkzpEjR1ZsPWrl/BRJHfRb\n",
"pePfV7OORpLP58fkcrkxacvRSFp9zcY6PA0YaZqeBS740k+HHtPK6+6KVv9dd0et162VcwDinHWH\n",
"0vElyb5qPzvrSeCG30Cy1+znR15JR3eBGy4NC+yhYPJyZM5S1Mo5DVgBuEkrZxUkPGJ/4GLgKq2c\n",
"XUwygCOBO63nqaUVCdzwm4gzzWAk4PpK4C4/8jrz+Xyaolmam6Y8U/Qj75HADccDUwI3LCsssByF\n",
"CBlTiiYe8WLz9oRE1xil40VaOfsA47VylktCW0NsdQxLagRuuDryJL+7aXoO8Swta9vIYumGplSK\n",
"AH7k/Tpww7LCAv3IKzcsMFtKUen43/Tg/KN03GMS2mrRyukHKOSHOqOX4Vkjn7YAKZBPW4BaYraI\n",
"zgBWR6zDq5GSPJ8VDc03WLQskE9bgJTI13i+pna08SPvUWC7wA17DAss1zpMUtWZolEig4FOpeP3\n",
"Kp6ohlSzL66VMwy4A9BKx9VmcbdYSiJww8GIdfgt0zQZOM+PPJ2eVJZWRCvn/wG/B6YqHR+e7Mvy\n",
"mWIjKctSNE4v30P2a7+KnL91mL4lwLtIzrn7gFsLmWmaCLt1amkoxjo8HakP9wliHd7RhXVosdSC\n",
"sjLaBG64OVKsty+wCrJzcYHpWy4XtR95cxLXnoIUGAa41Y+8sTVZQZ0pOU5RK+dI4AWkLM0jwPcR\n",
"pbgKklz2i0hS4juB7YHntXLOMtZkszDMvNqcp5a6ErjhoMANLwQuRRTiZCTu8E9WIVrqSEEplpoQ\n",
"/CbgST/ydka8oH8auOFegRv2R3JRn+5H3k6Ibri2cFHghnsARyPHXTsCRwVuuGeN1lBXerUUjXX4\n",
"O+CfwNd7sP7eNl9TgT9o5QwADgNu18o5Rum4GYqbWs9TS90J3HA35OzQWoeWRlNu7tPhwBgAP/Lm\n",
"BG74GrAFojsW+5G3TC7qwA3X9CNvNpJRZoIfeYsAAjecABwL3F+TVdSRUrZPTwUuNE4wJaN0vBD4\n",
"nVbOvcgHwE8rkK/R2O1TS90I3HAQ8v+0h2l6ETjXj7xmc+qyNC9LvU+1cjpMtrCeuA/YG7g/cMON\n",
"gE2AicC+iFMLAH7kzQjcMJmLejsSeazN2NG1WUJ96XX7VOm4bIVYdP27SsfNoBABHPPadNunJnNF\n",
"W9FMazaVwP+EKMRPgLGAX4lCbKZ114p2XDPUft1Kx58iFYdWQIot9MbRwKaBG74OPA0c70feC0hC\n",
"lXlFY5O5qIewbK7qeuSprgsV5z7VyvlxF237auWsUJ1I6WByra6F/MHMSlmcSsilLUAK5NIWoDfM\n",
"2eH5wOVIsvkpwMF+5N1axXZprlbyNRG5tAVIiVwd5iyEZZSyhXoX8Hc/8jZG0mxeYpzDSslFnb10\n",
"aSVQjqPNZlo5nlZOYYuxq3jBV4BLtHI2qIl0jUWZ17eVjpekKomlJQjccBfgNsQ57RNgHHBMNTFU\n",
"FksNWACw73szv9HR0TGm8FU8yHiejkSyKWF2NSLgx0gmmcFFlyRzURf3D6YOxkbghv0CN1w7cMO1\n",
"ajVnOSEZ6wDnAhto5cTAAq2cHwFPKh1PA1A6ftmkabsQOUdsJgpK0caGWaoicMPVgFOAvUzTFCTu\n",
"sOm25S0tyQKAu9Za559Kx3cUGidNmnRO0bjC9uqnibbFwAAk09JBhcbADR0kIcBk0/QcsFniuuFI\n",
"7t6qMF6vXYYFBm64XFigH3llhwWWbCkqHT+udLwRMBRxFhiIHJz+SyvnPa2ce7RyTkHOTJwepsoq\n",
"Q82rdXqwVEzghjsjZ4d7AQsR69C3CtGSIUrNavMKYiQcAhC44arAKMSR5gGgr9kNAZOL2niegoRn\n",
"HBy44YDADQci8YrXUgWBG5YdFhi44VmBG5YVFlh2mjel47eBP2nljFI6PlQrZzVkK3Vn5Ad2OFL3\n",
"rdmwlqKlYrqwDv+BWIfWk9mSNT42rz0qRT/yFgZuuA8wLnDDgxHlcwfwWz/yPjN9442Ftkwuaj/y\n",
"Hgrc8Abgb6bpht5qIHaHsQ6XhgX2YP0tExYYuOHSsMDADY/xI6+ksMBqcp/eDqB0PA9JvvpgFXNl\n",
"gYKl2KxKMZ+2ACmQT1sAgMANdwJ+AayNWIfjka2bep1N5+s0b5bJpy1ASuTrMGfJjjZ+5E0Gdu2m\n",
"r8dc1H7kjUN2SqrlVOBCP/LKioLwI28h8LvADcsKC8xkPcVqqDR/n1bOHUhGm4OUjl+vuWCWlsNs\n",
"J52M7JCAPMmea61DS5bRyjkH+Zs9T+n43kK7zX0q9HqmqJXzw2rCLLRy+mjlnND7yPTQyukDrGfe\n",
"2hI9ll4J3HAHxLN0FLAIuAr4kVWIliag3Kw2mSRww+XCAgM33Ddww6rCAkvZPn0cuFkr5zJTuqlk\n",
"TH3E85FA5SyzNuJpNVvpeEFvgy3ti7EOf4pk+QD4F2IdTk9NKIulPJqypmLghpshmXIeN45rOyAJ\n",
"yZO8gsRSjvcj781K7lNKRps3gROB47RyHtbK+YFWzobdjdfKWU8r5yCtnLuQp+efKx0/V4lwDcQ6\n",
"2Vh6JWEd7o1Yh1cDR1uFaGkymlIp8nlY4PTADd9C6ir+yMRTAuBH3svAacAxld6kJEcbpePZwI+0\n",
"cjYHTgDO08pZHQnQnIsUeFwDydixCPgrcLXScb5SwRpMszvZWOpI4IarINZhocbmVMQ6rOhJ1GJJ\n",
"maYsNOxH3uPARoEbrgfshEQ5jAauCdxwLvAk8ASSZ7XisMCy0rwpHU9TOh5t4hW3RbTxWCTjwY+B\n",
"HZWOldLxEZUqRK2cEVo5r2vl/LCo/R2tnEcTX+dXMn83FCzFpo1RbMfckI1Yc+CGX0OsQxcJYv41\n",
"cFSaCtH+rtuHOq27WS1FAPzIe9uPvD8hJa22QQwyD3gJOeP/FVVEQ1QTkvEWMF/p+PEq5lgGrZy9\n",
"kUDRD1g+b94DSsdH1OpeRbTC9mmO9nNbz1GnNQduuDJiHe5jml5CrMP/1ON+ZZLD/q7bhRy1X3dL\n",
"ONpgwgL9yKtpWGA1SnE6sK5Wzp+R7OkfAHeardZKeVHp+F6tnEermKMS7PapZSmBG34VOAv4AmId\n",
"BsAf6hh3aLE0kqa2FAv4kXdv76PKpxqleBLwltLxCwDmjPEArRyUjoNKJlQ67ilR8nCtnPuBVYGX\n",
"gTOrVMDL3Nq8Nu32qaV6jHX4E6RWHMjf2bl+5L2RnlQWS81pKqUYuOEPgbDSh9LADfsAx/mR9+tS\n",
"xldcOkrp+O6CQjTv3zfK8EGtnHrUT5wKHKB0vDNilVaUMqgYrZyVgdWRTCS1UrKWJiNww68gZ4f7\n",
"IkmPxwNHWIVoaUGazdHmceDmwA23LvfCwA03BW5BdjNLomJLUStnAJL5/CWl40WFdqXjt7Ryav7D\n",
"Vjo+KvF2DDBPK2f7GoR7JEtGVVrfztKkGOvwRCS5MIjn2hirDC0tTFNZin7kvRm44YlI/OEw4Ebg\n",
"qe7O94136i7AAUii8GPLSapRzfbpPUjwZIdWToQcck41c46oYt5eUTpeoJUzB8mKvpxSzOVy+cL3\n",
"I0aMmD5q1KjpQD7ZXmDu7rvvufLkyestGjr003w+PybR1eV44w2W60KstMcPa3L5KxmfXHPZ8796\n",
"xYwFyNnhuoh1eN3Go9d7a4UBfQ7N55efKgPrLTAsn8/nMiRPI8bP7aKtmeSvdPwy19Ri/r5XXzlg\n",
"/dPOWHWFjz8uu6xSWpjqG4WYxBOA8wI37DUs0I+8fLn3qjj3qVbOL4CLgd2AHwDfRSrXfwIcoXR8\n",
"W0UTy9yPAr9XOr7ZvN8NmKd0PNm8749sAeyqdPz35LXl5u/TyvGQ89FblY6znnnHUgO6sA5fQaxD\n",
"m/PW0vJo5fQDngKWAF9TOu6E5st9aqzG9ZGg/j5IEWNdbuLwYqqxFP+FJEO+Ren4SK2cDiRd2tzk\n",
"dmqFdJivAusjpamONu9PAF4HnqnyPmA9T9uKwA23R6zD9RDr8HfATX7kLU5VMIulQSgdf6qV8ynQ\n",
"D0lvuTBlkSrCZJKaXut5K1aKJnRibWAkYmV1IqZsxWjlbAmcg5xVHqeVs4PSsY8UlNxFK+cJRFnO\n",
"A0YpHdfCRd56nrYBgRuuhDxM7W+aXkWsw9fSk8piSY2PgMHIuWKPSjFww6ORIsKLgdWAE/3Ie9w4\n",
"vlyDWJxzgMP9yJuTuO4UpLgwSCm1mu7EBW54ORK0v3PBOgzc0AP+4kdel9vtpVCNpYjS8Szg1mrm\n",
"KJpvKp9/aCXbY+Co5a+oCdZSbHECNxwBnI08AC0GbgB+b61DSxuzgM+V4vvdDQrccH8km9POfuQt\n",
"CdzwMGCIKfwbAYf5kfdY4IZjgGsR5xYCN9wD2dnbCjFkpgRu+LIfeffXcA3vA9fxueMQwEPASYEb\n",
"3uVH3j8rmbQqpdjsmJJY6xbepimLpfYEbrgicDxwoGl6FYk7rOrMwWJpAUrNanM2cEwhRtCPvJsB\n",
"Ajd0gcV+5D1mxl2PJOpe0zjF+MAEP/IWmfETgGOBWirFTuAaP/KWlvvzI28WMCZwwwuQ+qZlU3Gc\n",
"YouwDvJgMEvpuCn31Qu0Y27IntZsrMNbEIW4GHmiPLwVFKL9XbcPdVx3r2EZgRuuA2wBbBq44aTA\n",
"DR8P3PBY0709Er4EgB95M8yc25qm7RAHtgLTTFstuQt4MXDDBwM3PD1ww68Hblgw9FasdNK2thRp\n",
"ra3THO2XGzJH0Zq7sA5fQ84OX22oZPUlh/1dtws56rPuUmIVh5nXA4FvI0lOng3c8H3EoJhXNH6u\n",
"aQcYgiRZ6aqvVpwH/BL5HP82YtV2BG74HvD7Sidtd6VonWxaiMANt0X+MYYi1uHvgf/zI+/TVAWz\n",
"WLJHKVltBpjXwJy/zwrcMEScbt5i+aINsGzUQGXxfqXzgh95vyu8CdxwAPA1pN7pQ5VOapWi0AqW\n",
"YttirMPjkCfaDiRc55wWsw4tllry8cRPPl71tx/OP+q5jo6dACZOnFg8puCA826i7X/IQ+dzwDZF\n",
"4wfzeQTCTPM+2TerBnInWRC44ZcK/+d+5C0EHgvc8FngTOBvlUza7meKrbR92pYEbrgNcnZ4EJLV\n",
"4nrgUKsQLZYe+eibA1ecf/da69zb2dk5prOzc0wXY15DtlmT255rIZ+XzwFfKjQGbuggVudk0/Qc\n",
"sFniuuHAs7UTH/zIuwr4duCGJyfk+AaizDeqdF5rKQp2+7TJCNxw4GpbrPR1YC8+tw7P9SNvWs9X\n",
"WiwWPj9T7NYhxY+8hYEb3gz8ELjX7MjsD1yCFGS4KnDDXfzIexzZUr3TeJ6ChGeMC9zwMuT/82Dg\n",
"lFovwo+8q4uanjD3eayL4SVhlaLQCpZiPm0BGkXghlsB53z42sfDkAP8G4EbCu7fbUA+bQFSIJ+2\n",
"ACmRr9O8pYZk/Ay4NnDDF4GPkexPEwACN9wHGB+44dLg/cJFfuQ9FLjhDXy+hXmDH3k1qWzUE8Z/\n",
"4LfVzFFx7tOsUmr+Pq2cVYFHkV/0LoX8f5bsErjhQODHwCHI0+cbiHX4cqqCWSxNhlbOAcDPgduV\n",
"jn8FzZf7tF60s6W41Eq0CjH7FKxDJA/uZ4hn6fVtZB1aLLXkY/PaFOWjGkk7K0XrZNMEGDfrHyOV\n",
"WDqA/yDW4UupCmaxNDeFkIzetk/bjnZWihuY15KLT1oaS+CGX0biDoch1uFNwO+M67XFYqmcXh1t\n",
"2hWrFMXysGQIYx0ei1iHfYA3kaw01jq0WGpDqY42bUc7xyluaF5bQim2Sm7IwA3/HzABONQ03QR4\n",
"XSnEVllzubTjuttxzZBu7tN2pS0tRa2cvnye1+/NFEWpJTma2G3dWIfHIMqwD1I8dIwfeVN7uCxH\n",
"E6+5CnK037pztN+aId3cp21JWypFwEHWrpWOP+5tsKW+BG64BTAG2dL+DLgZybdozw4tlvpQSu7T\n",
"tqRdlWJLbZ02K91Yh+f6kfevNOWyWNqApZaiVk6HDUv7nMwpRa2cEcBtwPlKxzcl2rcGrgGWZk9Q\n",
"Op5T4W0KefGsUkwJYx2egzygdAJ/AK611qHFUn+Ujj/VylmM6ID+gP2/M2RKKWrl7I1kK/mARNkR\n",
"rZz+QAQcpnT8mFbOGCS33gEV3spaiikRuGF/xDo8DLEO30Ksw4qqZFsslor5CBiEbKFapWjImvfp\n",
"i0rHB7F88crvAIuVjgtJXq8H9tXKWbPC+7SiUsynLUBvBG44HLEID0cC8UPgkCoUYr42kjUd+bQF\n",
"SIF82gKkRL6Oc1tnmy7IlKWodBx307U9MC0xboZWzgJgW+Dhcu6hldMPSRXWSet4npLL5fJpy9Ad\n",
"xjo8Gsm2vwLwX8Q6/Ec182Z5zfWkHdfdjmuGuq/bOtt0QaaUYg8MYXnrcS7L1vkqlfWRdc9QOv6k\n",
"WsEsPRO44ebI2eHGyIPIBOAaP/Lsz95iSRdrKXZBsyjFThJnjAkqyejeilunmSNww36IdXg4Yh3G\n",
"iHU4JU25LBbLUgrhaDarTYJmUYozgW2K2gab9uVIbjmMGDFi+qhRo6YDedO+jFI0GSNyXUyT72rr\n",
"wo7vffxHb32y38Ah/XKLP/psDTroXMkZMHWtHVa7ZvdR31xOIWZRfjvejm+H8ZuUsX0auOEmwMvA\n",
"SFNUmMANl4sI8CNvTuKaU5DiwgC3+pE3trf7ZIFM1lPUynkU+L3S8c3mvQtcrnS8sXnvIOeBQ5SO\n",
"Zyev7a0mmFbOr4CRwNlKx/fXaw3tiLEOjwKOQKzDGYh1+GKqglksluUwXvzfBc5TOr63p8/OwA1v\n",
"AUYBe/qR97jxE3gNOMyPvMcCNxwDDPcj7wAzfg/gCmArZEdvCnCKH3mZ/8zNmvdpgQ6W3Rp9AOir\n",
"lbOLeX8kcGexQiyRQoziG1XIlznSzg0ZuOGXkEw0RyMK8Vbg4HoqxLTXnBbtuO52XDPUfd0lWYqB\n",
"G24PzAdmJZq/Ayz2I2+ZiIDADQsRAT4wwY+8RSb2eAKS5D/zZEopauVsqZVzOzAcOE4r5zoApeNF\n",
"wD7AJVo5TyBep2X/gE28o4OkEpteK7kzQi6NmwZu2C9wQx9J3L0JYh0e40feWD/y6p1CL1fn+bNK\n",
"Lm0BUiCXtgApkavj3KUWGh4DnFvUtkxEgB95MxDHnW1N03bAK4nx00xb5snUmaLS8VRg/276pgA7\n",
"VHmL9TFOH0rHNli1Sox1eA6wqWm6DfhNA5ShxWKpnl7LRwVu+B1gqh95OnDDZFdvEQFDkCQsXfVl\n",
"mkshYrwAABTVSURBVEwpxQZgPU9rQOCGfZEt7CMxidWB8/zIm5yqYBaLpRx63D4N3LAP8HPge110\n",
"lxIRkD2HlRJoN6XYkueJjSRww00R6/BLpulPiHW4oPurLBZL1rhk3gcbLuzsXO9/S5Z85y8dHStO\n",
"nDixeMghwEN+5M1NtBWUXm8RATPN+2TfLJqAdlOK1lKsEGMdHo440vQF3kasw+fTlMtisVTG6asN\n",
"egb4PvCc0vGYSZMmnVM0ZCdgy8ANv23efwG4MnDD6cCNwEGFgYEbOojFWdgteg7YLDHXcODZWq+h\n",
"Hlil2Drk6zVx4IYbI4fthT/yO4CrM2Ad5lO+f1rk0xYgBfJpC5AS+TrO3aOjjR95yzgzBm74JvAT\n",
"E5IxALgqcMNdTNzikcCdfuQVIgKuBcYFbngZYl0eDJxSj0XUmkzGKVZDd7E2xvP0SfN2Z+to0zvG\n",
"Ovwh8CPkAep/iHX4XKqCWSyWqtHK2Qq4AZiqdHx4d5+dgRtuBZwFfBuxBO/2I+8qE7w/nmWD999P\n",
"XHcysgULcIsfeZfXd0W1oZ0sxS9iShVZhdg7gRtuhLhhF6zDOxHr8KPur7JYLE1ESXGKJnH/fl20\n",
"9xgR4EfeOGBcNQKmQTspxVbeOq0ZgRuugFiHxyB/H+8g1mFTnAdYLJaSsQnBu8AqRctSjHV4DnIo\n",
"DtY6tFhaGasUu6CdlKINx+gGYx0ehliH/RDr8Hw/8p5JVTCLxVJPlipFrZwObr4xTVkyQ6bSvNWZ\n",
"lrYUK82RGLjhhsD/AcchCvFu4MBmUIg2H2b70I5rhvqu26TPXIwYR/3rdZ9moy0sRa2cAcBQxEvq\n",
"vymLUy9ylOG+bazDQ5HEvQXr8AI/8p6uh3B1Ikd7uurnaL9152i/NUP91/0RMAhYsY73aCraQikC\n",
"wxCreLp5OmprAjfcADk73NI03QNc6Ufeh+lJZbFYUmABohRtoWFDuyjFlt46LRVjHf4AqTDSH3gX\n",
"sQ6fSlUwi8WSFtbZpgirFNuEwA2HIVlpCtZhBFxhrUOLpa2xSrEIqxRbnC6sw5mIdfj3VAWzWCxZ\n",
"oNfyUe1GuyjFdgjHyBc3BG74ReTs8Mum6V7EOpzfQLnqST5tAVIin7YAKZBPW4CUyNd5/kIMsnW0\n",
"MTSVUtTKuRFJ15bku0rH3QaXa+UMBBTietyqnqfkcrl84XtjHR4M/BgYgJRsucCPvL+lI119SK65\n",
"nWjHdbfjmqEh6y4kBbeWoqGplCLQqXS8W5nXDEOytMdKx5/WXqRsYazDs4GtTNNfgHF+5BVXybZY\n",
"LJaS8p+2E82mFCuhLc4TjXV4EDAasQ7fQ6zDJ3u80GKxtDPW0aaIplOKWjnXAFsAHwKXKh3ne7mk\n",
"5c8TjXV4FrC1aboPuNxahxaLpResUiyi2ZTiNGCS0vFkrZztgEe1cnZSOv5HD9e0rKVorMMDkRRt\n",
"BevwIlP002KxWHqjW6Vo6qkeD7jIEVR/4Jd+5D1i+rcGrmHZeopzEtefgvg2ANzqR97Yei2iljRV\n",
"7lOl40uVjieb759HLCK/l8taUikGbugAAXAyMGDAOv2mIjlL20Yh2nyY7UM7rhkasu6eQjKGAicC\n",
"e/uRl0N8FaLADdcN3LA/Eut8uh95OwEvANcWLgzccA/gaKTe4o7AUYEb7lm3VdSQplKKXRCzvDfq\n",
"UrRyVqTFPE8DN+wTuOHBwC3Idul7wMnDfjBkuh95H6QrXcPJpS1ASuTSFiAFcmkLkBK5Os/fk6PN\n",
"POCsQgiXH3kTgU8QJfcdYLEfeY+ZsdcD+wZuuKZ57wMT/Mhb5EfeQmACEiudeZpq+1Qr5+dKx5cm\n",
"moYAunhcwY15Q0etdMr7c4cNXHudd+ILzttJdRHzY57Ecl3cLt+VO3Sa41+9YsbryNPatqbtgS8e\n",
"ss7TA4f0/waQy+fzY7Isfx3GJ9ecBXkaNT6Xz+dzGZKnEeOHddHWTPJndXy326dmK3RC4X3ghoUt\n",
"1JnAt4BXEmNnBG64APlsehjYLnktcvQ1ugs5MkdHZ2dn2jKUjFbO/4AvKx3P0srZAHgRGKV0/ERh\n",
"zKRJkzpHjhzZYcbvBZwLPKx0fEYqQteAwA37APsDJwADgdnAxX7k5Qtj8vn8mFwuNyYVAVOiHdcM\n",
"7bnudlwz1H/dWjlbATcA/3rl5hsPL3x2dkXghjngKmSH6jpgZT/yDkn0/xc4w4+8CYEbLgL28iPv\n",
"YdM3EnjIj7zMG2LNtn06FrhbKycP/Ak4LqkQu6DpzxMDN1TIYfapiEJ8CDggqRAtFoulQkryPg3c\n",
"cCBwIeJM0wkUvopJKtXmsbgSZF5rJ1E6vhy4vIxLmjYcw1iH+yEH3QMR766L/ch7NFXBLBZLS9DR\n",
"0ZFbu0+fUfusuNJ6Czs7Vzusm3Fm2/Q6JAnIi6Z5JrBN0dDBpr3QP7iob1aNRK8rTaUUK6ApLUVj\n",
"HZ4NjDBNDwGX+ZE3t4fL8vWWK4Pk0xYgJfJpC5AC+bQFSIl8vSbu7OzMa+VMAXYF5r0Cm3Qz9HLg\n",
"aT/y7jRep0OA55BkIcBSb/iVgMmm6Tlgs8Qcw/n/7d19kFxVmcfx74NsgiiKskuUZlxUXiMKApFd\n",
"RRhAy7ddOrHWg2AvRNaioUqKXbOrhcKa3S0KKBeMVYo2aiHQKalbCjbgSySRLAS0wsawta5RgyU4\n",
"tktGSXCxeEvI2T/O6cydSc9M99DTt/ve36eqq3vuPd19npme+/S597zAxh6HMC+G6ppiJ1rXFJul\n",
"kf2Be4GdwCml5tjzGVdtVqnW4SWECXp3EFqHP8i0YiKSS83SyALgAWDXz27+2slTrynWyvVPAIcA\n",
"nyScGn0NoX/DVcAvgPOqjcq9tXL908DiaqNydnzeu4DrCB1vjDBkY0W1UflufyKbu2G7ptiN18b7\n",
"R4ckIZaA64GPExLi94EPKCGKyDzaSRiyttdZw1q5fiQh+V0CPEkYovETwFcbleeApcDVtXL9PkLy\n",
"2zPkotqorCF04Lkf2AB8dRgSIuT79Gnr1OlAX0+MrcP3A5cy0Tq8utqorMu0YiKSe6XmmG+WRp4C\n",
"XjZ1X7VR+QUzNJyqjcpDhMH50+2/jtBaHCpFSIq/yrQWM4itw8uBJXHTWuCaaqOyI7taiUjBtE2K\n",
"RVWEpDhwnWzajDt8gpAM7860YiJSRNOuR1tEeb6mOJDDMeKKFjcwMe6wde3wBSXEIs4NWcSYoZhx\n",
"FzFm6FvcT81epDhy2VJslkZeAryKcBH5NxlXB9izosW5hIvRCwmz0lzdw3GHoxSv2/ooxYsZihn3\n",
"KMWLGfoTt5JiSi6TIhM9Tx8ZhJ6ntXL9dYRxh8fGTd8mDIQt2gTeIjJ4lBRT8poUB+J6Yq1cXwh8\n",
"GFhO+F2PA1dWG5X7s6yXiEiKkmJKXpNi5tcTa+X6W4DLgJG46VvAqmqj8ses6iQi0oY62qTkNSlm\n",
"1lKM64n9A/DuVB2uSs0ZKCIySJ7OugKDREmxR2JHmjJhmMUBwLOEhTfr1UZlZx+qsL4P7zFo1mdd\n",
"gYysz7oCGVifdQUysr4P76GWYkpek+IiQlLaawHi+VAr15cAH2NiQt0HCOMO+/L+MLGwcpEUMWYo\n",
"ZtxFjBn6FreuKabkNSlCH3qe1sr11xCmZzstbnqMsAjn2rjmmIjIoFNSTMlzUpy3U6e1cn0RoVfp\n",
"UsLv8GngRmB1tVF5dr7eV0RkHuj0aYqSYhdiMlxOSIZ/QlhZugF8sdqo/L7X7yci0gfqaJMyVEmx\n",
"WRrZD/gScBSh7p8sNcemmx6tZ8Mx4sTdHwKWMZEM1xCWQxm4uVVFRLowbUuxVq7vdczN+xzNwzb3\n",
"6UrAl5pjf0mYMu3WZmnk4GnKvqBkVSvXrVaun1Qr168ljDF0hA/FGuDsaqPyqUFKiEWcG7KIMUMx\n",
"4y5izDAQc5+uJKyfuOeYWyvXpzvm5sLQJMVmaWQf4O8IC1dSao5tBTYDlTbFnwF+O5f3qZXrh9bK\n",
"9QuB2wnfkE4jLML5beCDg5YMU0azrkAGRrOuQEZGs65ABkazrkBGRvvwHm2TYlzNZ88xt9qozHTM\n",
"zY1hOn36OuAg4GepbT8FTmpT9pFSc2x3Jy9aK9cXAG8C/iLejk7t3kZoJd5WbVQen0ul++XOO+88\n",
"bHR0NOtq9FURY4Zixl3EmKFvcU/XUuzmmJsbw5QUF8X79CTafwAWtyk76XpinIN0EXBwvLUeHwoc\n",
"T1jxvuVpYB2hZfjjaqOS+YTindi0adNhWdeh34oYMxQz7iLGDH2Le7qk2M0xNzeGKSm2TB3/Z1ML\n",
"bDz8/MXj5foqJpLggbO85sPAj+Jts4ZViEiBPEe4RDSdWY+5eTJMSXE83h8I/D71eNvUgtsPOOyN\n",
"TCwfBeEPPh5v2+Kt9fN/VxuV381TnUVEBlqpOeabpZF2rcWOj7l5Yt4Px8QrsaPNOLC01BzbELet\n",
"A+4qNcc+2yq3bt264QhIRGTAnHnmmXtagbGjzTiwtNqobIjb1gF3VRuVz07zEkNvaFqKpebY7mZp\n",
"5MvABcCGZmnkCOA44Jx0ufQfVURE5qbaqOyulet7jrm1cr3tMTdvhqalCNAsjSwkDJM4mpDQLys1\n",
"x9ZmWysRkXyKnRQnHXOrjUquj7lDlRRFRETm09AM3p+Nme1nZl8zsx+a2YNm9s6s69RrZravmf29\n",
"md1jZuvN7AEzOyO1//gY/wYzu8PMXpllfXvNzI4ws51mdmpqW95j/kj8O99rZg+1Ys9r3GZ2TPx8\n",
"32dmm83s8tS+3MRsZiea2cNmdv6U7TPGaGYrzOw/4+0f+1vrgvDe5+IGXA3cGB8fATwOHJx1vXoc\n",
"42GE6esOiD+/A3gSeDWwAHgUOC3uWwkkWde5x/F/HfgjcGr8OdcxAx8A7gReFH8+L27LbdzARuDf\n",
"4uNXxv/j9+UpZuAs4FZgE3BeavuMMQLvBrbEcgvj4/dmHU/ebkPT0WYmZtaajmgZgPd+q5m1piO6\n",
"Lsu69dj/AVd4758E8N6vNbNngLcBO4Fd3vv/iGW/AjxiZgd57wd6Np5OmNkSwheA9PCZ95DjmIF/\n",
"Bi703j8P4L2/GcDMyuQ37sWEZID3fruZbQXeQLielZeYN3vv7zCze6Zsn+3zXAVWe++fAzCz1cBF\n",
"wHf6VfEiyMvp00JMR+S93+69X9362cyM8K1xHFhCKn7v/W8IM1Wc0O96zpOVwL9M2baE8G0ZyFfM\n",
"ZnYwIRkcaWbr4unTi+Lu3MZNmEnqLAAzez3hrM9achSz935sml2zxXgSk49xW8jZMW4Q5KKlSEGn\n",
"IyJMVv4IcB/wt0yOH+AJwow+Q83M3gP8xHvfDN8D9lhEaD2n5SJmwqlygLOBdwGvADaa2Q5CfHmN\n",
"+yNAw8weBl4OfNR7/2Mzu5j8xtwy2+d5EZP/x/MW/0DIS1JsKcx0RGa2H3AlsNx7783Ms3f8MOS/\n",
"g3hq/OPEU+NT5DLmaGG8r3nvdwG/M7M6YczYo+Q37tuAB7z3Z5jZocD9ZraNfP+tWzqJUcMF5lle\n",
"Tp+mpyMi9TiX0xHF06Y3ANd57zfHzePsPcfrgUz8bobVucAa7/0TqW2tg0ReYwbYEe/Tn+H/JUxi\n",
"n8u4zewY4ExgFew5fdgALianMU8xW4xT9x/I5Gvs0gN5SYq/BLYzedmnxcCD2VRn3l0L/Mh7/00z\n",
"W2BmI4RYj2oViNv2J/RwG2anAH8Vu+nfA7wKWGVmt5PfmAG2Eq4npU+P/SnQJL9xL4j3O1PbdhFa\n",
"zXmNOW22GB9k72Pcxr7VriBykRS997uB1nREmFlrOqLVMz1vGJnZJ4AXATeZ2UuBwwlxfxfYNzWG\n",
"7wLgm0PYM28S7/1F3vtTvPene+9PBx4DLvXeLwO+Rw5jBvDePwvcDJwPYGYvJgzHuImc/q0JnUia\n",
"hLMDmNkBwF8Dd5PPmI3Jp0Zni/FLwDlmtjBePjknbpMeys2MNma213RE3vtcTUdkZkcyufdZy0rv\n",
"/b+a2fHA9cDzhJbzcu/9jjblh46ZHQdcQeh0sgm43Xv/uZzHvD/hM/1Gwjqft3nv/z3uy2XcZnYi\n",
"YRiVAS8F1gCf8t7vzkvMZnYs8GkmOso95L2/MO6bMUYz+xjxSwPwde/9tX2seiHkJimKiIi8ULk4\n",
"fSoiItILSooiIiKRkqKIiEikpCgiIhIpKYqIiERKiiIiIpGSooiISKSkKCIiEikpivSBc+4M59zZ\n",
"XT7nCufcyHzVSUT2pqQoMgfOuVOdc0fPXhKcc5cAXyCse9mNzcAPnXMndls/EZkbJUWRubmRDlY9\n",
"d86dTpjL82+SJPltN2+QJMldhGWU7nDOvXxOtRSRrigpinTJOXcI8Frg3g6KfwZoJEnyP3N8u88T\n",
"lk76pzk+X0S6sG/WFRAZQqcAY0mS/HqmQs65NwAnALU2+/4MuAo4mbB+4D6E9fI+nyTJf7XKJUny\n",
"jHNuPbAcuLxH9ReRaailKNIh59xZzrlbgGuAp5xztzjnLp7hKW+P9z9vs68OjABvTpLkBMIyQscA\n",
"S9uU3QIc4pw7Yu61F5FOqKUo0qEkSe4gXN/bDFyfJMmXZ3nK4fF+vM2+twK3JkmyK772H5xzlwEv\n",
"blN2W+r1tnZfcxHplJKiSBeccy8jLPq7oYPirc4xz7TZ933gAufcK4BbgLVJkkzXO/XpKa8nIvNE\n",
"p09FuvNW4IkkSbZ0UHZ3vLc2+z4IrACOBG4Hxp1zN8VrjVO1/k93t9knIj2klqJId95OZ61EgO3x\n",
"fv+pO5Ik2UkYbrHKObcYqAIfBf4cGJ1S/CXx/vFuKysi3VFLUaQ7byMOwnfOLXbOvX+Gsq3W5Kun\n",
"7nDO3dp6nCTJT5MkuRT4DnBcm9dpPb+T1qmIvABKiiLdOQjY4pzbB/gw0Jih7D2AB45ts8855z6U\n",
"+mERsBhY06bsm4At3Q7+F5HuKSmKdOca4FzgSkIP1OenK5gkyRjwPdoPs1gBVJ1zD8XerD8AvgFc\n",
"kC7knDuIcMr2i72pvojMxLz3WddBJLecc0cBm4BlSZLcPYfnryIkxbfMlIBFpDfUUhSZR0mS/BxY\n",
"BtzQ7cTezrkVhEH971NCFOkPJUWReRZbiKcy0Yu0U48CJydJ8ljvayUi7ej0qYiISKSWooiISKSk\n",
"KCIiEikpioiIREqKIiIikZKiiIhIpKQoIiISKSmKiIhESooiIiLR/wOf2fr2NunhJgAAAABJRU5E\n",
"rkJggg==\n"
],
"text/plain": [
"<matplotlib.figure.Figure at 0x7f0ba453c450>"
]
},
"metadata": {},
"output_type": "display_data"
}
],
"source": [
"# plot v(t) and x(t)\n",
"fig,ax = plt.subplots()\n",
"c1 = \"#e41a1c\"\n",
"c3 = \"#984ea3\"\n",
"tts = np.r_[0,ts]\n",
"ax.plot( tts, tracev*mphfact, lw=2, alpha=0.8, c=c1 );\n",
"ax.set_xlabel(r\"$t$ (s)\");\n",
"for tl in ax.get_yticklabels():\n",
" tl.set_color(c1)\n",
"ax.set_ylabel(r\"$v(t)$ (m)\", color=c1);\n",
"ax2 = ax.twinx()\n",
"ax2.plot( tts, xs, lw=2, alpha=0.8, c=c3 );\n",
"for tl in ax2.get_yticklabels():\n",
" tl.set_color(c3)\n",
"ax2.set_ylabel(r\"$x(t)$ (m/s)\", color=c3);\n",
"ax.set_xlim((0,ts.max()));\n",
"ax2.grid('off');"
]
},
{
"cell_type": "code",
"execution_count": 237,
"metadata": {
"collapsed": false
},
"outputs": [
{
"data": {
"image/png": [
"iVBORw0KGgoAAAANSUhEUgAAAZgAAAEmCAYAAABf+4ZQAAAABHNCSVQICAgIfAhkiAAAAAlwSFlz\n",
"AAALEgAACxIB0t1+/AAAIABJREFUeJztnXmYXEXVuN8TliSyBQirLAEMYdMoARU/gY4RFcaMo0LJ\n",
"vquICy6IuCABFf0+MbixCIKCwk9LkGFkRCADzRqUJaCssoU1EDAQAkKAzPn9UdX0TdPd03emu29P\n",
"93mf5z59b91zq06fnunTVXXqlKgqhmEYhlFvxmStgGEYhtGemIMxDMMwGoI5GMMwDKMhmIMxDMMw\n",
"GoI5GMMwDKMhmIMxDMMwGoI5GMMwqiIi40XkYREZzFoXY3RhDqYDEJE1RGSWiEwdYT1fFpGP1Uuv\n",
"RL2bi8iFInKfiNwtIgMi8p6UdRwpIv8UkTtE5F4R+Z6IrFwis76IfENE/iEid8XjVhH5YqlslJ8o\n",
"IqeKyP1Rr4dF5BciMmGk7zkLRORTIvLHaKN5IvJvEfmbiMwY4tHjgE2BmhfNicgmIvJ9EZkrIrdE\n",
"W98Ty1Yf0RsxRg+qakebH8AkYBA4cIT1zAfOqbNuGwILgN8AY2LZN4D/AjvWWMcJwAvAO+P1BsCD\n",
"wIUlcmcAC4GpibLdgVeB/hLZNYEHgFuBdRL13gncBqyc9ec6zM/vAmB8vB4Xr5cBH6jwzBTgGeBy\n",
"YDBFWwcDLwLvT5TtBLwEXJO1LexozmE9mM5CWqSOJLOACcBXVHUQQFX/F3gS+NmQyohsCnwLOF1V\n",
"b4/PL4j1fkJEPpQQV+DnqnrHGwWqlwEXAruLyDsTsl8ANgeOU9VnEvUeB7wT+Mxw3mzG3Agcq6ov\n",
"A6jqK8D/ET7Tj1d45lTgJMKPgDQsAP5PVa8vFKjqXGAOsLOIrJ2yPmMUYg6mzRGRw4D+eHliHBqZ\n",
"JyIfSMi8T0SuEJFHROQBEfm7iOyZuD9VROYRfsF3J+o4ZoS6jQEccIuqPl9y+0rgvSKy2RDV7Ams\n",
"AFxRUl643i9R9gXgB2XqeCy+rpko2ym+3lEie3ui3aqIyEoi8iMReTIO210vIh8Tkfki8h8RuU1E\n",
"JkbZ/ePQ4LxYfouIfHqoNtKgqvuq6qMlxWvE10fK6L83sC7w82G0dbmqnljm1uqE3uaias+LyM0i\n",
"skBEBkVkGxH5axymfEhEviAiY0TkOBG5QUQeE5E/JZ1WvHdXfP4oEfl1tOkzInKjiLyvTJsHxc/p\n",
"sTiM+i0ROVdEXo2fy+5p7dDxZN2FsqPxB2H8vOwQGfAhwhDR8YmyPYHXgS+XyD5MhSEyYF6Nx5mJ\n",
"Z94W9fptmfq+Fu99fIj39vsoN6nMvSXAHTXY5y+EL7xVE2WXxno3KZHdJJYvrKHe3xKG+naJ12OB\n",
"PwEvl9oRuBn4XMln9mCyLJbvkMLWnxlCv22BW4A8cdgscW814HFg58R7qXmIrExb44EvE4bNXI3P\n",
"HB9tfRZxSBL4aiy7BJgey9YkDP+dWfJ84e/+2cRnsAJwfvxc3pmQPTjKHp0oOyb+DT3UqP/Ndj8y\n",
"V8COJnzIVeZggPuBh8uU/yX+E05MlFV0MMPUa6eo18/L3Pt0vDfUl+TlUW6tMveeAJ4c4vkpBAdb\n",
"+kX+7VjvJ0vKPx7Llw5R71ZR7uyS8sKXXqmDmQJISdmPgH814O/hkGibZcA5yc84IXMKcF7ietgO\n",
"hjBn9Ur8+/lEiudmRVtNS5RNjGV/KZE9FXi0pKzwd1/6GawLLAUujddjoj0eKJETgpM3BzPMw4bI\n",
"OhgR2RLYAripzO25hEngXDN1SlCPNN9V6xCRVQmT3Ger6uklt08lfCGeJCJbRfktgRMJjvflIdqe\n",
"Hl+Xs62qPgIsLiP/KnB6HMa5Iw5JHkTo5dUVVf2Nqr4VeCuhV3W3iLy/cF9E3gEcABxdp/a2B94C\n",
"fBE4S0TOT1nFPYnz/5QpK5RvUOH525MXqrqQ0OPJxaIp8dm/l8gp8K+UuhoJzMF0NuvE1/+Uufds\n",
"iUwjKLRRLmy1MDfwzAjrKPu8iKxGmJv6h6p+rvS+hjmh9xJ6SBeJyJ3AT4HPEwIQ3jRnUUJhPqDc\n",
"XMNy800ish5wA/AO4KOqOlVV30WIehs7RDvDRlWfIgwNvQacG3UR4HTghPhFXK+2BlX1UsKw1z4i\n",
"ckCKZ/+bOC/8aHixRGyQMPxVjnIO/T/AeBEZz9CfVb0DWzqGFbNWwMiUwpdvuYieifG1pi+Z+Iu7\n",
"Fm5W1UIE1oOECd8tysgVfrkPVe+twD6xjvkJfdYDVin3fJxYvwy4XFW/U6liDdFjXyp5diXCPMxZ\n",
"Q+hVcNrlbLtWyfVMYH3gC/FL/43myui+Qw1tFzhdVc+Mz40hzGO8khRQ1ddE5G5ghoisRehpbA4c\n",
"LiKHJkQ3ifUU7Pk9Vf1zpYZFZBzwqsbIwASF53cEflfj+xgp5dYtrQ28rKovi0i1z2pN6tOb7kjM\n",
"wXQGr8XXMfBGaO+GqjpXRB4gRGtJ4tchhPmRl4GrS+op1PEW4IOq2gcQf3GnQlUHRcQDB4jIhNhr\n",
"KPyK3g2Yq6rzC/KxfH0N4cIFLiLMVewGDCTKC+HJyw3HiMjGhF7Jr1T1Z4nyzwSV9Kx4vRKwe+H9\n",
"Jdid8Ev5jCHeXkGXnYAzE+1MIkygJyn0Ukq/yDYsLVPVW4DUtgZ2AX5FGA56g2jTSYTP+kVVXUSZ\n",
"oSYR+Q1hDu9NbcdewPj4bIG/xfb+X4l4ISrwWZpHMvwcEVmf4EQvi0X3EeZg3lMiNwZ4ezMUbFds\n",
"iKwzeJowb7BJvD4SKPw6/QJhLP64grCIfILwRfrtki+NhxJ17ArMroNuxwPPAbNFpDDE8XXCl9xR\n",
"JbKnAU+IyF6Fgjin8UPgSImZCkRkg1jvRap6ZeJ9bUUYinoYeDaGBu8vIvsDHyF8oRdYA+iVROYC\n",
"EXk7YW3ON1T1rmpvSlX/TZhA/5SI7BKfH0cIky4d3rmM8AX/5Th0R3wv+1RrYxhMFpGjC3aOTvQH\n",
"hN7fz1X11SrPCpWHim4DHo3Ou4AC3xaRzd+oQGQ74HsE53L28N/GG/rUynQR2TXqsCLh7/ZV4t98\n",
"7GV9E9hMRL6eeO7rhOg3Y7hkFV1AmED+LWEy+WZgtyqyqxK601dXuH84YRHZtYQJvV2yjp5otYMQ\n",
"OfQQYYjiWhJhvYRf2ZcT5hUeJExM71mmjncD/ySsZr8d+FCddNuMEL57H3A34df/u8vIfZfgjKaX\n",
"uXckYUL2DuBewhfZSiUyFxEipwbLHMuA7yZkxxNCoB8kTCjfSlgkuHuK97UCYZHik1GnqwkTyw/z\n",
"5simXPxfWEAIG/4dIbvBYPzMZo7QxqsSfkwMxM/vn4T1P9cCB1d57hdR3yXRRg/HvyNJyMyJ5cmI\n",
"w/cS5nIKn8m/oy3PpCT0u0K7l0dbLIvvvyce82LZAuCySrKxfFK032cJa3n+QRgWvgHYqUyb+8fP\n",
"+rH4WXw+fgYWRTbMQ6Jhm46I/AhYT1UPEZHJhC+1rbVkYlFE1iB8MTwETFbV6SX39wIOJPxRLROR\n",
"A4FXVNU35Y0YRkpEZAkhTLm0h2bUkTgc+RDBgZ43zDr+QnCII8rj16lkMkQWxzYPI3aTVfV+wi+P\n",
"/cuIv05Y7X1Dheq+C5ykqstiXeeZczFagbgS/N0lZVsQgg9uzUYroxwisruIfLakrDAHY5/VMMlq\n",
"DmZzQsTGvYmyuwmrlJdDVV/SMA9QLqJmXcJq5C0lpNm4VkSOaJDOhpGWyYT0PKsAxNdTCEOBf8xS\n",
"sQ6jlvma9YDvxF5PIfjh24RoypMbplmbk5WDWS++JuPTFxNW2KZhUnz9FPBh4JPAN0TkUyPSzjDq\n",
"w5+Icygi8i/gLsIcwC6qujRTzdocETmOsM5JCU6+d4hHbgKuAa4QkTsIIe87ET6ruxupazuTdZhy\n",
"6QRQ2gVNhfDOX6nq68AzIvJ7QoSU/UI0MkVV/wr8NWs9OhFV/R4h0KNW+XspP0RvjICsHExhIn8C\n",
"xXj4CYRw2jQ8F1+Tzy0ANioVHBgYsMVShmEYw2DGjBnDymaQlYN5kJCWYSugsF/ENoQMtmm4n7C+\n",
"Izm0NpGwaOpNDNdI7caUKVN+e9999x2ctR6tgNmiiNmiiNmiyEh+nGcyB6NhYdNZxMV+MUx5KnC+\n",
"hG1qrxORNavVEetZCpxHSApYWFG8FzGvklGeDTbYYFLWOrQKZosiZosiZov6kOVK/lmEYI25hIy2\n",
"e8c1MOOBLUmsoBWRMwgL6baRsLFQMib9aOClmCNpADhXVdNmazUMwzDqTGaT/LH3cUiZ8scoRpkV\n",
"yiqGHmvItHpg3RVsY6ZNmzY/ax1aBbNFEbNFEbNFfajZwTjnJhPSWbyNkKdpkJAx9h5gwHufdoLe\n",
"yIiZM2fOz1qHVsFsUcRsUcRsUR+GdDDOufcSFhq9aQ/rBIPOuV7gaO/9/DrpZhiGYYxiqs7BOOcO\n",
"A64j7NlxFCEl+jsIK5SnANsDXYQVrxsDdzjndm6kwkZdyGetQAuRz1qBFiKftQItRD5rBdqBisku\n",
"nXPvI0Ro7eO9v7mWypxzPYRd/97TakNmAwMDamHKhmEY6RjJd2e1HsyWwIdrdS4A3vteQsjwdsNR\n",
"xjAMw2gfKs7BeO9/O5wKvffXDFsbwzAMo22o2zoY59xF9arLMAzDGP1UjSJzzr0VWOq9f9Y5dxBv\n",
"Tk5ZQCiTat8wDMPoXIbqwdxGSHkNYevQ31Y4fkOZBJNGa5LP53NZ69AqmC2KmC2KmC3qw1AO5geE\n",
"vawhLKjcjLBZWLnj3nIVGC1JLmsFWohc1gq0ELmsFWghclkr0A5UHSLz3v88cdnvvX+kkqxzrr/S\n",
"PcMwDKPzqHmS33t/zBAiF45QF8MwDKONqGc25TPqWFdD6O7t2i9rHQzDMDqFNMkuxwNfAT4CbJB4\n",
"VglRZBvUXbv6sydgqfwNwzCaQJp0/T8D9gGuBeYTsiknmVknnRrJylkr0CLks1aghchnrUALkc9a\n",
"gRYin7UC7UAaB/MRYFvv/aPlbjrnbq+PSg3FHAyQy+XyWevQKpgtipgtipgt6kOaOZgFlZxLZPuR\n",
"KtMEVspaAcMwjE4hjYO50Tn39ir3z0rTsIiME5HfishcEblZRHarIruqiPxORK6uIjNZRF4TkV2q\n",
"NGs9GMMwjCaRZojsm8C5zrmFhEWXLybuCfDBlG3PAlRVdxKRycBNIrK1qi5MConIGsBFwEND1Hci\n",
"sHQImZW7e7ukr6e/UsobwzAMo06kcTDdwCep3Oup+UtbRMYAhwEfB1DV+0VkHrA/MLtE/HXAEYII\n",
"Jleob0dgCfBMDc2vCLxWq66GYRjG8EgzRDYLOA14N7AFI0sVszmwdskzd1MmYaaqvqSqiwi9pGq6\n",
"nVBj2x0/TGZ5loqYLYqYLYqYLepDmh7MSt77L1W66Zw7NUVd68XXxYmyxcA2KeoAQER2B+5U1SdE\n",
"atp0zSb6Q56lfMY6tAo5zBYFcpgtCuQwW4yYNA5m/hD3bxlG+6XDaqm25YxDbccQh9pqZGyaNgzD\n",
"MIzhkcbBnOicOwX4gff+2TL3z6D2UOXCRP4E4NnE+dMp9AHYF7hcVZ9PlFV0UssWDU757yWv9OZ+\n",
"mls6bdq0+TNnzpwP5MvFvMcucq5MNaNevkzZqNK/zvK5fD4/q4X0MfnWkC/8XbSKPk2Tnz59Osl7\n",
"c+bMKSNWG6Ja29y8c+5hYE1gdYJTeJHwZf5Gqhjv/biaGg09j4VAj6peH8sGgEtV9ZQKzxwMHKSq\n",
"0xNlZwDbUZy0fy9hXme+qi7XqxkYGNCfLZl9K+D6evqHikhra/L5/KxcLjcraz1aAbNFEbNFEbNF\n",
"kYGBAZ0xY0aq0aUCaXowqwMXU7mHUHOqGFUdFJGzgEOB62OY8lRgHxGZGNvpVtXnhqjniOS1iDwM\n",
"HKWq11Z5zOZgDMMwmkAaB/Oo9/6QSjeHkSpmFnCGiMyNeuytqgtFZGNgS2A88By80VPZHthURP4E\n",
"fF9V7yhUJCJTgeOAicCJInKxqv6sQrsdH0WGTV4myWetQAuRz1qBFiKftQLtQBoH86Eh7k9L07Cq\n",
"LgXe5LBU9TGKUWaFsiNK5Uru30HIlFwLHe9gLM9SEbNFEbNFEbNFfUiz4VjVRYze+2UjV6cp2BCZ\n",
"YRhGE6jnhmOjhY7vwRiGYTQDczCGYRhGQzAHYxiGYTSETnQwHT8HY3mWipgtipgtipgt6kMnOhjr\n",
"wZRfwdup5LJWoIXIZa1AC5HLWoF2oKYwZefcCsAuwDuBdWLxM8A84Frv/WBj1GsIHd+DMQzDaAZD\n",
"OhjnXDfwS2CjCiKPOee+6L3vq6tmjcN6MIZhGE2g6hCZc24P4M/A/YQdLfcCPkxYdLlXLHsA+LNz\n",
"bvfGqlo3zMEYhmE0gaF6MMcB+3vv/1BF5n+dc58CvgtcVjfNGoc5GMMwjCYw1CT/qkM4FwC8938E\n",
"Vq2PSg3H5mAsz1KSfNYKtBD5rBVoIfJZK9AODOVgxjrnhkzBH2VGy0ZeHd+DsTxLRcwWRcwWRcwW\n",
"9WEoB3MdcIlzbutKAs65rYDeKDsa6HgHYxiG0QyGmoM5FrgGuMs59wTwIPBCvLc68DZgQ+Ae4IBG\n",
"KVlnbIjMMAyjCVTtwcQMyjsC3wGeB3YGPhqPnYFF8d6OQ2VbbiGsB2MYhtEEhlwH471/CTgJOMk5\n",
"NxZYK95a5L1f2kjlGoT1YAzDMJpAqlQx3vul3vsF8RixcxGRcSLyWxGZKyI3i8huVWRXFZHficjV\n",
"JeUrisiXReRqEcmLyI0i8oEqzXZ8D8byLBUxWxQxWxQxW9SHrFPFzAJUVXcSkcnATSKytaouTAqJ\n",
"yBrARcBDZerYCPgSMFVVl4jIB4FLRGSKqj5ZRr7jHQwhz1I+Yx1ahRxmiwI5zBYFcpgtRsyQPZiY\n",
"KuZhYAD4CWHi/9h4fhXwcJRJhYiMAQ4DzgZQ1fsJDmv/MuKvAw64ocy9F4DjVHVJrGcO8AqwU4Wm\n",
"zcEYhmE0gSxTxWwOrA3cmyi7G9ihVFBVX1LVRYCUubdIVc8vXIuIEJxIpaADm4MxDMNoAlmmilkv\n",
"vi5OlC0GtklRRzl2Bear6rUV7lsPxjAMowm0QqoYLbl+Uy+lVkRkHPAD4OAqYuZgDMMwmsBQPZix\n",
"zrlx3vtXqgkNM1VMYSJ/AvBs4vzplPUAbwyNnQnMVtV55WSWLRqcoi/rZrlcLj9t2rT5M2fOnA/k\n",
"y6WFiFEkuTLVjHp5ykxejib96yw/KZ/Pz2ohfbKUf+O5FtEnS/nC30Wr6NM0+enTp5O8N2fOnDJi\n",
"tSGqpR2IIs65swlRWl/23t9TQWYr4KfAE977w2puOEzyLwR6VPX6WDYAXKqqp1R45mDgIFWdXube\n",
"bOABVT1NRFYG1lPVxwr3BwYG9GdLZt8KPNfX018xHNowDMMoMjAwoDNmzBjWyFJmqWJUdVBEzgIO\n",
"Ba6PYcpTgX1EZCJwMdCtqs8NVZeIfANYAThXRFYFNiEEIZxQRtwm+Q3DMJpA1qliZhFGt+YCFwB7\n",
"xzUw44Et4ysEoTOAI4FtRORPIjI1lm8J/BD4IrCE4ADv5M1zOwVGS9ZnwzCMUU3VIbJSRnOqmMQQ\n",
"GcCOfT39tb9xwzCMDqWRQ2TLER3KguE01GKsBLyatRKGYRjtTKpcZG1ER8/DWJ6lImaLImaLImaL\n",
"+lA3B+Oc+2C96moCnb4WJpe1Ai1ELmsFWohc1gq0ELmsFWgH6tmDOauOdTWaTncwhmEYDafiHIxz\n",
"bpMU9Ui1ulqQjh4iMwzDaAbVnMJ8QqhvrdEDoykqy0KVDcMwGkw1B3MFsCnw9xrr+uTI1Wka1oMx\n",
"DMNoMNUczGeA64BjvfdPDVWRc27XumnVeDp9DiaftQItRD5rBVqIfNYKtBD5rBVoBypO8nvvHwX+\n",
"FzijxrrK7TbZqnR0D6Zc8rtOxWxRxGxRxGxRH4ZKFXMacLxzbshoM+/9jLpp1Xg6vQdjGIbRcIaM\n",
"/PLe39EMRZpMR/dgDMMwmsFQWyaf5Jyb1ixlmoj1YAzDMBrMUD2YLuBY59yjhPT5FwPXee9HU0hy\n",
"OczBGIZhNJih5mCmApOBU4H3AFcDC5xzZzrndnfOjdahpo52MJZnqYjZoojZoojZoj7UMgfzIPBj\n",
"4MfOuQ2Aj8fjEuBl59ylwJ+By7z3/22ksnWkox0MIc9SPmMdWoUcZosCOcwWBXKYLUZM2nT9C4DT\n",
"gNOcc2sC3cAngN8BOOeuAP7svT+v3orWmdHa8zIMwxg1DDt/mPf+OeBc4Fzn3CrA7gRn83Og1R1M\n",
"p/dgDMMwGk5dElR6718CLgQudM7V9OUtIuMIizinRD2+papXVpBdFTgd2EhVpw+3ngTWgzEMw2gw\n",
"NTsY59xUoAeY672/wjm3DvAnYCpwOXCo9/6/3vtad4qcBaiq7iQik4GbRGRrVV2YFBKRNYCLqJwp\n",
"oKZ6SrAejGEYRoNJsx/MF4DdgJfj9cnAzsD1wI7A8bVWJCJjgMOAswFU9X5gHrB/GfHXAQfcMMJ6\n",
"knR6DyaftQItRD5rBVqIfNYKtBD5rBVoB9I4mB2BLu/9dXHOZU/gl977mcD7gI+mqGtzYG3g3kTZ\n",
"3cAOpYKq+pKqLqL8tgE111NCR6frtzxLRcwWRcwWRcwW9SGNgxn03i+O5x8ExgO/AvDePw3UOjQG\n",
"sF58XZwoWwysm6KOkdTT6T0YwzCMhpPGwazknCvM2RwA3Om9vztxv9aNyZKUZgQYTh3DqcfmYAzD\n",
"MBpMmiiyvwP9MW3Mx4GvFW445/YDXkhRV2ECfgLwbOL86RR1pK5n2aLBKQDLnli27teu+9rGM2fO\n",
"nA/ky3WH40reXJlqTN7kTd7k21Z++vTpJO/NmTOnjFhtiGptacVi1NjvCfMtVwB7e+9fc879gTAJ\n",
"/zXv/Sk1NRom5xcCPap6fSwbAC5V1bJ1iMjBwEHJMOU09QwMDOjPlsy+NV5e29fT/9Wa3rhhGEYH\n",
"MzAwoDNmzBjW6FLNPRjv/TPAh8uU7w3snaZRVR0UkbOAQ4HrY3jxVGAfEZlISKrZrarPDbeeIVTo\n",
"6CGyfD6fs0nMgNmiiNmiiNmiPqSZg6mKc+4zKR+ZBYiIzAUuAPaOa1fGA1vGVwhCZwBHAtuIyJ9E\n",
"ZGoN9VSj0yf5c1kr0ELkslaghchlrUALkctagXYg9Up+59wKhOit5LMCHAWcWWs9qroUOKRM+WMU\n",
"o8MKZUekrWcIOroHYxiG0QzSrOTfDPg1sCvlez6jaY8YczCGYRgNJk0P5lfAasC3gWeAwZL7J9RL\n",
"qSZgDsYwDKPBpHEwWwFbVdrzxTn3/vqo1BQ6fQ7GMAyj4aSZ5H+s2oZi3vvD66BPs+j0Hkw+awVa\n",
"iHzWCrQQ+awVaCHyWSvQDqRxMH91zn2o0s24s+VooaMdjIVfFjFbFDFbFDFb1Ic0Q2QrAWc55+4E\n",
"7gJeTNwTYFo9FWswNkRmGIbRYNI4mO/G140Ju1eWMpqiyMZ393aN6evpLw1UMAzDMOpEGgdzh/f+\n",
"XZVuOufm1UGfRrOEEAk3BlgDqJopwDAMwxg+aeZgfjnE/dEQpvyfxPlamWlhGIbRAdTsYLz3Zyev\n",
"nXPjS+731kupBpLssayZmRYZEzOpGpgtkpgtipgt6kOqVDHOuS2AHxGSXq7inHsJuAz4pvf+oQbo\n",
"V28WJc47uQeTw8IwC+QwWxTIYbYokMNsMWJq7sE45zYn7AnzMeCpeP4UYW+Yv8f7rY45GMMwjCaR\n",
"pgfzfeAq4Eve+6cKhc659YGfxfv71le9umNDZIZhGE0izST//wD7J50LQLw+ABgNqWKsB2MYhtEk\n",
"0jiYl733r5a7Ectfro9KDcUcjGEYRpNI42Bed85tX+6Gc+5dwOv1UamhmIMJ5LNWoIXIZ61AC5HP\n",
"WoEWIp+1Au1AmjmY3wBXOud+CdxIWFOyNvA+4PPASfVXr+7YHAyWZymJ2aKI2aKI2aI+pHEwpwDv\n",
"AI4rc++8eL9mRGQccAYwJerxLVW9soLsfsBXCOlorlHVoxP3tgZOi3WsClykqt+v0Kz1YAzDMJpE\n",
"zQ7Gez8IHOScOwvYA1iHsPFYv/f+hmG0PQtQVd1JRCYDN4nI1qq6MCkkItsBJwPbEnogAyJypKqe\n",
"FkXOBS5X1eNEZC3gfhG5XVXLZXd+EXiNkOzyLd29XeP6evpfGYbuhmEYxhCkWmgJ4L2/Hrh+JI2K\n",
"yBjgMMIaGlT1fhGZB+wPzC4RPwzoV9VF8dlzgGMIvRaAbQjOClVdJCL3x7I3OZi+nn7t7u1aBKwX\n",
"i9YCnhzJezEMwzDKk2aSvyrOubkpxDcnzN/cmyi7G9ihjOwOJXL3ANuKyNh43Q90A4jIFsBkYE6V\n",
"tm0exjAMowlU7ME4546n9hT8AkxK0W6hB7E4UbaY0PMoJ5uUez62NxF4AjgcuEREHiBkSP6Cqt5W\n",
"pe3kPMzaKXRuG/L5fM4mMQNmiyJmiyJmi/pQbYjs+JR1DWc/mNJnJEXdBdk/Azeq6gdEZCPgBhF5\n",
"WlWvqlBX0sF0ag8mh4VhFshhtiiQw2xRIIfZYsRUczBV938pJeV+MIWJ/AnAs4nzpyvITkhcTyA4\n",
"nGdiBNkMYB8AVX1cRC4BPkdIa7McuVwuP/a9K711xS1XXH/c+LFL3zFh6iH5fH5xuV8qMZtqrow+\n",
"+dEuX6ZsVOlfZ/lcPp+f1UL6mHxryBf+LlpFn6bJT58+neS9OXOqzTgMgaqWPfbaa6/DKt0bqTxh\n",
"7udZ4P2JsgHgK2VkTwF+nbg+ALgjnk8FBoE1EvdnA5eU1jNnzhxVVWZevMcBMy/e45Z4fC3Ne2yX\n",
"4+qrr56VtQ6tcpgtzBZmi+pH4btzOEfFSf7S/V9qYFkKpzYInAUcChDDlKcC54vIRBG5TkQKw1e/\n",
"BvYQkbVi9NnBhPUzECb/nyAm2RSR1YCZVJ/kt03HDMMwmkDdosiAo1LKzwJEROYCFwB7a1gDMx7Y\n",
"Mr6iqncBRwNXADcBt6nq6fHeUqAH2FtErgWuAS4ETq3SrkWRGYZhNIFqUWSDhLmOShPvpaSa5I/O\n",
"4ZAy5Y9RjDIrlF1AcELl6rkV2DVF07aa3yYvk+SzVqCFyGetQAuRz1qBdqDaJP/ThKGoWh3MZ0eu\n",
"TlPoeAdNDqYGAAAfs0lEQVRj4ZdFzBZFzBZFzBb1oZqDedx7f0KtFTnnPloHfZrB84nzCd29XSv0\n",
"9fTXPH9kGIZh1Ea1Sf4d01TkvS+3Cr/l6Ovpf42ikxlDWLBpGIZh1Jl6poqptnq+1Xgscb5JZloY\n",
"hmG0MamSXTrnNiCEBL8NGJu4JcCmddSr0TwKvD2ebwLcnKEuhmEYbUnNPRjn3FRCQsqjCetXpgMf\n",
"APYGDgL+2wgFG8QjifOO68HEVbwGZoskZosiZov6kGaI7CTgS977DYC7vfebee8nAasB3wN+0QD9\n",
"GkXSwYymnle9yGWtQAuRy1qBFiKXtQItRC5rBdqBNA5mQ+/970oLvffLvPfHAx+pn1oN59HEeSc6\n",
"GMMwjIaTxsEkQ3lXdM6VPrtRHfRpFslJ/g27e7tWykwTwzCMNiWNgxnjnJsUzx8FvlW44Zw7imHs\n",
"jpkVcZvkp+LlCsCGGapjGIbRlqRxCgPAXOfc+4DTgT7n3NeJ2YxJv39M1jwKrB/PN2X5eRnDMAxj\n",
"hKTpwXwf2A140nt/KSGr8b+ABwi9mR/UXbvG0skT/fmsFWgh8lkr0ELks1aghchnrUA7IKrD2Yhy\n",
"9DEwMKAzZsx4I69ad2/XPsDX4uXFfT39o81BGoZhNJzS7840pFkHc9pwGmhhOnotjGEYRqNJMwez\n",
"m3Nulyr3FXiRsEZm6cjUagoWqmwYhtFA0szBbAFcTRibLHdcA9wK/Mc5N6teCjaQBcDr8Xxid2/X\n",
"KlkqYxiG0W6kcTAHEtaPnAB8gpAm5pPAicCdwJ7AXsApwOedc5+vr6r1Jabon58ompqRKoZhGG1J\n",
"GgczE+jy3p/gve/13ue99xd772cREmB2e+8v8t4fB3wI+Fy1ykRknIj8VkTmisjNIrJbFdn9ROSW\n",
"KHdymfuHi8iNInKtiNwuItWG8pL8PXFe6zOjHsuzVMRsUcRsUcRsUR/SOJi3ee/vKnfDe38nsF3i\n",
"eh4w1Or4WYCq6k4EB/UHEVm3VEhEtgNOJjitdwPbi8iRift7AR8DdlbVXYDZFNe3DMW1ifP3d/d2\n",
"DStSYhSSy1qBFiKXtQItRC5rBVqIXNYKtANpHMxmzrnVyt1wzq0ObF5S/FqlikRkDHAYcDaAqt4P\n",
"zAP2LyN+GNCvqos0xFSfAxyRuP9d4CRVXRbrOk9VfW1viduBF+L5+sCWNT5nGIZhDEEaB/N34G/O\n",
"uZ2dcysDOOdWds7tCvwNmFsQdM4dDrxapa7NgbWBexNldwPldsXcoUTuHmBbERkbezzbAluKyEAc\n",
"IjuiTB1lifMwNyaKOmaYzDAMo9GkcTBfJmw0dg3wsnNuCfAyIbJsEnAUgHPuEuBMgtOpxHrxdXGi\n",
"bDHwpiGyKJuUe56wwdnE2C7Ap4APE4IOviEin6rxPcHyw2Q7p3jOMAzDqELN62C89/c5594BfIWw\n",
"2djawH+AOcAp3vtno9zHUrRfmkag0hxIuXQDQnFXzV+p6uvAMyLye8KGaH+sUYcbCeHKKwLbdPd2\n",
"rdPX0/9Mjc8ahmEYFUiVAdl7/zRwbB3aXRhfJwDPJs6friA7IXE9geBwnkmUJ59bQIWtA3K5XL5w\n",
"Pm3atPkzZ86c/9UJX8/Pfv7Ht1McntseuDxGkeTKVJNP1vNG4SiSp0yepdGkf53lJ+Xz+VktpE+W\n",
"8m881yL6ZClf+LtoFX2aJj99+nSS9+bMmVNGrDYyyUUWJ/kXAj2qen0sGwAuVdVTSmRPAVZT1cPj\n",
"9QHA0ao6VUTGEnpR+6pqX7z/XeD9qvqhZD3V8ul093Z9Fvh0vPxjX0//j+v1Xg3DMEYzDclF5pz7\n",
"SUzHXzPOOXHO/c45t281OVUdBM4iDGUhIpMJCx3PF5GJInKdiKwZxX8N7CEia0XHdDBwRqxnKXAe\n",
"cFCsZzxhsee5afQmZIUu8I6UzxqGYRhlqDbJfzxwsHPuj8657arIAeCcyxEmzFfx3l9QQ9uzABGR\n",
"ucAFwN6quhAYTwgXHg+gqncBRwNXADcBt6nq6Yl6jgZeEpF5hD1rzlXV82toP8mdifPJ3b1d41I+\n",
"bxiGYZRQcQ7Ge/+ic2468Afgn865BcB9hKGtVwiT7G8BNgC2IcyH/IYhVvAXiL2PQ8qUP0YxyqxQ\n",
"dgHBCZWr57+ENDbDpq+n/4Xu3q6HCOHTKxLez20jqdMwDKPTqTrJ771f6JybAXQRhqF2ZvmJISXk\n",
"87oIONN7f3Nj1GwK/6K4WPQdmIMxDMMYEUNGkXnvFbg0HjjnViVskTwIPOe9f6WhGjaPfxJSzgC8\n",
"PUtFGk0+n8+ViyzpRMwWRcwWRcwW9SFVmDKEoTPCvi/txj8T51O7e7ukr6e/Xbf7zGFbwhbIYbYo\n",
"kMNsUSCH2WLEpFnJ3+48AiyJ5xOosJbGMAzDqA1zMJG+nv5Blg9XHjJyzjAMw6iMOZjluSdxPjkz\n",
"LQzDMNoAczDLc3/i3FL3G4ZhjABzMMvz78R5O/dg8lkr0ELks1aghchnrUALkc9agXZg2A7GOTfU\n",
"jpWjkccJWxAArN3d27V2lso0Cgu/LGK2KGK2KGK2qA+pwpSdc1sRUsjsRthQbEPn3I+Am733FzVA\n",
"v6bS19M/2N3b9QDFdTBbkthIzTAMw6idmnswcS+YfxBW9T9OcUvkucBs59zM+quXCZ0yTGYYhtFQ\n",
"0gyRnUTINba+9/6dwCIA7/0lwB6EpJPtQNLB2ES/YRjGMEkzRLat9/6j5W547+9yzq1SJ52yJhlJ\n",
"Nrm7t2sFYIW+nv5Xs1LIMAxjNJKmB/P6EPfXGYkiLcQDFLdo3gK4Cvhrd2/X5pUfGV3E3ewMzBZJ\n",
"zBZFzBb1IY2DWeScO7zcDefcPoR5mVFPX0//f1n+vaxCSB3z8Ww0agi5rBVoIXJZK9BC5LJWoIXI\n",
"Za1AO5BmiOxHwEXRyVwDrOOcO46wl30X0NMA/bLi38DGJWWTMtDDMAxj1FJzD8Z7fzHwWWBr4OvA\n",
"hsAJwC7AZ7z3lzZEw2y4pkzZpGYrYRiGMZpJtdDSe38W8FbgI8AB8XUj7/05aRsWkXEi8lsRmSsi\n",
"N4vIblVk9xORW6LcyRVkJovIayKyS1pdynAZcCTwGWBZLNvAtlI2DMOoneHuB3NFHdqeBaiq7iQi\n",
"k4GbRGRrVV2YFBKR7YCTgW2B54ABETlSVU8rqe9EYGkd9CLuA/MPgO7erieATeKtTQnbRhuGYRhD\n",
"ULODcc5tRtgy+R7v/c0xLPlUYCpwOfAt7/1gLXWJyBjgMOLEuareLyLzgP2B2SXihwH9qrooPnsO\n",
"cAxwWqK+HQl7uTxT6/tJwXzaz8Hks1aghchnrUALkc9agRYin7UC7UCaIbIvA98hzL1AWHh5IMFJ\n",
"fRY4NkVdmwNrA/cmyu4mBAyUskOJ3D3AtiIyNlE2izAf1AjmJ84nNaiNpmJ5loqYLYqYLYqYLepD\n",
"GgezM7Cr9/4S59zKBOdyvvf+7cC7gX1S1LVefF2cKFsMrFtBNin3PCDARAAR2R24U1WfSNF+GuYn\n",
"zic1qA3DMIy2I42DUe/9gni+K7AG8HMA7/39FCfD01C6573UKAcgcajtGOCHw2i7Vh5JnG/awHYM\n",
"wzDaijST/Mn0/PsAD3nvb06U1TT/EilM5E8Ank2cP11BdkLiegLB4TwD7AtcrqrPJ+5XclLLdXun\n",
"TZs2f+bMmfOBfLnucFzJm/v06keM/X9Lzt8QYAUZs+6cq+dM/+D0D15dSb5Ms1XrN3mTN3mTbyX5\n",
"6dOnk7w3Z86cMmK1IarlOgdvxjnnCRPpDxFS9n/fe39ivJcD/s97/+6aGg09j4VAj6peH8sGgEtV\n",
"9ZQS2VOA1VT18Hh9AHC0qk4VkTOA7Shmdn4vYb5mvqout/J+YGBAZ8yYUdH5VKO7t+tKYM14ObOv\n",
"p39BNXnDMIx2YSTfnWmGyI4F/gf4HvBP4BQA59zPCVFkfbVWpKqDwFnAoRDWsBCi0c4XkYkicp2I\n",
"FL7Qfw3sISJrRcd0MHBGrOcIVX2/qk5X1enAU8BRpc6lDsxPnH+4u7drWndv17AM3gpYnqUiZosi\n",
"ZosiZov6kGYl/0Pe+62Adb33O3jvl8Rb3wemENaqpGEWYR5lLnABsHdcAzOekCZ/PICq3kXYCuAK\n",
"4CbgNlU9PVmRiEwVkQsJE/8nishRKXUZivmJ8y8AvyI6x1FKLmsFWohc1gq0ELmsFWghclkr0A4M\n",
"Z6HlsyXXCwGcc9sCd9Vaj6ouBQ4pU/4YxSizQtkFBCdUqa47gD1rbXsYzC9TtjNwdgPbNAzDGNWk\n",
"ShUzBL+rY12txg0U53kKbJSFIoZhGKOFNCv5r6Z8uDCEyK223V64r6d/fndv156EIbgzgRWACd29\n",
"Xav39fS/kK12hmEYrUmaHswOBEcyJnGsAWwPbAXcUnftWoi+nv4n+nr672D54TLrxRiGYVQgzRzM\n",
"A9776aWFzrlxwDeB2+qmVWvzOGGnSwh7xtydoS7DJZ+1Ai1EPmsFWoh81gq0EPmsFciK7t6utQmp\n",
"t54DHjtqta8Ou640DmaPcoXe+1eA451zeeCSYWsyeng0cV66KdmowPIsFTFbFDFbFOlwW6xDWFMI\n",
"YQv5YZMmTHmoxYWdMlz0WOJ8VDoYwzCMKiQzpzw3korSTPJvUqZYCN5uP+DFkSgying8cd4pTtUw\n",
"jM5hzcR5cxwM5deCFFgMuJEoMoqwHoxhGO1MJg7mceA4lk8muQxYAPzDe98p4bpPA68CKwNrdfd2\n",
"rdLX0/9SxjoZhmHUi+QQ2fMVpWogjYOZ470/dySNtQN9Pf2DcRvlzWLRxiy/IVrLk8/ncx0+ifkG\n",
"ZosiZosiHW6LuvVg0kzyj+bcW/VmtA+T5bJWoIXIZa1AC5HLWoEWIpe1AhnSfAdjLMdodzCGYRiV\n",
"yGSIzCiSdDDd3b1d04DVgNXjsSLg+3r6f5mFcoZhGCPAejAZk3QwGwHvAbaJ56sDbwEOjitiDcMw\n",
"RhPmYDLmTmrrOtrwmWEYo4bu3q4VCT+SISQ3HlF0sDmYYRDDkg8i7O75HeBLhJ02PwlckxB9a9OV\n",
"q4181gq0EPmsFWgh8lkr0ELks1YgI9ZInC/u6+lfNpLK6jYH45z7hff+i/Wqr9Xp6+l/AniitLy7\n",
"t+thYNd42ZIOpoPDL9+E2aKI2aJIB9uibsNjUMXBOOd2pfL+L6UI8JG0jYvIOOAMwpbLKwLfUtUr\n",
"K8juB3wl6nSNqh4dy1ckbGP8sajHysB3VPWqtPrUiaTTaUkHYxiGUYHmOBjg6pR11eqMkswCVFV3\n",
"EpHJwE0isrWqLkwKich2wMnAtoQ3PSAiR6rqaYSJ9S8BU1V1iYh8ELhERKao6pPD0GmkmIMxDGO0\n",
"0jQH8yBwOMunhqnGWWkaFpExwGHAxwFU9X4RmQfsD8wuET8M6FfVRfHZc4BjgNMIk1DHqeqSWM8c\n",
"EXkF2Am4KI1OdSLpYCwZpmEYo4m6rYGB6g5mjvf+mir3l8M5Nydl25sDa7N8mpW7CTtnlrIDy+81\n",
"cw+wrYiMjU7n/MINESkMkz2TUp968RTwOsG2E7t7u8b19fS/kpEuhmEYaahrD6ZiFJn3/nNpKkor\n",
"D6wXXxcnyhYD61aQTco9T+hZTSwjuyswX1WvTalPXYhRF08lijbMQo9q5PP5XNY6tApmiyJmiyId\n",
"bIukg2loD+ZNOOfGEsJxdwPGeu9nOuf2B2703j80TB1K524qDcmVm+NZTjYGDfwg6pglT1AcHtsI\n",
"GK5tGkWOzg3DLCWH2aJADrNFgRydaYumzcEsh3NubYLBt41FhV/pmwI/cc59wHt/V4q2CxP5E4Bn\n",
"E+dPV5BNjg1OIDicN4bB4tDYmcBsVZ1XrsFk6OG0adPmz5w5cz6QLxeSGH/B5MpUM6T8VittPeXx\n",
"1x/fEGD9Fdf/IPCm3tRI6h+pfJmyTPXJWD6Xz+dntZA+Jt8a8oW/i1bRpynyE8ZM2PqZJc+s9vrL\n",
"r69297n37ikfl53mzEk7+1FEVGsL/nLO/YqQDuXbwD+Bq73374r39gT29t7vWXPDYZJ/IdCjqtfH\n",
"sgHgUlU9pUT2FGA1VT08Xh8AHK2qUxMys4EHVPU0EVkZWE9V30jpMjAwoDNmzKg1YGFEdPd2HUiI\n",
"bAP4Q19P/8nNaLdW8vn8rFwuNytrPVoBs0URs0WRTrVFd2+XJ8yPA+zb19P/75F8d6ZZyb8b0OW9\n",
"v9Z7v9zYnPf+Qor7o9SEqg4SIs8OBYhhylOB80VkoohcJyKF7tqvgT1EZK3omA4mrJ8hPvsNYAXg\n",
"XBFZFXhbod6MsFBlwzBGI82Z5C/D0iF2rVx1GO3PIoxuzQUuAPaOa2DGA1vGV1T1LuBo4ArgJuA2\n",
"VT2d8PCWwA+BLwJLCGHLdzK8dTn1whyMYRijiu7erjEsnyqm6ZP823vvbytTPpWwfXIqVHUpcEiZ\n",
"8scoRpkVyi4gOKFS2X/TejnVlnMw3b1d22emSRl2Gvc/T8zu/XFL6ZQVZosiZosiCVv8F/h3X0//\n",
"YNY6NYHVKX6XvtjX0//aSCtM42DOBa5yzp1BmLQe75zbmbBG5evAT0aqTLvQ19O/pLu36wXCBzaW\n",
"EHzQMsx95YasVWgZzBZFzBZFSmxxJfDNjFRpJnUdHoN0DubHwNsJK+iPiWWFhZi/482r7zud+4Ad\n",
"s1bCMIwRs1t3b9eP+nr6Fw8tOnqIQ2IbEYK3tgbelbjdXAfjvV8G7Bd7MLsD6xDChPu99/bT5838\n",
"BPg0sFbWihiGMSw2pzgnMZUyyw1GC929XeOALeIxmRAItTWV587LLRdJTep0/d7764Dr6tF4O9PX\n",
"0/8A8I2s9TAMY3h093Z9BdgvXo4KB9Pd27UyoVcyieBECsfG1J5X8hHCqNSIqed+MBd57z9Zr/oM\n",
"wzAy5g6WdzAtQXdvlxBGkDaNxyaJ8w1JF/T0PCEHZOG4p6+nv255HNOs5D+IyqG/QvkklUYLks/n\n",
"cx28odJymC2KmC2KxJXudySKtunu7Vq5r6f/1Wa0393btRrBWbw1vibPNwDGpaxyEHgUeAC4P/G6\n",
"oK+nv2FLOtL0YH7TKCWMppOjM/MslSOH2aJADrNFgVxfT3++u7frMcLw0sqEOYs7qj82NN29XWMJ\n",
"PZD14uu68XxdgvN4K7DaMKtXYAFhmOtBgiN5AHi4r6d/6cg0T08aB3MPsAfLj+OtBmwHHAB8v456\n",
"GYZhtAJ3EBwMhGGyig6mu7drRUJQT+GYSNGJJJ3JhEp1pGAJwYkUjvnx9fEsHEkl0jiYn3vvHylT\n",
"/i/n3JXAqcCN9VHLMAyjJbgd+Gg83zWub0s6kbUT52uUrWF4vELoiTxJWLj9ZPK8r6d/SR3bahhp\n",
"wpR/VeXes865KfVRyTAMo2VI9limUp/J/tcJGeQXljmeIjiRRY2cG2kWdYkic85tT326fYZhGK3E\n",
"fMJ6v3VqkB0kRGUtAv4Tj3JO5Lm4MWHbkyaK7GrKbw62DrAVlipmNJHPWoEWIp+1Ai1EPmsFWog8\n",
"QF9Pv3b3dh1PmGeGovNYVOb8+U5xHLWSZj+YJcAtLD/Jv4wwLnglcH5c7d+SNHM/GMMwjHZhJN+d\n",
"FXswzrkPA1OAX3rvB4EHvPfTh6mjYRiG0WFUW/F5MiHdQIEvNlYVwzAMo52o5mDUe//V2HsB+EW1\n",
"ipxz36ufWoZhGMZop5qDWcE5Nz5FXR8dWsQwDMPoFKpFkV0L3O2cmwssBTZxzp1TQVYICddqRkTG\n",
"AWcQ5nlWBL6lqldWkN0P+Aohiu0aVT16OPUYAcs5VcRsUcRsUcRsUR+qOZijgZeAGYSdzlYFKk3y\n",
"C7BKyrZnAaqqO4nIZOAmEdlaVRcuV7HIdoT5oG0Jm+AMiMiRqnpamnqM5chhIakFcpgtCuQwWxTI\n",
"YbYYMRUdjPf+JYKTAcA5N897/65K8s65ebU2KiJjgMOAjwOo6v0iMg/YnzfvjHkY0K+qi+Kz5xB2\n",
"1DwtZT1G5C9/+cukXC6XtRotgdmiiNmiiNmiPqTZN+BbI7yfZHNCDp97E2V3Uz7l/w4lcvcA24rI\n",
"2JT1GJFbb711UtY6tApmiyJmiyJmi/pQs4Px3l82kvslrBdfk3tcLyZkGi0nm5R7njAkV8hUWms9\n",
"hmEYRhNJ04NpBOVSz9QiVypbaz2GYRhGk6g5VUxdGw2T8fcB66rqs7HsF8Daqrpviez1QK+qnhyv\n",
"pwH/AN5CiFyrqZ6BgYFRn5nUMAwjC+qeKqbBPEhIDrcVcH0s2wa4tIzszVGOhNydqrpURGqux/KQ\n",
"GYZhNJdMhshUdRA4CzgU3ujRTAXOF5GJInKdiKwZxX8N7CEia8WosYMJ616q1tPEt2MYhmGUIZMh\n",
"MoAYBXYGofexIvBNVZ0jIhsTsja/S1WfjLL7Al8l7Ldwjap+fah6mvpmDMMwjDeRmYNpBPXKDtAO\n",
"1GoLEXk/8A1gPGHTuL8Bx2kb/WEMJ9uDiOSBh1X1kMZr2DxS/o9sTdjnaTXCYutLVPXbzdK10aT4\n",
"H1mRYIf3E7KaPAscUfgB3A7Eue0/At9T1XOryKX73lTVtjmAHwG/ieeTCZsBrVtGbjvCftdrESLO\n",
"rgKOzFr/jGxxNfDReL4KYZ3Rl7LWPwtbJOS7COHw52Ste4Z/F2sAdwFT4vVawHlZ65+RLY4E7gfG\n",
"xutzgQuz1r+OdugG/gDcChxYRS7192bWYcp1I7Gq/2wIq/qBwqr+Ut7IDqDBcucARzRL10aT0hYX\n",
"q+qlUe4lQoDEh5qkasNJaYuC/NcIfxNtFRiS0haHA9er6n1RdpGqHtgsXRtNSltsA9yuqkvj9U2E\n",
"1FXtwjxV3Rt4YQi51N+bbeNgqF92gHagZluo6s9LisYT9g1vF9JmeziA4GQXV7g/mkljiw8AL4jI\n",
"H0TkehH5tYhMaIaSTSKNLS4D3iMia8fhst2B/sar2BxU9bEaRVN/b7aTg6lXdoB2YFgZDkRkBULv\n",
"5fQG6ZUFNdsi/qMcDpxKm/VeImn+LjYDPgOcoKqFuYf/11j1mkrNtlDVfuCnhC/Xh4BXCPOWnUbq\n",
"7812cjAF6pUdoB1Im+HgaMKQ2c0N0idLarHFF4Cz41BI2wQ5lKEWW4wFrlLVe+L1bODDIrJ+QzVr\n",
"PkPaQkT2JyyPeBuwKfAyYXlEJ5Lqe7OdHExhWCfZjZ8APF1BtlROgWcao1rTSWMLAERkD2B74NgG\n",
"6pUFNdlCRNYAPgacVyhqvGpNJ83fxaKS8gXxdaMG6JUFaWzxRUKAw+I49/Az4CARWbXBOrYaqb83\n",
"28nBJFf1F9iGkAmglIrZARqnXlNJYwtE5L3Al4ADVFVF5G2NV7Fp1GqLHYHVCfsNXQ0cBHxERK6K\n",
"9mkH0vxd3M7yw0WFYZB2Cc1NY4uVgNcT168TfoCs1DDtWpP035tZh8jVOdzuh8TQUkLY4bOEf5KJ\n",
"wHXAmvHetoR/lLUITnYA+FzW+mdki62BvxN+ma4aj99lrX8Wtih5ZhYxhLWdjhR/F+8ljLG/NV5/\n",
"F7gya/2bbIsJ8d7/xesVE9e3Zq1/A+yRBw5KXI/4ezPzN1VnA40FfgPMJXjbD8byjQld3w0TsvsS\n",
"Mgb8A/hx1rpnZQtCDrdlhCwJheOhrPXP6u8ilp9FWAPyMPCnwhdLOxwp/0cOBv4JXBPtsE7W+mdh\n",
"C0Ji3TOizA2EqLItsta/jnbYLn6+C+P34ZlV/iZSfW+21Up+wzAMo3VopzkYwzAMo4UwB2MYhmE0\n",
"BHMwhmEYRkMwB2MYhmE0BHMwhmEYRkMwB2MYhmE0BHMwhmEYRkMwB2MYTcQ5t4Vz7pgmtHOcc27j\n",
"RrdjGNUwB2MYTcI59z7Cqvh/1LnePznn/l5SPA+Y65ybVs+2DCMN5mAMowk45yYClwA/8N7n61z9\n",
"E8D8ZIH3/lLCHiZ9zrk16tyeYdTEilkrYBgdwrHAazRgHxHv/Zcr3PplbPfrwHfq3a5hDIXlIjOM\n",
"IXDOnQfsQcgi+29gT0Km2dMJW+/eBXR77x+v8LwATwFzvPf7Jcp3B04ipD2/gLDX+37AFoR08l8k\n",
"pJSfDWxJyHT9C+/9TxN1XBXlNwYmee8fLWn7QuC93vt22cfFGEXYEJlhDIH3/kCgi5B1+gbv/Z1x\n",
"mOsq4Dve++0rOZfIFGAd4L6Sei/z3r+LkAJ9N+Bl7/0uBIcxFuglOKDDvfdTgWOA2XEup1DHBwip\n",
"9CtxD7Chc25ymvdsGPXAHIxh1ID3/u/AD4BDnHOfdM59FNjae//jGh4vbOC2sMJ9AZ7z3p8X23qF\n",
"kD59U6DXe1/YB/3PwH+BD5d5vhKFHRrbaRM5Y5RgDsYwaud7hAiwXwE/AQ6s8bnCJPsrVWTuKbn+\n",
"T2m5915j+QY1tgth//ikDobRNMzBGEaNeO+XAYcS5mIWDjEslmQwvlbqaSjwYpkyypQPki44p/A/\n",
"PlhVyjAagDkYw0jHIYRdQP/HOfeVGp9ZFF/f0hiVqrJKfP1PVSnDaADmYAyjRpxz04H3A9OBC4GT\n",
"nHNvr+HRwjBXmqGtelFos3QIzjAajjkYw6gB59yahLDkA+JQ2RGEnsn5zrmx1Z6NocMPEvY+L4dQ\n",
"efistDyNLMA7gHu8909W09EwGoGtgzGMIXDOnUiY0F8DuMR7f7Bz7rvA5wnrYR4GfuK9P71KHV8H\n",
"vg1s4r1/IZa9mxAwsDXwEvAIsANwNvABYCPgXuBU4JYKsgOEtTgF2d97738Y6187yh3rvf9lvexh\n",
"GLViDsYwmkDs5dxGCDv+dpPa/CmwM/Du2OsyjKZiQ2SG0QS890uB3YEe59ynG92ec+5rwK5AlzkX\n",
"IyvMwRhGk4hzMTsCzzShuUeA93jvn2pCW4ZRFhsiMwzDMBqC9WAMwzCMhmAOxjAMw2gI5mAMwzCM\n",
"hmAOxjAMw2gI5mAMwzCMhmAOxjAMw2gI5mAMwzCMhvD/Adh63DsvhCR1AAAAAElFTkSuQmCC\n"
],
"text/plain": [
"<matplotlib.figure.Figure at 0x7f0ba45469d0>"
]
},
"metadata": {},
"output_type": "display_data"
}
],
"source": [
"# plot fuel consumption\n",
"fig,ax = plt.subplots()\n",
"c4 = \"#4daf4a\"\n",
"consump = r(v, traceu[:-1])/v * dx * 100 * gallonperj\n",
"ax.plot( xs[:-1]/1609, consump, lw=3, alpha=0.8, c=c4 );\n",
"ax.set_ylabel(\"fuel use (gallons/100 mi)\");\n",
"ax.set_xlabel(\"x (mi)\");\n",
"gallons = consump.sum()/100.0\n",
"mpg = 100./consump.sum()\n",
"ax.set_title(\"tot={:0.2g} ga={:.1f} mpg\".format(gallons, mpg));"
]
},
{
"cell_type": "code",
"execution_count": 250,
"metadata": {
"collapsed": false
},
"outputs": [
{
"data": {
"image/png": [
"iVBORw0KGgoAAAANSUhEUgAAAbAAAAEbCAYAAAC7ldEHAAAABHNCSVQICAgIfAhkiAAAAAlwSFlz\n",
"AAALEgAACxIB0t1+/AAAIABJREFUeJzt3Xm8I1Wd9/HPD5B9XwVBGAEhiKPYoGxC2h5U1MaVE8Iu\n",
"MgqCy4yDMz4j2m6Dz+Mjo7iAuKAohBzRkUXRx75QKLgBtgsYQdBWEVlkaUCgoeE8f1SFVF9y7829\n",
"ndSpJN/363VfXamcVH7963R+91SdOsdCCIiIiAyb1WIHICIiMhcqYCIiMpRUwEREZCipgImIyFBS\n",
"ARMRkaGkAiYiIkNJBawHZra2mX3ZzH5sZteY2UGxYxIRGXdrxA5gSCwCQghhHzPbGfiJmVVCCHdG\n",
"jktEZGypBzYDM1sNeBPwRYAQwu+AJcCRMeMSERl3KmAzexawGfDb3L7fAHvGCUdEREAFrBdbZX8u\n",
"y+1bBmwZIRYREcmogPVu8qSRFiUKEREBVMB60R6osXFu38bAHRFiERGRjEYhzuwW4B5gV+CqbN9u\n",
"wKX5RhMTE5rWX0RkDhYsWDCnM1oqYDMIITxhZp8HjgOuyobRPw+oT25759knXwe8ot5sjfXw+l12\n",
"2eXLN95447Gx4ygD5aJDuehQLjpW5Zd/nULszSLAzOzHwPnAYdPcA7Z+YVGV1NZbb71D7BjKQrno\n",
"UC46lIv+UA+sByGE5cAbe2y+3iBjERGRlHpg/Tf2PbB58+YtjR1DWSgXHcpFh3LRHypg/Tf2PbCF\n",
"CxcujR1DWSgXHcpFh3LRHypg/Tf2BUxEpAgqYP039qcQgSR2ACWSxA6gRJLYAZRIEjuAUaAC1n9j\n",
"3wOrVqtJ7BjKQrnoUC46lIv+UAHrP/XAREQKoALWf2PfAxMRKYIKWP+pByYiUgAVsP5TARMRKYAK\n",
"WP+N/SnEJEmqsWMoC+WiQ7noUC76QwWs/9QDg2rsAEqkGjuAEqnGDqBEqrEDGAUqYP039j0wEZEi\n",
"qID1nwqYiEgBVMD6b91GrbJ67CBEREadCtg0zGwNM3unmV1hZomZ/cjMXtLDS9cdeHAiImNO64FN\n",
"b1vg7cDzQggPmNk/AReZ2S4hhNumed36wAOFRFhOSewASiSJHUCJJLEDKJEkdgCjQD2w6d0PnBpC\n",
"eAAghLAYeATYZ4bXjfV1MM3z1qFcdCgXHcpFf6gHNo0Qwj3Aee3HZmbAmsBdM7x0rAuYiEgR1AOb\n",
"nQOBpSGEH8zQTveCiYgMmApYj8xsbeAjwLE9NFcPTERkwMb6FKKZXQbsP8XTV4cQXp61M+Bs4PQQ\n",
"wpKpjnfvitV3Afjl39c7/dp3vevabNnwpNv57mwqmWqXw6i92qu92o9s+/nz55N/bvHixV2a9cZC\n",
"CHN+8bgws9OBm0MInzWzNYGtQgh/zreZmJgId5598nXZwzPqzda5hQdaEkmSVHWROqVcdCgXHcpF\n",
"x8TERFiwYIHN5bU6hTgDM/t3YHXgK2a2PrATcNwMLxv3U4jV2AGUSDV2ACVSjR1AiVRjBzAKVMCm\n",
"YWbPBk4D3kZ6X9f9wPXATN1WDeIQERmwsb4GNpMQwk3MrciPew9MRGTg1AMbDPXAREQGTAVsMNQD\n",
"ExEZMBWwwRj3ApbEDqBEktgBlEgSO4ASSWIHMApUwAZjrE8hanhwh3LRoVx0KBf9oQI2GOPeAxMR\n",
"GTgVsMEY6x6YiEgRVMD66/Hsz7UatcrTokYiIjLiVMD66++5bZ1GFBEZIBWw/ro/t71RtCgiyyby\n",
"FJSLPOWiQ7noDxWw/rovtz22BQzN85ZXjR1AiVRjB1Ai1dgBjAIVsP5altveOFoUIiJjQAWsv/IF\n",
"bJx7YCIiA6cC1l86hSgiUhAVsP5SD0xEpCAqYD0ys53N7DEzO2CaZroGlkpiB1AiSewASiSJHUCJ\n",
"JLEDGAUqYL37ILB8hjbqgaF53vKUiw7lokO56A8VsB6Y2V6kKzLfNUNTXQMTESmIClhvFgEf6KGd\n",
"emAiIgVRAZuBmR0MXB9C+EsPzXUNTESkICpg0zCz1YB3A6f1+JL8KcSNG7WK9T8qEREBWCN2ADGZ\n",
"2WXA/lM8/SPgXOB7IYR8YZqyKH3ujq2+9/pN7955dQurrbPWWsufedSiDydJ8v1uF2yzudCqXQ6T\n",
"DHt7eOpF6mGKv8/tdwCWliieaO2TJKnmt2PHE7n9DqSfi7LEU1j7+fPnk39u8eLFXZr1xkIIc37x\n",
"qDOzs4DdgceyXXsDvwWWhhBem287MTERFixYYI1a5VLg6dnuV9ebrV5OPY6UJEkWVavVRbHjKAPl\n",
"okO56FAuOtrfnXN57Vj3wGYSQjgh/9jM/gC8I4Twg2letoxOAdsIGLsCJiJSBF0D64GZPc/MLgQ2\n",
"Bz5oZu+YprlGIoqIFEA9sB6EEH4JvKHH5roXTESkAIUUMOfczsBOpF/oTwB3Ay3v/W1FvH/BNJRe\n",
"RKQAAytgzrmNgFOAY4FtpmjTAs4EzvTePz6oWAqmU4ia5y0viR1AiSSxAyiRJHYAo2AgBcw5Nw+4\n",
"GFiL9B+qBdwBPEx63W09YGvgH4GPAm90zr3Se3/7IOIp2NifQtQ8bx3KRYdy0aFc9EffC5hzbjvg\n",
"IuC/gU977x+Zof1mwKnAZc65F3nvH+13TAVTD0xEpACD6IEdC5zgvb+0l8be+7uBdzrn3gUcAlw4\n",
"gJiKpGtgIiIF6HsB895/aI6v+3i/Y4lEPTARkQJEvw/MORc9hj5baT7EaFGIiIy4MhSPa2MH0Gdj\n",
"P4gjmwdNUC7ylIsO5aI/iroPbF/gZaQjD/PvacD2RcRQoIeAFaR/z3Uatcpa9WZrppWcR00VDRNu\n",
"q6JctFVRLtqqKBerbOAFzDn3TuB00i/2u0lvZG4z0iH1I6PebIVGrbIM2CzbtRFwZ8SQRERGUhE9\n",
"sLcBhwLf9N4/Zep759ySAmIo2n10CtjGqICJiPRdEQXsQe/9N6Z5/i0FxFC0e3Pbm0aLQkRkhBUx\n",
"iOOmbFqpqexZQAxFuzu3rQImIjIARfTA/hX4tHPuW6RTSj2Ye86Ak4DPFhBHkfIFbPNoUcSTxA6g\n",
"RJLYAZRIEjuAEkliBzAKiihgmwL7A0dM8Xzpl4Q2s+OB40hHF24IvH2GRS3Hugemed46lIsO5aJD\n",
"ueiPIgrYp0gHMXyG9It9csH6QAExzJmZHQq8GnhxCOFxMzuazorLU8kXsM2mbCUiInNWRAHbEdhx\n",
"qkl9nXMHFhDDqngf8OYQwuMAIYRze3jNPbltFTARkQEoYhDHLTPMSP+mAmKYEzPbEngO8GwzmzCz\n",
"H5jZCT28dKxPIYqIFKGIAnaec+7oaZ4v81RSO2R/1khnEnk98O9mVpvhdeM+iENEZOCKOIW4N3CQ\n",
"c+4Uuo9CLPNUUmtlf34uhLACuMvMvkY6oKM5zevuJb3WZ8BGjVpljXqztWKwoZZHkiRVXaROKRcd\n",
"ykWHctEfRRSww4HbgA2AF7LyII6oU0mZ2WWkIyS7uRp4V7Z9R27/X4Ftu72g84HciqO2fXib9ddc\n",
"3cJjjzwQHn14E+CufNtsMs9ql8Mk3T7Yw9SeLvO8DVP8fW5fTZIkmUX7QccTs32V7HNRknhitm9/\n",
"LsoST2Ht58+fT/65xYsXd2nWoxDCQH8OPfTQJavyfMwf0h7Yg8AhuX3vA/7f5LaLFy8O+cfnu10v\n",
"ON/tem32s2vsv0uRP1dcccWi2DGU5Ue5UC6Ui+l/Jn93zuaniGtg/2uG50s7lVQIYTlwLnAMgJmt\n",
"Qzqv41d6eLkGcoiIDFDfC5hzruac26b92Ht/2XTtvfc/y143zzl3QL/j6YN/A/5uZkuACeArIYTz\n",
"enidBnKIiAzQIHpgdwETzrnn9PoC59xBwIXAHwYQzyoJITwUQjg6hLBHCGHfEML/7fGl6oGJiAxQ\n",
"3wdxeO8vd859FviFc+4i4HvAjaSzcTxCOnBjXdLFLXcHXkM6oe9rvfd/7nc8EY3zzcxJ7ABKJIkd\n",
"QIkksQMokSR2AKNgIKMQvfefcs5dD3wM+Nw0TQPwXWCe9/7GQcQS0dj2wDQ8uEO56FAuOpSL/hjY\n",
"MHrv/RXAntmpxBcDO5GuTvwEae+kBVwxYr2uvL/ltnUNTESkzwZ+H5j3/gbghkG/TwmN8ylEEZGB\n",
"K2IY/bga21OIIiJFUAEbnGXA49n2ho1aZc2YwYiIjBoVsAGpN1tPkM6J2LZJrFiKlk0jIygXecpF\n",
"h3LRHypggzWuAzmqsQMokWrsAEqkGjuAEqnGDmAUqIANlmbjEBEZEBWwwcrPQL9ltChEREaQCthg\n",
"3Znb3iJaFCIiI6iI9cDGWX4dsa2iRSEiUiKNWuVQ4JXAGlu++dNzPo4K2GCN6ynEJHYAJZLEDqBE\n",
"ktgBlEgSO4BYGrXKgcC/9+NYOoU4AzOrmNkVZvZDM1tiZu+dxcvzPbCxKWCa561DuehQLjrGNReN\n",
"WmUr0kWB+0IFbGZfAa4KIbwYWAD8i5m9qsfX5q+BbdmoVazv0YmIDIFGrbI68EHSOXEBbgeOW5Vj\n",
"qoDNbDfgxwAhhHuA32X7evEg6RIyAOsA6/c9OhGR4XAcMC/bfgI4td5s/WpVDqgCNrNvA4cAmNmO\n",
"wM7A4l5eWG+2AmN6GlFEpK1Rq+wB/HNu1+frzdaSVT2uCtjMjgeebWY3Az8BTg4h/HwWr1/pNGJf\n",
"IxMRKblGrbIh8CE69ebnwJf6ceyBj0J0zj2XdD2wF5EOJd+M9BzoMtKZKu4gPUX3w2zplbL5JvCj\n",
"EMJLzGxb4GozuyOEcHmPrx+7ApYkSXVcL1JPplx0KBcdY5aLdwNPz7aXkZ46fHya9j0bSAFzzm0B\n",
"vAs4nHQ+wGuB60iDv4/02tCapNeEtgF2AY5wzm0LnAd83Ht/b5dD95WZXQbsP8XTVwP/Qjpwow4Q\n",
"QrjVzC4CTgTmUsDG5V6wKmM8THiSKspFWxXloq3KGOSiUascBLw8t+vD9Wbrjqnaz1bfC5hz7p+B\n",
"Y4Dzgd299/fP4rUbAq8BvuWc+6L3/tx+x5cXQjh4uufN7HnZ5mO53StIi+9T5H+jmjdv3tKFCxcu\n",
"XXvHF4RHbnnyjOOTPbBsNupql8Mk3X4zG6b2XfYNVfx9bl9NkmRRieJR+3K0b38uyhJP39uvuO+O\n",
"V6y+0ZaHhhWPrQWw5tP/4cZNXvrPwcxWOtbixT0NKejKQghzfvFkzrkPA7d6789axeMYaRF8pvf+\n",
"g30Jbg7MbC3gZuC/QghnmtkGpOdvzwghfCrfdmJiIixYsOApw+QbtcoBwOnZw6vrzdY7Bh13bEmS\n",
"LKpWq4tix1EGykWHctEx6rnIbhk6A9gn23U7cFi92Xpwctupvjt70e8e2Fne+1tX9SDe+wB8OTul\n",
"GE0IYbmZvQY43czqpKc8LwQ+M4vDjOMpRBEZb6+jU7wC8P5uxWtV9XUU4myKl3Nuxot4/SiGqyqE\n",
"cF0I4cAQwgEhhBeEEN4TQnhiFocY1+mkRGQMNWqVZ5KOH2g7v95sXTeI9ypkGL1zbu0uu8dlVop7\n",
"Sa+bAWzYqFW65WLUJLEDKJEkdgAlksQOoESS2AEMQjbbxgeA9vfcLcBnB/V+AytgzrljnHOV7OHe\n",
"zrmjp2l7nnPuvc65F2bXv0ZGvdl6gjEbSj9Gw4NnpFx0KBcdI5yLY4DnZtsrgPfVm63lg3qzQd4H\n",
"tjdwiHNuZ+B64H5gqlGFPyQ91XYk8Cnn3FLgW8AF2fWwYXcn6e0CkBawP0WMRUSk7xq1yi7Am3O7\n",
"zq43WzcO8j0HeQrxUu/964Hnk3Yhr5qm7b3e+294798O7Af8Gtga+NcBxlckDeQQkZHVqFXWIJ1l\n",
"vt0p+jXpROgDNcgCtp9zbnPv/RPe+6tIZ92Yyk7OueOdc+t671cAN3rvTwf+MsD4inR7blsFTERG\n",
"zdGkE1IALCcdddiX2TamM8hTiF8EFjvnWsAvSU+hXTZF248CHwb+4py7HfiGc+4SYNMBxlekfAF7\n",
"+pStRESGTKNW2YGVJ+o9q95sFXKZZGA9MO/9LcC+pLO5Lwf+a5q2j3vv3wNsAeztvX8vsDGwx6Di\n",
"K9hYFbDsrn1BuchTLjpGJReNWmU14FTgadmu35DOwlSIgU7m671/CPjaFM89pXhmpw+XZdu3s3JV\n",
"H2b5ub+2jhZFcaqM6DDhOaiiXLRVUS7aqoxGLg4F2lPurQA+WMSpwzYtp1KMlXpgWplZRIZdo1bZ\n",
"Bjg5t+ucerN1c5ExFF7AnHMDnaC3pO4HHs621wE2jBiLiMgqyX4J/1+k32cAvwfOKTqOGD2wLSK8\n",
"Z1TZysx/ze0a+etgIjLSXkl6ry+kcx1+qN5sPVp0EDqFWBwNpReRodeoVTZj5Xt0G/Vm69cxYlEB\n",
"K06+gI36QI4kdgAlksQOoESS2AGUSBI7gFVwCp3LIH8BzowViApYccZmKP0Iz/M2a8pFh3LRMay5\n",
"aNQq+wL/lNv1kXqz9fBU7QdNBQwws3lmdrOZHdPlueeb2Y/N7Cozu9jM5npz9dgUMBEZPY1aZS3g\n",
"3bld3643Wz+LFQ+ogGFmh5B2iZeRXozMP7cmcBHwHyGE/UlXY57ratMqYCIyzI4F2osM3w98Ml4o\n",
"qbEvYMCSEMJhpP8gkx0MrAghXJk9/gLwOjPbbA7vowImIkOpUatsT1rA2j5db7buiRTOk8a+gIUQ\n",
"/jzN03sBrVzbW4GHgBfM4a3upNPD27xRqzxtusYiImWQ3fP1bjrTRV1PutxVdGNfwGawFU/tmd3H\n",
"HBalrDdbK0jXPIN0NeqRXdhyVOZ56wflokO56BiyXBwEvCjbfgI4LVuoN7oYBeyLEd5zrgKTrotl\n",
"5joV1LicRqzGDqBEqrEDKJFq7ABKpBo7gF40apX1WfmeLz/oRSpno/AC5r2/sKj3MrPLzOyBKX6+\n",
"28Mh7iSdFT9vY1ZeoHI2NBuHiAyTE4HNs+2/EfGer276Ohu9c+4V3vvv9PF4h3jvL57r60MIB69i\n",
"CNcAh7UfmNl2wLrAdd0a5+/tmDdv3tKFCxcuBZLc/id7YGtuV6kmSbJXl8Mk3e4RyU45VIehfZd9\n",
"QxV/n9tXkyRZVKJ41L4c7dufi7LE85T2jVplV9LZ5rE119lgg70Wttb7x/mnJEnStX2vx58/fz75\n",
"5xYvXtylWW8shG5nyObGOXcI8DLgA977ufZScM5tBZwG/I/3/pJ+xTcdM0uAc0IIX8ntWxO4CTg6\n",
"hPADM3s/sFsIoTb59RMTE2HBggXTnlps1CqvI50AE9J7KN7ft79AiSRJsqharS6KHUcZKBcdykVH\n",
"2XORrfP1JWD3bNdPgZOzeV37qpfvzqn0tQfmvb/YOfc74Dzn3B+BC4Afeu+Xz/Ra59wGwH7AG4HN\n",
"gJO89wM/12pmuwPvB3YDTjKzfUMIbwEIITxqZq8BPmtmjwP3sPJQ0tm6Nbe97ZStRETiehWd4vUY\n",
"8L8HUbxWVd8XtPTet4CDnHMvA94CnO+cu4d06OXfgAdIlxZZD9gI2IS0eGwGXAF8wXv//X7HNZUQ\n",
"wvVk3eQpnv8F6crS/TAuBSyJHUCJJLEDKJEkdgAlksQOYCqNWmU94KTcrnPrzdafYsUznb6eQuzG\n",
"OWfAc4AXkg5c2AzYgHR4+t2k14V+CrS896Wr8L3q8RTi6sDVdH5xOKDebD008OBERHrUqFXeDhyd\n",
"PbwTeP0g5zsszSnEbrKidH32M9bqzdbjjVrlNuCZ2a5nAL+LGJKIyJMatcozgcNzu86IOVnvTHQj\n",
"c/HyM3+M8mlEERk+76TTsfkF8L2IscxIBax4+etg20WLQkQkp1Gr7AMckD0MwMfLOHAjTwWseH/J\n",
"bT8jWhQiIplGrbIG8K7crovrzVZrqvZloQJWvJEfiThk87wNlHLRoVx0lDAXhwI7ZNt/Bz4bL5Te\n",
"qYAVbxyugVVjB1Ai1dgBlEg1dgAlUo0dQFujVtmE9Janti/Um627Y8UzGypgxbuNzgTBW2tZFRGJ\n",
"7ARg/Wz7T6QTUAwFFbCC1Zut5XQmA14N2DpiOCIyxhq1yrOB1+Z2nV5vth6LFc9sqYDFMfLXwURk\n",
"KLyDTh24ut5sXRUzmNka+I3MAM65jwNHAi/23t+U7TsSuNR7f18RMZTMrcC8bFsFTEQKlw2bzy9U\n",
"+cmI4cxJUT2we4Gzgfy0Sd8D3umc+8eCYiiTfA/smVO2Gl5J7ABKJIkdQIkksQMokSTmm2fT2r0j\n",
"t+tb9Wbr97HimauiClgAzvTeP/nF7b2/y3u/CHAFxVAmf8ht/0O0KAak2/pA40q56FAuOkqQi1cC\n",
"O2XbDwOfixjLnBVyChH4JrDEObeE9DePK4FrvPcrgHUKiqFM8r/p7BgtChEZO41aZW3SlZbbvjos\n",
"w+YnK6oH9kHgvaSzzr8MmAAecM79mfSmuejMbJ6Z3Wxmx0zav4OZfcnMJszsx2Z2vpltPtVxevQX\n",
"oL1G2uaNWmWjVTyeiEivDge2yLb/BnwtYiyrpKge2M+9959vP3DOrQXsDRxCCSaLNLNDSP9Rl9G5\n",
"R6vtWOCeEMJxWduvkl7Pe91c3y+blf4PwK7Zrh2Bn8/1eCIivWjUKpsC+V/SPzfMSzoV1QN7yDm3\n",
"S/uB93659/5K0l7ZywuKYTpLQgiHka5RNtl1wGdyj5vAQX14z1ty28/qw/FERGZyPOliwpBei784\n",
"YiyrrJAC5r3/JPAy59y/tvc5515COjox+jWgEMKfp3nukhBCftDF2sBdfXjbfAGLnoN+KuE8b9Eo\n",
"Fx3KRUeMXDRqle1Z+czRGfVm6/Gi4+inok4h4r0/Y9KuH5DOfnxlUTH0yatYuUc2V6M8kKOKhky3\n",
"VVEu2qooF21Vis/FyXS+868Dhuqm5W76XsCccz8iHVmYZD9XTnGz8onAr733Q7NSs5ntSzrs/U19\n",
"ONxKpxAbtYqVfe0dERlOjVrl+cD83K5PjML3zSB6YBeRFqdtSAc7bO6c+xVPLWhnAi92zp3mvX/P\n",
"AOIAwMwuA/af4umrQwg9XYMzs+2B04A3hBC6drvz93bMmzdv6cKFC5cCSbd7Pp7+lk/vcldj0Rbh\n",
"8cefBmyz2av/5aNJkjw8VfvslEO1y9uWrn2XfUMVf5/bV5MkWVSieNS+HO3bn4uBx/PXM0+8krT3\n",
"1fbdrU88c6skSWr9OP5s28+fP5/8c4sXL+7SrDcWQn+LsHNuU+Ak7/2HnHMG7E56s/LbgUeBTYFf\n",
"AVeQdmNf470/tK9BzJGZXQGcE0I4d9L+zYFvAMeEEJaa2XbAHSGER9ttJiYmwoIFC2w279eoVb5M\n",
"mh+At9abrZ+t0l+gJJIkWVStVhfFjqMMlIsO5aKjyFw0apX96EwTtQJ4Xb3Zuq2I9+7FXL472/o+\n",
"iMN7f4/3/kPZdvDe/xpYk3TOvy2B5wFfALYDzgAa/Y5hFVj209lhtj5p8Xof8Lfs8Yn0Zxb5kR3I\n",
"ISLxNWqV1YC35nZ9o0zFa1UVNYjjQe/9A9n29dnPZ5xzh9AZ0hmNme0OvB/YDTjJzPYLIbw5e/o9\n",
"wItJe4xtATirD289qgUsiR1AiSSxAyiRJHYAJZIU9D4LgPYtTI8AXyrofQtR1H1gWzvnnjLnoff+\n",
"YuDZBcUwpRDC9SGEQ0MIW4YQXpgrXoQQ/jOEsNqkn9VDCH/qw1uPZAErwTxvpaFcdCgXHUXkIpuw\n",
"94TcrguGdcqoqRTVA3s/kDjnTgC+Dlzhvf+tc259RnM29l5pJKKIDMorge2z7QeBc6dpO5R66oE5\n",
"557tnJvzulXe+7tIRwLeBPw38Bvn3GOkKxMncz3uCLibzuwf6wFbRYxFREZEo1ZZE3hzbte59War\n",
"20xDQ23GAuacO5V01OAvVuWNvPf3eu9PIB1e/yrgMGBn7/1XV+W4wyzrbWlKKRHpt9cDT8+27wEu\n",
"iBjLwPRyCvHfgCPo0/Uy7/09wHf6cawRcQuwR7a9E/CjiLGIyJBr1CrrAsfldn1pmCfsnU4vRekb\n",
"pDfwbr0qpxFlSiM3pZTmvOtQLjqUi44B5+IwYJNs+3bS9RhHUi89sONJR7K8B/iEc+5G4GrStb2u\n",
"I50O6rHBhTjyRvEUYpXxvraZV0W5aKuiXLRVGUAusrUFj87tOrvebD06VfthN2MB894/AXzWOXc2\n",
"8FLgDaSLUra7qI85534D/BL4Nek9Xjd4728dTMgjJ1/A/qFRq6w+7DNEi0g0RwHrZ9t/BL4dMZaB\n",
"63kYvfd+Bem1q+8AOOd2Bg7Mfg5g5aofnHPLSIvZr0mL2y+AX3rvlyNPqjdb9zVqlbuBzUiXatkG\n",
"mHJ5FxGRbrLFKg/L7Tpr1H8ZnvN9YN773wG/I50WCufcM4F9s599gOeTDp3PT6S7wjnXAq4FfgZc\n",
"A/wqK47j7BbSAgbpaUQVMBGZrSNJfwmG9Lt5ImIshejbjcze+z8BfyIbrumcWxfYE9gr+3NP0kEK\n",
"z81+3pi99BHn3BLS62pXAVd770fqbvEe/B54Yba9I8O3RpqIRJT1vvKzHZ1db7aeiBVPUQY2E4f3\n",
"/iHSRSt/0N7nnNuItGe2R/bnc4EKaY9tH9Ih+8E5dxNpMbsSmPDe/3VQcZbEqE0plcQOoESS2AGU\n",
"SBI7gBJJ+ny8fO/rJsbkl+C+L6cyW8651UgXiXwu6dIiu5POWL8znWH+vyW99ua996VccmRVlgRo\n",
"1Cq7A1/OHt5ab7Ze07fARGSkNWqVTYBL6BSwU+rN1hXTvKRUVuW7s6i5EKeUjXK8Jfv5Vnu/c24d\n",
"0kK2F2nv7CjgX51zPwPe5r2/JkK4g3IjsBxYC9i2UatsUW+27oock4gMh8nXvsai9wXFzUY/a977\n",
"h733P/Hef8p7fzjptCgvJv2y/6FzrttqokOp3mw9Rjpis22PqdqKiLRlva/8d+Hnx+HaV1tpC9hk\n",
"2eKYV3vvjyH9B/tYv45tZvPM7GYzO2aaNh8wsz/06z27WJLbfv4A30dERsfk3lcSL5TiRT+FOFvO\n",
"uZNJ1xBbsx/HM7NDgMOBZaQLVXZrsxXpjCSDvIctX8DUAxORaY177wuGqAeWsy/pP9rJfTrekhDC\n",
"YXSWNenmVOCzwJwuNPbo10D7psOdGrXKBgN8r4HSnHcdykWHctHRp1yMde8LhrCAee8P995v5b2/\n",
"sB/HCyFTuUN3AAAQcUlEQVRMe9Owme0EbE06rH9gstmib2y/LekAlmFVjR1AiVRjB1Ai1dgBlEh1\n",
"VV6c9b7y932NXe8LhrCARfAB4H0MtvfVptOIItKLI4F1su2x7H2BCti0zGxP4NEQwg0FvWV+0VAV\n",
"MBF5ikatsjHqfQFDOIhjNszsMlaeizHv6hDCy2c4xAeAt/b6ftVqNWlvz5s3b+nChQuXAkl+f1t2\n",
"Drya37fFER9e+29f/8gG4dGHHwB2a9Qqa9WbreVTtW8fqtfjF9W+y76hir/P7atJkiwqUTxqX472\n",
"7c/FXP5/Vej0vm5Om5f+7/vkU/Pnzyf/3OLFi7s06030mTjKwsyuAM4JIZybPd6AdLLh9jRWGwO7\n",
"Aj8BmiGEs/KvX5W7yfMatcqFwA7Zw7fUm63rVvWYRUuSZFG1Wl0UO44yUC46lIuOueaiUausR7pE\n",
"SnvJlPfUm63v9zO2og31TBwlYuSuc4UQHiAtWOmTZlXSAjd/wHH8nE4B24N00dBhk8QOoESS2AGU\n",
"SBI7gBJJ5vi6Q1l5va/L+xLNkBr7a2BmtruZfR3YDTjJzD7Xpc07gUXA5mb2dTMb5AjBob8O1u2U\n",
"wrhSLjqUi4655KJRq6wNHJHb9ZVRX+9rJmPfAwshXE/6W810bT4BfKKYiFYaifhcrdAsIplDgE2y\n",
"7duByyLGUgpj3wMrm3qz9VfSDyfAusAuEcMRkRJo1CpPY+VV77+azaE61lTAyil/GlHzIorIy0kn\n",
"NAe4F7goYiyloQJWTrqhWUQAaNQqqwPH5nadX2+2HokUTqmogJVTvge2V6NWWTdaJHOgOe86lIsO\n",
"5aJjlrmYD2yfbT8IfL3vAQ0pFbBy+j3QnqNxfeDVEWOZi2rsAEqkGjuAEqnGDqBEqr00atQqBrwx\n",
"t6tZb7YeHEhEQ0gFrITqzVYAzsvtOjw7jSAi42UfOgO5HgEuiBhL6aiAldclwH3Z9tbAP0WMRUQK\n",
"lvW+jsvt+p96s3VvrHjKSAWspLI5EH1u11HZB1pExsMedEYhrwC+FjGWUlIBKzdPetoA0mmt9ooY\n",
"i4gU65jc9rfrzdYd0SIpKRWwEqs3W/eRnkpsOypWLLOUxA6gRJLYAZRIEjuAEkmme7JRq+wI7Jc9\n",
"DMC5gw5oGKmAld95QHutn30atcrOMYPphea861AuOpSLjh5ycWRu+8p6s/XHAYYztFTASq7ebN0K\n",
"XJHbdeRUbUVk+DVqlS2Bg3O7vhorlrJTARsO+dMHL2vUKltFi0REBq1GZ6L1X9abrV/GDKbMVMCG\n",
"QL3ZuoF0nTBIP9j1iOGIyIBkC1a+PrdLva9pqIABZjbPzG42s2O6PLeRmX3RzK42syVmdpmZxbip\n",
"OP9Bfm2jVll/ypYiMqxey8oLVv4gYiylN/YFzMwOAU4BlpGO9pnsfOD7IYT9gBcAfye3cnOBrgb+\n",
"kG2vB7wuQgw90Zx3HcpFh3LR0S0XjVpl8tmVr9WbrScmt5OOsS9gwJIQwmHA/ZOfMLN5wM4hhAsA\n",
"QuoNIYQVRQeZfZDzvbB6tkZQGVVjB1Ai1dgBlEg1dgAlUu2y76VA+/r23cB3CotmSI19AQsh/Hma\n",
"pxcAt5jZx7JTiJeY2XOKiq2L7wJ/y7a3IP3Ai8iQy2bZyS9Y2cxm45FpjH0Bm8EOpEXshuwU4leB\n",
"K8wsyvWnerP1KNDM7TpC00uJjIS9gZ2y7YeBb0SMZWiogE1vLeCBEMKXAUIIHlgOLIwY0zfpTC/1\n",
"bGDPiLGISH/kZ9m5qN5sLYsWyRBZY+Ymw8vMLgP2n+Lpq0MIL5/hEPfSOWXXdhvwjG6N83fXz5s3\n",
"b+nChQuXAkm3u+6zi7jVLoeZtv3WJ57JfRNffnj5X256FoCtscapjVrlvye3X3f3A5+3xqbbPH/y\n",
"/hX33PaLh66/8in3lfSz/RqbbrND48wT55clnpjtV193w+d/+4Yf7FCWeGK23/CA+pOfizLEE7N9\n",
"+3ORtf8T8MLsqceB8+f6/TAM7efPn0/+ucWLF3dp1hsLodvAu/FjZlcA54QQzs3tOwI4I4SwWW7f\n",
"LcCpIYTz86+fmJgICxYsKOR0XqNW2R64kDijIWdka66zQXj04Qdix1EGykWHctExTS6+V2+2/rPw\n",
"gCJale9OnULsMJ5aEC4CVpjZqwHM7ABgA+DSgmNbSTYv2pUxY5iOvqQ6lIsO5aJjilwEdOPyrIz0\n",
"KcRemNnuwPuB3YCTzGzfEMJbAEIID5rZq4BPm9kpwGPAwSGEpwy5j+A00tObm8cORERW2ePA5fVm\n",
"67exAxkmY1/AQgjXA4dO8/w1wIuKi6g39WbrbuCjseMQEYlFpxBFRGQoqYCJiMhQUgGTvtOcdx3K\n",
"RYdy0aFc9IcKmAxCNXYAJVKNHUCJVGMHUCLV2AGMAhUwEREZSipgIiIylFTARERkKKmAiYjIUFIB\n",
"k0FIYgdQIknsAEokiR1AiSSxAxgFmsy3T4qczFdEZFRoMl8RERk7KmAiIjKUVMBERGQoqYABZjbP\n",
"zG42s2O6PPcmM1tiZlea2Y/NbL8YMYqIyMrGvoCZ2SHAKcAy0gXl8s/tDpwF1EIIBwJnA/9TeJBD\n",
"RvO8dSgXHcpFh3LRH2NfwIAlIYTDgG6LVFaA+0IIN2WPfwJsbmZbFhbdcKrGDqBEqrEDKJFq7ABK\n",
"pBo7gFEw9gUshPDnaZ6+CljdzPbJHi8Ergsh3Dn4yIbXJZdcskPsGMpCuehQLjqUi/4Y+xWZpxNC\n",
"+KuZvQH4hpndDywHXhU5rNK77rrrdogdQ1koFx3KRYdy0R9j3wObjpntCjSAl4YQdgU+BXzPzNaM\n",
"G5mIiIx0ATOzy8zsgSl+vtvDId4IXBNCuB4ghPAF4BnAQYOMW0REZqappDJmdgVwTgjh3Ny+04Fn\n",
"hRBek9t3N3B8CGGl0YgTExNKpIjIHMx1KildA+uw7CdvMXCemT09hHC7mb0CWBP48eQXax5EEZFi\n",
"jX0PLLvX6/3AgcBS0mH1b8k9/07gaOABYG3g1BDC/4sQqoiI5Ix9ARMRkeE00oM4+s3M1jazL2dT\n",
"Sl1jZlMO5jCzI8zs2qzd/y0yziL0mgsz29/MLjGzxVk+PmxmI3W6dTafi9xrEjM7p4j4ijTL/yMV\n",
"M/uOmf3QzK43s48UGeugzeL/yBpm9kkzu87MfmRmF5vZNkXHO0jTTdc3qd3svjdDCPrp8Qf4KOlA\n",
"D4CdgbuBLbu02x34K7Ap6XW1y4G3xo4/Ui6uAF6Vba8HtIC3x44/Ri5y7V8J3Ad8KXbsET8XGwE3\n",
"ALtkjzcFzo0df6RcvBX4HbBW9vgrwIWx4+9jHg4BLgCuA46ept2svzfVA+uRma0GvAn4IkAI4XfA\n",
"EuDILs3fBHw7hHBPSP9lvgScUFSsgzbLXPxPCOHSrN3fgUuBlxYU6sDNMhft9u8i/UyMWk90Nrk4\n",
"HrgqhHBj1vaeEMLRRcU6aLPMxW7AL0IIy7PHPwGeU0ScBZluur68WX9vqoD17lnAZsBvc/t+A+zZ\n",
"pe2ek9q1gOeY2VqDC69QPecihHDGpF3rAKM0FddsPhcAR5EW8WUDjiuG2eTiJcD9ZnaBmV1lZl8w\n",
"s42LCLIgs8nFZcCLzGwzM1sDOBj49uBDLEaYfrq+vFl/b6qA9W6r7M/8F88yoNvEvltNancf6W/b\n",
"mw8mtMLNJhdPMrPVSXtfZw4orhh6zkX2H/F44DOMWO8rM5vPxT8AbwY+EELYn3SatsZgwytUz7kI\n",
"IXwb+ATpl/fvgUeAfx90gCU06+9NFbDZmzxsc6ovom7DO0ftS6vXXLT9G+kpxWsGFE9MveTiZOCL\n",
"2amiUR7+20su1gIuDyG0ssenAy8zs6cPNLLizZgLMzsSOBbYCdgeeBj4/MAjK6dZfW+qgPWufdor\n",
"f5pjY+COKdpObheAuwYTWuFmkwsAspvAXwD8xwDjiqGnXJjZRsCrgfZML6P2ywzM7nNxz6T9f83+\n",
"3HYAccUwm1y8jXQAy7Ls2s8ngWPMbP0Bx1g2s/7eVAHr3S2k/+l2ze3bDejWm7imS7vrcxdph91s\n",
"coGZ7Q28HTgqhBDMbKfBh1iYXnOxF7AhMJFNW3YM8HIzuzzLzyiYzefiF6x8Oq19mui2wYRWuNnk\n",
"4mnAitzjFaS/4DxtYNGV0+y/N2MPsRymH+A0sqHPpMNi/0b6n3Bz4IfAJtlzzyH9j7gp6S8JE8CJ\n",
"seOPlIsK8FPS36zXz36+Gjv+GLmY9JpFZEOsR+lnFp+LvUmvcTwje/w+4Pux4y84Fxtnz/2f7PEa\n",
"ucfXxY5/APlIgGNyj1f5ezP6X2qYfkjP259DOhfiNcA/Zfu3Iz01sE2u7eHAtcDPgI/Fjj1WLkgX\n",
"BX0ceCL38/vY8cf6XGT7P096D9QfgK+3v7hG4WeW/0eOBX4FXJnlYYvY8cfIBbAucFbW5mrSUYk7\n",
"xo6/j3nYPfv3vTP7Pjx7ms/ErL43NZWUiIgMJV0DExGRoaQCJiIiQ0kFTEREhpIKmIiIDCUVMBER\n",
"GUoqYCIiMpRUwEREZCipgImMEOfcjs65dxfwPqc657Yb9PuITEcFTGREOOf2JZ3V4md9Pu7XnXM/\n",
"nbR7CfBj59y8fr6XyGyogImMAOfc5sBFwEe890mfD/8XYGl+h/f+UtI1rC52zm3U5/cT6ckasQMQ\n",
"kb74D+AxBrCOlPf+nVM89ensfU8B3tvv9xWZieZCFInMOXcu8ArSWbhvAt5AOlP3maRL098AHOK9\n",
"v3WK1xtwO7DYe39Ebv/BwH+RLktxPvAT4AhgR9LlPt5GuuTH6cCzSVcK+JT3/hO5Y1yetd8O2MF7\n",
"/6dJ730hsLf3flTW8ZIholOIIpF5748GXkk6a//V3vvrs9OAlwPv9d6/YKrildkF2AK4cdJxL/Pe\n",
"70G6RMVBwMPe+wNIC9JawLdIC9zx3vvnAe8GTs+upbWP8RLSpU6m0gK2cc7tPJu/s0g/qICJlID3\n",
"/qfAR4A3Oude75x7FVDx3n+sh5e3Fwi9c4rnDbjXe39u9l6PkC5vsT3wLe/9sqzdN4GHgJd1ef1U\n",
"2isMj9IipTIkVMBEyuNDpCMIPwd8HDi6x9e1B1E8Mk2b1qTHd0/e770P2f6te3xfgIcnxSBSGBUw\n",
"kZLw3j8OHEd6LezOGU4b5j2R/TlVTykAD3bZR5f9TzC7wV3t75Anpm0lMgAqYCLl8kbSVaz3c879\n",
"S4+vuSf7c93BhDSt9bI/7562lcgAqICJlIRzbj6wPzAfuBD4L+fcc3t4afs04GxO/fVL+z0nn6IU\n",
"GTgVMJEScM5tQjps/qjsVOIJpD2r85xza0332mxo+y3A7lM0MaY+vTh5/2zaAvwj0PLe3zZdjCKD\n",
"oPvARCJzzn2QdMDGRsBF3vtjnXPvA04ivR/sD8DHvfdnTnOMU4D/BJ7pvb8/2/dC0gEhFeDvwB+B\n",
"PYEvAi8BtgV+C3wGuHaKthOk96K1237Ne39advzNsnb/4b3/dL/yIdIrFTCREZD10n5OOiz+Pwt6\n",
"z08ALwZemPUaRQqlU4giI8B7vxw4GHiNc+6fB/1+zrl3AQcCr1TxklhUwERGRHYtbC/grgLe7o/A\n",
"i7z3txfwXiJd6RSiiIgMJfXARERkKKmAiYjIUFIBExGRoaQCJiIiQ0kFTEREhpIKmIiIDCUVMBER\n",
"GUr/H88QTj/YM6n/AAAAAElFTkSuQmCC\n"
],
"text/plain": [
"<matplotlib.figure.Figure at 0x7f0babd21e10>"
]
},
"metadata": {},
"output_type": "display_data"
}
],
"source": [
"#plot -a/g\n",
"fig,ax = plt.subplots();\n",
"c5=\"#a65628\"\n",
"ax.plot( xs/1609, np.r_[0,(-f(v,traceu[:-1])*dx/g).cumsum()], lw=3, alpha=0.8, c=c5 );\n",
"ax.set_xlim((0,1));\n",
"ax.set_xlabel(\"x (mi)\");\n",
"ax.set_ylabel(r\"$\\int\\, dx\\, \\left(-\\frac{a}{g}\\right)$ (m)\");"
]
},
{
"cell_type": "markdown",
"metadata": {},
"source": [
"## Dynamic Program for Arbitrary Hill"
]
},
{
"cell_type": "markdown",
"metadata": {},
"source": [
"Now we want to solve for the optimal hill, this just means adding another dimension to our grid to optimize over"
]
},
{
"cell_type": "code",
"execution_count": 203,
"metadata": {
"collapsed": true
},
"outputs": [],
"source": [
"# new acceleration\n",
"def f(v,u,hp):\n",
" return u - A/m*(v>0) - B/m*v - C/m*v**2 - g*hp\n",
"\n",
"# new fuel use\n",
"def r(v,u,hp):\n",
" return m*v*u*(u>0) + P0"
]
},
{
"cell_type": "code",
"execution_count": 205,
"metadata": {
"collapsed": false
},
"outputs": [
{
"name": "stderr",
"output_type": "stream",
"text": [
"/home/alemi/anaconda/lib/python2.7/site-packages/IPython/kernel/__main__.py:37: RuntimeWarning: divide by zero encountered in divide\n"
]
}
],
"source": [
"# define our grid\n",
"XNUM = 161\n",
"VNUM = 100\n",
"VMAX = 50/mphfact\n",
"dx = 10\n",
"HMAX = 20\n",
"HMIN = -20\n",
"HNUM = 51\n",
"HZERO = HNUM//2\n",
"\n",
"# set up our matrix\n",
"costs = np.zeros((XNUM, VNUM, HNUM))\n",
"uvals = np.zeros((XNUM, VNUM, HNUM))\n",
"args = np.zeros((XNUM, VNUM, HNUM), dtype='int')\n",
"vs = np.linspace(0, VMAX, VNUM)[:,None]\n",
"hs = np.linspace(HMIN, HMAX, HNUM)[None,:]\n",
"\n",
"#set the endpoint\n",
"costs[-1,:,:] = np.inf\n",
"costs[-1,0,HZERO] = 0\n",
"args[:,:,:] = -1\n",
"uvals[:,:,:] = np.nan\n",
"\n",
"# fill in all of the other values\n",
"for i in xrange(costs.shape[0]-2,-1,-1):\n",
" # for each row going backward, fill in all of the costs\n",
" for j,vim1 in enumerate(vs):\n",
" # for each of our discritized velocities, compute the optimal cost\n",
" for k,him1 in enumerate(hs.flat):\n",
" # for each of our candidate heights\n",
" v = 0.5*(vs + vim1) #average velocity\n",
" dv = vs - vim1 # candidate differences\n",
" hp = (hs - him1)/dx # candidate slopes\n",
" # compute the candidate u values\n",
" u = ( v * dv + 0.5 * (dv)**2 )/dx + A/m * (v>0) + B/m * v + C/m * v**2 + g*hp\n",
" u[np.isnan(u)] = 0 # fix issues with zeros\n",
" dcost = r(v, u, hp)/v # new costs\n",
" # put in our constraint on the us\n",
" constraint = np.piecewise( u, [ u < -b, (u <= a)*(u >= -b), u > a ], [ np.inf, 0, np.inf ])\n",
" # all new candidate costs\n",
" newcost = costs[i+1,:,:] + dcost*dx + constraint\n",
" # pick the best\n",
" mincost = newcost.min()\n",
" costs[i,j,k] = mincost # record it\n",
" if np.isfinite(mincost): \n",
" pk = newcost.argmin() # record what choice we made\n",
" args[i,j,k] = pk \n",
" uvals[i,j,k] = u.flat[pk]"
]
},
{
"cell_type": "markdown",
"metadata": {},
"source": [
"Now trace out our optimal path"
]
},
{
"cell_type": "code",
"execution_count": 206,
"metadata": {
"collapsed": true
},
"outputs": [],
"source": [
"traceargs = np.zeros(XNUM, dtype='int')\n",
"traceargs[0] = args[0,0,HZERO]\n",
"for i in xrange(1,XNUM):\n",
" traceargs[i] = args[i-1].flat[traceargs[i-1]]\n",
" \n",
"traceu = np.array([ uvals[i].flat[arg] for i,arg in enumerate(traceargs) ])\n",
"tracev = np.array([ vs[np.unravel_index(arg, (VNUM, HNUM))[0],0] for i,arg in enumerate(traceargs) ])\n",
"traceh = np.array([ hs[0,np.unravel_index(arg, (VNUM, HNUM))[1]] for i,arg in enumerate(traceargs) ])\n",
"tracecost = np.array([ costs[i].flat[arg] for i,arg in enumerate(traceargs) ])"
]
},
{
"cell_type": "code",
"execution_count": 207,
"metadata": {
"collapsed": false
},
"outputs": [
{
"name": "stdout",
"output_type": "stream",
"text": [
"92.9024849766\n"
]
}
],
"source": [
"v = 0.5*(tracev[:-1] + tracev[1:])\n",
"dv = tracev[1:] - tracev[:-1]\n",
"dh = (traceh[1:] - traceh[:-1])/dx\n",
"vdot = f(v, traceu[:-1], dh)\n",
"dt = dx / v\n",
"ts = dt.cumsum()\n",
"print ts.max()\n",
"xs = np.linspace(0, XNUM*dx, XNUM)"
]
},
{
"cell_type": "code",
"execution_count": 227,
"metadata": {
"collapsed": false
},
"outputs": [
{
"data": {
"image/png": [
"iVBORw0KGgoAAAANSUhEUgAAAZYAAAEbCAYAAAD51qKQAAAABHNCSVQICAgIfAhkiAAAAAlwSFlz\n",
"AAALEgAACxIB0t1+/AAAIABJREFUeJzt3Xu8JEV99/FPAbIri3JngSCigjBcArhioiFm1lUjiYs+\n",
"JqnjxERQvCUmeeIr0fgkYjYxmufR4GOeaIy3SMjlZEpjIqjo4y60ATSKCInAgHIzGFjuAst9ofJH\n",
"99B9ZuecM32mZ6p6+vt+vc5r51LT/dvf7s5vu6q6ynjvERERqcpOoQMQEZHZosIiIiKVUmEREZFK\n",
"qbCIiEilVFhERKRSKiwiIlKpWhcWY8xqY8xZxphvGGMuMca8JHRMIiJNt0voAMa0CfDe++cbYw4H\n",
"/s0Y0/Le3xY4LhGRxqrtFYsxZifgdOBTAN777wOXAb8SMi4RkaarbWEBngnsA1xdeO0q4LlhwhER\n",
"Eah3YVmb/XpP4bV7gP0DxCIiIpk6F5a+wcXOTJAoREQEqHdh6Q/Q71l4bU/g1gCxiIhIps6zwq4D\n",
"7gKOBC7KXjsK+EKx0ZYtW7R8s4jICmzYsGFFPUC1LSze+8eNMZ8AXg9clE03Pg7oDLa97eO/cXGn\n",
"2ztp2jHG5ogjjjjrmmuuOS10HDFQLnLKRU65gPm51iuAM/Z/04fXrfQYde4Kg/Q+FmOM+QbwD8Cr\n",
"F7mHZfX8XKu2RbQqBx544KGhY4iFcpFTLnLKBQAHjHuAWn/Zeu8fBl43YvM1LJxBJiIiOxq7sNT9\n",
"iqWM3UMHENq6detuDB1DLJSLnHKRUy4AFZZSGl9YNm7ceGPoGGKhXOSUi5xyAaiwlNL4wiIispT5\n",
"udZO5Defr5gKS7MkoQOISBI6gIgkoQOISBI6gMD2AnYd9yAqLA3SbreT0DHEQrnIKRc55WL8bjBQ\n",
"YRERkdzY3WCgwiIiIjldsZSkwiIisjQVlpJUWERElqbCUlLjC0uSJO3QMcRCucgpFznlQoWlrMYX\n",
"FqAdOoCItEMHEJF26AAi0g4dQGAqLCWpsIiILGJ+rrUK2Dt7+tg4x1JhERERWDjVeNgq8SNTYRER\n",
"EVjYDbZ1nAOpsIiICKiwrMiT5+daO4cOIrAkdAARSUIHEJEkdAARSUIHEJAKywo1+qpF6yDllIuc\n",
"cpFreC6KYywqLCU0urCIiCxBVywrpMIiIjKcCssKqbCIiAyYn2sZVFhWbE3oAEREIrQnsCp7vK3T\n",
"7d0/zsGaVlgafcWidZByykVOucg1OBeVXa2ACkvTtEMHEJF26AAi0g4dQETaoQMIRIVlDE0vLCIi\n",
"w6iwjEGFRURkRyosY1BhERHZkQrLGFRYRER2VNld96DC0jRJ6AAikoQOICJJ6AAikoQOIBBdsYyh\n",
"0YWl4esgLaBc5JSLXBNzMT/X2hXYN3v6OHD7uMdUYRERabb9C49v63R7Y+0eCSosIiJNV2k3GDSv\n",
"sOyRrYkjIiKpYmG5tYoDNqWwbMt+XUW6Jo6IiKR0xbJCxWQdsGirGdfgdZB2oFzklItcQ3NxYOGx\n",
"CksJKiypdugAItIOHUBE2qEDiEg7dAAB6IplhVRYRESGq/TmSFBhERFprKo3+OpTYRERaa49gNXZ\n",
"4/vJJzqNpYmFZe2irUREmmXB1Uqn2/NVHLSJhaXJVyxJ6AAikoQOICJJ6AAikoQOYMoq7waD5hSW\n",
"20nXwAHYN1sbp3GauA7SYpSLnHKRa2AuVFhWKlv75rbCS/sv1lZEpEEqv+seGlJYMuoOExFZSFcs\n",
"YypWYxUWEREVlrHpikVEZCEVliJjzFnGmAsGftYs8ZHGF5aGroM0lHKRUy5yTcrFwAZfnoXj0GOp\n",
"bWEBvPd+/cDP/Uu0b3xhoZnrIC2mHTqAiLRDBxCRdugApmi/wuPbO93e9qoOXOfCUpYKi4hIbiLd\n",
"YFDzwmKM+agx5l+NMV8yxrSXab6gsGjDLxFpOBWWIXrAJ733LwTeDZxrjDluscadbu8+8nVwVgNP\n",
"nXyIIiLRUmEZ5L1/v/f+0uzxt4EvAm9e5mPqDhMRSU2ssOxS5cGqYow5Dzhpkbcv9t6/bMjrNwFH\n",
"DftAf5mG9U9dfdghu++0erfVqx5Z/czjXwZcM9g2mxXSHnKYZNhyD3Vqz5B1kOoUf8XtD02SZFNE\n",
"8YRs/8TnIoknZPv+34tY4plY+10PPvLk7XffehDA6medcEA2nPDEsTZv3jzksKMx3leymOXUGWPe\n",
"4b1/f+H52cBD3vs3Fdtt2bLFb9iwwQDMz7X+F/AL2Vsf6HR73akFLCISkfm51meAZ2RPX9Pp9hb8\n",
"R7v43VlWbbvCgLcZY/YDMMY8AzgF+NtlPqOuMBFpvElt8NUXZVfYiP4M+GdjzHZgDfBW7/2Fy3xG\n",
"hUVEJJ289OTs8YPAvVUevLaFxXt/JnBmyY8VC8uBFYYjIlInE9ngq6/OXWEroSsWEZGFO+lW2g0G\n",
"zSssjd7wq0nrIC1HucgpF7kG5WJi4yvQsMKSrYVze+Gl/RZrO6PaoQOISDt0ABFphw4gIu3QAUxJ\n",
"sbDcUvXBG1VYMuoOE5Gm0xVLxVRYRKTpJrIlcZ8Ki4hI8+iKpWLaolhEGmt+rrUL+fhypRt89TWx\n",
"sDT5iiUJHUBEktABRCQJHUBEktABTMH+QH+pljs63d6jVZ9AhaVBhi1K11TKRU65yDUkFxPtBgMV\n",
"Fm34JSJNo8IyAfcBD2SPn4w2/BKRZlFhqVq2Jk5ju8NEpPEmupwLNLCwZIrJXLtoKxGR2aMrlglp\n",
"5BVLg9ZBWpZykVMucg3JhQrLhDSysNCcdZBG0Q4dQETaoQOISDt0AJOUTVYqbhmiwlKhphYWEWm2\n",
"pwC7ZY8fouINvvqaWlh0972INNFEN/jqa2ph0RWLiDTRxMdXoOTWxNbaNcALgMOAPUg3zboT6AHf\n",
"dM49VnmEk3Eb6Ro5Bthvfq61S7ZXi4jILIunsFhrDwX+BPhFYLFdF++x1n4KeI9z7p5qwpuMTrf3\n",
"6Pxc6w7ShdgM6do5N4eNaiqS0AFEJAkdQESS0AFEJAkdwIRN/B4WGKErzFp7MvBd4HjgI8AbgVOA\n",
"lwA/C7wKeCvwj8Ac8F1rbWtSAVeocd1hDVkHaSTKRU65yDUgF+GvWKy1RwEfB17nnPvscgez1v5P\n",
"4NeBc6y1z3HO3VdNmBOxFTg2e9yIwiIijRe+sAAvBX7BOfetUQ7mnHsE+JC19hbS+eDnjhfeRDXu\n",
"ikVEGi98YXHOfWglB3XOdVcWzlRpWRcRaYyBDb5gAht89VU63dha+54qjzdhumIRkSbZj/w7/45O\n",
"t/fIpE5UaroxgLXWAIeSLgtQ/LwBXg2cUUlkk9e4wpIkSbsBg5MjUS5yykVuxnMxlW4wKHnFYq09\n",
"DrgKuA64iHRqXv/nAuCZlUY3WcXEHtiQDb/aoQOISDt0ABFphw4gIu3QAUxQsbDcumirCpS9Yvkr\n",
"4BrgD4E7SG+QLPpEFUFNyb2ka+WsJl07Z3fSTcBERGbR1K5YyhaWtcBPOecGCwoA1lo3fkjT0en2\n",
"/Pxcaytptx6kSVdhEZFZNfFVjfvKDt7fuFhRyXx0nGACaNw4i4g01tGFxzdM8kRlC8tHrLXvstYu\n",
"dqVzzrgBTZkKi4jMvPm51u7A4dnTx0lXU5mYsl1hXwBOB2611l4HbCu8Z8gDr4umFZYkdAARSUIH\n",
"EJEkdAARSUIHMCHHkl9IfK/T7d0/yZOVvWL5IOn6YFuBB7PP93/qOKuqUYVlhqdRlqZc5JSL3Azn\n",
"4vjC43+f9MnKXrG8CvgJ59y3h71prb18/JCmSnffi0gTFAvLxL+ny16x3LJYUcm8fJxgAmjUFYuI\n",
"NM/8XOtJLBy4n/gVS9nCcrm19uAl3n/zOMEEUFwrZ//5udbOwSIREZmMI0jv1wP4r063N7E1wvrK\n",
"doV9Huhaaz9Humvk4OB9nZZ0odPtPZJt+LUvaZHdjwnP7xYRmbKpdoNB+SuWfwaeD3yAdIZYQn2X\n",
"dOlrTHdYkiTt0DHEQrnIKRe5Gc3FCYXHE+8Gg/KF5TpgPfCiRX6urzS66SiumTPThYXZXgeprHbo\n",
"ACLSDh1ARNqhA6hStgbicYWXpnLFUrYrbLNz7muLvWmtrcM+LIMac8UiIo3zdGDP7PE9wI3TOGmp\n",
"Kxbn3K8t8/67xgsnCBUWEZlVC8ZXOt3eUktyVWbJwmKtfbG19illD2qtPdRae8LyLaOgwiIis2qq\n",
"N0b2LXfFshfwZWvtfsu0e4K1tgV8lXRJ+jpQYRGRWTX1GWGw/J73n7HWPg+43lr7MeArpPux3Oac\n",
"eyjbTXI30uWYjwFeSTrl+I3Oud5kQ69MkwpLEjqAiCShA4hIEjqAiCShA6jK/FxrX6B/3+HDwNXT\n",
"Orfx3i/byFp7GvA+Fn7x9j9YXCPsKuA3nHNJRfGNbcuWLX7Dhg2LrmOWzZq4CFiVvdTudHvbFmsv\n",
"IlIH83OtDcD/yZ5+p9PtvanM55f77lzKSIP3zrmzSDfE2gi8H/gccD6wGegCm4Cfds4dU2VRMcas\n",
"M8Zca4w5dch7xxtjvmGMucgYc44xZu+VnKPT7Xm0ZpiIzJ4g3WBQYrqxc+4R4IvZz8QZY04Bfpl0\n",
"ipwfeG9X0lUAXuu9/5oxZhPptsl2hafbSjotD9KrsutWeBwRkVgEKyxlb5Ccpsu8968m3Zt+0MnA\n",
"du99/56aTwKvMsbss8JzNWmcRURm3Pxcaw3pGmGQ/sd8oht7DYq2sHjvb1ri7RNJ1yrrt/0h8ADw\n",
"nBWerkl334vI7Dua/Pv92k63d980Tx5tYVnGWna8kvkRsP8Kj9eIK5YZXQdpRZSLnHKRm6FcBOsG\n",
"g/oWFs/AuEtmpbtYNqKwMGPrII2pHTqAiLRDBxCRdugAKhK0sJRdK6wSxpjzgJMWefti7/3LljnE\n",
"bSxcsRPS9XCG7jNQ3G503bp1N27cuPFGICm8/kRh2Wm3px6dJMmmIYdJhm1bmv0Pp12H9kNeq1X8\n",
"FbdvD/w5h45H7eNo3/97EUs8pdv77Y/uZFbtdpJ/+IFHspcuX6p9/63169dTfG/z5s1Dmo1mpPtY\n",
"QjLGXAB82nt/duG1VwBneu8Py54/DbgBWOu9v7P4+VHmYs/PtVYBF2dPHwNe0On2HqvwtxGFJEk2\n",
"tdvtTaHjiIFykVMucrOQi/m51lFA//tya6fbW9HOvhO/jyUww45dXOcBuxhjXpg9fz3wT4NFZVSd\n",
"bu9h4K7s6c6kG3+JiNRR0G4wKNEVZq19NnAq6aXSYcAewOPAnaQztL4KnOWcu72KwIwxxwB/CBwF\n",
"vNUY8wLv/ZsBvPePGGNeCfylMeYx0qJw2pin3Ar0b7I8gIUzxURE6mLq+68MWrawWGt3Bf4v8Jbs\n",
"pf8CbgK+R3rFs4b0N/JiYJO19t3OuTPHDcx7fwXwS0u8fznwgnHPU7CVtIjB7N59n4QOICJJ6AAi\n",
"koQOICJJ6ADGkS1RFfcVi7V2J+Ac4CCgA3zZOTfshkWstWuBVwG/b61d65x7R9XBTtjMzwwbNvjX\n",
"VMpFTrnIzUAuDgb6N4pvI9CuvsuNsbyN9Av3OJcaWlQAnHO3Ouc+CrSAk6y17erCnIqZLywiMvMW\n",
"7L8yrY29Bi1XWD4PvN45N/LUMefcNuClpF1ldaLCIiJ1F7wbDJYpLM65a51zT1S8rGtsUdbaV2af\n",
"2+acu7maEKdGy7qISN0FH7iH8tONv73M++9baSAR0BWLiNTW/FxrL9LtTQAeJd0fK4iyheVwa+0O\n",
"M7WstQdYa79CvppmHd0N9O9UfWq2OuhMmaF1kMamXOSUi1zNc1G8Wrkquz8viLKF5Tbgtdbas6y1\n",
"uwNYa/8H6ZLML2D4Eve1kA1yzXp3WDt0ABFphw4gIu3QAUSkHTqAMUQxvgLlC8sZzrmNpF1i/2at\n",
"/Tvgn4Dvk/6muhXHN223FB7P6r0sIjKbFswICxYF5Reh/HL26yXA75Lu8HgRsN459xj5TZR1VRxn\n",
"OQT4eqhARERGNT/XWg0cWXgpaGEpe8WSWGvPAC4kXc5lI+mX8WestXtba+s8eA8Lp0j/eLAoRETK\n",
"OZr8QuH6Trd3T8hgyhaWY4A/Av6B9KbJLzrnLOn9Ll8HTq84vmm7rPD4hGx5BBGR2EUzvgLlC8s2\n",
"wDrnTnPOPbHVpXPub0j3oa/7UvPXAvdnj/cDDgwYyyQkoQOISBI6gIgkoQOISBI6gBWqdWH5lnPu\n",
"s8PecM7dAHxp/JDCyfZg+W7hpeMXa1tHM7AOUmWUi5xykatjLubnWjuzsOu+XoXFOffiZd5/w3jh\n",
"RKH4h3Lcoq1EROJwGOkq85DeEnLLEm2nYrklWt5irT267EGttT9nrV1ue+FYFQvLTF2xiMhMWtAN\n",
"1un2gm8LvNwVy7eAL1lrTx7lYNbanay1bwT+IvtsHV0JbM8eP2t+rrVHyGBERJYR1fgKLHMfi3Pu\n",
"O9batwH/ZK29CvgKcA3p5dZDpFsG70Y6yH0M8HLSS7KTnXN3DT9q3Drd3oPzc62rSX8/kPZdXhgw\n",
"JBGRoYZs7BX0/pW+ZcdYnHOfA04AbgTeDpxFOkh/PrAFOBf4OOlU468AxzrnoqiaYyj+4cxMd1jN\n",
"10GqlHKRUy5yNczFQaQzWCGd0XptwFieMNKd9865a4BftNbuATyfhXve30W6iua3nHPBFj2r2OXA\n",
"a7LHszSA36a+0ymr1ka56GujXPS1qVcuit9P/5HNbA2u1JIuzrl7yJd1mWXFK5aj5+daq0KuFCoi\n",
"sojoxleg/H0sS7LWbqryeKF0ur27gB9kT59Eut2yiEhsoiwsZReh7O8i+XTSAfvi5w1p99GmSiIL\n",
"73LS3yekf3jR/KGJiGQzVp+ZPd1OOqM1CqUKi7X2BNKl8Q9bpEnw+dMV+nfgFdnjmRnAF5GZURxf\n",
"ubrT7T0ULJIBZbvCPkJaFV8NvBh40cDP9ZVGF9aCO/Dn51qVdhsGkoQOICJJ6AAikoQOICJJ6ABK\n",
"iGJ/+2HKdoUdBJzknHt82JvW2vnxQ4rGTaQz3vYGngI8A7guaERjquM6SJOiXOSUi1zNchHl+AqU\n",
"v2L5z8WKCoBz7t1jxhONbFkELe8iItGZn2utIt2DpS+KGyP7yhaWv7PWLrrQpLX2O2PGExsVFhGJ\n",
"0VHkPU43drq9u0MGM2jJrjBr7adZOCBvgJdky7xcSbo/S/G9pzNbZvIOfBGpveiWcSlabozll4Gb\n",
"B17bTroe2PPYseisYbZcQ7om2mrgwPm51tpOt3dr4JhERKIdX4HlC8tVzrkTRj2Ytfay5VvVR6fb\n",
"2z4/1/oucGL20nHA/w8Y0liSJGnXbHByYpSLnHKRq0Mushmq0c4Ig+XHWH6/5PHevNJAIlYslnXv\n",
"DmuHDiAi7dABRKQdOoCItEMHMIJnArtnj+8EfhgwlqGWLCzOufPKHMw5V9c9WJZS/N/AyFdvIiIT\n",
"Et3GXoNm4aa/SbsS6K8Yetj8XOspIYMRkcaLeuAeVFiW1en27ge+lz01wLEBwxERiXrgHlRYRqX7\n",
"WUQkuPm51gHAAdnTB0lnrkZHhWU0s1JYktABRCQJHUBEktABRCQJHcAyirPBvhvLxl6DVFhGM7jx\n",
"167BIhlD7NMop0m5yCkXuRrkIvpuMFBhGUmn27uDfErfKuDIgOGISHOpsMyYWekOE5Eaymak9vfC\n",
"egy4ImA4S1JhGd2C/VmCRSEiTXUs6cxUgGs63d4DIYNZigrL6GZx4y8RqY9adIOBCksZPwB+lD3e\n",
"kxqu5JwkSTt0DLFQLnLKRS7yXKiwzJps2YS6L6PfDh1ARNqhA4hIO3QAEWmHDmCYbCZqtBt7DVJh\n",
"KUcD+CISwpGkM1IBbup0e3eGDGY5KizlaABfREKoTTcYRF5YjDHrjDHXGmNOHfLeVmPMBYWf90wh\n",
"pKuBh7PHB8/PtfabwjlFRFRYqmCMOQV4O3APC3eq7DvPe7++8HPGpGPqdHuPsnDuuK5aRGSi6rCx\n",
"16BoCwtwmff+1cC9oQMZUOdxliR0ABFJQgcQkSR0ABFJQgcwxKHAHtnjHwH/GS6U0URbWLz3Ny3T\n",
"5ChjzJeMMRcaYz5mjNlnKoHVeGZYDdZBmhrlIqdc5CLNRfQbew2KtrCM4ArAeu9/mrS7rNRul2P4\n",
"D/KuuWfPz7XWTOm8ItJMteoGgxoXFu/96d77bdnTTcBzjDEnTvq8nW5vG/D97OlOwDGTPqeINFqt\n",
"Bu4BdglxUmPMecBJi7x9sff+ZWWO571/wBhzF+nd8JcMvl+8vF23bt2NGzduvBFIhl32Znfetoec\n",
"ptj+MuDZALsefKRNkuTkZdqXPb7aq73aqz3b7966Zqc1e53oH33oPv/Ig7czsLFXlfGsX7+e4nub\n",
"N28e0mw0xvu4u+uMMRcAn/ben114bT1wr/f+0uz5rsD9wM94779e/PyWLVv8hg0bDBWan2u9FHhf\n",
"9vSSTrf3a1UeX0QEYH6u9RLgT7Onl3a6vTdP69zjfHfWoSvMkK/o2XcIUPwy/03gWuCbU4qpOIB/\n",
"7PxcK8iVX1mRr4M0VcpFTrnIRZiL2nWDQcSFxRhzjDHmM8BRwFuNMR8rvH1+2sRcaIy5CNgAbPTe\n",
"T2Wbzk63dytwc/Z0NXDENM5bgXboACLSDh1ARNqhA4hIO3QAA2o3cA+BxlhG4b2/AvilRd67CTh9\n",
"uhHt4HLgoOzx8cCVAWMRkRmTzTh9dvb0cdIZqbUQ7RVLDdT5RkkRid+x5N/R3+90e/eHDKYMFZaV\n",
"W3Cj5Pxcq9IJAiLSeLUcXwEVlnHcQL7czF6kEwpERKpSLCxR778ySIVlhTrd3uMs/MOuw4KUSegA\n",
"IpKEDiAiSegAIpKEDgAgm2lavPlahaVBajXOEuk6SEEoFznlIhdRLo4gnXEKcHM2E7U2VFjGU6vC\n",
"IiK1UdvxFVBhGVcPeCR7fMj8XGtaKyyLyGxTYWmqTrf3CHBV4aU6jLOISMSyGaa1HbgHFZYqFP83\n",
"ocIiIuM6hHSmKaQzT28IGMuKqLCMrzbjLBGugxSMcpFTLnKR5GLBMi7ZDNRaUWEZX3GZhSPn51pP\n",
"DhbJ8tqhA4hIO3QAEWmHDiAi7dABUPPxFVBhGVun27uXdGVlgJ3Rxl8iMp5aj6+ACktVFizvEiwK\n",
"Eam1bGZpfxWPR0hnntaOCks1ajPOIiJRK46vXJnNPK0dFZZqFAvLsfNzrZ2DRSIidVbL/VcGqbBU\n",
"oNPt3QJszZ7uBhweMJylJKEDiEgSOoCIJKEDiEgS+Py1H7gHFZYqRT/OEtE6SMEpFznlIhcyF9mM\n",
"0iOzpx74bqhYxqXCUh2Ns4jIOI4hnVkKcF0247SWVFiqs6CwaOMvESlpJrrBQIWlStcD27LH+wIH\n",
"BYxFROpHhUUW6nR7j1GDcRYRiU82k/TYwksqLPKEqAtLJOsgRUG5yCkXuYC5OJx0RinA1k63t3Wp\n",
"xrFTYalW7AP47dABRKQdOoCItEMHEJF2oPOeUHhcy2VcilRYqnUlsD17/Iz5udaeIYMRkdqYmfEV\n",
"UGGpVKfbexht/CUiJWQzSGfijvs+FZbqxd4dJiJx+THSmaSQziy9PmAslVBhqZ4Ki4iUsWCZ/GyG\n",
"aa2psFSvuPFXa36utTpYJDtKQgcQkSR0ABFJQgcQkSTAOWu//8ogFZaKdbq9H5HvUb0LcHTAcBbQ\n",
"mlA55SKnXOQC5WKmBu5BhWVSin85NIAvIkPNz7X2Ag7Nnm4nnVlaeyosk6FxFhEZxY8XHl+VzSyt\n",
"vV1CBzCjBhekfFuwSEQkZscUHs9ENxiosEzKzcAdpFMIdwNeEzYcEamBmSks6gqbgE6354Gvho5j\n",
"kNn1yU8JHUMslIuccpELmIu7ge8EOnfldMUyOR8mHYjbJ3Qgfbsd/cJX3X/ZVz4XOo4YKBc55SIX\n",
"KBfbgYs73d62ZVvWhArLhGSDcF8OHUdRkiSHn/LOD/196DhioFzklIucclENdYWJiEilVFhERKRS\n",
"KiwiIlIpFZZmSUIHEJEkdAARSUIHEJEkdACzwHjvQ8cwUVu2bPEbNmwwoeMQEamTcb47dcUiIiKV\n",
"UmEREZFKqbCIiEiloiwsxphdjDG/bYy5wBiTGGO+box50UCb440x3zDGXGSMOccYs3eoeEVEJBdl\n",
"YQEOBn4LOMV73wbeDXzeGHMQgDFmV+DzwDu99yeRrrHzV4FirY0kSdqhY4iFcpFTLnLKRTViLSz3\n",
"Amd47+8D8N5vBh4Cnp+9fzKw3Xv/tez5J4FXGWOiWZcrUu3QAUSkHTqAiLRDBxCRdugAZkGUhcV7\n",
"f5f3/on1eowxBtgVuD176USgV2j/Q+AB4DnTjLNuzj333ENDxxAL5SKnXOSUi2pEWViG+BngRu/9\n",
"v2bP15Je1RT9CNh/qlHVzKWXXnpo6BhioVzklIucclGN6AuLMWY18F7gtMLLPvvZofk0YhIRkcUF\n",
"KSzGmPOMMfct8vPlQjsDfBz4oPf+ssIhbgP2HDjsntnrIiISUNRLuhhjPghc673/y2wm2Frv/U3G\n",
"mFcAZ3rvD8vaPQ24IXv/zuIxtmzZEu9vUEQkYitd0iXawmKM+T3gIOD3Sbu4DgF+yXv/R8aYVcA1\n",
"wGu99/9qjPlD4Cjv/Vy4iEVEBCItLMaYZwNXD3lrk/f+j7M2xwN/CTwG3AWc5r2/e3pRiojIMFEW\n",
"FhERqa/oZ4WNwhiz2hhzVrbEyyXGmJcs0fY1xphvZ+3+bJpxTsOouTDGnGSMOdcYsznLx59kkyVm\n",
"Qpm/E4XPJMaYT08jvmkq+e+jZYz5kjHmQmPMFcaY904z1kkr8e9jF2PMnxtjLs2WlDqnv/LHLDHG\n",
"rDPGXGuMOXWZduW+N733tf8B/jfw6ezx4cCdwP5D2h0D3ALsTTpucz7w66HjD5SLC4CXZ4/XkN5w\n",
"+luh4592Hgrtf570Xqi/Dh17wL8TewBXAkdkz/cGzg4df6Bc/DrwfWBV9vxvgM+Gjr/iXJwC/CNw\n",
"Kel49WLtSn9v1v6KxRizE3A68CkA7/33gcuAXxnS/HTgiz69s98Dfw28ZVqxTlrJXPyz9/4LWbv7\n",
"gS8AL51SqBNVMg/99r9D+vdhZq7aoHQu3gBc5L2/Jmt7l/f+tdOKddJK5uIo4HLv/cPZ838Djp5G\n",
"nFN0mfeSRjwUAAAGW0lEQVT+1ex4s/mg0t+btS8swDOBfVg42H8V8NwhbZ870K4HHJ3NMpsFI+fC\n",
"e///Bl56MrNzH1CZvxMAv0paWO+ZcFwhlMnFi4B7jTH/mK0a/kljzOD9YnVWJhfnAT9hjNnHGLML\n",
"6fqEX5x8iNPjvb9pxKalvzdnobCszX4tfincw/DlXdYOtPsR6f9Q951MaFNXJhdPMMbsTHq18tEJ\n",
"xTVtI+ch+8fxBuAjzNjVSqbM34lnAG8C/sinq4Y/DMxPNrypGjkX3vsvAh8i/UK9nnQR3N+bdICR\n",
"Kv29OQuFpW9wettiXxJNWApm1Fz0/S5p19glE4onlFHy8BvAp7Iuj1meIjlKLlYB53vv+wu8fhD4\n",
"WWPMARONbPqWzYUx5ldIl5E6DHg68CDwiYlHFq9S35uzUFj63TfFS/Y9gVsXaTvYzpOvmlx3ZXIB\n",
"gDHm50hXhX7nBOOatpHyYIzZA3gFcHb/pcmHNnVl/k7cNfD6LdmvB08grhDK5OI3SScu3JONK/w5\n",
"cKoxZvcJxxij0t+bs1BYriP9B3Fk4bWjgGH/+75kSLsrCgN0dVcmFxhjfpJ0Q7Vf9d57Y8xhkw9x\n",
"KkbNw4nAU4EtxpgLgFOBlxljzs9yMwvK/J24nIXdQv2ujpsnE9rUlcnFk4DthefbSf/j8aSJRRev\n",
"8t+boae8VTRt7k/JpomSTiG8g/QfyL7AhcBe2XtHk/4j2Zu0qG4Bfi10/IFy0QK+Sfq/0d2zn78N\n",
"Hf+08zDwmU1kU1Fn6afE34mfJO0//7Hs+buBr4aOf8q52DN77/3Z810Kzy8NHf+EcpIApxaej/29\n",
"Gfw3VVFiVgGfBr5BWl1fnL3+NNLL3IMKbX8Z+DbwLeADoWMPlQvgItLlcB4v/FwfOv4Qfyey1z9B\n",
"eg/HDcBn+l8os/BT8t/HacB/AF/L8rBf6PhD5ALYjXS780uAi0lniT0rdPwV5+KY7M/4tuz78ONL\n",
"/L0o9b2pJV1ERKRSszDGIiIiEVFhERGRSqmwiIhIpVRYRESkUiosIiJSKRUWERGplAqLiIhUSoVF\n",
"ZAqstc+y1r5jCuc5w1r7tEmfR2QpKiwiE2atfQHpnezfqvi4n7HWfnPg5cuAb1hr11V5LpEyVFhE\n",
"Jshauy/weeC9zrmk4sP/F3Bj8QXn3BdI9xE5x1q7R8XnExnJLqEDEJlx7wQeZQJ7eTjnfnuRtz6c\n",
"nfftwLuqPq/IcrRWmMgirLVnAz9Huqrr94BfJF359aOk29xeCZzinPvhIp83wFZgs3PuNYXXTwbe\n",
"R7r8+D+Q7qf+GuBZpEu7/ybp8u4fBJ5NuvL0XzjnPlQ4xvlZ+6cBhzrn/nPg3J8FftI5Nyt7qUiN\n",
"qCtMZBHOudcCP0+6CvTFzrkrsu6s84F3Oeees1hRyRwB7AdcM3Dc85xzJ5AuRf4S4EHn3AtJC8Uq\n",
"4F9IC88bnHPHAe8APpiN1fSP8SLSZe0X0wMOstYeXub3LFIFFRaRJTjnvgm8F3idtfYXrLUvB1rO\n",
"uQ+M8PH+xmm3LfK+Ae52zp2dnesh0mXMnw78i3Ouv8/454AHgJ8d8vnF9HdFnJXN26RGVFhElvce\n",
"0hldHwPOBF474uf6g+cPLdGmN/D8zsHXnXM+e/3AEc8L6R7txRhEpkaFRWQZzrnHgNeTjrXctkz3\n",
"V9Hj2a+LXVl4YNuQ1xjy+uOUm2zT/7f9+JKtRCZAhUVkNK8j3XXzp6y1bxvxM3dlv+42mZCWtCb7\n",
"9c4lW4lMgAqLyDKsteuBk4D1wGeB91lrjx3ho/3urDJdWFXpn3Owq01k4lRYRJZgrd2LdHrxr2Zd\n",
"Ym8hvRL5e2vtqqU+m00Bvo50b/FhDIt3kw2+XqYtwI8DPefczUvFKDIJuo9FZBHW2j8mHajfA/i8\n",
"c+40a+27gbeS3s9yA3Cmc+6jSxzj7cAfAIc45+7NXnse6USAFnA/8APgucCngBcBBwNXAx8Bvr1I\n",
"2y2k99L02/6dc+5Ps+Pvk7V7p3Puw1XlQ2RUKiwiE5Rd1XyHdPrwH0zpnB8Cfhp4XnaVJTJV6goT\n",
"mSDn3MPAycArrbVvnPT5rLW/A/wM8PMqKhKKCovIhGVjLScCt0/hdD8AfsI5t3UK5xIZSl1hIiJS\n",
"KV2xiIhIpVRYRESkUiosIiJSKRUWERGplAqLiIhUSoVFREQqpcIiIiKV+m9u7w/hSrxvaQAAAABJ\n",
"RU5ErkJggg==\n"
],
"text/plain": [
"<matplotlib.figure.Figure at 0x7f0b97d50110>"
]
},
"metadata": {},
"output_type": "display_data"
}
],
"source": [
"#plot h(x)\n",
"fig, ax = plt.subplots()\n",
"c5=\"#a65628\"\n",
"ax.plot(xs/1609, np.r_[0,traceh[:-1]], lw=3, alpha=0.8, c=c5);\n",
"ax.set_xlim((0,1));\n",
"ax.set_xlabel(\"x (mi)\");\n",
"ax.set_ylabel(\"h(x) (m)\");"
]
},
{
"cell_type": "code",
"execution_count": 208,
"metadata": {
"collapsed": false
},
"outputs": [
{
"data": {
"image/png": [
"iVBORw0KGgoAAAANSUhEUgAAAb0AAAEfCAYAAADV+hihAAAABHNCSVQICAgIfAhkiAAAAAlwSFlz\n",
"AAALEgAACxIB0t1+/AAAIABJREFUeJzsnXmYJVV5/z/39jYbzMCwzpnCYRsQUFBEShS9OGpFEVyI\n",
"BhUUAbcYotFoFAPVpfFnEsW4JIiCiRoXDAKCCpTSUCwJFQVBgywiitQchAGG2Wd6erm/P07VdHV1\n",
"3Xvrbl23us/neeapvlWnTn27+k699Z7zvu8pVatVNBqNRqOZD5TzFqDRaDQazWyhjZ5Go9Fo5g3a\n",
"6Gk0Go1m3qCNnkaj0WjmDdroaTQajWbeoI2eRqPRaOYN/XkLSEMK41DgPmCNkMGt4b7HgftjzW4X\n",
"MrggD30ajUYzHzBtdwFwCXAYyl6c7zvWz1LafQN4VmL3S4B9fcda322dzdCTRg/4JDCa2He9kME7\n",
"8xCj0Wg085RhoOo71otM2z0U8E3bfbbvWOsS7bb6jnVS9MG03WOAi3rN4EEPDm9KYRwHbAaezFuL\n",
"RqPRzFdM2y0D5wBfB/Ad6yHgbuCMZFvfsd6f2HUG8O1ua2yFXvT0hoF3A69M7D9CCuM6YDfU0Of5\n",
"QgZPz7I2jUajmS8cBCwHHojtuw94Qb2TQmP5OsDpnrTW6SlPTwrj1cC9QgYy5fC9wJuFDE4ENgLX\n",
"z6o4jUajmV/sG243xvZtBPZpcF4FuNN3rM3dENUuPePpSWGUgY8Cb0g7LmRwTuzjMLBJCuM4IYNf\n",
"zII8jUajma8kCzSXGrQ/A/jPLmlpm17y9N4KuEIGG2L7Um+ukME2YD0zo4U0Go1G0xmiYJVlsX3L\n",
"gCdqnRBGe54I3NBFXW3RM54eKrz1KCkMK/y8H/AFKYw/Al8ENgkZ3AUghTEI7AE8luxkZGRELxuh\n",
"0Wg0LbBmzZq4o/Ewyrk4HLg93HcE8OM6XbwWuM53rMnuKGyfnjF6QgbvjX+WwvgD8AEhg1ulMN6B\n",
"ens4Nzx8HvA74H/T+kr84XqSww477BsPPvjgWXnraEQRdBZBIxRDZxE0gtbZSSKNSYfBd6xJ03Yv\n",
"Bc4Gbg9TFo4G3mLa7l7A1cCpvmM9EzvtbcCnZ0t7K/TS8CYAUhhHS2H8ANgL+KQUxgeAm4CSFMZt\n",
"Uhi3A2uAU4QMJvLU2g7777//qrw1ZKEIOougEYqhswgaQevsJA00DgMl03bvAL4LnB7m6C0EVodb\n",
"AEzb3QM4yHesO7untn16xtOLEDL4FfDnKYfOSdmn0Wg0mi7hO9YoMKMoiO9YAVPRndG+Z1CeYE/T\n",
"c57eXMK03UWm7falHTv22GMfmWU5LVEEnUXQCMXQWQSNoHV2kiJo7CTa6HUJ03YPQ034XpR2/JRT\n",
"TnlkVgW1SBF0FkEjFENnETSC1tlJiqCxk2ij1wVM212OMna7A4fkLEej0Wg0IdrodRjTdoeAz6FS\n",
"LgBShzcBb1YEtY+Xt4AMeHkLyIiXt4AMeHkLyIiXt4CMeHkLyICXt4DZpFStzq20tpGRkWpeKQum\n",
"7ZZQ0U4no8r1LAXWX3nZuSfTuHSPRqOZ22wXMui5VQci8nx2ziY9F71ZcM5EGbwdwMeBiwfGdy4E\n",
"rgL2z1OYRqPJnaoUxsVCBv+Rt5D5jDZ6HcK03ZeikuYBLgTuK01OlldsWncY8CiwAdiWlz6NRpM7\n",
"+wPvl8JYK2QwYyFWzeygjV4HMG33EOAfULVCL/Yd66az3/3lxU8u2XNFeXJiIfAIcLaQwaY8dWo0\n",
"mvyQwngr8CFgWArjMSGD3+StaT6iA1naxLTdpcDngUWAC/wHwMd/+qV3LRgf3W2i3DcJfEgbPI1m\n",
"3vM94IfAEHCRFIae588BbfTapwKsAB4EPuk7VlUK45Sh8Z1nVClVn1yyPBAyeDR5kud5lVnW2RJF\n",
"0FkEjVAMnUXQCMXUKWRQBf4JuAtVZvHzUhgLcpK2i6Lcy06hjV77LAq39/iONSqFcQzwiXJ1ko0L\n",
"d398++DCHWFUZ5LK7Elsi0reAjJQyVtARip5C8hAJW8BGankLSAjlfgHIYMx1Lqha1GrFzjhWqJ5\n",
"Usn5+rNK3jd7LhDl4Y1LYQhUjl5/Cb63ecGSp8Nj+j5rNBoAhAw2An8DbEEVz393vormF/ph3D59\n",
"AAt3bu9Dze0tA+4AvgBEq0DogCGNRrMLIYM/oNKaJoFzpTD+LGdJ8wZt9Nqnn2qVNQ/e9hrgYOAP\n",
"wMfDZY8io1erKotGo5mnCBncwVRt3gulMI7KU898oSeNnhTGoVIYY1IYL43tO0YK4w4pjNulMK6V\n",
"wtgzT40x+vbYtnGfPbZtPAhVheVvhAy2hMe00dNoNPX4L+BKYBAV0blvg/aaNulJowd8EhiNPkhh\n",
"DALXAB8TMngJ8Evgklonm7a7rOsKQ4567IHDl+zcurxcnZwAPipksDZ2eDzcphk9r+viOoOXt4AM\n",
"eHkLyIiXt4AMeHkLyIiXt4CMePUOhhGdnwV+ASwH/p8UxmyXAvNm+Xq50nNGTwrjOGAz8GRs96uB\n",
"cSGDW8LPlwFvlMJYXqObw7socRoHPv3o8QCP7ilGhAzuShyu6elVKhWvy9I6QhF0FkEjFENnETTC\n",
"3NIpZDAOfAxVteloYFaHOYtyLztFzxk9VMFmJ7HvOOD+6EPoTW0Dnl+jj8GuKEsghXHIwp3bjSql\n",
"yV8c8LyfpzTRgSwajaYhYUTnNeHHN+WpZa7TU0ZPCuPVwL1CBjJxaF8gWdFkA7VXLpgVowe8abLc\n",
"V9o2uHDjlgWLt6Yc13N6Go0mK1cBVeCVPRSzMOfoGaMXJmh+FPhMyuFq+C9JrbHvrhs9KYwlwGsm\n",
"S6XS5qHFzzBl4OJE+3rmPms0mt4kfNm/HRgAXpeznDlLLw27vRVwhQw2xPZFRm0d8LxE+2Xh/hn0\n",
"P/Djf65UPvNRgGOPPfaRU0455RHASxu7DkvwVFK6qdt+zze+4cjdvVsO3rHXPtVFiweXr1paPiyl\n",
"j4kFfex29tEDH/Y8b2Mz/TerR7fX7XX74rff7b3v3n35f/1gRWli4t1SGN8KU59mTc98oGcWkZXC\n",
"uAQ1gTsW7jKBB1ArFHwDuEjI4JCwrYHKh9tXyODpeD8jIyPVT9w6/he+Y/1XF7WWUKHGB/7dqef/\n",
"5nf7HHQkqu7mtfF2pu3+F3AQ8Be+Yz0cP+Z5XqUIX7oi6CyCRiiGziJohLmrMxzxuhIwUIXqb+2W\n",
"tohIY9oisqbtLkBFyh+GcpLO9x0rdVkk03afjco73A3YA7jGd6xPdFd98/TMsJuQwXuFDF4iZHCS\n",
"kMFJwOPAB4QM3gDcAPTH8vbOBq5MGrwY3R7efAFwIPDk7/d6VhDuG09pV29Or9IFXd2gkreADFTy\n",
"FpCRSt4CMlDJW0BGKnkLyEilmcZCBpPAD8KPb+64mnQqdY4NA1XfsV6EGo273LTdGbEU4WozPwD+\n",
"xnesE4GXogx3z9FLw5sASGEcDVyAqkL+SSmMq4UMviiF8XrgYimMCWA9cFadbrpt9KLoqqsny32r\n",
"wp/rzenpQBaNRjMN03b7UBHoJnCb71j3hId+BLwPMKUwniVk8Mec9JWBc4A3APiO9ZBpu3cDZ6BK\n",
"LsY5F7jdd6wHw7brgbfPotzM9JzREzL4FfDnKfvvAU7I2E3XjF5YMeFlKM/uKuAj4aE0o1cvOV2j\n",
"0cwzwhVXjgQs4JWol3uA5wLvAhAy2CSF4aKCWU5jpoGZLQ5CJcw/ENt3H2qkK8nLgftM270cWBme\n",
"87e+Y21IaZsrPTO82WG66em9EWXEbhYyeIqpF4dmhzc1Gs08wbTdxabtvg+4GhWj8BaUwXsmbLIw\n",
"cUoUk3CqFEby2GwRlUSLB+FtJD1V7EDUahGO71gvQVXU+l535bWGNnpNIIUxALw+/HhFuI2MXr3h\n",
"zZ7zqDUazazyRtRQ4UrgKeC7wDuA94fHpz0jhAweBH4NLAHyXoEhGe2Ylio2BNzkO1ZUROTzgGXa\n",
"7n5dVdYC2ug1x8tR7v7vgLvDfa16el5HlXUPL28BGfDyFpARL28BGfDyFpARL28BGfHC7ZJw+33g\n",
"ZN+xPu871m+oPwUSvVi/qRv1OEulUqVUKg2/5z3vWVUqlYZTmkQpYfFaxsuAJ1Lark/s/1O4Xdm2\n",
"0A6jjV5zRAEsV4SFYmHqy9pUIEsRwq2hGDqLoBGKobMIGqGQOqOX4yd9x4o/K+q9GI+ghj9Xo+b8\n",
"Okq1WvWq1erwgw8+eFa1Wh1OafIwypjFaxkfgSqOneQepg97RnOVj3VCayfRRi8jUhirgWNQqx1f\n",
"Hzu0a+X0lNN0RRaNRgO1R4TGE8d3IWSwEzUHCLOXvrAL37EmgUtRKWKYtnsoqiD2d0zb3cu03dtM\n",
"290jbP514OWm7Yrw81nAiO9Y2ujNEt3w9CIv7ydCBtti+7PM6elAFo1mflPL6DV6RlyFWl19TZ1V\n",
"ZaZh2m6fabtvNm33wOZlzmAYKJm2ewdqHvJ037HWoQJvVodbfMfygQ8C15u2ewvwHFReX88xVwMs\n",
"Omr0pDB2Qy1vBFPj7BFZPL25ep81Gk02ar0c101rEjJ4XArjVlQC+etRHlUjnouqY3wL8OGmlcbw\n",
"HWsUeGfK/oCp6M5o3zdQkak9zVz19IY63N8pwALg50IGjySO6ZQFjUbTiEaeXr0X4yh94Y1SGFme\n",
"JYvD7dKM2uYVc9XoDXSqo7AWXpQsn/TyoMVAlrAQbM9TBJ1F0AjF0FkEjVBIna0Ob4IKHHkE5Vm9\n",
"LMNlo+f6ggxtC3MvO8VcNXqdHN58IXAAqhZoWvHXep5evaGLStvKZodK3gIyUMlbQEYqeQvIQCVv\n",
"ARmp5C0gI5VwW2t4s6GnF0aKR/U4sywwGz1vMhk9inMvO4I2eo2JvmRXJZf5CGnJ09NoNPOKWnP/\n",
"WUsV/hjYDhwnhdEoQCV6rnd6mmdOoI1eHaQw9gdORH0xr6nRTFdk0Wg0jWiUslDX6AkZbAGuCz/O\n",
"qE2coFlPb16hjV59TkPdoxvrLGOkA1k0Gk0jaj0nJsNtX1iMuh5RTMFrpTAW12mnjV4dtNGrgRTG\n",
"EDPrbKaRZXhzrt5njUaTjVSj5ztWlYwvx0IGUfnDxUylUKWxa3gzgyGdd/TcsJsUxnmoJTVKwN7A\n",
"14QM/jU89jhwf6z57UIGF6R00wlPbw2qzlxU+LUWuvZm/nh5C8iIl7eADHh5C8iIl7eAjHjhtl4+\n",
"73h4vK/G8ThXAM9D1eO8MlYOMU50rTIqkn1nRo3zgp4zeqhK5K8QMnhKCuMQ4H4pDF/I4E7geiGD\n",
"GYmSKXTC6EVlf66o8cWKaCk5vYC1A3uWImiEYugsgkYopM6sc/+jDbq8GbVKw8GoBWjvSmkTH1la\n",
"QAOjV5R72Sl6cdjtjHCdusidfwZY1WQf/eGqxC0hhfFs4ChgM3BDg+ZZjJ6e09No5jcdmfsXMhhj\n",
"qh5nrfSF+HNdR3Am6DmjJ2Rwb/SzFMZpKMPz03DXEVIY10lh3CaF8dUGtejaSVCPvkzXChnsaNBW\n",
"197UaDSNyGL0so68XR2eU5HC2DvlePx5o4NZEvSc0QOQwjhKCuM3wOeAtwgZbAoP3Qu8WcjgRNQK\n",
"vtfX6oMW33CkMJYCVvjxynptw0linaen0Wga0WhOL96mLkIG61DzcP3AG1KaJIc3NTF60ugJGdwr\n",
"ZHAk6g96jRTGceH+c8J8FVDVv58fHUuhVU/vVJTBvEPI4NEGbXcZvDAKK0lTX2aNRjNnabVyUy3i\n",
"9TiTHqL29OrQi4EsuxAyuEcK4yfAXwNnJo5tk8JYDzyLxKKGpa1PHTZw7xXXViqf2Xnsscc+csop\n",
"pzwCeGkTtmHduQoAExOlAw444C/KW7bsvuWFxz0kko0T7YdPHOz7l5/vXFEuMeF5XiWl/4kFfez2\n",
"3H3Lr/U8b5/EsQ2VSuULdfUkDjXU3532H2T6ysl560lrvwr4Rg/pSW0fq3HYE3pqtF+FqvPYK3pS\n",
"20f/33pFT532q4BHdh9kv00qnGQ82X6/xaVDto9Xd3/HcwY+4nne5kz9/+c3MS4cXtC3ceNzn3nt\n",
"yf/+kOf9e6z9Lmdmzaq+EzzPO62e/hrPrrlLtVrtmX9rV6zcc+2KlW9M7PvS2hUrr1q7YuVJa1es\n",
"PDa2f3DtipVja1esPCHe/sYbb6wef+ENdx5/4Q2rWrj+i9euWHnn2hUrf7R2xcq+Ru2Pv/CGxeG1\n",
"bq1x/Izw+N8kj918883Ded/vLP+KoLMIGouiswgai6jz+AtvuDZ8FqxMtjn+whuuCo89q5m+165Y\n",
"+abwefW1RH9vD/u78/gLb3hJVo033nhjNe/7NRv/em14c3fgAimMhQDhJO3rgBtRRZ/fF2t7HvA7\n",
"4H9r9NXKnF6UpvCDGnU2k9QLYoFYtYUWtGg0mrlDljm9ZkfergO2oqZ5Dkm5FujhzRn0mtH7E/Aj\n",
"YEQKwwN+hlo08SvATUApjNy8HZU8fkod49TUnJ4UhgBOQOW01KqzmaTeFzm+v9fus0ajmV06Xq5Q\n",
"yGAr8JPwYzx9Qacs1KGn5vSEDEaBC8N/SQJU4npWmv1j/zmqCszPhAw2ZDynkaenozc1Gg10r0bv\n",
"D1AjVK+RwvhyGOinPb06zGUPJLOnJ4UxgIrahKmoqCw08vT0KgsajQbqvyC3OryJkMHvgTuBhcBr\n",
"wt3a6NVhLhu9ZkqRHQMsBX4vZPCbJs6r9/YGuvbmbOHlLSAjXt4CMuDlLSAjXt4CMuKF225Wbro2\n",
"3Earqjebp+e1eN1Coo2ewgy3/9PkNeolpsf3z/gyFyVEuAg6i6ARiqGzCBqhkDo7nacXxw+3z5PC\n",
"WECTc3pFuZedYi4PuzVj9F4Ubls1eo0CWfScnkYzTwkrN3WtXKGQwXopjPuBZ6OKUHdseNO03QXA\n",
"JcBhqN/hfN+xfpbSroIKOHw8tvtffceqW9UqD+a90ZPC2AtYDewAftXkNXQgi0ajaUTkeU36jjWZ\n",
"crwTc/93oIyemdjf7pzeMFD1HetFpu0eCvim7T7bd6x1iXZV4DO+Y32rzet1HT28OfUluSuMHm2G\n",
"rHN6c/nlQqPR1Gc2nhPREOcJTH/JbjllwbTdMipi/usAvmM9hFrE9owapxRiwdq5/DDO+seOhjbv\n",
"aOEaOjldo9E0opHR68Q0yK9Rieqrlm3bcP+GRbsqB7bj6R0ELAceiO27D3hBjfavN233Hajf1wX+\n",
"0XesLEU+ZpW57Ok1TFmQwugDjg8/tmL0GgWy1Pwyx+ow9jRF0FkEjVAMnUXQCIXT2U6UdyaEDMYJ\n",
"axAbz/xpVexQQ6NX517uG243xvZtBJJ1hKP9/wO8Anh1uP2nRtfOg7ls9LJ4eoejiin/CWi0okIa\n",
"Wb/Mafe50sL18qCSt4AMVPIWkJFK3gIyUMlbQEYqeQvISIXGI0It5+kluANgz20bVsX2ZfH0Kg2O\n",
"J1eQmTGM6TvWPb5jfdZ3rEnfsbaiDN57M1x71pnLRi9LcvquVAUhg7SlgRqRNWVhLg8jazSa+nTd\n",
"0wvxAZaMbllJddfjrObLf6lUqpRKpeHh4eFKqVQaTmkSBavEV1lZBjyRQUsALDJtd68MbWeVuWz0\n",
"snh60XyeX7dVbbJWZNFzehrN/GU25vQQMpDAH0swNDS+c2G4u6anV61WvWq1Ojw8POxVq9XhlCYP\n",
"A+tRI2IRR5BYyg3AtN3zTNuNP3P3RdUxfrrJX6PrzGWjV9fTk8JYAjwHZZhm/BEzMltvcBqNprjM\n",
"ZpS3P0mJhWM7loSfWw5kCdMrLgXOBghTFo4GvmPa7l6m7d5m2m7kBT6fcJUa03b7UKvgfKfG4tq5\n",
"MpeH3Rp5ei9EGaO7Y6uxN4v29DQaTSNmcxrkjslS+UND46OLgSfpTJ7eJabt3oHSd7rvWOtM2zVQ\n",
"+c2LgA0o4/gJ03bfCSwBfgn8bTsXDo3sIagSkZMor/F+37Eea6ffuWz0Gs3pRfN5rURtRmSdoNa1\n",
"N7uLl7eAjHh5C8iAl7eAjHh5C8iIx+yOCN01Ue6bHJwYW1ienOibLPe1VXvTd6xR4J0p+wOmojvx\n",
"Het/gJNb0DsN03aXAh8BzgJW1GhzP6r6y1daSYnoOaMnhXEeauHYErA38DUhg38Njx2D+mUnUGPN\n",
"ZwkZrK/RVU1PTwqjhErihNbn80DX3uwJiqARiqGzCBqhYDpvdo8IP3a9XKGQwfYtZ356HbBs4diO\n",
"xVuHFjcsuNEr99K03WNRxbOHUIb4flTQzHbUVNxiYH/gucA/Au80bfdk37EeT+2wBj1n9FAVAF4h\n",
"ZPBUuBrw/VIYPir58hrg7UIGt0hhDKNqwr25Rj/1PL0DgP1QbvkDddo1otEb3GSinUajmX/MSiBL\n",
"xPpFyx4HVi8YG12ydWjxJtN2+3oxSTxOOFx6DfAvqJqdOxq0Xw5cAFxv2u7xvmPtzHqtXnwYnyFk\n",
"8BSAkMHvpDCeAVYBAhgXMrglbHcZ8IgUxnIhg7QIoXpzetGb1z1CBmm18LKia29qNJpGzOrc/1NL\n",
"9ly3aGw7gxPTIji3dqLvLnIW8F7fsX6cpbHvWE8DHzRt98OotVB/kPVCPWf0hAzujX6WwjgN2Az8\n",
"FPgoyt2N2q2VwtiGihqaUfWb+rU3Dwy3D7cpV6+yoNFoGpH15bgjz+NnFi3btHDjjmr/5MRgeXKi\n",
"HM7r9bTR8x3rUy2ed1Gz57R8k6UwUiNrhAzaiqwJ+z4K+D4qMujNQgabpDD2BTYlmm4gvSQO1B/e\n",
"PDjctmv0sn6Z53JqiEajqc+spjZNlsvlsb7+HYMTYwsHx3cu3DG4UK+eHqOph7EUxlIpjH+QwlgL\n",
"PAj8BPgucDnK21orhfEbKYy/CutatoSQwb1CBkcCbwCulcI4DlUKJy3no1Zl73rDmweF29+3qjEk\n",
"67DFjJeLgtUO7GmKoBGKobMIGqFwOrPO6XVq5K28s29gO8DQxNgCGqRvFeVedorMN1kKo+nIGimM\n",
"k4UMmoqsiSNkcI8Uxk+Avwb+CDwv0WQZU6VydlHa+tRhpZ1bV1UqFe/YY4995JRTTnkE8CqViieF\n",
"MQSsRBmkR8M/eCXl8l5aVFO8/YlG3zG/Xjex4rDl5Vd6nnd3SvuJBX3stttQaQ/P84YTx1aREirc\n",
"jp5utAfOqvGfIhc9NdpXPM8b7iE9tdpXPM+jh/Skta94SmSv6KnVvqKa94yeWu0rbzis/5Hrfze+\n",
"246JmUbP87zKyw7oO/WeJyZWrN6zfLLnecs7oKe8s39wBzu3sWBybOkbnzvwYc/znqzTvkJxUkDa\n",
"plStNk6Yl8IwUPlsXwD+VcigbmSNFEYUWfMy4Hghg0yRNVIYewIVIYOrYvu+hDJS3wQuEjI4JKbp\n",
"D8C+8UCWkZGR6iduHb8LeNJ3rFenXOMw4DvAI0IGf55FVy1M2z0HeB/w775jXZxyfCkwAmzyHevl\n",
"8WPhQ3q4nevPBkXQWQSNUAydRdAIxdL5sZtHb0c5AiO+Y/1dso1pu28FPgRc7jvW59q9pmm7lw1M\n",
"jL1wv03rDp4olccfW7b/Sb5j/bKexkqlMjwyMlJds2ZNIdbEa4esw5tnAe8VMvhcI4MHIGTwtJDB\n",
"B4FvoyJrsrI7cIEUxkIAKYy9UTl7NwLXA/1SGC8N254NXFkjchNqz+lF83ntDm2CDmTRaDSNmdWU\n",
"BaA8Vu7f2TcxMdZXnew/7PGH9utQv3OCTMObQgYtRdYIGTQbWfMn4EfAiBTGTtTw5deFDC4GkMJ4\n",
"PXCxFMau5PQ6fdUax44iNzth9HTKgkajacSsGz1KJSiVNgJ7HfT0o6uB6zrUd8cwbfcE4MWo0mI/\n",
"DvetQi0E8KjvWP/djev2VMqCkMEocGH4L+34PUxVUmlELU+vU0Es0NjT08npGo2m5cpN7VxvvNz3\n",
"TInqXku3bz60Q/12DNN234+qzfkL1IrrHwde5zvWI6btgoqs74qz0FIovRTG0VIYthTGq8LPe0th\n",
"eFIYz0hhXC6FsaizMpumCvSH1b6TdCpdAdrz9LwOXH828PIWkAEvbwEZ8fIWkAEvbwEZ8fIWkBGP\n",
"2V1lAcLnzWj/4JMAC8d2HFi/eS738oXA4b5jvdl3rBcD7we+bNruPsAOakflt02r+WN/BbwSFbkJ\n",
"8DngROB24DjAbl9aW0T15qZ5e1IYCwgru9DaSulJGhm9yNMrmbY77V73Sr27RhRBZxE0QjF0FkEj\n",
"FE7nbBu9MsDWoUWPA/RNTqys1zine/mrsJg1oFZeR8Vp/CWqTGTXaNXoHQecLGRwmxTGYuDPUVGd\n",
"p6CGH1/bKYEtMhZuk/N6q1BvEIGQwRjtU3d4M1xLSgezaDTzm9me0+sD2DawcP0kpcnJUnk3KYw9\n",
"OtR3pwhM232nabuBabtHAfiOtd13rGFUla12ykPWpVWjNylksDH8+RXAQuCrAEIGT6BWzM2T6A0i\n",
"WYps2nyeabtl03Y/bNqu1eJ1Gnl68WO6KotGMz+Z7Tk99awplbaO9/WPjvUPlJl69vUEvmNdAdyK\n",
"Wmz2wcSxS0jPRewIrbrTA1IY/UIG48CZwL1CBvfFjued6xF5cXWNHnAo8Jbws9vCdRoFssD0oYuG\n",
"y3xoNJo5x2xXZImeS1vHy/2jY+X+EurZd1eH+u8IvmM9TI3YCt+xbu/WdVu9yf8L/EQK41FUqbAP\n",
"RwekMN7GzBqZs00mTw81v5fWLiuNvsyg0xY0mvnOrNbeZGpUadtYX//ozv6BQXrM02uEabtHA68H\n",
"7vAd66em7e4NXAEcjXJQzvYda1srfbc65PbxcHs68EPg3wCkMC4H/hO4usV+O0VWT2//cNtolfVa\n",
"NOPpTftCF6XeXRF0FkEjFENnETRC4XTOdhGLmNEbGB3rG4g8vVR69F52LVgys9GTwviuFMZpUhgL\n",
"hQyeFDKwhAx2EzI4LQoKETI4XcigLGTwL60K6hAzPL0akZvRcvSteryNxuqh9he60uI1Z5tK3gIy\n",
"UMlbQEYqeQvIQCVvARmp5C0gIxVmeWkhpp4123b2DYyOlfvLTKVqpVHp0HU7yXHAyb5j3Wba7q5g\n",
"Sd+x2g6WbMbTW4zy4p6WwrhaCuPtUhjLWr1wl0nz9Fah5hofjUVuRuV5Wv2yZQlkiaKQ9PCmRjM/\n",
"yWt4c+vNxQmnAAAgAElEQVREuW98dGBoHFjWgxGc9Zj0HSs1WNJ3rLaCJTMbPSGD1wF7A+9AuZxf\n",
"AtZJYfxUCuN9Uhi9VN8tuiFxoxe593+I7Ys8vVaHN5uZ09NVWTSa+UkuyenAdkoltgwtjhaQLdK8\n",
"3oBpu9H9OBO413esjgRLNnWThQy2oiYTr5DCGERZ4DcAw8C/SmH4wFXA1UIGnSjz1SqR0Ysbs7RK\n",
"LNGcXjc9PR3IotHMb2a/9qZiK8DmKaN3ME1GcJq2uwC4BDgM9Xuc7zvWzxqc4wF/8B3rnc1cK8H/\n",
"Aj8xbXdGsKRpu20FS7acOyZksFPI4Dohg3ehjMfLgTtRa9/9TgrjHimM1Bqas0Bk9OLJ6ckcvd2A\n",
"JeG+bgay6OR0jWZ+M9uF6XcFsgBsHlocrYzTiqc3DFR9x3oR8Fbg8rBUWCqm7Z4MHEP6gt/NkBos\n",
"adpu28GSHXGnhQwmgVvCfx+QwngB8EaU4E924hpNUm94M/JAV8SOlUzb7fMdq57HlkY7np7X5LXy\n",
"wstbQAa8vAVkxMtbQAa8vAVkxMtbQEY84KTw59nO0xsF2Da0aAfKAahl9Ly0nWHpxHNQnha+Yz1k\n",
"2u7dwBnA52u0/zDw70Bb84e+Yz0JzCga4jvW6Si70jLdmme6S8jgTuD8Zk6SwuhHhaq+DjVmOwj8\n",
"vZDBTeHxx1ErtkfcLmRwQUpX04Y3w/X5kpGb+yfO6ae+8Uqj5ZSFgtUO7GmKoBGKobMIGqFgOm92\n",
"Xxl+nK1AlmlGb+vgwiiaPbXwdJ17eRCwHHggtu8+4AU12p8J/BjYjRaMnmm73wWuBK7zHWt7o/at\n",
"0jWjh6qf1iwrUcOjRwsZbJbCeAVwjRTGaiGDPwHXCxlkGSeOojOjYctV4fbRsIoMpBu9ZiumZElZ\n",
"0HN6Gs38ptHLcbeGN0cBdvYNTobX3kMKY0GWhcBD9g23G2P7NgJHJBuatjsEnIuK8/h48nhGogwB\n",
"TNt1UUOY1/qOtaHF/lJp2ehJYZyAcj/3T/RTAp7VYrebgAuEDDYDCBncKIWxA5WXcWUT/SSNXtoa\n",
"emlGr1l0RRaNRtOIRtMgXR3epFTqBx5HORX7An9ssr/k/Fxa5ORfAV/3HWvUtN2W5vN8x3pdmJP3\n",
"GtSQ6peAy8LAmKuBq33HeryVvuO0dJOlMD6IGtPdBjzN9IrYJZTFbhohg/XAd2LXiYY4nwx3HSGF\n",
"cR3Kfb4POF/I4OmUrpLRm9FSFY/E2sTn9KC1e6EDWTQaTSNmLWXBtN0SU0YpGrnqB55AGb39CY1e\n",
"qVSqEEtMv/HGG5PdrQu3y4CnYj8/kbjmUtSUVNRXy+kEvmPtyhAwbXdGhoBpu7syBHzHailDoNWb\n",
"fB7wJuAqIYMZVl0K4+4W+03yMuARIYNbw8/3Ah8QMtgihfHPwPWoxQiTRF+uKJDFCLfxNfSSnl4r\n",
"EZw6OV2j0TRiNpPTo6HNSaa//P8p/HlXPnW1WvWIBbGMjIwkS3s9DKwHDkeV/wI1tPnjRLvjgN2B\n",
"kXDV81XAAtN2b0KlOPit/CK+Y+0ErgOuM233PagyZG9ETYF91rTdXwNX+Y7VVLBkqykLW4QMrkwz\n",
"eCHvabHfXYRlwz4NnBXtEzI4R8hgS/hxGHi+FMZxKadHw5vRly0yemtjbSKjty3Rthl07c0eoAga\n",
"oRg6i6ARCqdzNmtvRn1E83gw3eglX/Zr3kvfsSaBS1GLu2La7qGogs/fMW13L9N2bzNtdw/fsW70\n",
"HesY37FO8h3rJOCbwA2+Y728VYOXpsV3rFt8x/qA71jPAl6EMohNR3K2avR+K4WxtM7xWtE9mQiH\n",
"Nb8GfF7IINVrFDLYhnoLSZs/3JWyEPY1zeiZtrsE9Waygym3vZ05vSyBLMn+Ky1cLw8qeQvIQCVv\n",
"ARmp5C0gA5W8BWSkkreAjFTIPqfXSU9vgulGL5oLS6ucVanT3zAqpesO4LvA6b5jrUOVBVsdbndh\n",
"2u6lqFHAimm7V8SqqnSar/iOdb7vWDOCahrRqqAPoSqw/BCVQrAldqwEvB+4uMW+AS4CfCGDK8PK\n",
"L/sChwCbhAzuAgj37wE8ljy5/yH3YxMrjjXKT//2ry95/akvPOfW21eXd+5c17dlyzNhk+ht5/Gl\n",
"Q+wJrHjzswc+5Hne+lg3Xloob/hWVAHYZ1Hp8NGJ6qJzjh74kOd5P0lrv/ei0n47J6orXrGq/1zP\n",
"814VO7Qq7ReP95881EhPN9oDqzzPG+4VPTXaVzzPq/SQniK3r4R/717RU/T2lWctLS19Ykt10Y6J\n",
"mZ6e53kV+8TBV3zh5ztXlEuMx/6vtapnl6fnO9ZE5VPu4qE+lq79y788Zd/LL18xtu++J3ueV63V\n",
"fxLfsUaBGRHzvmMFTEV3xve/q1GfWTFttxvBkpSq1eYDbaQwjkZlyde6cFXIoKW3FimMv0MFmZyP\n",
"+uUOQL05PAKcKGRwbtjuw6gQ2aOEDHa9QY2MjFQ/cev4m4CPAVdeedm5PwK+ATwgZHAGgGm7L0QZ\n",
"5TtRVVkOB870HSueA9gQ03Z/hjK8r/Ida32NNheh5ib/1ncsL9rved5wpVIZbuZ6eVAEnUXQCMXQ\n",
"WQSNUCydH7t59GDg2cA7fMf6TbKNabsDwB3AuO9YZjvXCytN3Qxs8R2rYtrufwNDn7vKecuB64Pv\n",
"AY8JGZya1FipVIZHRkaqa9asyXsBcABM220ULLmf71gLWum7VU/vy6jInn8LBSUtp9NKp1IYq4HP\n",
"hB/Pix0aBm4CXiqFcRvql94EnBI3eDHiKQsro+5jx6PyZKNAdOO6PbypA1k0mvlJ1jy9ftN2S75j\n",
"tVPCKx7IEl1z6JrnWk9/0LsMYB8pjL4az81eYlewZNr9CCvDtESrRu9g4OBaSY5SGC9rpVMhg99S\n",
"f57xnIxdxY1eNJ8XxI5HUZ2jTB/3bpaWA1k0Gs28oW70pu9Yk6btTqKefWWarwwVJx7IAuGz8LZD\n",
"zOoHvcvWA3sCe5FIO+hBtviOVS83u+VgyVYDWR5ukNWf1Th1i/h6epGnl2b0xmgvMbSd5HSvhevl\n",
"gZe3gAx4eQvIiJe3gAx4eQvIiJe3gIx4zG4Ri3ggS/ya/aSkLYR4bV6zG/w2zP+rRcvBkq16et+R\n",
"wni7kMG3ahy/k9bKkHWKeMpCZPTi6Qppnt6spiwUqnZgj1MEjVAMnUXQCAXTebP7ofBjo+fEAOo5\n",
"1PICqcz09JIRnEeijN6vpmnsPT6ESkbveLBkq0bPBF4phfGRGoJajqzpEPFVFtJy9IZi7ZI5fZkI\n",
"Kx8kv2Bp6OFNjWZ+k6VGb6fSFpLXik/11Etb6DX2BF4CvK3G8ZbnPVs1em9FpQrshqqIEhfQchmy\n",
"DjIO0D8xthB180aZKmUGnfH0dn25Gkw8a6On0cxvsgxvdsroJQNZ4i/1NRPUe5CuBEtC60bvPiGD\n",
"59U62MEyZK2yE2Dxzu3RmLAM1/yL6MScXpahTehgXT2NRlNIZnNOLzWQJdQQeXpFMHoHAwf7jpUa\n",
"O2LabkvBktB6IEujdfLaLkPWJmMAQ2OjkdELEscjoxcf3mw2ejNLukL8uPb0NJr5STNz/+2+HEfP\n",
"9PHENl6KbEZSeQ/ycC2DF9JysGQmoyeF8RdSGLtWJRAyuL5eeyGDn4fnHSuF8dJWxbXBGMDgxNju\n",
"4ee1ieOdGN7M8vYWP65rb3aJImiEYugsgkYonM52yhU2S63hzfic3v5heca4xl7jO6btvr3O8Ttb\n",
"7TjrDX4SGJHC+HMhgxkVBdKQwnglqn5mbkavf3J8McqwJUuVxQNZ2p7Ta9CulqdXoTdDhZNU6H2d\n",
"FXpfIxRDZ4Xe1wjF0pnn8Gbc09uEqje8GFgEbI1p9Nq8bqcxgVeattvxYMlMD3ohg5ukMC4G7pHC\n",
"uAZwgQdRE407QhGLUGPFRwGvR+VRvEHIIDm0OBuMAZSr1YXMDGKBqaHMTgxvZp3T08ObGs08Y1KV\n",
"ecwjkCUZvdkvZFCVwngSFdG+D/CHNq/VTboWLJnZuxEy+LIUxr3AZ4Gv1mlaBW4AjhUyeLBVYW0y\n",
"BlBSRm8DU4shRvSCp6fRaOY4k9VdC6o2ivLu1OrptTy9qN91KKO3N71t9O7zHatmsOSslSETMrgZ\n",
"eIEUxpGoBf0OAZaibvB6lBt6c07eXZzw7aYaGbekpzebc3ra6Gk085TxyRmeVy26VZElOZIVPQv3\n",
"bvM63aZrwZItvVWE83qZ5vZyYifVKiVl3KqoPI848ejNVmtvNpuyoI2eRjPPiBm92XpO1MvTgymj\n",
"t0+b1+kqvmPVDZb0HevnrfbdaspCrzPWNznRP17uKwNPCxkkv3BpRq9VT6/VNzivyevlhZe3gAx4\n",
"eQvIiJe3gAx4eQvIiJe3gCxsGK3+T/hj1ijvbg1v1vP0vDav2Tam7f6FabsrGreccd6xpu02FSw5\n",
"V43eeP/kRP9Eqa80OXNoEzpj9Nry9Hq03t0MiqCzCBqhGDqLoBGKo/Nf7xzzwx9nq4hFrYLTkdGL\n",
"4ht2eXo9ci+fBEZM2z0y6wmm7b4S+AFNzk32VJUQKYx+4K+A16EidAaBvxcyuCk8fgzwFdQfdD1w\n",
"lpDBjMVbfceaWPOxq/oowUS5/6mUS7Vde5PmPb2eutcajWZWmO0iFvXy9GDKCdirzet0FN+xbjJt\n",
"92LgHtN2m8oQCFdxz0yveXorgb8GThUyqAAXAtdIYewvhTEIXAN8TMjgJcAvgUtqdTQwMV4C2DEw\n",
"lLaieSeHN/WcnkajqcVsPyei6xVuTs93rC8DrwJWoTIEPOA+4PfAw8D/AT8FLkLlGB7rO9bPmr1O\n",
"r3kfm4ALhAw2AwgZ3CiFsQN4MWGdTCGDW8K2lwGPSGEsFzJIBqowMDFWBtg6uOiZlOt0cniz0Rtc\n",
"p/JvNBpN8WirclML1MzTC7eR0Vveiyuo+451M/CCcJizZoZAs95dnJ4yeuFQ5Xeiz2GpnEGUi/sq\n",
"4IFY27VSGNtQ6/bNsPaDE2N9AJuHFtczeqPo2psajaZ7ZJ3771QgS3J4c9qcnpDBmBTGM8Ae4b+0\n",
"6Z/c8R2raxkCvTa8meRlwCPAbagiqRsTxzdQw00fGh/tA3hqyfLkOdADgSw9Wu9uBkXQWQSNUAyd\n",
"RdAIxdH5+tX9J4Q/ztbLcXIEKi0la1owS717adruAtN2v2Ha7h2m7f4iDB5Ja3ecabtXm7brmrb7\n",
"S9N2Lzdtd4/Wf43u0ZbRk8IYkMLYWwqj45OiUhgLgE+jglWqqHy7tIoGpZR9DI6PDQAEe+y/Ie1w\n",
"uM279mYRqOQtIAOVvAVkpJK3gAxU8haQkUreArKwx4LSi8IfZztPr9bwJkx5d1HaQqVOf8NA1Xes\n",
"F6FKg11u2m6ao3Eq8CPfsSxUgMnewD80pXyWaMroSWEMhisufE8K4/eoqJongHVSGGNSGGulMH4o\n",
"hfEuKYzdWhUVDmt+Dfi8kEFUbmYdsCzRdBkzS4wBMDQ+OgDwm/0P25xyuJNLC+lAFo1Gk8pEtZo1\n",
"OX228vQgJW0hDdN2y6glfL4O4DvWQ8DdwBkpzb8JfDdsN4kKNGy5KHQ3yXyDpTDOBj6EWtLhJuCf\n",
"UZOi61EP9uWoVcqfh5qA/FspjG8D/yhkMJbaaW0uAnwhgyvDqM19gV8Ap8f0GKgQ1ruSJ7/6xJfe\n",
"euCB1h6TkxOl0tJNw57n/QrwKpWKZ9puiVjKwotX9h1175MTK/ZdXHqJ53nDsW68tPyVcCig8rrV\n",
"/Qd6fxxfsWJJ+fjwvNT25oryUfc/Pbli/yXllyb6X5X2i0f9px2qp6db7YFVCd256qnRvuJ5XqWH\n",
"9BS5faXe97kA+nuq/cL+0nFLh1jRV+KJlD52tX/O3uUTH900ueJFou9Mz/NemLX/jaPVoaVDpdFY\n",
"+2me3pF7lw9du2lyxcF7lF/ted5SgL1f9apjd7vttt36tm9vNEJ3EOq5/kBs330oT24avmP9LvrZ\n",
"tN39gdcAf9+g/1woVav1aqAq7w64FPg18LUosrIRUhhDwNuBk4F3CxmkemQp5/0dsAJVe60EHAC8\n",
"CfgM8Fvg7UIGt0ph2MARQgZ/ET9/ZGSkevjbzzrwn9e8785H9xR9f1q630t9x/q/6LhpuwPAHcC4\n",
"71imabsnAF8C7vAd67wsGsN+LNTwq+s71ifqtKsAnwNu8R3rw9F+z/OGK5XKcNbr5UURdBZBIxRD\n",
"ZxE0QnF0fvH7I1/73n3jzwfu8h2rZr1I03YvQOUnf9p3rKuz9G3a7ltRjsh5vmPdEe57DfBJ4Hrf\n",
"sS4wbfc04OPAVb5j/T8AKYw3AJ8AfiRk4ET3cmRkpLpmzZpSrP8Xo+IphnzHGgv3fQp4ke9Yr6ih\n",
"6QrAAv7Jd6xPZ/k9Zpssw5sfAT4tZHBRVoMHIGQwKmRwKaow6MeznCOFsRpl3M4DNqNSGO4FqkIG\n",
"O1EJif8ohXEbKmrzvTW62qd/cqI6WeobZ2ooMyL6HHmf7dbe1MnpGo0mlYlq5tqbTaUsmLY7CLwz\n",
"/Hho7FByeDNtTi/T8GaMpGeUGkcB4DvWm1AJ5CeYtvvZjP3PKg0fxEIGbVlrIYMngL/J2Pa31DHE\n",
"Qgb3ACfUOh5jr77JiepEuTzBzN8xnq4AuvZmI7y8BWTAy1tARry8BWTAy1tARry8BWThsc3VXwHH\n",
"0Pk8vVeiUg5g+rOrURkyCHP1frR927PfVyoNr169etVvf/vb4RtvvDF5jcg4LmMq+GUZpA/VRviO\n",
"tdW03c8AN5u2+wnfsXZm+YXSCEfmlqGCaTqSXtGy9yGFcRFqQvPE0FghhXEG8GMhg7SIydlkn77q\n",
"RHWi3DdGbU8v+kN0uyJL6pe5R+rdNaQIOougEYqhswgaoTg6b187EU2tdKz2ZhiXcHpsV/yczIEs\n",
"pyxcVHrv+qeHo50jIyN24lIPo2I2DgduD/cdAfw4RdO7fcf6WmzXtlDLAqaetQ0JPdg3oEb1jkcF\n",
"w5TCYxMog3sn8BPgct+xMo8+RrSTsvAMKsJyW2yfC3xQCuO5bfTbCfbum5yoTqjhzeSwZbzuJrRe\n",
"e1MvIqvRaBrRjSjv5wDPTrkGZEtZ2Bju3y1MDUsljMK8FDgbwLTdQ4Gjge+YtruXabu3mbYbRdSf\n",
"HsZHREb5XOBO37E2Zfh9CM87GxX1+WpUsORpKKO3BFgY/vxnwJXAccCdpu1eEHqDmWlnnqkKfEXI\n",
"4LFoh5DBk8CwFMY/oAJf8mJ5/8REdaJcHmfm7xjdoNny9LTR02jmL80+J7I8h6LgvS0ogxB/6Dcq\n",
"OI2QQVUK42lgP1R0pqxzrWHgEtN27wi1ne471jrTdg1gNSqCfgMq4v5Tpu0C7AasBd6c4XeJvLso\n",
"WPJFdby3x8J/9wL/adpuFCx5RehpZgqWbMfoXQXcLYVxN2p8/RbgF+HadQvb6LcTLO2vTlQnSuUJ\n",
"Zg5vJj29vJLTNRrN3KejL8em7e4NrAnbXwO8jenPrmTB6VrPtw0oo7eMOkbPd6xRpgJm4vsDVCpZ\n",
"9PknqCHHVvgIKmr1t82cFGq71LTda1HBkpliR9oxep9E5WGsRIWoXgiUpDCeAv6jjX47wbLy5GR1\n",
"spw6vJkMZOl2cnr05dNGT6OZf3S69uZpYZsRlDcF6Z5ecngz+XyLyjMubXC9rtNuaoPvWJmDJaG9\n",
"Ob1fChlcKmRgCxmchIok+jPgv1Bze3mytH9yvDqpPL1aRq9Tnl5LgSxFqR1YBJ1F0AjF0FkEjVAc\n",
"nc/dp3x0+GPbI0LhMOAbw4/fJz1IpVbB6TRPD2BpUe5lp2jH6G2Twjgs+hDm5d2C8v7+rG1l7bGs\n",
"fyploVtGL5oA1rU386eSt4CMVPIWkIFK3gIyUslbQBb2XlR6TvhjJ1IWXoGqevVbVDmwtCCV5LRL\n",
"LU8vMnrL6NF7adruRabtPmHa7urYvjNiwTMt0bLREzL4ImBJYXwo2ieF8XJUVOfB7YjqAIPl6uRY\n",
"tVSu0jhlodXhzejL/Lu6rfScnkYzb5nMnpxe9zmRSFP4vu9YVdJf2LPk6UEPDW/WoWaGgGm7LWcI\n",
"tFUlRMjgS4ldtwEfRgW15EpVrawLjVMWmvb0wqih56rLzKz9mUBXZNFo5ikdrMjyXFSO3AbghnBf\n",
"PU+vXkUWmO7p9SpV4Cu+Y+3KEPAd60lg2LTdljMEOvogDgtL/1sn+2yDrajfrxsVWZ4T9vOg71hp\n",
"6/XF0Z6eRjNPiXl67ZYrfEu4vTKMWoRsc3q1jF7c0xulN7kKuNu03WkZAr5jtZUh0HB4UwrjHVIY\n",
"LT+wpTDKUhiZCzl3iiqlLeGP3ai9eVy4vTND22bLC2k0mjnC5GTTw5szjJ5pu/sBJ4V9/CB2KG1q\n",
"Jmv0ZhE8vShD4H9RGQIjwGbTdgOmRvKaJsuc3q3At6Qwjmm287CA9PcAv9lz26ZUqjW8Oc3T8x1r\n",
"AuVGl0zbzWqYoqU1shg9XXuz+3h5C8iIl7eADHh5C8iIl7eALDyxbfKh8Md25vTeFO6/MRzei0gb\n",
"pcpShgyme3peA2158UvfsS71Hcv2HatjGQINjZ6QwR+AvwbeL4XxMymMt0lhHFSrvRTGCimM06Uw\n",
"rgK+CHxUyOAXrQpslapapQEaz+lBE0Ocpu0uAo5EfUl/mUFK6pe5KLUDi6CzCBqhGDqLoBGKo/Ph\n",
"Z6p/CH9saU7PtN2FTKUpfK/GOfU8vVpGb5en18P3cptpu7syBHzHGvUdq+0MgUzRm0IGTwsZvAtl\n",
"/F4M/EwKY70UxgNSGL4Uxv+EPz+JWuzVAr4kZPBqIYM/tiquHSZL5ajmW/KPnSxDBs3V3zwmbHe/\n",
"71hZXGydnK7RzF+yrsZS68X7NaiyXr/2Hes3iWNpz62m5/SkMGouFZQnvmN9EbBM292VIWDabtsZ\n",
"Ak0FsggZ3A/8JYAUxirUAq/7oG70k4CMVlxoFSmMY1GJl58SMvhmbP/jwP2xprcLGVxQq5+Jcl9k\n",
"9BqlLEBzwSzNDG3G+15g2u6ZGc/RaDRzgyPDbdPDm6btlpkKYLk85Zx2hjd3oKZ4hlA5x9sb6MsF\n",
"37E6niHQTvTmH4HNQga3ttHHNKQwTgXeinoLSS5ceL2QwYwacLUYL/dHbzLJ37Gt4U2mjF7WIdux\n",
"sP8B4AMZz9FoNHOLLQ2Op02DHA+sQi2nc1PKOfU8veTwZr9pu6Uwvy8qOr0R5bQspUeNXpJwBfe2\n",
"MgTaMXqPAPtLYfwIFaiyEbhSyODpNvq8W8jgWimMm9voA4Cd/QOR0WtUexMyJqibtrsbam2pceBX\n",
"WXT4jrXTtN2Po5bk0Gg0849NpButOGkv3pGXd0UYpl/rnPhza5qn5zvWpGm70Woz/Uw960DN6+2D\n",
"iuB8vIG+rmHa7juAb4dBha2cXwbe7zvWl7O0b8fofRD4o5DBLwGkMPYA3iyFgZDBV1vpUMggqHP4\n",
"CCmM61Dj2/cB59czsDsGhqKJ2kYpC5Dd03se6k3qV75j7WjQdhe+Y90MTDPknudVengCeRdF0FkE\n",
"jVAMnUXQCHNS57SUBdN2VwEnoIYhr65xTtrLetrqL7WM3kaA9aeecpKABzJo7Ba3At8ybfezvmPd\n",
"08yJYYmyTwGfy3pOO2XIro4MXvj5mdDY3SCFkbnidRPcC7xZyOBE1B/r+nqNNw8tiYxeFk8vq9Fr\n",
"Jj+vEZUO9DEbVPIWkIFK3gIyUslbQAYqeQvISCVvARmpZGyXHN6MSo5dV6cARr0yZJMp7XY9C03b\n",
"/avPvPK8509SYnzZshMyauwKvmPtyhAwbfdnpu2+zbTdmhkCpu2uMG33dNN2d2UI+I6VOUOgZU9P\n",
"CmMIVRbnN0IGu+bHhAz+KIWxqNV+ayFkcE7s4zCwSQrjuFrpEOuWLI+8wCwpC1nrb3bS6Gk0Gk3E\n",
"LqNn2u7uwGvDz2kBLBH1hjfjnt6051u40vgZv9/rALF+8bKn+rZsqbl6+mzhO9bTwLtM2302cB7w\n",
"SdN29wDWoYZhJ1HFtpejnt0/Bb7kO5bX7LXaGd78Icr9LklhXIOqB3dv2OexbfTbECGDbVIY61HL\n",
"x88wesHg4Oonf+9+uXT4KYftPlTaw/O8YcALhxmmeXqe51WM3UtHbBqt7nXa4QMf8DzvqfC4Fx+W\n",
"CP8AhyzsZ+hjJwy+3vO8UxKX9dKGMcJlOyopv8aqtN+tTvtm++9Ie2BVeP96Qk+N9pVaw0i9dj8L\n",
"0L6S+P+St56it890P888qn/fax8aX7HHglLfAUvL3/y5nNh7xwQ3+o71+1r9f/yEwaGL79q5or9c\n",
"Wh77/k8LZPE8r7LPotKq0Ynq4rOPHviY53lbzz16YPm37x3bY7JUntgxsKC8ZMvW3I1ehO9YuzIE\n",
"wiHeZIbAWt+xHqrZQQZK1WoySDIbUhifAD6DKo/zNtSbyV6oMeh3Chl8v1VRYSDLfwgZfCv8fBKw\n",
"ScjgrvDzIKoMzcuEDP4nfu7IyEj18Lef5Z527mXnoZI5f+c7VjRUgGm7X0UZ5fdFLrFpu98AjgLe\n",
"6TvW/6VpMm33FcA/Aj/3HesvW/3dIjzPG65UKsPt9tNtiqCzCBqhGDqLoBHmnk7Tdo8EvomaW1uG\n",
"WtX8A75j/Xedcxah5sN2+I71knDfp1F50n/vO9YN4b5rAAG8zncsadruqcCFu2/fvOffu1/Ytvez\n",
"jceeffn3XjUyMlJds2ZNT+bsRYTafd+x1rXaRzvr6f0f8CHgASGDs1FLx+8HLGvH4IWUwn8RBwDv\n",
"i30+D7Wkz//WOH8DtYcs6+Xp1Rve1EObGo2mW0TDkYehnqN/BO5ocE6WVRZg5vPtMICJcnl8+8CC\n",
"cnn79p7x9OKYtnueabunhKNsEfcBbzJt9z2t9ttOIMu1qDeTE8PPVSGDdfH5vWaRwjhKCuMK1Fzh\n",
"+6UwoijQm1DDqLdJYdwOrAFOETKoFeK6kcZGr9lAlmjItlMl1bwO9dNtvLwFZMDLW0BGvLwFZMDL\n",
"W0BGvLwFZMTL2C56lkUv+9/3HWuyVuOQaTl44c9pqzokn4WHA0yWyhOj/YPlQSnbSTPrJkcB/wE8\n",
"ZQfNEyEAAB3oSURBVNrur03b/TfUc/g2VA5jS7S7nt6T1J9obba/e1HFVZP7A+CcmWfUJO7pJVMW\n",
"6gWypN4P03b3Rs3BbWN6VZiWKUK4NRRDZxE0QjF0FkEjzEmdcSO1BfhxoxN8x6qm5ODV8/T6w6L6\n",
"qwEmSuWJ0f6hwcHg8ZZXLOgmvmO9x7Td96KM34nhv88C+wOfbrXfubqwadzTS/6OabU3G3l6URWW\n",
"u2skiWo0Gk07xJ8rP/Qda1vNljPPixu9Rp6eQbgW3WS5PDHaP1CmzvJCpu0uAC5BDYn2A+f7jvWz\n",
"lHYvAf4u7HsZKrDxgqgCTKuE5/9f+O/i8FqvY/r0V1O0M6fXy2xgyqhl8fSyGj09n6fRaLpB9Aya\n",
"RC2d0+x50ct8ozy9XasWTJT6Jkb7h8qoMmS1GAaqvmO9CFUi8nLTdvdJafcp4Ku+Y70CeBlwGir2\n",
"omVM2x0ybfd5pu1Oe4b7jnUNbVS4mqtGbyO1g1PqlSGrZfSiIJZZXyJJo9HMC/6EqrzyZd+xHmvi\n",
"vOSzK214M94mMnpj1VJpckf/EMDCMO96GmF5r3OArwOEqQJ3A2ek6Ljad6wfh+22ooZnX9XE75HG\n",
"Nag50cdN273EtN1XmLa7NDS6q1rtdK4Ob06L3owXWqXJ6E3TdgWwAlU/r638EI1Go0kjfD61Mk9V\n",
"y9NLG97sJwxiAX5DqXTMtsFF0ct/mrd3ECoZPF6i7D6mRr52kbIawkJUYnk73I5aWqmC8jK/jcrZ\n",
"G6MNL3LOenph5NO0YcswwqnZpYWiqM1ftloQNY0wybTnKYLOImiEYugsgkbQOmNk8fSi59sgU57e\n",
"LwG2DS4Ym1i4cDfSjd6+4TZeBi1amaEmYbDMq4CvNBLfgHuBjwIP+Y51ru9Y+wF7A8t8x/paq53O\n",
"VaP3p3CbfAvqQ/3OEwkDVm94s1tDm5UO99ctKnkLyEAlbwEZqeQtIAOVvAVkpJK3gIxUutx/8oW9\n",
"ntGLjNsoKg+Q7QMLqpPK6NXL1UsGozQKIvlb1HBnW89M37F+CFxGLD3Bd6ynfcdqaxmkOTm8KWTw\n",
"cPjjTtQfMzJ6aUEsUGN4M/QMdRCLRqPpVZI5ePWGN/cKt88A28Z3bNntF9u3Ld30+J8W37jhmb9K\n",
"ccui4cllwFOxn5+oJca03dcAz2eqYHZb+I71FPCDTvQVMVc9vYjkFyItiAVqD28egHKn1wO/R6PR\n",
"aHqL5ChVPaO3d7jdAGztX7Bk8+o993vyo/vtv/nmffb7ZkrfD6OefYfH9h1BjVEv03ZN1GoJZ4Y5\n",
"hIc096vMDvPF6A0mtmOJdrWM3i4vr918E41Go+kCaVM4kD68Gc3FbSBcKX20fzAaqpwRvRnGRVwK\n",
"nA1g2u6hqFSB75i2u5dpu7dFJcLC1RG+CJwLDJq2uwSw2/vVusN8MXpZPb1k9Kaut6nRaHqZ5At7\n",
"luHNDajqUuzsH4zaJ/OZI4aBkmm7dwDfBU4Piz0vRFV2WRi2uxTlJDyKinTfBLy4+V+n+8zJOb0Y\n",
"0dxd0ujVmtPbdT/CHJVuzud5XeizG3h5C8iAl7eAjHh5C8iAl7eAjHh5C8iI1+X+s3h6yeHNZ4g8\n",
"vb6BvvL27ZtJ8fQAfMcaBd6Zsj9gKrqTaJWHIjBfPb2k0UuL3jyIqUnboNPC5mDtwNwogkYohs4i\n",
"aAStM0aWOb3IMC4PtxtQS7Mx1jfQ36eMXi1Pb84x141e8i0os6dHbGhTz+dpNJoepdbwZtqcXuQF\n",
"7vL0xvoGon2pnt5cZK4bveTwZqOUhbjR06kKGo2m10mOZtUb3oyIKlaNT5TLpbFyXwlt9PJDCuNY\n",
"KYzfSWG8I7H/GCmMO6QwbpfCuFYKY88M3WUNZJk2RBBWFIgqsWijp9FoepVmAlkiNoSjV9urlKo7\n",
"BhaU0cOb+SCFcSrwEVSpm2ps/yCq+OjHhAxegiqhc0mGLmulLDRKTj8MWAKs9R3rT2g0Gk1v0qqn\n",
"B7C1WipVw7QFbfRy4m4hg9NR4a5xXg2MCxncEn6+DHijFMZy6pOc5M06vNn1oU1dO7BzFEEjFENn\n",
"ETSC1hkji6eXXAP0mXC7vUppctuSpbujhzfzIVwhPY3jiK1YLmSwFpVn8vwGXSbX1ItySpK125LG\n",
"cTbm8ypd7LuTVPIWkIFK3gIyUslbQAYqeQvISCVvARmpdLn/ZpLTQY2gRU7FNkql6uiixUvQRq/n\n",
"2JeZ3t8GGlT7ZuYXYlG43VqrnWm7/cDzws96Pk+j0fQyzQ5vbowV298+WSpVR/v08GYvUmVmpW9o\n",
"XO07+YWIjF7S04sPERyJ8gj/EBY71Wg0ml6l2UCWDbGft1UpTe7sG5hX0ZtFqciyjinvK2IZNRYp\n",
"jBJC+w6sGAMrj1764oN2e4vneeuZGt7cFrX1PK/ypsP7z/zpH8ZX7L2odMJ+i8sH/d+TEyuWDpXu\n",
"Sus7HKOvpB1KS0St035Vl/vvSHtgled5w72ip0b7iud5lR7SU+T2lfDv3St6it6+q/fz+BXl4x54\n",
"enLFfovLR4a7phk9z/Mqpx7a/9ZbHh1fAbDnglI50gNso0R1xzwLZClVq72Xdy2FcTPwH0IG3wo/\n",
"vw64SMjgkPCzAfwB2FfI4On4uSMjI9U1a9aUAEzb/RvgbcAXfMf6tmm7HwdOA/7Jd6wronNM230h\n",
"cDFTw5kvAD7qO9ZN3fodPc8brlQqw93qv1MUQWcRNEIxdBZBI2idEabtvgd4F/A137G+ZtrubaiX\n",
"+5f5jrU1bPMy4KJIku9YfxvuP3/Bzu1nvuueq/et/Pqmyx/41jfeFz075zK9OrxZYvrQ5fVAvxTG\n",
"S8PPZwNXJg1eCjvC7cLEdluiXTREsAh4bvjzL5tS3Dxel/vvFF7eAjLg5S0gI17eAjLg5S0gI17e\n",
"AjLidbn/ZtbTg6nITYDt1VJpcke1PMo88vR6yuhJYRwlhXEFas2m90thfA1AyGAn8HrgH6UwbkNF\n",
"bb43Q5dRwMqixLZW9OZq1B//t75jbaCL6NqBnaMIGqEYOougEbTOGM2snA4z5/Sqo5RH0XN6+SBk\n",
"cC/wphrH7gFOaLLLyOgtDreNPL3ofrS1zL1Go9HMEskI9UZ5enFPb5tKTh/SFVnmEEmj1yh6MyI1\n",
"iEWj0Wh6jF3Dm6btlkgvOF07erNUmhxVa+rNG09vrhu9yKNbEm4jTy+Zpxf/UkzS/fk8jUaj6QTx\n",
"UaroeV5NrAxTy+htr1Kq7tRGb06R9PSibT1P737fsbZ0VZVGo9F0hng1qbShTag3p1cqVXf2Dejh\n",
"zTlEZLyiYc1Gc3owS1VYdO3AzlEEjVAMnUXQCFpnjDRPbzLRplb05rZqqTQ5Nji0AO3pzRlqzekl\n",
"jV78SzFbQSyVWbpOu1TyFpCBSt4CMlLJW0AGKnkLyEglbwEZqXS5/3jKQlrkJjSI3hwbGBpEG705\n",
"wy6jF9bUHER9IZKrLESfx4FfzZI2jUajaZf48GZk9JLDm1GbHb5jxad2tlMqMdo/CI1LOs4Z5ovR\n",
"W0LMy0tM8uI71mbgq8A/J74UGo1G08vEUxZqDW8+AzwB/DqxfxvAzv4B5hM9lafXaXzH2mna7hjq\n",
"C7E03J1q1HzHunTWhGk0Gk1niBu9VE/Pd6wx03bfyMzUrG0Ao321Y1hM212AWrD7MJS9ON93rJ/V\n",
"aLsE+Aqw0nesk5r7NWaPue7pwZS3t3fis0aj0RSdtECW5PAmvmONxpYUilBGTxWcrsUwKgXiRcBb\n",
"gctN252xpJtpu0uBH1LDqegl5oPRiyI49wq3vfJH8fIWkBEvbwEZ8PIWkBEvbwEZ8PIWkBEvbwEZ\n",
"8brcf9qcXnJ4MxXfscaA8XHK4zvL/TMMn2m7ZeAc4Oth+4eAu4EzUrobB94M/HdT6nNgTg9vhkSe\n",
"XfR2kozczAVdO7BzFEEjFENnETSC1hkjk6dXh20TVXaODgylOUAHAcuBB2L77kOtQjONcEWHrWFV\n",
"mJ5mPnh6yeHNXvH0NBqNpl3iKQu1Alnqsa1aKlV39KcavX3D7cbYvo1MORCFZD4ZvWh4syc8PY1G\n",
"o+kALQ9vhmyDUnVHuqcXkVx0tee9uXrMB6MXGblem9PTaDSadklLWcg0vFkqlSqbH/3N6vXbNg79\n",
"29at+6Y0WRdul8X2LUOlPxSWQs3pSWF8A3hWYvdrhQzqRWRGgSzR8Kb29DQazVwhLWUhk6dXrVY9\n",
"03afd/ll534OeNkDU8/IiIeB9cDhwO3hviOAH7cnOV+K5ulVhQxOSvxrlILQk4EsunZg5yiCRiiG\n",
"ziJoBK0zRluBLL5jVUcPOCDNy8N3rEngUuBsANN2DwWOBr5j2u5epu3eZtruHi0rz4miGb1WiIze\n",
"gnDbE0YPXTuwk1TyFpCRSt4CMlDJW0BGKnkLyEily/23G8jCzv33W1nn8DBQMm33DuC7wOm+Y61D\n",
"Fe9fzVQRf0zbvQT4S+AI03avMG336GZ0/P/2zj1arqq+458fCUk0oUYIBBwuhLchLB4J4NXaMBLp\n",
"iAJqS7dUqCiwUIvUBxRajdwMLhsWCsRVqsKSR2OlslEQApUrprkQhCkQwmqlwcpDHIZKwkMalESS\n",
"nP6x9yQnJ2funbnzOOfM+X3WOuucsx9nf+fMnP2b/Ti/3Ssy1b0JUCsMfAuYg+u2vLxQq46MkSXa\n",
"EtQxPUVR+oVwS69en7dk9IKJE6OeWrZSKZc2Ap+ICa+ybXZnPexTrZSbFFlr6a0BvlOoVecDlwDL\n",
"aoWBsf5NRI1eWlp6iqIo7dLMenqjEkyY0FL6rJMpo1eoVS8v1Kqr/PEjwF3AJ8fIpkZPUZR+JdzS\n",
"G88rCwQ775wro5e57s0IVdxsou0Ie0GYfWzpteenz33bxk2s37CZ9US6N/1AczHm2iNx3hQ6mH5W\n",
"TFiSemLTA7NGRkYWpUVPg/TFkZGRYor0ZDl90X/fadGT9fRdvZ+XvWcylz2wcc8Nm/jdhs1bVz/f\n",
"3Ch97PUnTMyV0ZMgiL53mF5qhYGLCrXq5aHzpcCGQq16bj1s+fLlwYIFC7a+PDk4NDwH+OfQZc6u\n",
"lEuJr5nXqJJOG1nQmQWNkA2dWdAIqjPM4NDw/biJep8HrgJWVcqlsXrAtvKLk05ePG31Yyc8sfTG\n",
"eeG6s1/JVPcm8PlaYWB3gFphYD/gFOC7Y+RJ5USWLDywkA2dWdAI2dCZBY2gOiPUuzjrq5+31L05\n",
"bfVj0XX2+pqsdW9+HbitVhjYBEwFzivUqivHyBM1erq0kKIo/UR9Mku9e7Mlowds7KCW1JMpo1eo\n",
"Va8ArmgxWypbeoqiKB2i3tKrv4vc6hhdroxe1ro3x8PrbO8wVWdvKorST9RbeuPq3gT+0EEtqafv\n",
"jV6lXArY1toLyNm/GkVR+p56S2+H2ZtNkqs6se+Nnqdu9H7vjWDiqO/AzpEFjZANnVnQCKozQlst\n",
"vZdPPumwzspJN3kzemkazysmLaBJikkLaIJi0gKapJi0gCYoJi2gSYpJC2iSYg/KaMvobdh//8M7\n",
"Kyfd5M3o6Xieoij9RvSVhdbckE2alKuX09XoKYqiZJu23tMLJqvR60fU6CmK0q/UjV59mZ+WjNgW\n",
"ben1JfXV09M0pqcoitIJ6mN6+/n9b1rJHEye0nBpoX4kL0YvjS29kaQFNMlI0gKaYCRpAU0ykrSA\n",
"JhhJWkCTjCQtoElGelBG3Wgd4PdPtZJZNm64t7Ny0o0avYRQ34GdIwsaIRs6s6ARVGeE8Jp60KLR\n",
"2+eSRf/eWTnpJi9G7//8/rVRUymKomSPN0LHm4Bft5hfX07vQ+7GrcZwS9JCFEVROkx4TO7XlXLp\n",
"jYYpYyjUqptp3YtLZsmUw+nxUimXXgG+kbQORVGULhA2ek+P8xq5ae1lyujVCgNTgG8Dh+C0f7FQ\n",
"q96TrCpFUZRECRu9lsbzQsQ6nR4cGt6hzq2US7F17uDQ8Om4hWwD4N5KuXThOLV0lax1by4CgkKt\n",
"+k7go8D3a4WBPZKVND7Ud2DnyIJGyIbOLGgE1Rkh3J3ZstHzGhu19BYBQaVc2lrnDg4N71DnDg4N\n",
"H4Zb7/RPgWOBuYNDw3/dqpZekBmjVysM7AScDVwHUKhVfwmsBs5IUlcbFJMW0CTFpAU0QTFpAU1S\n",
"TFpAExSTFtAkxaQFNEmxB2W029IrEtPSGxwa3q7OrZRLo9W5ZwN3Vcqll71T/+uBT41DS9fJjNED\n",
"9gd2A54Ihf03cHQyctpj2bJls5LW0AxZ0JkFjZANnVnQCKozwhuh/XOtZvYa41p6rdS5R0fSrQHm\n",
"DA4NT45JmyhZMnoz/f7VUNirQCa7N1etWjUraQ3NkAWdWdAI2dCZBY2gOiPUjd4zlXKp5VmYXmOc\n",
"0Wulzp0ZSfdbQIAZrerpNlkyenWi6+FJIioURVHSQb17c7wzN2H01dObrXPj1ipNXf2cJaO31u+n\n",
"h8KmAy8koEVRFCUt/BzndWqkjWvEGb1W6ty1MekCYF0bmrqCBEEqFhIfEz+RZS3woUKter8PWw7c\n",
"WahVr6qnW758eTY+kKIoSspYsGDB1paZn8iyFvhQpVy634ctB+6slEtXhfMNDg1fBexSKZfO8ed/\n",
"BVxYKZeO6Jn4JsmM0QOoFQYWAzMLtepZtcLAQcCDwKGFWnXtGFkVRVGUFhkcGl4MzKyUS2cNDg1v\n",
"rXNxa/bdBpxSKZdeGRwangPcAxyGG8+7B/hBpVz6VkLSG5Kl7k1w74xIrTDwIHATcJoaPEVRlK6x\n",
"CJDBoeGtdW6lXFqLW7vvYL+nUi49DlwI/ASoAI+m0eBBxlp6iqIoitIOWWvpNUREpojIjSLyoIg8\n",
"LCInJKBhooh8TkRWiMiIiDwgIseH4o/0+u4XkTtEZNdI/gtE5BG/9cSFj4gcJCJviMj8NOoUkXP8\n",
"fbxPRB6r60yZxtn+O18pIqtFZGEoLjGdIjJPRJ4UkTMj4ePWJCK7+jwr/TWO6oZOEZklIteLyHJf\n",
"zk0iMiOSL3GdkfiyiDwTE94znaN8528RketE5Gf+N/pjEZmQhMbECYKgLzbgMuAGf3wQ8BKwR481\n",
"zMJNG97Fn78XWA/sBUwCngWO83GLABvK+z7cC52TgMn++P090PyvuCWX5vvz1OgE/gJYBkzw5x/z\n",
"YanR6Mt7CPiKP97V//Y+kKRO4BTg+8Aq4GOh8LY0+WsO+eMibhmbSV3QuQj4euj8u8CtadMZip8J\n",
"1ICnI+E90zmaRuAu4DR/LMAPgIlJ3Mukt8QFdORDuBbrOuDdobCfAl/osY5dgdMjYeuAU4EPAk+F\n",
"wvfGvV+zmz+/DVgYil8I3NFlvccA1wLPsM3opUYn8F/AO2PCU6PRX/+1SCVRAS5KUicw4Pcr2N6Y\n",
"jFuT/31vAvYOxT8J/FkXdJ4M7Bc6PwlYHzpPhc5Q/NXAl4BnIuE90znKvZwH/M8o+Xp6L5Pe+qV7\n",
"MxUuyoIgeDkIgu/Vz0VEcP+e1uIMzBOhtM/hVnKf64Pi3Ph0W/8ioBwJO8aXDSSnU0T2AOYAB/su\n",
"rvtEpO7LLxUaQ9yF+5eNiByA62n4aZI6gyCoNohqR9NcYKPPU6et56yRziAIlgVBEO4qnML273yl\n",
"QieAiByI6825Pya6ZzpH0bgAeEpEvua7N5eJyJwkNKaBfjF6aXVRdhzwK2AlO7rpATe1t64xzo1P\n",
"1/SLyInAz4MgqEWiZrJtpfk4Lb3SOcvvPwKUgD8HLhaRj/jy0qCxzjk44/wkrpX3mSAIHiU99zJM\n",
"O5rG+g13k5OAfwqdp0lnGbiEeO8jadA5C2f4Hg+C4I9xXcUrRGRqijT2jH4xenVS46JMRKYAXwU+\n",
"Hrg+gfq2Q9LQcU+m0orITrjut8Ux0WnRWXdUe00QBJuCIFgH/Atw1igaen4vPbcCDwRBcCBwFHCZ\n",
"uAlMabmXYdrV1PNnTETeBewHLBlDy2hxXdEpIkcDfwiC4PEWtIwW1w2dk3FdwzcCBEFgcb42Tx5F\n",
"B6PEpc61WCv0i9FLlYsy3615LXBlEASrfXDUTQ/+fG2D+Ol0z4XPR4HhIAh+Gwqr/5DTovMVvw9/\n",
"h/+LG4NKi0ZEZDbuX/QS2NpdeDvw6TTpDNGOprXAW0bJ23FEZF/cn7NTgyAIO1NOi84ybpigEWnQ\n",
"+QrwYiTsedyzlBaNPaNfjN5TwMvA20NhhwIPJyOHK4BKEAQ/FJFJIjLgtRxST+DD3oybaYWPj+p/\n",
"qEv63g2c5KfZrwD2BJaIyG0p0vlL3FhTuBtlBm6GXFo0ghuzhe0X8tyE+3edJp112tH0KDBFRPYO\n",
"xc+mS8+Zf0VhKXBmEATrRGRAROr3O3GdIrILcABwo3+OrgL29M/VJ9OiE7cGXnS1gxk4w5cWjb0j\n",
"6Zk0ndpw/wav98cH4f7Z9PSVBV/2xcA3gKnANNwPaAhXOf6KbbMkh4CbQ/lKwOO4ynIKbrD4xB5p\n",
"Ds/enJwWncA3gR/64zfhZnOenqZ76cuoAp/257vgDPb5adCJc0J8Zui8LU24V1wu8cfH4V5/aHv6\n",
"eozOacC9voxpfvsHYN806YzEFdlx9mbPdTa4ly8AH/Tn83EttT9K8l4mtSUuoGMfxH1hN+B8wz0M\n",
"vDcBDQfjfNJFt/oP5kjgAdzEltuBt0byfwF4xG8X9EDvEbj3ddb7B+WzadKJa4Esxf1TfQC4MBSX\n",
"Co2+rHm4Cvo+3D/jxcBOSerE+UC8xVduDwHXduLeAW/1eVYCPwOO7LDOa3z4V2Oeo83APmnSGYr/\n",
"nH+G1vt0R/Ra5xjf+THAf+BmmK4A5iX1nSe9qRsyRVEUJTf0y5ieoiiKooyJGj1FURQlN6jRUxRF\n",
"UXKDGj1FURQlN6jRUxRFUXKDGj1FURQlN6jRUxRFUXKDGj1FURQlN6jRUxRFUXLDxKQFKEoWMcYc\n",
"D+xurb25x+WeiPNBeShwk7X2E2Ok/zJwo7W24SKoipIntKWnKC1ijDkft6Dpyl6Xba39sbX2KJyH\n",
"/GZ8CK4GHjTGzOuuMkXJBmr0FKUFjDHvAa4ETrXWPj9W+qSx1t6JW+vvDmNMdF00RckdavQUpTW+\n",
"BtxurR1tpey0cTVuFZK/TVqIoiSNjukpSpMYY+YAc4FrIuHbjbMBFdy6fwfgFjg+H7fI8ZW45aem\n",
"Af9orV3i858NfAa31NNlOAM1n22rxH/ZWntHjCQxxpwLnAHsg+vyPM9auzqcyFq7wRgzAnwcWNjO\n",
"PVCUrKMtPSV3GGM+a4z5e2PMLcaY2caYLxljFhpjrhgj65/4/S/CgZFxthOA162183FGbzLwI5xR\n",
"PMdaewRwEXClMeZdPv91Pj84A/mQtfYYa+1ewN3ArcaY90e0iC9riy9rf9xCoTcbY+Ke6zXA24wx\n",
"B43xGRWlr1Gjp+QKY8zfAHdbaxcDP8Et/HkDzjidMUb2A/1+bYN4AV6x1i4F18LCLeq5L/Aja+2r\n",
"Pt2twO9xK1ZHeSgyI3QhrpV4eUzaV6213/FlbcGtcH2gLy/KC5HPoCi5RI2ekjcmWmvrLbW9gdV+\n",
"Qso1bGvJNaI+EWTDKGnWRM5fioZbawMfvldM/sfCJ9baN3AzMA81xuweSRsdV3zR7+Ou+7rf62QW\n",
"JdfomJ6SK6y1V4ZO5wPDPvy5JrJv8XtpEB8Ar8WEERO+hfjn79WYsLrhnAGsG6Wsur646+4USaMo\n",
"uURbekouMcZMBt4B3NtCtpf9/s2dV7SV6TFhu+GM3Isxcc0y1e9fGjWVovQ5avSU3GCM2dl7UgEY\n",
"xLXYHvZxuxtjxhrTq3dRxnUfdoojwyfeOM8DHrfWrovP0hR1zdHuV0XJFWr0lDxxLvBvxpg3AacA\n",
"L1prN/m484A7x8i/AtfiOqxBvNC46zMa3ijt240xpwEYYwS4FDcOd3EbZQEcDqzJwgv1itJNJAia\n",
"8WSkKNnHGHMY8EXcu3N342ZPTsVN8rDW2v9s4hp3AVOttcVQ2LG4iTCzgd8BzwJHA9cBx+MmzDyB\n",
"c132SEzaedbawBizBWfkAN6Hm4X5Au49vWWNyrLWzjXGXAr8Je7VhaeB71lrF/k8u/ly/s5ae3VL\n",
"N01R+gw1eorSAsaYQ4BVwIettfd0+NpbgEXW2kvHTNzadZfgZqYea63d3MlrK0rW0O5NRWkB/7rD\n",
"h4Frs+DE2RhzAXAc8AE1eIqiRk9RWsa38OazbUZkJ2k0TjdengXeYa39TYevqyiZRLs3FSVhQr43\n",
"D8d5e6laa49NVpWi9Cdq9BRFUZTcoN2biqIoSm5Qo6coiqLkBjV6iqIoSm5Qo6coiqLkBjV6iqIo\n",
"Sm5Qo6coiqLkBjV6iqIoSm5Qo6coiqLkhv8HmlwESyFoQpQAAAAASUVORK5CYII=\n"
],
"text/plain": [
"<matplotlib.figure.Figure at 0x7f0ba478d110>"
]
},
"metadata": {},
"output_type": "display_data"
}
],
"source": [
"# plot v(x) and u(x)\n",
"fig,ax = plt.subplots()\n",
"c1 = \"#e41a1c\"\n",
"c2 = \"#377eb8\"\n",
"ax.plot( xs, tracev*mphfact, lw=2, alpha=0.8, c=c1 );\n",
"ax.set_xlabel(r\"$x$ (mph)\");\n",
"for tl in ax.get_yticklabels():\n",
" tl.set_color(c1)\n",
"ax.set_ylabel(r\"$v(x)$ (m/s)\", color=c1);\n",
"ax2 = ax.twinx()\n",
"ax2.plot( xs, traceu, lw=2, alpha=0.8, c=c2 );\n",
"for tl in ax2.get_yticklabels():\n",
" tl.set_color(c2)\n",
"ax2.set_ylabel(r\"$u(x)$ (m/s$^2$)\", color=c2);\n",
"ax.set_xlim((0,1609));\n",
"ax2.grid('off');\n",
"align_yaxis(ax,0,ax2,0);"
]
},
{
"cell_type": "code",
"execution_count": 209,
"metadata": {
"collapsed": false
},
"outputs": [
{
"data": {
"image/png": [
"iVBORw0KGgoAAAANSUhEUgAAAcUAAAEdCAYAAAB9rHzGAAAABHNCSVQICAgIfAhkiAAAAAlwSFlz\n",
"AAALEgAACxIB0t1+/AAAIABJREFUeJzsnXmYHEX5+D+bkysQzghFQ5T7+CqSIPy4MiGigIRCUM5W\n",
"EZBGEEQRxSMQDm+IFwKtIocd5RCwQYhIAgMBFTGAGgmnBJtCCCbhCEdCyP7+eGuyncnu7MzszHTP\n",
"bH2eZ59Nd1V3v1OZrbffqvfo6u7uxuFwOBwOBwzJWgCHw+FwOPKCU4oOh8PhcFicUnQ4HA6Hw+KU\n",
"osPhcDgcFqcUHQ6Hw+GwOKXocDgcDodlWNYC9IZR3jbAo8AkZZJ77bkXgHmpbvcpk0zJQj6Hw+EY\n",
"DIQ6GgdcB1wQxP7VqfMnAJ8DXgVGAF8KYv/+VPuZwNH28Nog9i9KtW0AXAWsj+igU4LYf7jJH6Vq\n",
"8mopng8sLTs3Q5lkYurHKUSHw+FoEqGODgHOAl4BulPndwYuB44MYn8C8DPg5lT7AcCJwJ7AXsAJ\n",
"oY4OSt36UmBOEPv7AF8F4lBHI5r8caomd0rRKG834DXgpaxlcTgcjkHMw0HsH4VYg2l2AF4OYv8J\n",
"e/wXYKNQR5vY4wCYHsT+siD2lwLTgZNhpZX4MeAKgCD2i8Ay4OBmfpBayJ1SBKYC5/VyfkejvNuN\n",
"8mYb5YVGeRu2WC6Hw+EYNASxn/TRdB8wNNTR/7PHkxHLb4E9Hg88luo/z54D2BVYGsT+c6n2R1Pt\n",
"mZMrpWiUdyAwV5nE9NI8FzhCmWQfxJyf0VLhHA6Hw0EQ+/9FrL0bQx09BhwLHJbqMgaZo0u8DGzS\n",
"R1t5e+bkRika5Q0Bvgx8u7d2ZZITlEmW2MOpwK52qdXhcDgcLSLU0fbAb4APBbG/PfAT4I5QR8NT\n",
"3Sol1S5v62qwiAMiT96nxwB3KJO8nDrX62Apk7xhlLcI2BJ4MN02a9Ysl+Hc4XA46mDSpEnVKKhP\n",
"Aw8GsT8XIIj9X4Q6uhjYH7gdWACMTvUfTY+PyAJgvbL7jQb+ORC5G0melOLewM5GeR+2x+8CfmiU\n",
"9yzwI+BVZZI5AEZ5IxB33ud7u1GV/7GDkq6urqnd3d1Ts5Yjr7jx6R83RpVp1/GpwaAYDiwvO7cc\n",
"GGn//SCwfaptR+Cv9t8PAWuEOto8ta+4A/DL2iVuDrlZPlUmOVmZZO9SyAXwAvB5ZZJDgS2Az6a6\n",
"nwY8BTyQgahtzbbbbjs2axnyjBuf/nFjVJkOHJ8uVl21mwlMCHX0LgAbbjEC+LNtvxw4OtTRyFBH\n",
"ayDxipcDBLG/ELgBON5eOwFRsre14HNURW6UYgmjvPcZ5f0W2Ag43yjv88BdQJf1PL0PmARMViZ5\n",
"J0tZ25FNN910bNYy5Bk3Pv3jxqgynTI+oY52DnV0A2LpnRrq6GcAQezfjkQI3B7q6B7gXODwIPZf\n",
"sO13ICEX9yOeqlcEsZ92jDwFGBfqaDbwLUAHsb+sVZ+rP/K0fAqAMsnfEc+mck5otSydyLhx4+Zn\n",
"LUOecePTP26MKtMp42P3DD/eR9sPgR9WuHYaMK2PtsWAboSMzSB3lqKjuUyePHl+1jLkGTc+/ePG\n",
"qDJufNobpxQdDofD4bA4pehwOBwOh8UpxcFHMWsBck4xawHagGLWAuScYtYCOOond442gxGjvDHA\n",
"7q141jaAgUNa8ax2xI1P/wzCMZqnTPJktZ0LhUKxibI4moxTihljlKeACBiVtSwOh6NXuo3yTlUm\n",
"+Wv/XR3tjlOKGWKUNxL4LqIQHwWezlYih8NRxsbAHsC3jPKOVSZ5MWuBHM3FKcVs+RKSDuk54FRl\n",
"ktcylsfhcKQwyhuKpJksKcZAmaQ8xZmjg3CONhlhlHcw8FGkwOZXnEJ0OPKHzZo1BXgReB9werYS\n",
"OZqNU4oZYJS3DXC2PfyuMsnjrXp2sVgstOpZ7Ygbn/4ZbGOkTLIY+CqS9PoYo7xJlfoPtvHpNJxS\n",
"bDFGeesg+4hrALcok8QtFqHQ4ue1G4WsBWgDClkL0GqUSf5BT1qzc4zytqzQvdB8iRzNwinFFmKU\n",
"1wWcg1T9eBL4XrYSORyOGrgOuBNYG/iuUd6aGcvjaAJOKbaWo4H9gCXIPuJbGcvjcDiqRJmkG7gQ\n",
"mA9sDZxtX3QdHYRTii3CKC+9SX++Msl/spTH4XDUjjLJ68BXgLeAjyDOco4OIpchGdYR5VFgkjLJ\n",
"vfbcLsBlwDvAIuA4ZZJF2UlZPUZ5GwDfQcY7Uia5K2ORHA5HnSiTPG2U9y3gfOAso7x5yiTzspar\n",
"GYQ6GocsG18QxP7VqfPrIaWhtgfWQorCHxzE/ju2/UxkZQzg2iD2L0pduwFwFbA+MieeEsT+w83/\n",
"NNWRV0vxfGBp6cAobwQQA2crk+wNPISt5NwmnIoEAT8CXJKxLMWMn593ilkL0AYUsxYga5RJbgdu\n",
"RKrGf62sudhygZpAqKNDgLOAV4DusuZfA3cGsb8XsCvwOtBlrzsAOBHYE9gLOCHU0UGpay8F5gSx\n",
"vw/i1RuHOhrRzM9SC7lTikZ5uwGvAS+lTh8ILFcmucce/wI4zChvw1bLVyvW2/TD9vDCrAN/XV7G\n",
"yrjx6R83RiuZhiiMHYzydiyd7KDxeTiI/aOAV9MnrfW4TRD71wIEsd8dxP7HgtgvzW0BMD2I/WVB\n",
"7C8FpgMn22s3QIrIX2GvLSKx2ge34PNURe6UIjAVOK/s3G7AyuUJZZLngDeQN5S8cwASfjFHmWR+\n",
"xrI4HI4GoUyyFPi9PTw8S1kGSqijNcrPBbGf9NF9EvB0qKPvhzq6P9TRraGOdkq1jwceSx3Ps+dA\n",
"5uylQew/l2p/NNWeObnaUzTKOxCYq0xijPLSTWMoe1sBXgY2aZVs9WA90w6zhzdlKYvD4WgKNwHH\n",
"Ah82yvuBMsmSrAVKY5clN0S2bzayv3v7WaeG245FFON1QeyfFeroCODuUEfvDmL/dWS+fiXVPz1X\n",
"l7eVt2dObpSiUd4Q4Mv07s3Vzepr2mDXsHPMjsC2yH96MVtRHA5Ho1EmedYobw4wDtnmuSErWazz\n",
"yy6INbYLoIDRVV7+dg2PGgm8FsT+VQBB7F8f6uhiYDJwre3T23xNH225msdzoxSBY4A7lEleTp0r\n",
"DdYC4P1l/Ufb86uRXtMfN27c/MmTJ88Hir2t9duUTIVebtOI/oe9s+aao5Z84AP/W3DSiV99sljs\n",
"r3+z5XH9XX/Xv/H9b0SU4uHFWbNeYujQlsjT/U5312tPvbnp6/Pf2vKNZOkwYF3KFMyQkV1rDVtn\n",
"6LChI4e8MWTkkDeGrjHk9aFrDnnjnTdXPHDDLde/PHfB37d/6fUFr7/y1uKld55+5x69yNEbi4H/\n",
"lZ17Htjc/nsBqyrj0fT4iCwA1iu7djTwzyqf3XS6ursrKfTWYZR3ObAzPW8seyDr0vMR992LlUm2\n",
"tn094BlgjDLJwvR9Zs2a1T1p0qTM3zyM8kYBM5D9xMOVSZ7NWCRA/qg6yBGg4bjx6R83RqtilDcc\n",
"uB0JMTj+yem/2qBZ4xPqaChSkHwSMIFVlc8yYC7inf8QUopucRD7K6q5d19zZ6iju4Erg9i/xh4f\n",
"C/w4iP0NU32eBqYEsf/rUEc3I96lF9q2KcD4IPZ1qKMNkeTqY0v7iqGOngLOCmL/5lrGolnkxlJU\n",
"Jjk5fWyU9wzweWWSe23dwR8Z5e1r4xaPB24sV4g542BEIT6QF4VoKeCWcitRwI1PfxRwY7QSZZK3\n",
"jfJuAT5Fjw9BsZHPCHU0Flme/AiyN1jiP8BdwJ+Budbbs9F0saoFGgPTQh3pIPbjUEf7IjVhS05H\n",
"l9v279vrjgbOBAhif2GooxuQOfz8UEcTkLCW25ogd13kRimWsJlfpiD/8ecb5d2sTPIjo7xDgUuN\n",
"8lYG72coZkWsg83H7OFvs5TF4XC0hN8hSnH/oYsW39uIG1qrcF/gSFb1zvwPsgp1F/DvIPabstwX\n",
"6mhn4FzEN+LUUEd7BbF/UhD7S0IdHQxcEuroLGR178Ag9l8FCGL/jlBHVwD321tdEcT+jNStTwGu\n",
"CnU0G4mA0EHsL2vGZ6iH3ClFZZK/06NQ0ucfQYJB24HdgC2R9fOG/IE4HI78okySGOU9AOw+eubM\n",
"bTis/uxvNjziUMSrdVN7+k3gj8AtwD+apQjTBLE/F/h4H20PIsu4fV07DYnj7K1tMaAbIWMzyJ1S\n",
"7BBKSv0mW6TU4XB0PjcCu6/197/vaJTXZROIV02oo1GIEjqGnr3CBLgeuDWI/VyFe3QqTik2GKO8\n",
"McgG+HJkScXhcAwO7gUWDH31tV2Q1aK/VnORzfJyLPIyvbY9PRdxMLy3WkcZR2NwSrHxHAoMBe5U\n",
"Jil3W84DxawFyDnFrAVoA4pZC5BHlEmWG+XdOOTNN7dC9gErKkVrGfqIZViqzfhXRBk+2IolUsfq\n",
"OKXYQIzyhtGTfCCXDjbOlb4ybnz6x41RRX439M03TwT2NcrbTJnk+fIOoY7WBI4CPol4bQLMBn4R\n",
"xP6/WieqozecUmwsBcRr9t9InJDD4RhEKJMsNMq7EzgIWQ79canNepMeiiTM3sCe/hvw0yD2cxO8\n",
"PthxSrGxHGF//7bWTXaHw9ExXIcoxUON8n6mTPJWqKM9gTOA99g+c4FLg9ivat/R0TqcUmwQRnnb\n",
"0VNXLDeBqA6Ho7Uok/zLKO9fwE7Pr/9/x/1eKkj8P9tsEOvxLrdnmE+cUmwcR9nfsTLJ65lK4nA4\n",
"MmXx2t7tL4zecdIL6+/8Tbq7n6GrawlSB/b6PAWqO1Ynj/UU2w5b7PgAJPv7dRmLUxGbUNjRB258\n",
"+seNUd+EOho6/cwbz7t/+5NPeWH0jqOGrHh75Lpv/nc2cGgQ+5FTiPnHWYqN4XAkf989yiQma2H6\n",
"oYBzqa9EATc+/VHAjdFq2LRoZ7/1wrL96Bry/Mjlr89577M3b7TOWy8tLav+48gxTikOEKO8EfRk\n",
"sPl1lrI4HI7WY+sYnoqEY3UNGdm1hCWc+YEnr3xs2Iq3bwEmGuWNUSZ5MVtJHdXglOLA2R9xr34C\n",
"F4bhcAwaQh11IZUrTkfSsi0Hoi2O2ORdkw7Y7x7wMcqbBXwI8Tn4UXbSOqrF7SkOAFsN42h7eK0L\n",
"w3A4BgehjjYFLgHOQRTiHOCYIPYvGbrGkOWprtPt748a5a2NI/c4S3FgvB/YHqlEfUfGsjgcjiZj\n",
"rcOPIjGHawEvI9UgZvQWYmHDMx5G5gqN22LJPblTikZ5pyFfni5gY+BnyiSX2LYXgHmp7vcpk0xp\n",
"vZQrKYVh3KhM0ozins2gmLUAOaeYtQBtQDFrAbIg1NFmwDeAD9hTdwHfDWK/vNh5sew4QpTi0UZ5\n",
"17VT5ZxQR+MQj/oLgti/upf284BPBrH/7rLzZ5JaRQti/6JU2wZIftf1ER10ShD7DzfnE9ROHpdP\n",
"TwCOUiaZhFSx/oFRXqnA5gxlkompn8wUolGeQrzwlpPTPKe94fJWVsaNT/8MtjEKdTQk1NHHEOXw\n",
"AcQ6/CrwlV4UYm/jMxspDLwpsF9zpW0coY4OAc4CXkHCzcrbxwAnlreFOjrAnt8T2As4IdTRQaku\n",
"lwJzgtjfBxnHONTRiKZ8iDrIo1L0S9UllEmeQpYmx2YqUe/4yPj9IafVMBwOxwAJdaSQSfxspJLF\n",
"TOCIIPbvrDYjjTLJCnqWTY+1vgjtwMNB7B8FvNpH+xRkbMo/TwBMD2J/WRD7S5F91ZNhpZX4MeAK\n",
"gCD2i8Ay4OCGS18nuVs+VSaZW/q3Ud7hwGtIxWmAHY3ybkcyyz8KfE2ZZLU3tWZjlLc+cIg9/FWr\n",
"n+9wOJpLqKMhyOR9GqIMFyNLpTPrvOXvEcWwM/A+4JFGyNlMgthP+moLdbQ1YvnegFiFacbT42AE\n",
"suV1iv33rsDSIPafS7U/aq+5aaAyN4I8WooY5e1scwdeBBytTFJ6U5kLHKFMsg9i0s/ISMQjgJHA\n",
"bGWSpzOSweFwNIFQR5sDlwFfRhTiHxHrsF6FiDLJW8CN9tAfsJDZcx7iedub1TsGmZ9LvAxs0kdb\n",
"eXvm5FIpKpPMVSbZCfHyio3ydrPnT1AmWWK7TQV2LbW1CqO8NemphnFNK5/tcDiah907PBK4FhgH\n",
"LAK+HMT+14LYX9yAR1wPvA1MMMrbogH3y4RQR+OBZf3Ufqy0tFzelqvl5Nwtn6ZRJnnEKO82JDj2\n",
"E2VtbxjlLQK2BB5Mt6U3useNGzd/8uTJ84Fibw4CNo9joZfH99p/yW7jvzry6X/vsHzjjV587htf\n",
"108Wi7pS/1rv34L+ZyBxVXmRJ2/93fj03/84YH6O5Blw/1cff+NfSJD9++2pPwAXbfeFzXcpFotT\n",
"a7z/cfQ2PiYp2u0fDRwDfCfL8Zk4cSLptpkzqzaEz6NnObQ3FrDq39Bo4KVU23pl/UcDuakn2dXd\n",
"nZ94c6O8DYCCMslNqXM/BjYHfgK8qkwyx54fgZRpmqBM8qdS/1mzZnVPmjSpKW8eRnnDgN8B7wK+\n",
"pExSbMZzmkmxWJxaKBSmZi1HXnHj0z+dNEZ27/AoZJJfA1gIfNs6gNRFpfExyns3sg+3FDgkC5+I\n",
"vuhr7gx1dDdwZRD714Q6GoUYIf+1zaORWO2/IKEXYaijmxHv0gvt9VOA8UHs61BHGwIvAmNL+4qh\n",
"jp4Czgpi/+Zmf8ZqyJuluC4wxShvhjLJm0Z5GyNvVd8FtgD2oWdT9zTgKeCBFsr3QUQhPgvc28Ln\n",
"OhyOBhPqaEvEg3IXe+p24OIg9sv3vBqGMskzRnn3ABOAIxHvzbzTZX8IYv81RAkCEOqogCjMian+\n",
"lwPTQh193153NHCmvX5hqKMbgOOB80MdTUCKKeSmBm3elOJ/gVuBWUZ5y5C3kCuQTe/NgX2N8mYj\n",
"A/0qMLlVgbDWjfqT9vBX1s3a4XC0GaGOhtJjHY4E/gd8K4j9Vr3oXoUoxSOM8q7Oa/1VW/XjXGBH\n",
"4NRQR3sGsR+k2s8ADgU2soruwiD2/x7E/h2hjq4A7rddrwhiP+0UeQpwVaij2Yhfi85TSa1cKUWb\n",
"FeYc+1NOggT2Z8UewLbIH9DtGcrhcDjqxFqH5wLvtaduA6Y10zosR5nkn0Z5cxBnnsPJqcNeEPtz\n",
"gY9XaP8h8MM+2qYh6e96a1uMrADmklwpxZxTSun2G2WS3LzVOByO/rHW4TFIrOBIxPHjW0Hsz85I\n",
"pKsRpXiUUV7kVp7yg1OKVWCUtx5iKb4DxBmLM1CKWQuQc4pZC9AGFLMWoBZCHY1FrMP/s6duBX4Q\n",
"xH5fmVoGSrGKPn8Gngc2s3L9vUmyOGrEKcXqKABDgQfavYL2YMtbWStufPqnXcbIWofHItbhCCQc\n",
"4JtB7N9f8cIBUs34KJN0G+XdbeXbD6cUc0Mug/dzyAft77ozWjgcjtYR6ug9iJPe6YhCjIEjm60Q\n",
"a+Qu+3u/NsqH2vE4S7Ef7NLpbsjS6d0Zi+NwOCpgrcNPIEmphyMxcd8MYv9PFS/Mhn8ie5ubAjsg\n",
"OUAdGeMsxf6ZgLw8/K3dl04djk4m1NFWwJXA5xCF+DvEOsyjQixVzyi9aLdNSalOx1mK/VNaOp2V\n",
"qRQOh6NXQh0NQ2KIP4MowxeQmLm/ZCpYddyF5FLezyjvp8ok+UkxNkhxlmIF7NLpB+igpVObC9HR\n",
"B258+idPY2RLGF2FBIQPR8oPHZmlQqxxfB5GqkRsAWzVFIEcNeEsxcqUlk7/qkzSiCz5eaBAm7nU\n",
"t5gCbnz6o0DGY2Stw+OQtI/DkGxYFwSx/9cs5bIUqHJ8lEneMcorIplhPoikrnRkiLMUKzPJ/nZe\n",
"pw5HTgh1tC1iHZ6MKMQbgaNyohDr4U77e3/nhZo9zlLsA6O8dYHdgRU4y8HhyJxQR8OBTyPJpIch\n",
"we8XBLH/YMUL88/fgMVIGbxtgcezFWdw4yzFvtkX+cObo0yyKGthHI7BTKij7ZDUaCchf5fXI9Zh\n",
"uytEbFGDUszi/lnK4nCWYiVcwL7DkTHWOjwesRCHAQY4P4j9OZkK1nj+iCQH3995oWaLU4q9YJQ3\n",
"Csl1mo4j6hSKWQuQc4pZC9AGFFvxkFBH2yM5S7exp64DfhrE/huteP4AKNZxzSNIBR6FlGr6VyMF\n",
"clRP7pSiUd5pSFmRLmBj4GfKJJfYtl2Q2orvAIuA45q0tFlaOv1bpy2dtkveyqxw49M/zR6jUEcj\n",
"kDJxxyE5h59D9g7bwjqsZ3ysF+ospPDw/uREKYY6Goe8jFwQxP7V9txYpLzflsBawDPA6UHs/y91\n",
"3ZlIcWGAa4PYvyjVtgHiKLU+Ms+eEsT+w03/MFWSxz3FE4CjlEkmAYcBPzDKG2+UV8pfeLYyyd7A\n",
"Q0iF52ZQqiLtlk4djhYS6mhHpL7gCcj89Bvg6HZRiAPkj/b3B/PghRrq6BDgLOAVIL2cexywKIj9\n",
"SUHs/z/ESPlZ6roDkFCZPYG9gBNCHR2Uuv5SYE4Q+/sAXwVi+yKUC/KoFH1lkv8BKJM8hXhljQUO\n",
"BJYrk9xj+/0COMwob8NGPtx+Gd9nD9shI4bD0faEOhoR6ugUJE3b1khR8ZOC2L84iP03s5WuZfwT\n",
"me/ehVhhWfNwEPtHAeUltuYAP00dX8eqDkIBMD2I/WVB7C8FpiPhMyUr8WNIsnaC2C8Cy4CDm/EB\n",
"6iF3SlGZZG7p30Z5hwOvIW9QuwHzUv2eA94Adm2wCJshZv1iZFPf4XA0kVBHOwER4lAzBJlEj87T\n",
"klorsLlQS7GWu2cpC0AQ+0kf528NYv+Z1Kk1kMTmJcYDj6WO59lzIPP10iD2n0u1P5pqz5zc7SkC\n",
"GOXtjLx9rAUcoUzyqlHeGFZ/Y3kZ2KTBj9/Z/p7rPMAcjuYR6mgkkq/0k4gy/A9wXhD7g7m24APA\n",
"hxGleF3GslTLwaxqOY5BllxLpOfp8rby9szJpVK01uJO1rFmhlHeIciadm9KqtFr76Xq3P9s8H1z\n",
"QbFYLDhnkr5x49M/jRijUEc7I56l70b+riPgMrvc1tYMcHwesL/HGeUNUyZZ3iCxmkKooz2R/8MT\n",
"ypoqGRTlbXXN4aGOtkGW2tdDIgUWAvOC2H++nvuVyKVSLKFM8ohR3m1IodBngfeXdRmNVNNehfQX\n",
"cty4cfMnT548Hyj29kW1yXsLpePNt93m0GH/W7jJa/vsPUL1IlN5/3RTNffPuj9wXB8Ji9tCfjc+\n",
"ueg/tVgslrdVdf93lq4Y+tLsV8YPGdk1dsXS7teB+Ujc4T9aKH+z+9c9Pkz/Fd7Xv7HW8BcXjBr6\n",
"5ps7I6EaTZF/4sSJpNtmzqzNrzDU0ZbAt4GPBbH/TqppATI3lxhNz/LqAkSJUdZelRES6mg9xPnn\n",
"OGSrq7c+85AohcvK5KqKru7u/KwQGuVtABSUSW5KnfsxsDmSzeJiZZKt7XkPcQUeo0yysNR/1qxZ\n",
"3ZMmTarrzcMobyRwD+IGXlAmeb3uD5NTisXi1EKhMDVrOfKKG5/+qXeMQh29F3HlH4u82UdA2AnW\n",
"YZqBfoeM8s5CQjN+rkwSNkywfuhr7gx1dDdwZRD716TObYTknP1UEPvzQx15wItB7C8LdXQz4l16\n",
"oe07BRgfxL4OdbQhUvh5bGlfMdTRU8BZQezfXEk+Gx5yCzASiQWdZ+/1JrL8vjZSsPm9wD5IuryP\n",
"BLH/Qi3jkDdLcV1gilHeDGWSN43yNkZiFr8LzAB+ZJS3rzLJvcim/I1phdgAtkXG5OlOVIgORxbY\n",
"vcPPAsciS2XzgalB7M+tdN0g5gFEKe4OtEwpVqCL1BJnqKN1EIV4DvA/e/xZRNZnkVC5aaGOvm+v\n",
"Oxo4EyCI/YWhjm5A5u/zQx1NQEp+3VZJAKt0Y+AHwCVB7L/VT/8NgSnAjFBHuwexv6zaD5s3pfhf\n",
"4FZgllHeMsSsvkKZ5FIAo7xDgUuN8lYG7zf4+aX9RPfH6nA0gFBH76Mn0HsFEoP4s06zDhvMHGA5\n",
"4lexjjLJkiyESO377gicGupozyD2AyS2cB9WzfbVjY0bD2L/jlBHVwD327Yrgtifkep7CnBVqKPZ\n",
"iIWnq1BaxwEnB7H/+2pkD2J/IXCGTSJwCPDbaq6DnClFZZKlyB/QOX20P4IEhDaLkudpRzrZOByt\n",
"ItTRGoj1cAxiLfwb8SzNRaaWPKNM8rpR3lxgFyRUoZiFHNaS/3gv578OfL2fa6cB0/poW4ysANYi\n",
"ywW19E9dd3Gt1+QuTjFjOtrz1FLMWoCcU8xagDagWKkx1NH7kUw0xyLW4S8BfxApxGID7lHyQs08\n",
"XnGwkStLMUtsZpxNgZJHXEfiwg0q48anf/oao1BHayJLY0ch1uFTiGfpo62TLnsa9B36G5IZZpcG\n",
"3KvjsMvyhwJ/DmL/j6GONgZuQLKR3QEcX2/i+LqVolFerzEiyiQDihHJkNLS6aO2vpnD4aiSUEe7\n",
"ItsemyO5MK9C9pKqdnBwrMKjwNvA1kZ5o5RJXstaoJzxOWAHeupQXoTsc96OZD87F/hKPTeuSSka\n",
"5fUbI2KUtzJGpM2USynfaScvnTocDcVah59DvCVBrMOpQew/1vdVjv5QJllqlPcoMi/9H/CnjEXK\n",
"G7sBE4LYfyXU0dpIPtVLgtj/fKijMYiybK5SNMqrJUbkO8CnjfI+okxSU4xIhpQSAzxSsZfD4QBW\n",
"xo1NQazD5Ugy718Gsf92poJ1Do8gSnEXnFIsZ0UQ+6V0cR8E1sSGrwSx/2Koo7pXKKpSijZQfmWM\n",
"iDJJxRgRuz83BUnRtrsySa6XUIzy1kBM8RXAYM676HD0S6ijtYDT6PFMfALxLH08O6k6ktJc5PYV\n",
"V2d4qKNhQewvBz4BzC3bu647/We13qfHAScrk1zUn0IEUCZZqExyBpKx4pB6hWshOyMvCE90etB+\n",
"HynMHBY3PpUJdbTb0LWH3IkoxOVIHb1POYXYQwO/QyWluJNR3vAG3bNTeAC4LdTRz4GPIh7OAIQ6\n",
"OpbVi0dUTVWWojJJTTEiRnlDlElWKJPUHCOSEYNp6bSACzuoRAE3Pqth921OAz7W1dW1GRK4fV4Q\n",
"+09kK1kuKdCA75AyyStGec8gCbe3x/k7pPkqYnQdBfwOW6Uj1NG1wBHYDDr10KyQjL/R+DqHzaS0\n",
"PDGo6rd5qJcnAAAgAElEQVQ5HNUQ6ugDyHbIpsDyUdus+bfFDy/5lF26cjSXRxCluAuDXCmGOvo1\n",
"kl7u9iD2X0JKbK2CLYp81ECeM5CQjD2tUJuW3aeLfFSNrgqjvKGIcxAMDkvR4agKax1+HjjMnnoM\n",
"OG+Twuhjj5h6qFOIreERZHlwF+BXGcuSNWtjxyDU0R3AzcAtQey/3MiH1JXRxijvDOA+xET9MDCx\n",
"7GftRgnYArZHPJf+0+Dk4g5H2xLqaA+kyO1hyN7hpcBxQew/malgg4/Si/r7jPIaXTu2rQhiXwMb\n",
"A59Coh5+DCwIdfTHUEefDXX0rkY8p15LseR5dlNv1emN8tppGdItnTocFlvx4AwkWwhIEPl5Qew/\n",
"nZ1Ug5rngf8BGyErcPMzlSZjgth/Hclcc0OooxFIOMZHganAJaGO/gLcBNwcxP6/63lGvblPlyiT\n",
"3NibQrQEdd43CwaTkw04J5L+KGYtQFbYKurXIQrxbeAS4NO9KMRii0VrN4qNupGdY0tzkwvNSBHE\n",
"/rIg9m8PYv8zyDbefog/y+nAU6GOHgl11GtxiUrUayk+YZS3njLJK320jwf+WutNjfKGIdkxNLI3\n",
"OQL4hjLJXbb9BSRpQIn7lEmm1Pqc1PO6GGSWosvtWZnBOD6hjkYh1mGpcsG/EOuw1zftwThGtdCE\n",
"8fkHYhHtjHhatgyboOE64IIg9q9Ond8FyVy2soxfEPuLUu1nInUUAa4NYv+iVNsGSBrA9REddEoQ\n",
"+wOaf4PYX4EUiL8H+Hyoo/HI0v9RwPm13KteS/GLwCVGeYcb5e1olLdF6mdL4NQ677s5ouUPUSYp\n",
"ILkUY6O8TW37DGWSiamfuhWiZSxSs/F/gBngvRyOtiPU0V7IpKeBZcg+zfH1Lj05mkIpZd62rXxo\n",
"qKNDkLSeryD1EkvnRyDJXM4OYn9v4CFsLUXbfgBwIlLmby/ghFBHB6VufSkwJ4j9fZDQitjes5HM\n",
"CWL/a0Hs71jrhfUqxQ2AvZG13bnIOnfp5xnEeaUeXgWmlJLfKpPMBN6ieTUUV1qJFZaCHY6OI9TR\n",
"uqGOzgV+BGyC/B0fG8T+NUHst1PO4sFAKRZ0a+st3yoetiEO5YHwBwLLg9i/xx7/AjjMVrsH2T6b\n",
"bpc3lwLTgZNhpZX4MeAKgCD2i8jL2MENln1OvRfWu3z6E2ABEjC5kNRbhOW8em6qTLIIGUBg5fLm\n",
"COAle2pHo7zbgVGIA8DXBugxWgrF+McA7uFwtBWhjvYBvoZ48i1DlsF+7ZRhPlEmWWKUZwCFrG61\n",
"xOkpiP2kj6bdSG1jBbH/XKijN5DY9DuR7bPpqf7zkJJi2D5Lg9h/LtX+qL3mplrks3vgDQ8LrFcp\n",
"bgVs1VfKN6O8CfUKVMYEYL4yyb32eC7wefsl+R4wA/jAAO5fKirslKKj4wl1tC4SRvURe+ofSL3D\n",
"+ZkJ5aiWJxCluC0tUooVGMPq1uPLyIpDqf2VKtvK26si1NEZwDTgDcQwW5Fq7mIAYYH1Lp8+3U8O\n",
"1BPqvO9KbJLubyJ5VwFQJjlBmWSJPZwK7GqUt1ud918PeetaRs/yRMfjcntWplPHJ9TRvsD1iEJc\n",
"iiT3/0w9CrFTx6hRNGl8SnNUS/cV+6Cb1VcHYdUk3JW2o8rb6om/LIUFjgpif8sg9t+d+hnLqg6Z\n",
"NVGvpTjdKO+TyiTX9NE+oDRvdtn0Z8A0ZZJevZKUSd4wyluEmMkP1vGYnezvecokg6nUTQHnUl+J\n",
"Ah00PqGO1gO+hOwDgbj3XxDE/rMDuG2BDhqjJlCg8eNTSrieB6W4gJ5QthKj7flS++iytpdSbev1\n",
"cm2tKeyWBLF/Y4X2usMC61WKewD7G+WdhWjkJam2RqR5uxj4izLJjUZ5IxCTe2vgVWWSOQD2/PpI\n",
"cOsqpF2ix40bN3/y5MnzgWKZq/T/Aby5/XZvFovFqb3IUN5fTspbYKFd+wNjB9PnHczj8/I/lowd\n",
"scGwvVcs615rxfIVC1e81f1t4Lr03mG99+9ljDL/vHnq34TxeQJg+ahRexTvumsqQ1Zb5Kv5/hMn\n",
"TiTdNnPmzF669cqDpPKLhjrygLXocW55kFWdLXekJ0TvIWCNUEebp/YVdyBV5aJKngh1tF6qpmI5\n",
"dYUFAnR1d9fudGmUtxRRRiWzN32TLuBdyiRr1COQUd5XgM0QR4AuYAvETJ4P7KNMcqLtdybi9ruz\n",
"MsnKP/JZs2Z1T5o0qV9z3Cjvp8DuwJdLcZCDgWKxOLVQKEzNWo680gnjE+poNOJKX0qY/Aiyd/if\n",
"Rty/E8aomTRjfOzq2SxgXeAgZZIF/VxSM33NnaGOisCVpThFGz7xBPDJIPbvtV7MOwaxf6Rt/zCy\n",
"37crMoc/BJwZxP4M2/4bYF4Q++eHOpoAXANsE8R+1XV3rSL+FhK32ZthdnsQ+zv1dm1/1GspPqpM\n",
"Um4+r6TeNG9GedsC37aHp6WapgJ3Afsa5c1GPvSrwOS0QqzhOUPoWT6dW4+sDkceCXW0H3A2Ejb1\n",
"FpKV5nob3OxoU5RJuo3ynkAsoG3pWapsGqGOdgbORSy9U0Md7RXE/klB7C8LdXQocGmoo5XB+6Xr\n",
"gti/I9TRFcD99tQVJYVoOQW4KtTRbMSvRdeiEC2lsMBj+2ivO8SuXqX4tX7a61rPVSZ5gsrOPwN2\n",
"4LG8G1gHeKEZb1wOR6sJdbQ+Yh1+yJ6ag+wdPtf3VY4243F6lOJ9zX5YEPtzkVW63toeoUL8eBD7\n",
"0xBrsbe2xfRkT6qXpoQFQp1KUZlkRj/tda3ltpBSKMZgrE9WzFqAnFPMWoBaCXU0CbEO10eswx8D\n",
"v22idVhs0n07hWKT7lvyQN2uSfdvJ7YCtgpiv9coCLssWxdVKUWjvCOB2cokqzm19HPdOGDtVJxh\n",
"Xhi0StHlraxMO42PzQ7yZSQvJoh1eH4Q+01NWdhOY5QFTRyfPIVlZM3TfSlES92ritXGKb4EzDLK\n",
"q3rj0ihvf+C3SNq3vDFolaKj/Ql11BXqaH8k7vCDSG257wCfbbZCdGTKfKR6iWeU1041a5vB9FBH\n",
"n6zQ/rd6b1yVpahMcpdR3qXAI0Z5MXAHsr69AFmu6UJccjdFMrkfiqx9f1SZpK9UQZlglDcKeA/y\n",
"5Xq8n+4OR66w+SW/DEyypx5E9g5rWsVxtB/KJG8b5T2NhDtsDfw9Y5GyZA9g/1BHDQ8LrHpPUZnk\n",
"J0Z5c4HvA2GFrt3AH4BxyiR5VDo729+PKZPU6vHkcGRCqKMuYH9EIY5G0lv9ECmm6pLZDx6eQJTi\n",
"dgxupXgMEhY4Ckn1WR4WWLclXZOjjTLJ3cB4u4y6D/K2sh6Sd24RorHvzpt1WIbLd+poK6x1eDYw\n",
"0Z56ALgwiP3/ZieVIyPcvqLwaBD7fYYFhjqquz5jvd6n/0IKkbYjJUtxUMYnFovFgnOU6Js8jY+1\n",
"Dj+MhFqsB7yOWIe/y9I6zNMY5ZEmj4/zQBWaEhYI9ccptiU2K8QO9vDRLGXJkALOpb4SBXIwPqGO\n",
"NkIKsJZcy/+CWIcvZCfVSgrkYIxyTIHmh2VsbZQ3TJlkeZOek2vKkgH01l53WOCgUopIeZL1kWw4\n",
"zjHBkTusdXggksR7XcSB4AfALW7v0GHL5j0HbI4kIXkyY5FaQqijI4HZtTqUhToaB6wdxH7VYYH1\n",
"lo5qV0pLDo8rk7gJxpErQh1tjGQBOR9RiH8CjgxiP3YK0ZHiMft7+4q9OouXgFmhjqoOC7RhSzWH\n",
"BQ42S7H0JXqsYi+Ho4VY6/AjSAHgUYh1OA241SlDRy88hsSnbg/cmrEsLSGI/btCHV0KPBLqqKaw\n",
"wCD2a3L8HGxKsWQpOqXoyAWhjjYBvg7sZU/dD3wriP0Xs5PKkXNK89egcrYJYv8noY5qCgsMYr/m\n",
"sMDBphSdpegcJPqj2IqHWOtwMvBFJDn9a8BFSMmbvFuHxawFyDnFJt+/NNFvZ5Q3RJlk0FRACWL/\n",
"bmC8XUbtMyywVuswTV31FEsY5Q1HAom7lUn+V/eNGkhfNcGM8tYH7kRSYk0YTF8kR74IdTQGsQ5L\n",
"VQZmI9bhS31f5XD0YJR3G1J8/XBlkmcbcc9qa9F2OjVZirba/UeR9drdkVQ6XbbtHeBFJOfcbcC1\n",
"yiSv1Xj/YcDnkLIiXcAI4BulIsBGebsAlwEra3gpkyyq8vZpJxunEB0tx1qHhwBfQKzDVxHrcEYb\n",
"WIeOfPEYohS3BxqiFHsj1NEOwKWIrlgHuDGI/Qtt22rzcRD7i1LXngkcbQ+vDWL/ombJ2Uiq9j41\n",
"yjseqaB8IFLw93BEKa4DrGn/fQBwI7Ab8DejvCnWmqyWzYHTgUOUSQrAOUBslLepVcgxcLYyyd5W\n",
"lstruHdp6TSPqeccHU6oo3chNeCmIH8z9wBHBLHfDsuljvxRmsea7YF6NXBfEPv7IPl2vxDq6COh\n",
"jlbOx0HsrzYfhzo6ADgRWQ3ZCzgh1NFBTZa1IfRrKVpl9HMkLdr/q2D9PW9/5gK/MsobCXwSuMEo\n",
"76Qqi/m+CkwpPUOZZKZR3lvIoL4NLFcmucf2/QUw3yhvQ2WShVXc2+0nOlqOtQ4PBc5A8jG+gjgK\n",
"3OGUoWMAtMrZZkdgKkAQ+4tCHT0J7ITojuVB7K8yH4c62jCI/YVIRpnpQewvAwh1NB04Gbi9yfIO\n",
"mGosxbOAbyqTXFzLcqgyyVJlkp8jg/PVKq9ZpEwyvXRsM9CMQNxudyOl0JRJnkOSIu9apUgrl0+r\n",
"7O9wDIhQR5sClyD7h2sjDhhHBLH/B6cQHQNkpaVo58lmcRuy5E+oo62AbYCZyHw8r9QpiP3y+Xg8\n",
"qxog8+y53NOvUlQm+aYyyRP99atw/YvKJF+o8/IJSA2x2cj6+Stl7S8jWWoqYpS3DuAh1ua/65Sl\n",
"IygWi4WsZcgzjRgfW+/wMOA6ZO/9ZSRX41n2Lbqtcd+hyrRofBYAi5EkD5s28TknAtuGOnoKSTX4\n",
"uSD2H0Lm41fL+qbn4/L5uqq5Og/UndHGKO+zvZw7zChv6MBEWnmvNYBvIs403UjsSW9v19W8JZWs\n",
"xCcHa67AFIWsBcg5hYFcHOpIIY4JX0OCie9CstL8sYOsw0LWAuScQrMfYOfEVmS2uQn4UxD7WwPv\n",
"B74T6mg/qpuP2/L7XrX3qVHe9oj5e68yyX+QDdTLyro9BnzHKO9SZZKaUuuUPasL+BkwTZmkVAJk\n",
"AfKfkma0Pb8K6Qz148aNm3+cPmTUen+8c9TQN99cbT/RvtUVehGj2Fum+3bvD4wtFotT8yJP3vpT\n",
"5/h0r+juWvjAazsMX2/o7t3vMLz7ne7/vvPmirOBmWllmLfPW2//XsaoreRvdv9WjM/Gk/bbau2H\n",
"H9ns9XG7fvHJYnFFrfefOHEi6baZM2eu0sF6nk7CepAGsf+czSbzWWT5ttJ8vMAep9saHnIU6mhl\n",
"WGAQ+w0JC6w6TtEob1/gSiQJbYKsH08D7lMmmZfqNwTZg6xqH7GPZ00DnlImudQ6+oxB1qovVibZ\n",
"2vbxkJx2Y9KONr3F2hjlnYek0fq2MsmN9crVCRSLxamFQmFq1nLklXrGx1qH5wDj7KmZwPfS7umd\n",
"hPsOVaZV42OU90HgO8D9yiSfH+j9yufOUEfvAx4G1g9i/xV7bhoSMH8FcLG1IAl1tHI+DmJ/Yaij\n",
"m4E5qfCNKcD4IPb1QGS0Xq+9hgUioSGrhAUGsV9TWCDUsHyqTHKvMslWSNjEWcAawCnAP43y/meU\n",
"9zujvDORsAyvVkFKGOV9BRgKXG33ArcGjgdmAMOscsaeu9F5njqyItTRkFBHRyB7h+OQPZ6zg9g/\n",
"u1MVoiNXNNsD9THAIFXuCXU0CsnCdCd2Pg51tMp8nNozvxw4OtTRyFBHayDWZi0hdKsR6qjmsMBQ\n",
"R1OsNVk1Nad5UyZ5HrjeKG+yMsknjPLWRZZS90EG7Djgu7XeF8Aob1vg2/bwtFTTVGWSZUZ5hwKX\n",
"2kQBi+yz+rvnGoh1+w7wVD1yORzlhDraHLEOS952dwAXBbG/ODupHIMMgySP38gob6NGZxULYn9p\n",
"qKNDgWmhjo5GlM9vgZ8Gsb/Ctl0a6mi1+TiI/TtCHV2B5PIFuKK/Goh9Ya3DlWGBFay/VcICQx2t\n",
"DAsMdXRSEPvVhAUOKPfpDQDKJK8iyVf/MIB7Ye/1BBWsV2WSR+hJjVUtW9t7PqVMsnQA4nUKxawF\n",
"yDnFSo2hjoYARyCZl9ZAJoPvBLF/V/NFyw3FrAXIOcVWPESZpNso73FklWI7oOGpNoPYn0NPoevy\n",
"torzcRD705AttoFyFvDNIPZrioIIYn8p8PNQR7cgYYFVRUEMKPdpHilfFzfK+xhwNnCbMsm52Unm\n",
"aHfsvsk59DgY/AGxDl/OTirHYMYo7wvAscBlyiRXDOReLvep0O+eolHepwYSZmGUN8Qo77T+ezYN\n",
"t5/oGBChjoaGOjoG+A2iEBcCXwpi/xtOIToyplXp3nJHqKPVwgJDHR0W6mhAYYHVONrcC1xjk3HX\n",
"hN0j/A0S9JkV29rfdScgcAxeQh1tidRu+yKyXHo7kpWmmKVcDoelpBS3rdirAwh1tH2oIz/U0Rb2\n",
"VG9Lt48hsZTvrvc5/e4pKpM8Y5R3OhJ/OBa4CvizMkmvmWGM8jYD9kX2XdYETm5UaZNasRbuVvbw\n",
"ySxkcLQn9m3zKMTDeiSyX/OtIPbvzVQwh2NVnkUydSmjvLWVSV7PWqAmsglwHvDuUEcJ8Eaoo88g\n",
"CcvnAQSx/2ioo68giV/qCgusytHGhj18xihvB8Qr9Hxbn3ABkr5nBbABsCGwDPgj8GNlkmI9QjWQ\n",
"LZAJ7QXrEORw9Iu1Ds8F3mtP3QZMK8VqORx5QZlkuVHe08jy6TbAIxmL1DTsC+lWoY42A/ZGohxO\n",
"AS4LdfQycB+SEnQeAwgLrMn71AbpnwJgrcYtEO09BMlWYAaSJ7UJbGN/50mmTCkWi4U+MrkMekId\n",
"DV37PWtcgGT5GIF8p78VxP7sTAXLGe47VJkMxudJRCluSwcrxRJB7D8PXB/qaHIQ+58IddSwsEAY\n",
"WEjGs8BryiR5Xk4qrbO7pdMeCjiX+tWwexDnLl3w9oeQWKdbgR8Ese9WGFangPsOVaJAa8enNL9t\n",
"U7FX53EDgP0bbUhYIAxMKc4HNjXKuxVxpHmF6jPMtIrSl8SVi3L0it079JESZyOGjOx6nSV8Poj9\n",
"+/u51OHIC6X5bVApxSD2b2nGfQeiFM8AnlUmeQjA7jEeYZSHMknYEOkGjrMUHX1i68OdgxRNBYi9\n",
"wzde84MfmeQUoqOdKM1vWxvlDVUmeSdTaRpMqKNPAVEQ+3V9Lptw49Qg9n9STf+6S0cpk9xcUoj2\n",
"eLFVhn+wAaWZYpQ3GtgYSVxuMhbHkSNs3OGngQhRiC8Cpwexf8GwtYcuy1Y6h6M2rBPhC0jI0OYZ\n",
"i9MM7gWuCXVUc1hgqKOawwLrthSN8kYCOwL/UiZZOZEokzxrlLdWvfdtICUr8SllkhWZSuLIDdY6\n",
"PBf57gL8DvhhEPtLspPK4RgwTwLvQua9TELgmkUQ+8+EOjodiT8ciw0LDGK/17BA6526SlhgEPtV\n",
"j8lAlk9/h3j8dBnlxcgm51x7z3GVLmwRpfV1t3S6KsWsBciCUEfDkOTAJyHf0ReAC4PYL3+DLLZY\n",
"tHakmLUAOaeYwTOfQLwvt0GqWHQUtvrGZ2yNx9OA80Md9RsWWE+SjYEoxfuQGoUTkdx7PwA2At4C\n",
"Pj2A+zYKl8mmFwajK32oo20Q67CUCusm4EdB7K8W6DwYx6dW3BhVJqPxKb38d3RmGxukfwqAtRpX\n",
"CwusNXF4OQNRiv9EUl/9RpnkeKO8LmQP7+X0cmqGOCebQY61Do8DTkS+6/8FLghi/69ZyuVwNIFB\n",
"F5YRxP58JAqiodStFJVJbjHK2xiYBFyrTNKNmLIDwihvHFK09QJlkqtT519AMhWUuE+ZZEof9xiO\n",
"1FDsxtVQHJTYDfZz6SnAeiOynNLJabAcg5fnkFW6MUZ56ymTNCz7UqijE5EiwsuBdRGntHut48tl\n",
"SK3aRcBx6eLaoY7ORIoLA1wbxP5FjZLJ3v9iJJxqn5J1GOrIB34/kET9dXufAiiTvKRMcu1A7pHG\n",
"KO8QpHbWK4hCSzNDmWRi6qdXhWh5N6Lwn1MmeaNR8jnyT6ij4aGOTgKuQRTi88Bng9j/tlOIjk7F\n",
"hmGUDICGLaGGOvo4oBHFsy9SH3GMLfwbA2cHsb838BBweeq6A5AVmj2BvYATQh0d1Ci5LIuBnyER\n",
"BiXuAM4IdfTe3i/pnwEpxSbwsDLJUcBAs4i49G6DkFBH2wFX0+NMcz1wVBD7D2YqmMPRGkrzXSOX\n",
"UM9BUh2+AxDE/jVB7N8AHAgsD2L/HtvvF8BhoY42tMcBMD2I/WW22O904OQGygViOF0WxP5zpRNB\n",
"7L8UxP5UxPO0Lgayp9hwlEmSCs07GuXdDowCHgW+ViF7jttP7INOzFsZ6mg4cAKyfzgMiUs931YN\n",
"r4lOHJ9G48aoMhmOT0OdbUIdbYLE8W4b6uhCYDjw6yD2Lwd2I7WdFcT+c6GO3gB2RbxfxyOKsMRK\n",
"B5kGchPwcKijhxGP33uAB4PYX46EYtRF3izFSswFjlAm2QdZXp1Roa+zFPumkLUAjSTU0fbAr+hx\n",
"prkOOLoehWgpNEi0TqaQtQA5p5DRcxttKY61v48EPgwcDnwl1NGRiMdn+Yrey/Y8wBhknu6trVGc\n",
"D3wDeMDKNwt4zZaVqnurJFeWYiWUSU5IHU4FXjXK202ZpLelMWcpdjh2T6NkHQ5FHA0uGIAydDja\n",
"ndKe4nuM8oYpkywf4P1G2t+htb5eCnUUIU43z7K63wdAV+rfvbU3koeC2P956SDU0UhgD+AQZG+x\n",
"LtpGKaZRJnnDKG8RsCWwmlI0w4fvvnTIkOVTPPWbcWeeOX/y5MnzgWJvSxrFYrFA7292HdkfGFss\n",
"FqfmRZ56+r8+/62PrzFm+ITlr6/YgC661/JGzt1oj3Uv3V9/cDWFOBjHpxX9exmjtpK/2f0zGR+T\n",
"FI3ynkNSvY0FnqrUf+LEiaTbZs6cWd5nsf39Yurcf+39HwTeX9Z/ND0RCAvscbrtpV7kGAhvhDra\n",
"Loj9xwHs3uU9oY7+CnwNqCuHcVd3d7OVee0Y5d0NXKlMco09ngi8qkwyxx6PQMzjCcokf0pfO2vW\n",
"rO7tP3ncHGCOMknQYtFzT7FYnFooFKZmLUc9WOvwM0hmmqFAguwdPtyoZ7Tz+LQKN0aVyXJ8jPK+\n",
"B+wHnKNMcnst186aNat70qRJKy09a3ktBI4pVaQIdXQOUuD3p8DFQexvbc97wDPAmCD2F4Y6uhmY\n",
"E8T+hbZ9CjA+iH094A+ZwqZ/GxbE/jR7vB9wO3BTEPvH1HPPvO4pdrGqGb4F8NnU8WnIUsEDFe7h\n",
"lk47iFBHOyEJvD+NfG+nI3uHDVOIDkcH0LAgfmt5XQN8CiDU0ZrAxxEP7xnAsFBH+9ruxwM32nRs\n",
"IOEZR4c6GhnqaA0kXvFyGkwQ+z8uKUTLbOBM4Fv13jNXy6dGeTvTk6z5VKO8Pa21dxewr1HebERZ\n",
"vgpM7qdEinOy6Z1i1gLUgn1bPQn4BKIM/wOcF8T+35v0yGKT7ttJFLMWIOcUM3x2ad5rVKzil4DL\n",
"rYfnm8DVQexPBwh1dChwaaijlcH7pYuC2L8j1NEV9CxhXhHEfiXnyIYQxP7biBVbN7lcPh0IqeXT\n",
"TyiTzOv3AkduCXX0f8hL0lhk0346Epe0NEu5HI68YpS3GXALsEiZ5EO1XFu+fDpYyZWl2EDeAXot\n",
"K+LIP9Y6DJAUTkOQ/IbnB7H/jyzlcjjagP8CS4ANjPI2rBDL7eiDvO4pDpRnlUmcNdGG2PRM0xFn\n",
"GpA9jWOdQnQ4+sfmoB4UFTOaRadais7Jps2wm/GfBY5B9o2fQfYO52YqmMPRfjyBhEtsA/w5Y1na\n",
"jk5Vis7Jpo0IdfQ+ZO9wC6RY6NXAz93eocNRF85SHACdunzqLMU+sMG8uSDU0Rqhjr6IJBPeAtkH\n",
"/nQQ+5dkpRDzND55xY1RZXIwPo32QB1UdKpSfDRrAXJMIWsBAEIdvR/4DbJcugL4JeAHsf+vTAXL\n",
"yfjknELWAuScQsbPfxpxNtzSKG9kf50dq9KRy6fKJHUXmHQ0FxsAfCqSZLgLScJwfhD77kXG4WgA\n",
"yiRLjfLmA1sBWwNZv2i2FZ1qKTpySKijcYh1eBRiHV4BfNIpRIej4bgl1DrpSEvRkS9CHa1Fj3UI\n",
"Yh1ODWL/seykcjg6mseRQsDbZS1Iu+GUoqOphDoaD0wBFLAcuBL4pU3H5HA4moOzFOvEKcXBR7EV\n",
"D7HW4WlIAmGQP9LzSmVeckwxawHagGLWAuScYtYCkCo4bJQ3tJ880Y4UHZn71OXvy5ZQR7sh1uFm\n",
"iHX4S+BKZx06HK3DKO82YAxwuDLJs/31d3On4CxFR8MIdbQ2cDpwuD31OGIdumQKDkfreRxRitsB\n",
"/SpFh+C8Tx0NIdTRB4DrEIW4HKmd9imnEB2OzHD7inWQO0vRKG8cMrleoExyder8LsBlSFDqIuA4\n",
"ZZJF2UjpKGGtwzOAj9pTjyHWocsq5HBkS8OUYqijbZCkKJOC2L/XnlttTg5if1HqmjOR4sIA1wax\n",
"f9FA5WgFubIUjfIOAc4CXkHq55XOjwBi4Gxlkr2Bh2hCFWdHbYQ62gN5gfkoYh1eivxhOIXocGRP\n",
"yamtEZbi+cDK1IuhjlbOyUHsrzYnhzo6ADgR2BPYCzgh1NFBDZCj6eRKKQIPK5McBbxadv5AYLky\n",
"yT32+BfAYUZ5G7ZUug6gEXkZQx2tE+poCnAJ8C7kDfLYIPZ/GcT+8oHeP0tykLcy97gxqkyOxud5\n",
"pLbiRgOZK63j3GvAS6nTBwLLg9hfZU4OdVR6TgBMD2J/mc1jPB04uV4ZWkmulKIySdJH027AvFS/\n",
"54A3gF1bIVeHURjIxaGO9kSsQw28jSjGTwex//TARcsFhawFaAMKWQuQcwpZCwArayuWllAHEsQ/\n",
"FTVlqPwAABTASURBVDiv7Nwqc3IQ++Vz8nhkK6XEPHsu9+RuT7EPxrC69fgysEkGsgxKQh2NAr4A\n",
"HGJPzUVylv47O6kcDkc/PIEoqm2BP9V6caijA4G5QeybUEfppv7m5DHINlhvbbmmXZRiN6k9xhSD\n",
"PqamFYQ62gv4BrAxsAzZO5gexL4LCHY48k3dzjahjoYAX6bHiS5NNXNyWwbBt4tSXIBUkk4z2p5f\n",
"jUKhUCz9e9y4cfMnT548Hyimz5ew6/+FXm7Tkf2BscVicWo1/UMdrTtyk+E/HrbO0A8BDF936IKN\n",
"91nvvjU3G/keYB96ydyRt8/bzPFphTx57d/LGLWV/M3un5fxWfCpT663/q2/3+ydUaMOLRaLy9L9\n",
"J06cSPpeM2fOLL/8GOCOIPbTVYdKSq+/OXmBPU63vUQbkMuMNkZ5dwNXKpNcY481cLEyydb22AOe\n",
"AcYokyxMX+uyMlSmWCxOLRQKU/vrF+poX+BrwEaIdXgZ8OtOtw6rHZ/BjBujyuRpfIzyhgOzgaHA\n",
"BGWSN/rqWz53hjq6HNgZ8R0A2APZJ5wPXAVcHMT+1rbvyjk5iP2FoY5uBuYEsX+hbZ8CjA9iXzf2\n",
"EzaevFqKXaxqhs8AfmSUt68yyb3A8cCN5QrRURXFSo2hjtYDvgh8xJ76B7J3OL+5YuWGYtYCtAHF\n",
"rAXIOcWsBSihTPK2Ud6/keXTrZG/56oIYn8Vb9FQR88Anw9i/95QRyOBH4U62tfGLR4P3BjEfmlO\n",
"vhyYFuro+8hcfjRw5sA/UfPJlaVolLczcC4wAXkbeUSZ5CTbtgsSB5cO3l9cfg9nKdZPmXW4FBnv\n",
"azvdOnQ4OhmjvKnAwcB3lUlu6KtfX3NnqKP3IbmMPwzMAW4OYv9HNnh/lTk5iP3Fqeu+iCzBAvwm\n",
"iP2LG/SRmkqulGIjcEqxdqx1+CUk9gjgEeCCIPZdvkSHo80xyitZaTcrk3yzr35u7hTyunzqaBGh\n",
"jgrAV4ENgbeQN7/rnHXocHQMLgdqDTilOEgJdTQaSan3YXvqEWTv8D/ZSeVwOJpASSlu7Wor9k+u\n",
"Mto4WkOoo/2A6xGF+BZwEXCSU4gOR+ehTPIakvJtJDA2W2nyj1OKg4hQR+tfdcJ1EfA9YAMkie9R\n",
"QexfG8T+imylywc5yluZW9wYVSan49PI5OAdjVOKg4RQR5OA6995a8V+iHX4PeBkm7PQ0UMhawHa\n",
"gELWAuScQtYC9ILbV6wSt6fY4YQ62gBJ1fRBgOGjhz2/fMk7Rwaxb7KVzOFwtBBnKVaJU4odTKij\n",
"/YGvICmW3gR+7B2+0c4T95voFKLDMbhYWS3DKK/LVtBw9IJTih2ItQ6/Akyyp/6GxB2aYrG4U3aS\n",
"ORyOjHgRqVoxGqlW8WK24uQXpxQ7iFBHXcD+yHLpaKS+2Y+QDBTOkcbhGKQok3Qb5T2B1EHcFqcU\n",
"+8QpxQ7BVrz+CrCfPfVX4MIg9p8v61pspVxtSDFrAdqAYtYC5Jxi1gL0QVopzs5YltzilGKbY63D\n",
"DyHW4XqIdfhDxDpcbd+gj3JJDosbn/5xY1SZHI9Pydlmu0ylyDlOKbYx1jr8Kj0u4A8g1uF/MxPK\n",
"4XDkFeeBWgVOKbYh1jo8AEnTti7wOvADIO7NOnQ4HA7gWaQ26uZGeesokyzJWqA84pRimxHqaCPE\n",
"OpxgT/0Z+GYQ+y9kJ5XD4cg7yiTLjfKeBnYAtgEezlikXNJWStEo7ypgy7LTByuTvJ6BOC3FWocH\n",
"ItbhKGAJMA241VmHDoejSh5HlOJ29KMUQx0NAz4HaKRQ8AjgG0Hs32XbdwEuY9V6iotS15+JFBcG\n",
"qct6UWM/SnNotzRv3cokE8t+BoNC3BhRgOcjCvFPwJFB7N9Sq0LMaV7G3ODGp3/cGFUm5+NTS7q3\n",
"zYHTgUOC2C8A5wBxqKNNQx2NAGLg7CD290byKF9eujDU0QHAicCewF7ACaGODmrYp2gi7aYUBxWh\n",
"jrpCHR2MVLTYB7EOzwM+H8R+vXFGhQaJ16kUshagDShkLUDOKWQtQAVqcbZ5FZgSxP5rAEHsz0Ty\n",
"Ju+FrFotD2L/Htv3F8Bh1vkPIACmB7G/LIj9pcB04OQGfYam0lbLpwBGeZcBOyEK4nvKJMVsJWoO\n",
"oY42Ab6OfAEB7kf2DhdkJ5XD4WhzngK6kdqKw5VJ3u6ro10KnV46tls4I4AFSBjYY6m+z4U6egPY\n",
"FbgTGJ++FpgHnNLAz9E02k0pzgNmKZPMMcobD9xtlLe3MsnfsxasUdgv3mTgi8A6wGtIvcPb3d6h\n",
"w+EYCMokrxvlJcAWwLvpWU6thgnAfCTw/xNI2rg0LyMp5ADGlLWn23JNWy2fKpN8T5lkjv3334Db\n",
"EDO9Iwh1NAZJy3YOohBnA0cEsX+bU4gOh6NB1FxGKtTRGsA3EWeabsTa7G1O6kr9uy3nrHazFMtJ\n",
"gB3LT6YzSowbN27+5MmT5wPF3jJN2E3xQi/3bln/tHU4ZETXpkPXGjpi9HvX/tP6u67zSldX16nF\n",
"YrFh8gBji/+/vfsPlrOq7zj+/iIlGEFQpAGWVajhRwItiKBFQrzAOC2D8oCjB4G11dRxZaqlDtYf\n",
"FZjo6DAWxbRFZLGOSVhn4tNCXEJFSm5z0SgdMMbOWEIInWq3j8qNCVhHBUly/OOcvdnc7N27e3Nv\n",
"dp/dz+ufzT7P2b0n38l9vjnPc873jI0tn83+D1J7FJ+O2reIUa76P9ft+zk+r7j0Txe9dOzhI1dt\n",
"337pzWbnNo6vX7++xddOXJ/uAm4r10qNGavjwGsmNT06Hm+cP3rSue0tf0CfMe/zk8yzQvHDhaz+\n",
"d03vVwPPFbL6exvHRkdH/SWXXGItv6APxdHhjcD58dDDwC3lWunnc/HzxsbGRvq4DFXPKT7TU4za\n",
"6/f4ZIXiBYQ7UpsKWX3iTttU185KUr0NeKpcK90RZ50uIDw7/Fy5VloY2xSB/wEWlGulHZWkuhbY\n",
"VK6VPhXP3wScW66Vkrn++x2oXN0+BT6YFYrHAmSF4snA5cDdve3SzFSS6iGVpPpWwszS84kzvYAP\n",
"zVVChL6uy9gXFJ/pKUbt5SA+EzVQs0Kx7QCiklQ/ArwIWFVJqkcAC4FlwAPAoZWkujQ2XQbcU66V\n",
"dsT3dwJXV5LqvHjr9Wqalmz0s7zdPv0ssDYrFHcBLwH+spDVc1ftvZJUX0WYWXpOPDRGGB3umPJD\n",
"IiKzYwdhsf3LgeOByTvpAFBJqqcCt8S3H2g6tbxcK/22klSvAO6oJNWJxfuNBuVa6cFKUv0yYdY8\n",
"wJfLtdIDs/q3mCO5un3aiX6+fRorRPwZYVHrYcAzwK3AQ5pIIyIHS1Yo3g78MfChxrK2fr52Hkx5\n",
"GynmViWpnkG4PbowHloHrCjXSpOnNYuIzLWthKR4Gv27/2NPKCnOsUpSnU+o5PAOwjPcjLAI/9Ge\n",
"dkxEhpm2kZpC3iba5EolqS4BvgZcEw+tJtQs7VlC7PO6jD2n+ExPMWovJ/FprFXUhsOTaKQ4BypJ\n",
"9UTgBkK9UgjlkD5VrpWemPpTB80Iul3SzgiKz3RGUIzaGaH/41Mn1DE9LisUjypkdT3GiZQUZ1Gc\n",
"evxuwmSa3yPUZ70L+Fq5Vtrdy76JiDQUsvrurFB8CjiTcAv1sR53qW8oKc6CWPHhIkK90uPi4fuB\n",
"f9QyCxHpU1tRUtyPkuIBqiTV04HrgfPioSeAW8u10sAUKReRgaTJNi0oKc5QJameAFxH2FcMQkWa\n",
"LwBf161SEckBTbZpQUmxS5WkehThueFVhOeGLwBrgJU5WXM41usO9LmxXncgB8Z63YE+N9brDnTo\n",
"KWAPcHJWKM5j9coed6c/qKJNhypJdR7gCDX+joyHvwHcWa6VWpZJEhHpZ1mh+M+EfRXf+cTqlY+r\n",
"oo1GitOKyTAh1PVrbJL5KPAPfbLEQkRkpp4kJEU9V4yUFKcwRTJ8ErgdeES1SkVkAGwF/gQ9V5yg\n",
"pDhJ3B4lAa5lbzLcRlhv+HC5VtrTq76JiMyyxmQbjRSjXCXFrFA8nLAn12mEvv9tIas/NBvfXUmq\n",
"ryRMnnkLMD8eVjIUkUHWNinGgiT7XHPLtdKsXHP7Vd5qny4HfCGrn0+oJ7omKxR/v/1HplZJqlZJ\n",
"qq+LO0vfQ0iK84HvERbiX1uulTYMUkLMSV3GnlF8pqcYtZen+BSy+k5gO3sHApMtB3y5Vpq45laS\n",
"6oyvuXmQm5FiVigeAvwFcCVAIatvywrFzUAJuK2b76ok1eOBNwOXASfGw78l7Ca9plwrbZutfveh\n",
"EfIzZbwXRlB8pjOCYtTOCPmKz5PAsZMPVpLqPtfccq20rZJUZ3TNzZPcJEXgD4BjCBVjGh4Hzu3k\n",
"w3Gx/QhwMXB206lx4F7g3nKttHNWetrH1q1bd9LIyEivu9G3FJ/pKUbt5TA+W4ELWhw/oGtuXuUp\n",
"KS6Ir80L5H8BLG7VOO5jeBZwDnA+cHrT6ecJ/5NbBzw2TBVoNm3adFKv+9DPFJ/pKUbt5TA+W6c4\n",
"3tU1d1DkKSk2TF4Ksd9i00pSXQksAl7UdPg3wEZgA/Cdcq30q7nqoIhIjjw5zflpr7mDJE9JcTy+\n",
"Hg38vOnPT7doeyawG/gh8H1gE/C9cq30/Fx3UkQkZzKg1SChm2vuwMhNmbc40WYcuKKQ1TfGY6PA\n",
"/YWs/vlGu9HR0Xz8hURE+kxzmbc40WYcuKJcK22Mx0aB+8u10uen+Ircy81IsZDV92SF4pcItUc3\n",
"ZoXiKYRnhlc3t1PtPhGRA1eulfZUkurENbeSVFtecwdNbkaKAFmhOI+wkPR0QkL/WCGrr+9tr0RE\n",
"BlMsd7nPNbdcKw30NTdXSVFERGQu5a2izZTM7HAzW2lmj5jZY2b2pl73qZfM7FAz+2sz22BmY2b2\n",
"XTO7uOn82TFWG83sPjN7eS/72ytmdoqZvWBmS5uOKTaRmb0n/tv5lpn9oBEnxQjMbFH8/fq2mW02\n",
"sxubzg19fPJqYJIisRyR936iHJGZDXQ5ommcCPwVcLn3fgS4GaiZ2fFmdhhQAz7qvV9CmKF7Z896\n",
"2lufJKxbBUCx2cvM3k4ojn+h934poYrJAsVowipgo/f+QuAS4INmdpnik2+5mWjTjpntU47Ie7/N\n",
"zAa+HNE0/h+4yXv/SwDv/Xoze45QueIFYJf3/uHY9p+AH5nZMd77Hb3p7sFnZucBvyTUfmy4FMWm\n",
"4Wbgvd773QDe+9UAZpagGEFYxL4cwHu/08y2AWcQrquKT04NykhxKMsRteO93+m9/2rjvZkZcBhh\n",
"ivV5NMXKe/9/wK8J1X+GyXLgE5OOnQdsabwZ1tjEuyxnAKea2Wi8ffq+eFoxCv4VuBzAzF4NnAKs\n",
"R/HJtYEYKTKk5Yi69EbgR8C3gXeyb6wAnmXv/pEDz8wuBX7ovc/C/xcmLCCMspsNVWyik+LrVYRN\n",
"aF8GPGpmzxBioRjBewiPJJ4CjgLe773/vpldh+KTW4OSFBuGqhxRp8zscODTwLu8997MPPvHCoYk\n",
"XvF2+4eJt9snGerYNJkXXyve+13AdjOrEtas/RjFCMJGAt/13l9sZicC3zGzp9G/oVwblNunzeWI\n",
"aPrzQJcj6kS8bXoXcJv3fnM8PM6+sSK+H2c4XAM86L1/tulY44I17LFpeCa+Nv8O/ZQwgWvoY2Rm\n",
"iwiTa1bAxC3SGnAdik+uDUpS/G9gJ/vuhLEYeKw33ekrnwP+w3t/j5kdZmZFQlxOazSIx+YTasQO\n",
"gyXAm+N0+g3AccAKM1uLYtOwjfAcrPmW3ysIdTIVo/B8HsKktYZdhBG24pNjA5EUvfd7gEY5Isys\n",
"UY7oq+0+N+jM7COEnUJWmdkRwEJCjB4ADm1am7cMuGdYZsZ579/nvV/ivb/Ie38R8DPgeu/9lcA3\n",
"GeLYNHjvnwdWA38OYGYvBt5OWIYw1P9+oicI/0G4BsDMjgTeAjyE4pNrA1PRxsz2K0fkvR/ockTt\n",
"mNmp7Dsbt2G59/6TZnY2cAdhN5GdhOeNz7RoP7DM7CzgJsJEkk3AWu/93ys2gZnNJ/xO/SFh67V7\n",
"vfefjeeGPkZm9lrCki8DjgAeBD7uvd+j+OTXwCRFERGRAzUQt09FRERmg5KiiIhIpKQoIiISKSmK\n",
"iIhESooiIiKRkqKIiEikpCgiIhIpKYqIiERKiiIHgXPuYufcVV1+5ibnXHGu+iQi+1NSFJkB59xS\n",
"59zp07cE59wHgC8Q9rLsxmbgEefca7vtn4jMjJKiyMx8BTh3ukbOuYsI9THflqbpT7r5AWma3k/Y\n",
"mug+59xRM+qliHRFSVGkS865E4CTgW910PxWoJam6X/N8MfdTtiO6G9m+HkR6cKhve6ASA4tAepp\n",
"mv5vu0bOuTOAc4BKi3PHArcAryfsyXcIYR++29M0/c9GuzRNn3POjQHvAm6cpf6LyBQ0UhTpkHPu\n",
"cufc3cBngF875+52zl3X5iMXxtetLc5VgSLwmjRNzwHeCCwCrmjRdgtwgnPulJn3XkQ6oZGiSIfS\n",
"NL2P8HxvM3BHmqZfmuYjC+PreItzbwDWpGm6K373L5xzHwNe3KLt003ft637notIp5QURbrgnHsp\n",
"YdPdjR00b0yOea7FuX8DljnnXgbcDaxP03Sq2am/mfR9IjJHdPtUpDtvAJ5N03RLB233xFdrce4d\n",
"wA3AqcBaYNw5tyo+a5ys8Xu6p8U5EZlFGimKdOdCOhslAuyMr/Mnn0jT9AXCcosVzrnFQBl4P/Aq\n",
"YGRS85fE1x3ddlZEuqORokh3LiAuwnfOLXbOvbVN28Zo8vjJJ5xzaxp/TtP08TRNrwe+AZzV4nsa\n",
"n+9kdCoiB0BJUaQ7xwBbnHOHAO8Gam3abgA8cGaLc845d23TmwXAYuDBFm3/CNjS7eJ/EemekqJI\n",
"dz4DXAN8mjADdfdUDdM0rQPfpPUyixuAsnPuB3E2678D/wIsa27knDuGcMv2i7PTfRFpx7z3ve6D\n",
"yMByzp0GbAKuTNP0oRl8fgUhKb6uXQIWkdmhkaLIHErTdCtwJXBXt4W9nXM3EBb1X6aEKHJwKCmK\n",
"zLE4QlzK3lmknfox8Po0TX82+70SkVZ0+1RERCTSSFFERCRSUhQREYmUFEVERCIlRRERkUhJUURE\n",
"JFJSFBERiZQURUREIiVFERGR6HctU3jamxNXUAAAAABJRU5ErkJggg==\n"
],
"text/plain": [
"<matplotlib.figure.Figure at 0x7f0ba476de10>"
]
},
"metadata": {},
"output_type": "display_data"
}
],
"source": [
"# plot v(t) and x(t)\n",
"fig,ax = plt.subplots()\n",
"c1 = \"#e41a1c\"\n",
"c3 = \"#984ea3\"\n",
"tts = np.r_[0,ts]\n",
"ax.plot( tts, tracev*mphfact, lw=2, alpha=0.8, c=c1 );\n",
"ax.set_xlabel(r\"$t$ (s)\");\n",
"for tl in ax.get_yticklabels():\n",
" tl.set_color(c1)\n",
"ax.set_ylabel(r\"$v(t)$ (m)\", color=c1);\n",
"ax2 = ax.twinx()\n",
"ax2.plot( tts, xs, lw=2, alpha=0.8, c=c3 );\n",
"for tl in ax2.get_yticklabels():\n",
" tl.set_color(c3)\n",
"ax2.set_ylabel(r\"$x(t)$ (m/s)\", color=c3);\n",
"ax.set_xlim((0,ts.max()));\n",
"ax2.grid('off');"
]
},
{
"cell_type": "code",
"execution_count": 219,
"metadata": {
"collapsed": false
},
"outputs": [
{
"data": {
"image/png": [
"iVBORw0KGgoAAAANSUhEUgAAAZgAAAEmCAYAAABf+4ZQAAAABHNCSVQICAgIfAhkiAAAAAlwSFlz\n",
"AAALEgAACxIB0t1+/AAAIABJREFUeJztnXmYXEXVuN8TlhAgECCsCoR9CZ9BAiqI0BhAYcg4CpTI\n",
"vigiqIgEVBQJouinCJ8oi+zgDz4tQYaREYGMaRYNO6KsskX4EAghELYQIKnfH1VN37m53dN35nbf\n",
"O9PnfZ77TN+6datOn+65p6vOqVPinENRFEVRsmZU3gIoiqIoIxM1MIqiKEpTUAOjKIqiNAU1MIqi\n",
"KEpTUAOjKIqiNAU1MIqiKEpTUAOjKIqiNIWl8xZAaT4isjJwHHCtc+6BIbTzDeBp59x1mQnn290Q\n",
"+CnwX8Ai4HngJOfcnSnaOBo4CnDAaOD3wGnOuXciddYCDgH2BlYIxW8DlwG/jtU9FDgT+HdCd+sD\n",
"zzrnJjUqXxEQkc8DnwM2BxbjdfAU8DPnXF+k3gTgX8BDCc2sBqwDrOGcm9dAn3sC3wj3jcF/NrOA\n",
"LznnFgzh7SjDAeecHiP8ACbgHygHD7Gd2cAlGcu2Dt6gXAqMCmXfAt4CtmuwjVOB14Ctw/nawJPA\n",
"1bF65wNzgEmRsj2Ad4DeWN1Dkt4rIPiH8jfy/lwH+fldBYwJ58uF80XAJ2Pfl6dqtHEF0N1gf98E\n",
"ngYmRso+E76Lq+atDz2af+QugB4t+JCrBuaQIbYzG7g0Y9kuABYA42LlTwB/a+D+9YF3gf+OlR8U\n",
"3vPukbLzgO8ltHFVqLt1pGwD4GMJdfcCXgdWyvtzHYSurwLWi5VtHd77LyNlywOdCfePD5/VlAb6\n",
"2ioY7j0Trk0FlslbH3o0/8hdAD2a/AHDEfipjsX46Z77wxH9xboDcFO4/gRwJ7BP5PqkcM9C4OVI\n",
"GycOUbZRwKvAbQnXzgsybzBAG8eHelNi5WuG8ssjZUsBktDGf4e6uzQg8w3AuQ2+v2WAnwD/AR4F\n",
"bg+/4GcHPd4HjA91DwT6gl7vA+7BTyM1+/uxc3jv0xqo+y3goQbbPQd4kzAqHYRcd+NHtouBLYE/\n",
"AQ/jR49fDd+dk4G/As/ip0RXi9x/cuR7fyxwUdDpS8DfgB0S+jwkfE7PAncBJwGXB0N5P7BHsz+P\n",
"kXbkLoAeLfiQ/a/8xCkyYPfwD3RKpGwf4D1i00D46Y7EKbKI0RnouCByz8ZBrssS2qsYjs8O8N7+\n",
"X6g3IeHa68ADDejnj8A8YMUB6m0c9LJlg3q/DD/Vt1M4r/iGFsT1GB6oX4l9Zk9Gy0L5til0feQA\n",
"8k0MD90yYdqsTt1R4fM/usH3/k/gEWAKcCPwYCg7K2oIBmjjlPDZXggsG8q+GcquI/wgAFbBG+0L\n",
"YvdXvvdzI5/BUsCV4XOJjlgPJWZogRPDdyhxulCPBj7DvAXQowUfch0fDPA43nEfL/9j+CccHymr\n",
"aWAGKdf2Qa6zE659KVwb6CF5IzXm9IHngP8McP9meAP7lQbk/TnQ1+B7qzjSL46VVx56cQOzGbHR\n",
"FX70888mfB8OC7pZBFwS/Yzr3DMVmA+s0GAfr+NHMA8Bm4ayLcL37V/Ayg20MT3oanKkbHwo+2Os\n",
"7jnAM7Gyyvc+/hmsgR+NXx/ORwV9PBGrJ3gjrwZmkIeGKbcxIrIpsBFwR8LlWXgncKmVMkXIIs13\n",
"3TZEZEW8X+Ji59x5A9Qdg/+V+8sG+94l/O2nW+fcv/EP6jjvAOeJyD0i8oCI3I+fstm4wf4axjl3\n",
"qXPuA8AH8KOqh0VkxwFuOwbvf3uzwW7GhON7zrl/hX4fwU87bRzaa5RHIq9fTiirlK9d4/6/R0+c\n",
"c3PwI55SKNos3HtnrJ7Dj7qUQaIGpr1ZPfx9OeHa3FidZlDpY6WEayuHvy8NsY3E+0VkLNAL3OWc\n",
"+8oAfQDsj49UazREe7XwNymU99WYLGvifQkfAvZyzk1yzn0YH/U2usH+UuOcewFvNN/F+xoSEZGN\n",
"gV2BX6Vo/jW8gb8vVn5P+PvRFHK+FXld+dHwRqzaYvz0VxJJBv1lYEz44TDQZyWNyqr0R9fBtDeV\n",
"h+9qCdfGh79zGmko/OJuhLudc0eG10/iH0QbJdSr/HIfqN17gS+ENmZH5FkTv85jiftFZDzeWX+j\n",
"c+57Dcp9DHBe5AE3EBWjnaTbVWPnU4G1gK+Gh/77osZvFJFt8T6JRjjPOXdBuG8U3o/xdrSCc+5d\n",
"EXkYmCIiq7rktS1HAzc5555osF/wU2MfZ8kfsYvC31b+uB2XULYasMA5t0BE6n1Wq5DNaLotUQPT\n",
"Hrwb/o4CEJH1gXWcc7NE5AngYyIisYfn9nhn9MxYO5U2lgd2dc71AIRf3Klwzi0WEQscJCLjnHOv\n",
"hrYF2A2Y5ZybXakfytdyzj0faeYavK9iN3wUVoXdw98ro32KyLp4v82vnXO/iJQf6UVySzy8RWR7\n",
"vE/lohRvryLL9vhQ7EpbE4CxsbqVUUr8QbZOvMw5dw+QWtfATsCv8dNB7xN0OgH/WcdHBZXP+VDg\n",
"gFoNh1HAmJhxuhpvYD6M991V2Dr8nZX2DQyBraMnYcHthvgfGQCP4X0wH43VG4Vf/KsMEp0iaw9e\n",
"xDvs1wvnRwOHh9dfxc/Fn1ypLCKfwy9A/G7sofFUpI2d8Svdh8opwCvAmSJSmeI4AT8nfmys7rnA\n",
"cyKyb6Ug+DR+DBwtIpOC/GuHdq9xzt0ceV+b46eingbmisiBlQP4NP6BnsQxwFU1ft0nEvwOlwCf\n",
"F5GdQv/LAT9iyQf5DfgH/DfC1B3hvXyh0f4aZBMRmVbRs4gsE+TZCB9o8U7CPfsDc51zNyRcq3Af\n",
"8Eww3hXOB/4BTA8P9Mqo8jS8kz/NdFsSaaatdhGRnYMMS+O/t+8QvvPOucXAd4ANROSEyH0n4P1I\n",
"ymDJK7oA70C+DP9L5m5gtzp1D8DP3d4NnJFw/Yv42PZb8Q69nfKOnijagY8cego/ZXQrkbBe/K/s\n",
"G/HrYJ7EO6b3SWjjI/iHxoNBz7tnJNsG+PDdx/BrHfqAjyTU+z7eGC2xXgVvNP8JPIBfy3AascV8\n",
"+NHOIvx8ffxYBHw/od3V8elkth7E+1oKOJ3qOpiZeMfy0ywZ2VQK/wvP48OGf4PPbrA4fGZTh6jj\n",
"FfE/JvrC5/cP/HqPW4FD69x3LwNkLQBmhPc0Pla+Ct6QzA6f65P4IIlVGpD3xqCLReH9d4Xjfqrp\n",
"hG6oVTeUTwj6+zJwNn5ty0v4HxnbJ/R5ID544NnwWRwTPgONIhvkIUGxLUdEfgKs6Zw7TEQ2wT/U\n",
"tnA+wiNabyvgZnzM/iv4f5CrnXPnhuv7Agfjv1SLRORg4G3nnG3h21GUhhGR1/FhyvERmpIhYTry\n",
"KbwBvWKQbfwRn/1gWOWdKwq5TJGFuc0jgIsBnHOP4395HJhQ/Qh8nqh5zlvDS/BJDSt8HzjdObco\n",
"tHWFGhelCIjISSLykVjZRvjgg3vzkUpJQkT2EJEvx8oqPhj9rAZJXj6YDfERG49Gyh7Gr1KOs22s\n",
"3iPARBEZLSJr4Ec2m4pIn4jcKiJHJbShKHmwCfADEVkBIPw9Cz8V+Ls8BWszGvHXrAl8L4x6KsEP\n",
"38VHU57RNMlGOHkZmDXD32h8+nz8CtukutF6lbj08fg5VoDPA5/Cp2H/VkhLrih583uCD0VE/okP\n",
"3X0J7yNcmKtkIxwRORm/zsnhjXz3ALfcAdwC3CQiD+D9RtvjP6uHmynrSCbvMOW4A6jWL40kR5FQ\n",
"De/8tXPuPeAlEfl/+Agp/YWo5Ipz7k/4JI1Ki3HOnYYP9Gi0/qMkT9ErQyAvA1Nx5I+juhJ7HD6c\n",
"NqludKHUOLzBeSlSHr3veeCD8Ub6+vp0sZSiKMogmDJlyqCyGeRlYJ7Ep2XYHJ/CHHxK7usT6t4d\n",
"6hGp96BzbqGIPI5f3xGdWhuPXzS1BINV0khjs802u+yxxx47NG85ioDqoorqoorqospQfpzn4oNx\n",
"fmHThYTFfiFMeRJwpYiMF5HbRGSVUP0iYE8RWTVEdRyKX8RFmMe+Ap8UsLKieF/q5FVSYO21156Q\n",
"twxFQXVRRXVRRXWRDXmu5J+OD9aYhc9ou19YAzMG2DT8xTn3EDANvyHWHcB9rn/m22nAmyEXVh9+\n",
"g6l+6UEURVGU1pObkz+MPg5LKH+WapRZpewqvBFKauct/EJLpUEmT548O28ZioLqoorqoorqIhsa\n",
"NjDGmE3w6Sw2xqdBX4zPGPsI0GetTXLQKwVk6tSps/OWoSioLqqoLqqoLrJhQANjjPkYfqHRDnWq\n",
"LTbGdAPTrLWzM5JNURRFGcbU9cEYY44AbsPv2XEsPiX6h/ArlDcDtgE68Cte1wUeMMZ8opkCK5lQ\n",
"zluAAlHOW4ACUc5bgAJRzluAkUDNZJfGmB3wEVpfsNbe3Uhjxpgu4H+AjxZtyqyvr89pmLKiKEo6\n",
"hvLsrDeC2RT4VKPGBcBa240PGd5qMMIoiqIoI4eaPhhr7WWDadBae8ugpVEURVFGDJmtgzHGXJNV\n",
"W4qiKMrwp24UmTHmA8BCa+1cY8whJCedBJ94MinVvqIoitKmDDSCuQ+f8hr81qGX1TguJSHBpFJM\n",
"yuVyKW8ZioLqoorqoorqIhsGMjA/wu9lDX5B5Qb4zcKSjkeTGlAKSSlvAQpEKW8BCkQpbwEKRClv\n",
"AUYCdafIrLVnR057rbX/rlXXGNNb65qiKIrSfjTs5LfWnjhAlauHKIuiKIoygsgym/L5GbalKIqi\n",
"DHPSJLscAxwHfBpYO3Kvw0eRrZ25dIqiKMqwJU26/l8AXwBuBWbjsylHmZqRTErzKectQIEo5y1A\n",
"gSjnLUCBKOctwEggjYH5NDDRWvtM0kVjzN+zEUlpNqVSqZy3DEVBdVFFdVFFdZENaXwwz9cyLoFt\n",
"hiqMoiiKMnJIY2D+Zoz5rzrXLxyqMIqiKMrIIc0U2XeAy40xc/CLLt+IXBNg1ywFUxRFUYY3aQxM\n",
"J7A3tUc9tfKUKYqiKMOEzu6OTfHLTt4F/nXs2G8Ouq00U2TTgXOBjwAboalihi2aZ6mK6qKK6qJK\n",
"m+tiWWAlYLXwd9CkGcEsY639eq2LxphzhiKI0lJKaBhmhRKqiwolVBcVSrSvLpaJvH53KA2lGcHM\n",
"HuD6PUOQQ1EURSkG0YHHe0NpKI2B+YEx5ixjzPga1zVVjKIoyvAnOoIZkoFJM0V2BbAKcKwxZi4+\n",
"ikzQVDGKoigjiahdGNIUWRoDsxJwLd6YJKGpYhRFUYY/uRiYZ6y1h9W6mDZVjIgsh59W2yzIcZJz\n",
"7uYadQ/AJ9p0wC3OuWmRa5cB68du2cs592YaedqMct4CFIhy3gIUiHLeAhSIct4C5EhmTv40Bmb3\n",
"Aa5PTtn3dMA557YXkU2AO0RkC+fcnGglEdkKOAOYCLwC9InI0c65c0MV55zbJWXfbY3mWaqiuqii\n",
"uqjS5rrIzAeTZsOxlwa4vqjRtkRkFHAEcDGAc+5x4H7gwITqRwC9zrl5zjkHXAIc1WhfiqIoSipa\n",
"b2AyZkP8Ip7o4syHgW0T6m4bq/cIMFFERlcKROQ8EblVRP4kIqUmyKsoitIu5OKDyZI1w9/5kbL5\n",
"wJY16kbrvYoPNBgPPIc3OH3OuXtFZFtgpojs6Jx7IHuxFUVRRjzDfgRTIZ6/rFaEWlKeMwFwzv3U\n",
"OXdveH0P0At8OTMJFUVR2othP4KpOPLHAXMjr1+sUXdc5Hwc3uDU8gk9S/JIqJ/jbvLkybOnTp06\n",
"GygnOfRCLqJSQjPDvj4s6cQcTvJnXH8C/bNU5C1PbvXL5XIp+jpveXKuPwH/vSiKPC2rP+e+OVuN\n",
"XW/sOgAv3j1nd0xCrUZxzrX8wI+c5gI7Rsr6gOMS6p4FXBQ5Pwh4IHJ+Yqz+FcAF8XZmzJjh8niv\n",
"RTxmzpw5PW8ZinKoLlQXqov+x9Rr9/zy1Gv3vCccRw7l2dnQCMYYsxSwE7A1sHoofgkf+XWrtXZx\n",
"SqO2WEQuBA4Hbg9hypOAL4jIePyCzk7n3CvARcDNIrIq3v9yKP3T0hwnIpc6514SkQ3w2wrook9F\n",
"UZTB0bpUMcaYTuBXwAdrVHnWGPM1a21Pyr6nA+eLyKwgx37OuTkisi6wKTAGeMU595CITANuAhbj\n",
"F1qeF2nnDOBaEXkPWAE4xjl3W0pZFEVRFE9rFloaY/YE/gDcApwDPAG8hveBrAxsjF+A+QdjzFRr\n",
"7Q2NduycWwgskRnAOfcs1SizStlVwFU12vk58PNG+1UURVHq0jIn/8nAgdba39ap89/GmM8D3wca\n",
"NjCKoihKIWlZuv4VBzAuAFhrfwesOBRBlJZSzluAAlHOW4ACUc5bgAJRzluAHGnZhmOjjTHLDdRI\n",
"qDN6oHpKMWjzPEv9UF1UUV1UaXNdtMzA3AZcZ4zZolYFY8zmQHeoqyiKogxvMpsiG8gH8228g/8h\n",
"Y8xzwJN4Jz/4/WE2BtbBp2s5aCiCKIqiKIWgNaliQgbl7YDv4degfALYKxyfAOaFa9sNlG1ZURRF\n",
"GRa0bj8Ya+2bwOnA6caY0cCq4dI8a+3CoXSuKIqiFI7MwpRTJbu01i601j4fDjUuw5SQh0hBdRFF\n",
"dVGlzXXR2mSXWaeKUXKnRHuHYUYpobqoUEJ1UaFE++piRKSKURRFUYpHa5z8kVQxjwPfAfYFPoVP\n",
"D7NvKHsCnypmj6EIoiiKohQCTRWjKIqiNIWW7WipqWIURVHai5ZFkWmqmJFJOW8BCkQ5bwEKRDlv\n",
"AQpEOW8BcqRlBkZTxYxA2jzPUj9UF1VUF1XaXBctW2ipqWIURVHaC00VoyiKojSF1q2D0VQxiqIo\n",
"bUVrV/JXCAbl+aF0qCiKohSTzu4OoYVhysoIpM3zLPVDdVFFdVGljXWxVOT1op6u3iGlAcvMwBhj\n",
"ds2qLaXplPIWoECU8hagQJTyFqBAlPIWICcymx6DbEcwF2bYlqIoitJ6Mpsegzo+GGPMeinakXpt\n",
"KYqiKMOC1hgYYDbg8MajEdxQhVEURVFyJbNFllDfwNwErA/c2WBbew9VGEVRFCVXMvXB1DMwR+LT\n",
"v3zbWvvCQA0ZY3YeqjBKyyjnLUCBKOctQIEo5y1AgSjnLUBORG3CkKfIajr5rbXPAP8NnN9gW0+l\n",
"6VhElhORy0RklojcLSK71al7gIjcE+qdUaPOJiLyrojslEaOdqTN8yz1Q3VRRXVRpY11kekU2UCp\n",
"Ys4FTjHGDBhtZq2dkrLv6YBzzm0P7A/8VkTWiFcSka2AM/CbnH0E2EZEjk5o7weAZhZQFEUZPC2b\n",
"IgPAWvvAUDuJIyKjgCOAzwI45x4XkfuBA4EzY9WPAHqdc/PCvZcAJwLnRtrbDngd0HxoiqIogyfT\n",
"KLKBtkw+3RgzeaidJLAhsBrwaKTsYWDbhLrbxuo9AkwUkej+M9OBUzOWUVEUpd1oWZgyQAfwbWPM\n",
"M8C14bjNWjvUkOQ1w9/5kbL5wJY16kbrvYoPnR4PPCciewAPOueeE2k0olpRFEVJoHUr+a21k4BN\n",
"gHOAjwIzgeeNMRcYY/YwxixT7/4GiBuqWhYiyaBJmGo7EfhxI511dndclEK2EUsb51laAtVFFdVF\n",
"lTbWRct9ME8CPwN+ZoxZG+83+SxwHbDAGHM98AfgBmvtWw32Oyf8HQfMjbx+sUbdcZHzcXiD8xI+\n",
"OOBG59yrkes1hzHvPbuoq1QqbQwwefLk2VOnTp0NlJMiRsIXrJTQzLCvH8r73TOc5M+4fqlcLpdT\n",
"1G+2PHnWLxG+FwWRJ8/6le9FUeRpSf0PL7vNK3e9cufY9xa8N/a1f7++rXxWps+YMSOh2cYQ5wY3\n",
"22WMWQXoBD4HVEKMbwL+YK29om6nfuQxB+hyzt0eyvqA651zZ8XqngWMdc59MZwfBExzzk0SkfOB\n",
"raha2o/h/TWznXOfjbbT19fnfvH6mTN6unprhkO3C+VyeXqpVJqetxxFQHVRRXVRpV110dndsSvw\n",
"k3Da19PV+62+vj43ZcqUQfkfBp0/zFr7CnA5cLkxZgVgD7yxORuoa2Ccc4tF5ELgcOB2EdkEmAR8\n",
"QUTG4309nc65V4CLgJtFZFW8/+VQwtoc59xR0XZF5GngWOfcrTW6Xn4w71VRFKVNaM1CyzRYa9+0\n",
"1l5trd0fWGItSw2m4/0os4CrgP2cc3OAMcCm4S/OuYeAafjR0R3Afc6586INicgkEbka7/j/gYgc\n",
"W6PP0Z3dHUvVuKYoitLutCwXWT+MMZOALmCWtfYmY8zqwO/xI48bgcOttW9Za99ppD3n3ELgsITy\n",
"Z6lGmVXKrsIboVptPQDs0+BbWR6/ZkZRFEXpT+vWwcT4Kt7XsiCcnwF8Argd2A44ZajCtAidJmvf\n",
"PEtJlPMWoECU8xagQJTzFiAncttwbDugw1p7W/C57AP8ylo7FdgB2GuowrSItjcwbZxnaQlUF1VU\n",
"F1XaWBe5GZjF1trKgsdd8T6SXwNYa18EGpoaKwBtb2AURVFq0Lpkl/GOjTEV63YQ8KC19uHI9eGy\n",
"jF4NjKIoSjItTRUT5U6gN6SN+SxwfOWCMeYA4LWhCtMixuQtgKIoSkHJLUz5O+HvfkA3Pn0Mxpjf\n",
"Ar/Br10ZDqyQtwCKoigFJZ8wZWvtS8CnEsr3wxud4ULbj2DK5XKpjZ2Y/VBdVFFdVGljXRRvoSWA\n",
"MebIrNpqMjqCSc5Z1K6U8hagQJTyFqBAlPIWICfyGcFUMMYshV8IGb1XgGOBC4YqUAto+xGMoihK\n",
"DVqbTbmCMWYDfF6wnUke+Qx1j5hWoSMYRVGUZHKLIvs1MBb4Lj5V/uLY9eGyo6SOYBRFUZLJzcBs\n",
"Dmxea88XY8yOQxWmRegIRlEUJZncVvI/W29DMWvtF4cqTIvQEUz75llKopy3AAWinLcABaKctwA5\n",
"kdtK/j8ZY3avdTHsbDkcaPsRTJuGXyaiuqiiuqjSxrrIx8mPt2wXGmMeBB4C3ohcE2DyUIVpEZoq\n",
"RlEUJZncfDDfD3/Xxe9eGWe4RJHpFJmiKEoymS60TGNgHrDWfrjWRWPM/UMVpkW0/RSZoihKDXLz\n",
"wfxqgOsapqwoijK8yWdHS2vtxdFzY8yY2PXuoQrTIpbv7O4YLlsLNIVyuVzKW4aioLqoorqo0sa6\n",
"yM3JjzFmI+An+KSXKxhj3gRuAL5jrX1qqMK0iFHAaODtvAXJkRLtG4YZp4TqokIJ1UWFEu2pi3zW\n",
"wRhjNsTvCfMZ4IXw+gX83jB3huvDBfXDKIqiLEluyS5/CPwF+Lq19oVKoTFmLeAX4fr+QxWoRagf\n",
"RlEUZUny8cEAHwcOjBoXgHB+EDBcUsWAjmAURVGSyG0/mAXW2neSLoTyBUMVpoXoCEZRFGVJcgtT\n",
"fs8Ys03SBWPMh8nA2rWQdl/NX85bgAJRzluAAlHOW4ACUc5bgJzIbSX/pcDNxphfAX8DXgZWA3YA\n",
"jgFOH6owLaStDUwb51laAtVFFdVFlTbWRW5hymcBHwJOTrh2RbjeMCKyHHA+sFmQ4yTn3M016h4A\n",
"HIdPR3OLc25a5NrX8JFtAqwOXOCcG2hRaFsbGEVRlDhhfWA+qWKstYuBQ4wxFwJ74h/mLwG91tq/\n",
"DqLv6YBzzm0vIpsAd4jIFs65OdFKIrIVcAYwEXgF6BORo51z54YqRwC7OufmisjGwCMicqdz7u46\n",
"fauBURRF6U8/49LT1Tvk/JKpFloCWGtvB24fSqciMgpvGD4L4Jx7XETuBw4EzoxVPwLodc7NC/de\n",
"ApwIVAzMgc65uaGdJ0TkFWB9QA2MoihK42Tqf4F0Tv66GGNmpai+Id5/82ik7GFg24S628bqPQJM\n",
"FJHRAM65BysXRGRv4HXgpgH6VwOjKIrSn0z9L/EG+2GMOYXGU/ALMCFFv2uGv/MjZfOBLWvUjdZ7\n",
"NfQ3HngO3p9G+x3ecHzeOffaAP23tYEpl8ulNnZi9kN1UUV1UaVNdZFpiDLUnyI7JWVbg5mvi99T\n",
"KwllUtvv1w2jmIkisjVwg4h0qg+mLiXaNwwzTgnVRYUSqosKJdpPF5k6+OMNxqm7/0uclPvBVBz5\n",
"44C5kdcv1qg7LnI+Dm9wXopXdM79XUR6ga/jswv0Y9G8xZsBLPrPojWOv+34CVOnTp0NlJN+qYRs\n",
"qqUEeYZ9/YSyYSV/xvVL5XJ5eoHk0frFqF/5XhRFnqbXP2DFg1b6/Rt27JtvvsFbLy1YUUSmA8yY\n",
"MSOh2QZxziUe++677xG1rg21Pt73MxfYMVLWBxyXUPcs4KLI+UHAA+H1qsDnYvXPBq6JtzNjxgw3\n",
"9do97wnHr9K8t5F2zJw5c3reMhTlUF2oLlQX/ph67Z4TIs/IqyvlM2bMcINts6aTP77/SwMsSmHU\n",
"FgMXAocDhDDlScCVIjJeRG4TkVVC9YuAPUVk1RB9dih+/QzASsDJIjImtLM6fk1M3wAitPsUmaIo\n",
"SpzMo8hShynX4VjgshT1pwPni8isIMd+zrk5IrIusCk+X9grzrmHRGQaPjJsMX6h5XmhjeeBP+LX\n",
"xryDnz67GDiP+qiBURSlrejs7tgI+Dbwb+D0nq7exbEqrTMwxpjFeF9Ho7s/pnLyO+cWAocllD9L\n",
"NcqsUnYVcFWNNr4fjjS0u4Ep5y1AgSjnLUCBKOctQIEo5y1AE/gOsDXwYeCfwHWx660LU8Y73M+n\n",
"cQPz5aGL0zJaamBCCob18Gt6PoKfDnwZOK6nq3dOvXubQRuGX9ZEdVFFdVFlhOpi68jr3VnSwLQ0\n",
"TPn/rLWnNtqQMWavDORpFU3fD6azu2Mp/NbSHwW2A9aIVRkP7ErCyExRFKXJjE8oy3wEU8/Jv12a\n",
"hqy1Savwi0YlEGGZzu6OZZvc1zTgB0AHSxqXCis3WQZFUZQkkgxMoVPF3JdVW03kjcjrZk+TxQ3u\n",
"G8AtQDSlznJNlkFRFCWJpB+30Y0YF2bRSaooMmPM2sD+wMbA6MglwSeYLDpvUlXsCvi0M80iasC+\n",
"DtzZ09W7qLO7Yx9g+1CuO2sqitIqXgEqyz/o7O6QWMbk6Kjm5Sw6bHgEY4yZhE9IOQ2/fmUX4JPA\n",
"fsAhwFtZCNRk3oy8XrHJfUX9PA/1dPVWpueiW0vnYmDCKl4F1UUU1UWVEaqL+Khk1dh5fgYGv2Pl\n",
"1621awNEJSapAAAgAElEQVQPW2s3sNZOAMYCpwG/zEKgJhM1ME1z9IeosegIJtpv1MDkNUVWyqnf\n",
"IlLKW4ACUcpbgAJRyluAzu6OpTu7OzJzY9B/1gngA7Hz1SKv55IBaabI1rHW/iZeaK1dBJxijPkL\n",
"8NMshGoiLTEwwLJUjfc7PV29UYfZ25HXOkWmKArw/g/TjYCPh2MS8Hxnd8fhPV298zLoIh7YtA7w\n",
"j8h5dATTcgMTTQWztDFmVNjlssIHsxCoyUSd/M00MNG241OHRRjBKIpSAMII5ePAjuHvWrEqHwR2\n",
"AK7PoLuBRjCZG5g0w69RxpgJ4fUzwEmVC8aYY8k27UyzaNUIJjo9Vs/A6AhGUdqbk/AJffdmSeNS\n",
"YexQOwnr8uLP6HVi57mOYPqAWcaYHfC5vnqMMSfg84OtTPr9Y/JADYyiKEVi59j5G/ilDCvhF2lD\n",
"Ns+JpHV/749gOrs7lqG6LYrDR5wNmTQjmB8CuwH/sdZej89q/E/gCbwV/lEWAjWZIhiYIvhgyjn1\n",
"W0TKeQtQIMp5C1Agyi3qJxrNehQwpaer9zvAvZHyLNbsxafHoP8IJhpRNi8S9TokGh7BWGvnE9m6\n",
"2Fp7BXBFFkK0kFaFKRd6BDNC8ywNCtVFFdVFlVboIowaKqvn3wPujaxLiT43snhOJBmYNTu7O5YO\n",
"QUiZT49BunUw52bVaY7kMYJ5M3ZNnfyKokDsORFb9PhWjXqDJcnALEU1c33mIcqQzgezmzFmpzrX\n",
"HX7+8GFrbSZpBppANIqsVSOYqEGhp6v33c7ujvfwul+6s7tjmZ6u3kwSyymKMqyIPoPqzXRkYWBq\n",
"5V78APAcTRrBpDEwGwEzGTh9/1vGmDOstdMHLVXzaNUIJtp2fAQD3g9T+XKNIaPMpYqiDCvqzXRE\n",
"z5s1RQawdvib+Sp+SOfkPxh4FjgV+Bw+Tcze+IzBDwL7APviQ+6OMcYck5WQGdIqAxP9QiSl0Mnd\n",
"D6MoSu7U+yGa9QimloGpOPfz9cEAU4EOa+2p1tpua23ZWnttGKnsD3Raa6+x1p6M38zmK1kJmSF5\n",
"jGAWJFzP1Q8zQvMsDQrVRRXVRZUW6aJeMFDWPphaU2SV5L+5G5iNrbUPJV2w1j4IbBU5v5/+ewsU\n",
"hSI4+SH/EUwphz6LSilvAQpEKW8BCkSpBX00mvGjmSOYytqXqJM/symyND6YDYwxY621r8cvGGNW\n",
"AjaMFRfRr9CQk7+zu2MdfALPBcBJPV29r6Xsp94vE8jfwCiKkj9RA/NG7Fozw5QrQUZQoBHMncCf\n",
"jTGfMMYsC2CMWdYYszPwZyIbaRljvgi8k5WQGbIAH+0GsFxIn5DEefhEcx8Dpgyin4EMTBEWWyqK\n",
"ki+NrpfLeorsxcjrcSEfWlPClNMYmG/gNxq7BVhgjHkdr4SZwATgWABjzHXABXijUyhCnHndabLO\n",
"7o6J9E8Ct/EgukozgtG1MIrSnjTq5B+TQdr+6AhmTuT1yuGojGje6OnqzWyZSZqV/I8ZYz4EHIff\n",
"bGw1/FzdDOAsa+3cUO8zWQnXJN6gOj22AhCf/vpS7HwwU306RaYoykDU9MH0dPUu7uzuWED1+bBc\n",
"vE5KahmYcTRpegxSZkC21r4IfDtLAXKg5ggmjF52jNUfTDBA0afIyjn0WVTKeQtQIMp5C1Agyi3o\n",
"Y6BgoLeoPh+WJzsD81Lk9Vhgjch5pgYmy93Shgv1psi6EuoPZsV/vegQyHkEozmnqqguqqguqrRI\n",
"FwMtyM7SDxP1wbxFdeZG6B+g1ZoRjDHm58AL1tqfNdqYMUbwCTBvsNZelYF8zaCegUnaNG0wBiZq\n",
"NGqt5E+qqyhK+zDQD9EsI8miBuYdfOLilcL5JpFr0dHNkKk3gjkFONQY8ztjzFZ16gFgjCkBtwIr\n",
"FNi4QP2MyquxJIOZIiv0QktFUQpBI1NkSXUHQ/Q5sxB4NXK+aeT1C0Pspx81RzDW2jeMMbsAvwX+\n",
"YYx5HngM7yB6Gz+0Wh6fy2ZLvLPoUlKs4BeR5YDzgc2CLCc5526uUfcAfICBA25xzk0L5UsDXwU+\n",
"E2RaFviec+4vNbqtN4IZz5KkGsF0dncsS1Wv75Ecrq1OfkVR0kyRZTmCWUhk6xVgg8jraAjzkKnr\n",
"5LfWzjHGTAE6gEOAT9B/hasDZgPXABdYa+9O2f90wDnntheRTYA7RGQL51w0ygER2Qo4A5iI32mt\n",
"T0SOds6di5/W+jowyTn3uojsClwnIps55/6T0GfiYsvO7o7RVIeMUdKOYPrlIYul4K6gBkZRlDRT\n",
"ZFn6YCpTZBWi6wEzNTADOvmttc5ae721dl9r7Vr4h/C6+LUiK1hrN7LWfimtcRGRUcARwMUAzrnH\n",
"gfuBAxOqHwH0OufmOecccAl+9zfwzqqTnXOvh3Zm4EdY29foutYIJrqjW9RHktYHM9CXJt5+yw2M\n",
"5pyqorqoorqokkMusjynyKK0bgSThLX2DZZMazAYNsT7PB6NlD0MbJtQd1vgusj5I8BEERntnJsH\n",
"XFm5ICKVabJazqpaBiY6PfYM1XnJFTq7O0b1dPUurvNeogwUogz5+2BKaEhqhRKqiwolVBcVSjRf\n",
"FwNNkTVrBBOfIqvwDn6GKDPyDFOu7KQWfaPz6R+THa0brfcq3t+S5DPZGZjtnLu1Rr+1DEzUwT+H\n",
"wX+4aQ2MTpEpSpsR0lRV1qYsxj/042QZphxdB/MOySOYOTWm9AdN6hFME4i/oVobmiW98X51Q9DA\n",
"j4BDkxoolUrlpTdeapXROyy74XJjRi9cd+x6u5fL5VtDzHu/DXeWl+VHj2KpdQD2XnGfU8rlcsUw\n",
"lZNi5MOQurTH8h0fvG3BresArLLUKq5cLpcS6i9YltFjl5Vlx44dteIy5XJ5erSpeu0nvK3U9RPK\n",
"Mm1/mNUvtVr/Wn9Y1K98L5rS/pErHTX6qtevXOcd987r77DwP/EHe7lcLm0zetvd/vXOY+sAbLjM\n",
"RnuXy+X1hiBPvxHMpGW3Xv/Jd59cJ1pxlaXGUS6XS7vssgvRtmbMmJHQbGOId2m0nuDUfwxYwzk3\n",
"N5T9EljNObd/rO7tQLdz7oxwPhm4C1jeObcwlAlwOXCdc+6aeH99fX1uypQp0tndsQNwdii+s6er\n",
"9xiAzu6OL1NNE3MxfkO1SnTF53u6ep9s5H11dnd8EvhpOC33dPVOS6izEfC7cDq7p6t3n0bazopy\n",
"uTy9VCpNb2WfRUV1UUV1UaXZuujs7lgb+GM4faGnq3evhDpfAI4Pp7/r6epteE1iQlu/AbYIp4fg\n",
"p+Z/HavW29PVe0r83sqzczD95jlF9iQwD9g8UrYlkBQscHdCvQcrxiXwc+AO59w1IrKsiKxbo99G\n",
"fDAv02Bq/wQGctxB/j4YRVHypZGp9CwXWkanyN4meYos0zUwkKOBcc4tBi4EDof3RzSTgCtFZLyI\n",
"3CYiq4TqFwF7isiqIfrsUPz6GcK938KH2l0uIiviMyAfXqPrqOGoZWDmMvjNyaJfnKRFlvFyzUWW\n",
"L+W8BSgQ5bwFKBDlJrc/kIMfBumD6ezuWKuzu+Pgzu6O9SPF8TDlJAOTaQQZDMHAGGOy2LFyOn52\n",
"axZwFbBfWAMzBh/FNQbAOfcQMA24CbgDuM85dx7+5k2BHwNfA17Hhy0/SLLPBmobmPh+CIMdwaT9\n",
"4mgushxRXVRRXVRpgS4aWc4QfX409Jzo7O5YDbgMvzbwnEia/+hMyTssmUUemmBgUjn5jTGb41PI\n",
"7IYXch1jzE+Au621S/g9BiJMcR2WUP4s1SizStlVeCMUr/sv0hnKRqfI6qWUqUe/hZY16izEG0AB\n",
"lu3s7liqp6t3UYo+FEUZ3qSdSh9wBBMi035E9Vm2Fj5b8nz6j2De7unqfbezu+NN+j8D8xvBhL1g\n",
"7sKv6v8/qvukzALONMZMzVq4JhH90Fbo7O4YFax8dKFlfASTZopsoDxklY3Poost1Q+jKO1FIzMd\n",
"aZdKHMmS6wgrP47jU2Sw5DRZrj6Y0/G5xtay1m6Nd9Bjrb0O2BM/hVV4wkgh+sGtSPKOboMdwTTy\n",
"ywTU0a8o7UwjU2QNj2A6uzt2wmc8ibNSZ3eHsOQUGfRfW/hmT1dvFgvo+5FmimyitXaJUDoAa+1D\n",
"xpjBZB3Oi7nAeuH16vRfT1PZD2GwI5hGokMgf0e/oij5EX1O1HqwNxRFFpz5P6hxeSwQ9Ze/F5mO\n",
"j45gMp8eg3QjmPcGuL76UARpMVFlrsmS/hfIJky5kAZGc05VUV1UUV1UaYEuGhnBDDhF1tndsTzw\n",
"M6rPqOeB+yJVxrJkmpgK0RFM7gZmnjHmi0kXjDFfwPtlhgvRbM1rsmQEGTR/BJNnwstSi/srMqW8\n",
"BSgQpbwFKBClJrefOiluJCIMgDD1dQrVHSkXAifQ/1k8lv5rYKIGpukjmDRTZD8BrglG5hZgdWPM\n",
"yXinUgfJ2w0Xlagzaw36f5AVA5OFD6bREYz6YBSlvRjQV9vT1buos7vjbfzzQfCGIvrcOAiYEjn/\n",
"cU9X76Od3R2vR8riBia6P9UzkddPpJC9YRoewVhrrwW+jE83cAKwDnAqsBNwpLX2+mYI2CTiI5iB\n",
"psgaGsGEXxRjI0WFnCJTFCV3GokigxrTZCHl1Vcj12xPV2/lGVzPwER/TPfi9/K6hv7Z6jMj1ToY\n",
"a+2Fxpj/BXbA+1xeAv5qra2noCIS98FE5yIHNYIJxuVEfOw5wCJq77kAamAUpZ1pZIqscq2yhGJ5\n",
"4OXO7o4N8FG9lQHCA8BZkXuiiyjjPpj3RzA9Xb1v4RepN43B7gdzUxNkaSVxAxO18Ek+mLoGJhiX\n",
"acC+keJre7p6X69xC+S86ZiiKLkymOUMy3d2d6yMNyaVZ9ILwIk9Xb3vRupFn10rUXuKrOk0bGCM\n",
"MRvgt0x+xFp7dwhLPgefP+xG4CRrbaObcuVNPSf/v8PfhqbIgnE5Hvh8pPhGfGRHPfIcwZRb3F+R\n",
"KectQIEo5y1AgSg3uf3BTJGtBHwTv008+B+px/d09b4cuyc6glmR2k7+ppNmBPMNYA+8/wX8EO1g\n",
"4CG8b+a1UDYceA3/4SxH/4f7G1SNz0L8NNdSwOjO7o5lYr8SKsblm8B+keKbgO83kPolamDWiiWm\n",
"azZPn9n9s1b2V2RUF1VUF1WarYvB+GqPxyfyrfC9nq7exxLuic6crETtMOWmk8bAfALY2Vr7vDFm\n",
"WbxxudJae5AxZhPgDwwTA9PT1es6uzteBOJfoMcrG/+EOm/gV/mD/8Xxvk8lhAx+C9g7cv8M4OQG\n",
"84pFvzj7h0NRlPaj0RFM1Lic09PVW65xT3T2pV4UWdNJsw7GWWufD693xj94zwaw1j6O/7U/nEiK\n",
"+348dp7o6A9J5U6hv3Hpw/+iaFQPmef9URRl2DGf/v7YOEmjmz/hMybXYlhOkUXTDXwBeMpaG90c\n",
"bLj4XyrMSSiLx4Iv4Yfp7O5YBvgh/ePP/wScmjIjch+wHfChFPcoijJyWABc3tPVW+/ZGTcw/wR+\n",
"FN9iOcawnCJ71BhzMfAUcCD+IQuAMabEwKlkikbSCCZuYPqNYDq7O5bDLzjdMVJ+LX6BUyoDGxJq\n",
"nprmHkVR2o6osXgBmBaeHfVYiM92vwzeuKwUu9Yy0kyRfRv4OHAa8A9C3LUx5mx81FRP5tI1lyQD\n",
"82TsPDqC+QBwLv2Ny/8Cp6c1LnmjOaeqqC6qqC6qFEgXf8bPtjwPfDMhYmwJwugmapiiC8mLGaZs\n",
"rX0K2NwYM95aOzdy6YfAmQw/n0LcwDzX09Ubd7ZFz79D/2nCi4HzBxiqFpUSGpJaoYTqokIJ1UWF\n",
"EgXQRU9X79P47VDS8jrVBZpRA1PYKTIAYsYFa+0cAGPMRHzI8nAh7oNJysUTHcFUjIsDzuzp6v3f\n",
"pkilKIoydKIjmOg6v8JOkQ3EbzJsqxXERzDxCDJYMnzwXeC7alwURSk4w2uKzBgzE//rPQkBNslE\n",
"otbxOtXFlpA8gomOct7AO9juabZgiqIoQyQaqpzbFFmaEcy2eEMyKnKsDGwDbA4Mqwdv8J3MjhQ9\n",
"klDtz8CdwL3AkWpcFEUZJtTKpVhYH8wT1tpd4oXGmOXwDvD7lryl8JwNHAXc1tPV+1z8Yk9X73zg\n",
"mJZL1XzKeQtQIMp5C1AgynkLUCDKeQswRF6rUV7MKTJqRDJYa98GTjHGlGnSngLNoqer9y7grrzl\n",
"aDWlUqmctwxFQXVRRXVRZQToolYm92JOkUXSxNTigwNcVxRFUVrDGzXKizlFZoxZL6FY8BuPHUDt\n",
"N6QoiqK0llpTZMU0MPR3iMeZD5ihiaIoiqJkRK0psgU1yptCGgPzf8DJ+FFLhUX4FAZ3WWtrWcya\n",
"iMhywPnAZkGWk5xzN9eoewBwHD5U+hbn3LTY9cnA74DTnHOXp5VFURRlBJFkYN4geb1f00hjYGZY\n",
"a7N+cE8HnHNuexHZBLhDRLZwzvVbZS8iWwFnABOBV4A+ETnaOXduuN6J309lPrXX6iiBcrlcGgFO\n",
"zExQXVRRXVQZAbpIMjDlnq7eYu4HY609PMuORWQUcAQ+pxfOuceB+/GZmuMcAfQ65+Y55xxwCT68\n",
"uML9zrn9qD3vqPSnlLcABaKUtwAFopS3AAWilLcAQyTJwNzUaiGyTBWTlg3xOXIejZQ9jF/QGWfb\n",
"WL1HgIkiMhrAOfdss4RUFEUZhsQNzGvksCQjTwOzZvg7P1I2H1ijRt1ovVfxvqDxCXUVRVHamoTN\n",
"D+/o6ept+Z5deRqYCnGfiSTWSvat1KqrKIqiVEkMnmo2qdP1Z0jFkT8OmBt5nbQR2JxwjUg9B7yU\n",
"psOo027y5Mmzp06dOhsoJznzwoZDpYRmhn39hLJhJX/G9Uvlcnl6geTR+sWoX/leFEWe1PW3G/2R\n",
"Zx5Y+MCW77DwRuDWRtvfZZddiF6bMWNGQrUGcc7lcuBHT3OBHSNlfcBxCXXPAi6KnB8EPJBQbyZw\n",
"cFJ/M2bMcHm916IdM2fOLOUtQ1EO1YXqYiTrYuq1e44ZahtDeXZmNkVmjPllSsO2GLgQOBwghClP\n",
"Aq4UkfEicpuIrBKqXwTsKSKrhuizQ/HrZ+IIOm02IMM8/DJTVBdVVBdVRoouerp6W7qwMk7NKTJj\n",
"zM40vqZEgE8Pov/pwPkiMivIsp9zbo6IrAtsCowBXnHOPSQi0/BhdovxCy3Pe79zv07mFGBL4BgR\n",
"2cE59+VByKMoiqJkRD0fzMyUbaVe4OicWwgcllD+LNUos0rZVcBVNdp5ENg3bf+KoihK86hnYJ4E\n",
"vkjjU04XDl0cRVEUZaRQz8DMsNbe0mhDxpghhBooiqIoI42aTn5r7VfSNJS2vpIfIURRQXURRXVR\n",
"RXWRDanWwRhjRuMjuHYDRltrpxpjDgT+Zq19qgnyKc2hxPDfEjYrSqguKpRQXVQooboYMg2HKRtj\n",
"VgPuAc4DPgdMDpfWB2YZYyZmL56iKIoyXEmzDuZ0fA6wErAqYcW9tfZHwDHAqVkLpyiKogxf0hiY\n",
"3YAOa+2t1tpXoxestVcDG2QqmaIoijKsSWNgFg6wa+WKQxVGURRFGTmkShVjjNmmRvkk/PbJyvCg\n",
"nLcABaKctwAFopy3AAWinLcAI4E0UWSXA38xxpyPz8w5xhjzCfxmYCcAP2+CfEoTGCl5lrJAdVFF\n",
"dVFFdZENaQzMz4D/Ak4MB0BlIeZvgDMzlEtRFEUZ5jRsYKy1i4ADwghmD2B1/H4svdbavzZJPkVR\n",
"FGWYknrDMWvtbcBtTZBFURRFGUFkuR/MNVm1pSiKogx/Gh7BGGMOoXZKfsE7+5VhQLlcLqkT06O6\n",
"qKK6qKK6yIY0U2SXNk0KpdWU0DDMCiVUFxVKqC4qlFBdDJk0BuYRYE/67w8zFtgKOAj4YYZyKYqi\n",
"KMOcNAbmbGvtvxPK/2mMuRk4B/hbNmIpiqIow52GnfzW2l/XuTYX2CwTiRRFUZQRQSZRZCGFzLgs\n",
"2lIURVFGBmmiyGayZBSZ4Bdcbo6mihlOlPMWoECU8xagQJTzFqBAlPMWYCSQZgSzLd6gjIoci4H7\n",
"gcOBkzKXTmkKGn5ZRXVRRXVRRXWRDTVHMMaYT+H9Kr+y1i4GnrDW7tIyyRRFUZRhTb0RzBnAhMj5\n",
"15oriqIoijKSqGdgnLX2m2H0AvDLeg0ZY07LTixFURRluFPPwCxljBmToq29hiqMoiiKMnKoF0V2\n",
"K/CwMWYWsBBYzxhzSY26AqyXpmMRWQ44H+/nWRo4yTl3c426BwDH4aPYbnHOTRtMO4pH8yxVUV1U\n",
"UV1UUV1kQz0DMw14E5gCrAKsCNRy8guwQsq+pwPOObe9iGwC3CEiWzjn5vRrWGQrvD9oIvAK0Cci\n",
"Rzvnzk3TjtKPEhqGWaGE6qJCCdVFhRKqiyFT08BYa9/EGxkAjDH3W2s/XKu+Meb+RjsVkVHAEcBn\n",
"AZxzj4vI/cCBLLkz5hFAr3NuXrj3EvyOmuembEcJ/PGPf5xQKpXyFqMQqC6qqC6qqC6yIc06mIHW\n",
"uaRZB7MhsBrwaKTsYZJT/m8bq/cIMFFERqdsRwnce++9E/KWoSioLqqoLqqoLrIhTS6yG4ZyPcaa\n",
"4e/8SNl8YI0adaP1XsVPyY1P2Y6iKIrSQjLb0XKQJKWeaaRevG6j7SiKoigtQpyrtUllEzv1zvjH\n",
"gDWcc3ND2S+B1Zxz+8fq3g50O+fOCOeTgbuA5fGRaw2109fX1/o3qiiKMgKYMmXKoH60p9kPJkue\n",
"BObhk2TeHsq2BK5PqHt3qEek3oPOuYUi0nA7g1WQoiiKMjhymSJzzi0GLsQnyayMaCYBV4rIeBG5\n",
"TURWCdUvAvYUkVVD1Nih+HUvddtp4dtRFEVREshligwgRIGdjx99LA18xzk3Q0TWBe4BPuyc+0+o\n",
"uz/wTXz25luccycM1E5L34yiKIqyBLkZmGaQVXaAkUCjuhCRHYFvAWPwm8b9GTjZjaAvxmCyPYhI\n",
"GXjaOXdY8yVsHSn/R7bA7/M0Fr/Y+jrn3HdbJWuzSfE/sjReDzvis5rMBY6q/AAeCQTf9u+A05xz\n",
"l9epl+656ZwbMQfwE+DS8HoT4GV8AEC83lbA88Cq+IizvwBH5y1/TrqYCewVXq+AX2f09bzlz0MX\n",
"kfod+HD4S/KWPcfvxcrAQ8Bm4XxV4Iq85c9JF0cDjwOjw/nlwNV5y5+hHjqB3wL3AgfXqZf6uZl3\n",
"mHJmRFb1Xwx+VT9+M7QDE6q/nx3Aec1dAhzVKlmbTUpdXOucuz7UexMfILF7i0RtOil1Ual/PP47\n",
"MaICQ1Lq4ovA7c65x0Ldec65g1sla7NJqYstgb875xaG8zvwqatGCvc75/YDXhugXurn5ogxMGSX\n",
"HWAk0LAunHNnx4rGACMpj1vabA8H4Y3s/BrXhzNpdPFJ4DUR+a2I3C4iF4nIuFYI2SLS6OIG4KMi\n",
"slqYLtsD6G2+iK3BOfdsg1VTPzdHkoHJKjvASGBQGQ5EZCn86OW8JsmVBw3rIvyjfBE4hxE2egmk\n",
"+V5sABwJnOqcq/ge/re54rWUhnXhnOsF/gf/cH0KeBvvt2w3Uj83R5KBqZBVdoCRQNoMB9PwU2Z3\n",
"N0mePGlEF18FLg5TISMmyCGBRnQxGviLc+6RcH4m8CkRWaupkrWeAXUhIgfil0dsDKwPLMAvj2hH\n",
"Uj03R5KBqUzrRIfx44AXa9SN13PAS80RreWk0QUAIrInsA3w7SbKlQcN6UJEVgY+A1xRKWq+aC0n\n",
"zfdiXqz8+fD3g02QKw/S6OJr+ACH+cH38AvgEBFZsckyFo3Uz82RZGCiq/orbInPBBCnZnaA5onX\n",
"UtLoAhH5GPB14CDnnBORjZsvYstoVBfbASvh9xuaCRwCfFpE/hL0MxJI8734O/2niyrTICMlNDeN\n",
"LpYB3oucv4f/AbJM06QrJumfm3mHyGUcbvdjQmgpPuxwLv6fZDxwG7BKuDYR/4+yKt7I9gFfyVv+\n",
"nHSxBXAn/pfpiuH4Td7y56GL2D3TCSGsI+lI8b34GH6O/QPh/PvAzXnL32JdjAvXfhrOl46c35u3\n",
"/E3QRxk4JHI+5Odm7m8qYwWNBi4FZuGt7a6hfF380HedSN398RkD7gJ+lrfseekCn8NtET5LQuV4\n",
"Km/58/pehPIL8WtAngZ+X3mwjIQj5f/IocA/gFuCHlbPW/48dIFPrHt+qPNXfFTZRnnLn6Eetgqf\n",
"75zwPLygznci1XNzRK3kVxRFUYrDSPLBKIqiKAVCDYyiKIrSFNTAKIqiKE1BDYyiKIrSFNTAKIqi\n",
"KE1BDYyiKIrSFNTAKIqiKE1BDYyitBBjzEbGmBNb0M/Jxph1m92PotRDDYyitAhjzA74VfF3Zdzu\n",
"740xd8aK7wdmGWMmZ9mXoqRBDYyitABjzHjgOuBH1tpyxs0/B8yOFlhrr8fvYdJjjFk54/4UpSGW\n",
"zlsARWkTvg28SxP2EbHWfqPGpV+Ffk8Avpd1v4oyEJqLTFEGwBhzBbAnPovsv4B98Jlmz8NvvfsQ\n",
"0Gmt/b8a9wvwAjDDWntApHwP4HR82vOr8Hu9HwBshE8n/zV8SvkzgU3xma5/aa39n0gbfwn11wUm\n",
"WGufifV9NfAxa+1I2cdFGUboFJmiDIC19mCgA591+q/W2gfDNNdfgO9Za7epZVwCmwGrA4/F2r3B\n",
"WvthfAr03YAF1tqd8AZjNNCNN0BftNZOAk4Ezgy+nEobn8Sn0q/FI8A6xphN0rxnRckCNTCK0gDW\n",
"2juBHwGHGWP2NsbsBWxhrf1ZA7dXNnCbU+O6AK9Ya68Ifb2NT5++PtBtra3sg/4H4C3gUwn316Ky\n",
"Q+NI2kROGSaogVGUxjkNHwH2a+DnwMEN3ldxsr9dp84jsfOX4+XWWhfK126wX/D7x0dlUJSWoQZG\n",
"URrEWrsIOBzvi5kzwLRYlMXhb62RhgPeSCgjoXwx6YJzKv/ji+vWUpQmoAZGUdJxGH4X0I8bY45r\n",
"8J554e/yzRGpLiuEvy/XraUoTUANjKI0iDFmF2BHYBfgauB0Y8x/NXBrZZorzdRWVlT6jE/BKUrT\n",
"UVIm+FUAAAEtSURBVAOjKA1gjFkFH5Z8UJgqOwo/MrnSGDO63r0hdPhJ/N7nSQi1p8/i5WnqAnwI\n",
"eMRa+596MipKM9B1MIoyAMaYH+Ad+isD11lrDzXGfB84Br8e5mng59ba8+q0cQLwXWA9a+1roewj\n",
"+ICBLYA3gX8D2wIXA58EPgg8CpwD3FOjbh9+LU6l7v+z1v44tL9aqPdta+2vstKHojSKGhhFaQFh\n",
"lHMfPuz4uy3q83+ATwAfCaMuRWkpOkWmKC3AWrsQ2APoMsZ8qdn9GWOOB3YGOtS4KHmhBkZRWkTw\n",
"xWwHvNSC7v4NfNRa+0IL+lKURHSKTFEURWkKOoJRFEVRmoIaGEVRFKUpqIFRFEVRmoIaGEVRFKUp\n",
"qIFRFEVRmoIaGEVRFKUpqIFRFEVRmsL/Bx93WANw4iXZAAAAAElFTkSuQmCC\n"
],
"text/plain": [
"<matplotlib.figure.Figure at 0x7f0ba4c47fd0>"
]
},
"metadata": {},
"output_type": "display_data"
}
],
"source": [
"# plot fuel consumption\n",
"fig,ax = plt.subplots()\n",
"c4 = \"#4daf4a\"\n",
"consump = r(v, traceu[:-1], traceh[:-1])/v * dx * 100 * gallonperj\n",
"ax.plot( xs[:-1]/1609, consump, lw=3, alpha=0.8, c=c4 );\n",
"ax.set_ylabel(\"fuel use (gallons/100 mi)\");\n",
"ax.set_xlabel(\"x (mi)\");\n",
"gallons = consump.sum()/100.0\n",
"mpg = 100./consump.sum()\n",
"ax.set_title(\"tot={:0.2g} ga={:.1f} mpg\".format(gallons, mpg));"
]
},
{
"cell_type": "code",
"execution_count": null,
"metadata": {
"collapsed": true
},
"outputs": [],
"source": []
}
],
"metadata": {
"kernelspec": {
"display_name": "Python 2",
"language": "python",
"name": "python2"
},
"language_info": {
"codemirror_mode": {
"name": "ipython",
"version": 2
},
"file_extension": ".py",
"mimetype": "text/x-python",
"name": "python",
"nbconvert_exporter": "python",
"pygments_lexer": "ipython2",
"version": "2.7.9"
}
},
"nbformat": 4,
"nbformat_minor": 0
}
Sign up for free to join this conversation on GitHub. Already have an account? Sign in to comment